You are on page 1of 333

GRADE 11

GEOMETRY
GEOMETRY - 11

CHAPTER 1 CHAPTER 3
POLYHEDRONS ANALYTIC ANALYSIS OF LINES

A. PRISMS ................................................ 10 A. ANALYTIC ANALYSIS OF POINT .......... 216


B. TYPES OF PRISM ................................... 14 B. ANALYTIC ANALYSIS OF
C. SURFACE AREA OF A PRISM .................. 27 TRIANGLE .............................................. 229
D. VOLUME OF A PRISM ........................... 33 C. ANALYTIC ANALYSIS OF LINES ............ 236
E. PYRAMIDS AND PYRAMIDAL D. FURTHER APPLICATIONS ................... 276
FRUSTUMS ............................................... 57
F. TYPES OF PYRAMID .............................. 59
G. FRUSTUM OF A PYRAMID .................... 72
H. SURFACE AREA OF A PYRAMID ............ 76
I. VOLUME OF A PYRAMID ....................... 85

CHAPTER 4
ANALYTIC ANALYSIS OF CIRCLE

A. EQUATION OF CIRCLES ...................... 296


B. POSITIONS OF LINES AND
CHAPTER 2 CIRCLES ................................................. 312
SOLIDS OF REVOLUTION C. POWER OF A POINT .......................... 329
D. RADICAL AXIS ................................... 333
E. RADICAL CENTER ............................... 336
A. CYLINDERS ........................................ 109
B. CONES AND CONICAL FRUSTUMS ...... 133
C. CONIC SECTIONS ............................... 160
D. SPHERES ........................................... 169
CHAPTER 1

POLYHEDRONS
A. BASIC CONCEPTS
Definition Prismatic Surface
Consider a fixed polygon and a line d in
Prismatic k
space which is not parallel to the plane of surface d

the polygon. E
directrix
A solid prismatic surface is the surface A D
formed by all lines that intersect the sides generatrix
B C
of the polygon and are parallel to d. The
parallel lines are called the generators of
the surface and the polygon is called the element

directrix.

Definition Prism

A prism is a polyhedron formed by the intersection of a


closed prismatic surface and two parallel planes.
P1

An n-ssided prism is a polyhedron made of a polygon with n sides


(the base), its translated copy (the opposite base), and n P2

faces joining the corresponding sides of the bases.

Prisms are named by their bases. For example, a triangular prism has triangles as its bases
and a pentagonal prism has pentagons as its bases. All cross-sections parallel to the bases of
a prism are congruent.

10 Solid Geometry
The following figure shows the different elements of a prism.

Base (upper)

Lateral face
Altitude any face of a prism
any line segment which is not a base
which joins the two
bases and is perpendicular Space diagonal
to both bases any line segment
which joins two
Face diagonal
vertices not in the
any line segment
same face
which joins two vertices
of the same face Basal edge
Base (lower)
any side of a base

The length of any altitude is called the height of the prism. The height is the perpendicular
distance between the bases.
An edge of a prism formed by two adjacent lateral faces is called a lateral edge. The lateral
edges of any prism have equal lengths.
C‘ B‘
Note that the length of a lateral edge is not
D‘ A‘ the same as the height in slant (oblique)
prisms.


E‘
E‘ F‘
ge
Ed

h We usually name a prism using the vertices of


 he
height
eig
e ght
nt
Sla

its lower and upper bases, but we can also


C B
α name a prism by one of its diagonals. For
D A example, the prism in the figure is
ABCDEFABCDEF or FD.
E F
Obviously, a prism has at least one pair of parallel congruent faces which are the bases. All
the other faces are lateral faces. Because the bases are parallel, the lateral faces of a prism
are always parallelograms.

EXAMPLE 1 Find the height of an oblique pentagonal prism with 60° inclination if its lateral edge is 8 m.

Solution The lateral edge is 8 m and the edge makes a 60° angle with
the base.
8
l h Hence h   and h =   sin 60°,
3
60° i.e. h  8   4 3 m.
2

Polyhedrons 11
Note
An n-gonal prism has n + 2 faces, 2n vertices and 3n edges.
It has n(n – 1) face diagonals and n(n – 3) space diagonals.

EXAMPLE 2 Find the number of faces, vertices and edges of the prism at the right.

Solution The prism has a pentagonal base. By the result above,


it has 5 + 2 = 7 faces, 2  5 = 10 vertices and 3  5 = 15 edges.
These can be easily observed in the figure.

EXAMPLE 3 A prism has 21 edges. Find the number of faces and vertices of this prism.

Solution If a prism has 21 edges, this number is three times the number of vertices at its base. So the
prism is a heptagonal prism, i.e. its base has 7 sides. So it has 7 + 2 = 9 faces and
7  2 = 14 vertices.

EXAMPLE 4 A prism has 18 vertices. Find the number of space diagonals in this prism.

Solution An n-gonal prism has 2n vertices, so 2n = 18, n = 9.


By the result above, a nonagonal prism has 9(9 – 3) = 54 space diagonals.

EXAMPLE 5 The sum of all the edge lengths of a hexagonal prism is 86 cm. If the length of one lateral
edge is 5 cm, find the perimeter of a base.

Solution Let the basal edges be a, b, c, d, e and f.


Then the base perimeter is Pbase = a + b + c + d + e + f.
Since the prism has 6 lateral edges,
we have 2 · Pbase + (6 · 5 cm) = 86 cm.
Hence Pbase is 28 cm.

12 Solid Geometry
Remark Parallel Sections of a Prism
Parallel sections of a prism are two or more sections of a prism which are produced by parallel
planes cutting all the lateral edges. Parallel sections of a prism are congruent polygons.

A1 E1

B1 D1
C1
A E P1

B D

C
P2

If the parallel planes are also parallel to the bases, the polygons formed are congruent to the
bases.

Definition Right Section

A right section of a prism is a section made by a plane which intersects


all the lateral edges of the prism and is perpendicular to these edges.
D C

A
B

Conclusion

 The bases of a prism are congruent polygons.


 The lateral faces of a prism are all parallelograms.
 Parallel sections of a prism are congruent.

Polyhedrons 13
B. TYPES OF PRISM
Definition Right Prism, Oblique Prism
If the lateral edges and faces of a prism are perpendicular to the bases, the prism is called a
right prism. If a prism is not right, it is called an oblique prism.

In an oblique prism, some of the faces can be rectangles but at least one pair of faces must
be parallelograms.
In a right prism, the lateral faces are all rectangles and the lateral edges are all altitudes. If
we consider a rectangle to be a special type of parallelogram, we can consider a right prism
to be a special type of oblique prism.

Definition Regular Prism


A prism with bases which are regular polygons is called a regular prism. Some mathematicians
say that a regular prism can be either right or oblique. In this book, we will assume that a
regular prism is always a right prism.

EXAMPLE 6 The regular hexagonal prism in the figure will be made out of sticks. If the
prism has basal edge 5 cm and lateral edge 7 cm, find the total length of all
the sticks.

Solution The prism has a hexagonal base, so the base perimeter is 5 . 6 = 30 cm.
The sum of the lateral edge lengths will be 7 . 6 = 42 cm. So the total edge lengths will be
2  30 + 42 = 102 cm.

14 Solid Geometry
Regular prisms with identical edges are a type of semiregular polyhedron.
The following figures show the nets of some of these polyhedrons.

Regular
Regular triangular prism Regular pentagonal prism octagonal prism

Regular prisms with identical edges have very simple nets. These nets are all made up of two
regular polygons (the bases of the prism) connected by a ribbon of n identical squares.

EXAMPLE 7 Find the base area of a regular triangular prism if the length of a face
diagonal is 4 cm and all the edge lengths are equal.
m
4c

Solution The lateral face is a square. If the diagonal length is 4 cm then by the
2
Pythagorean Theorem, each edge is 2ñ2 cm. So the base area is (2 2 )  3 =2 3 cm 2
4

Check Yourself 1
1. Find the number of faces, edges and vertices of the adjacent prism.

2. A student uses 39 identical toothpicks to create the edges of a right prism. How many
square faces does this prism have?
3. If a prism has 9 faces, how many space diagonals does it have?

4. What is the height of the adjacent oblique prism? 12 m

60°

5. A prism has 10 vertices. How many face diagonals does it have?


6. A right hexagonal regular prism with identical edges has total edge length 90 cm. What is
the length of its longest diagonal?
Answers
1. F:10, E:24, V:16 2. 13 3. 7 . (7 – 3) = 28 4. 6ñ3 m 5. 20 6. 5ñ5 cm

Polyhedrons 15
C. SOME SPECIAL PRISMS

1. Parallelepiped

Definition Parallelepiped

A parallelepiped is a prism with six faces which are all


parallelograms.
We can group the faces into three congruent pairs.
If no face of a parallelepiped is a rectangle, no vertex of
the parallelepiped includes a right angle.

A rectangle is a special type of parallelogram. Therefore in some cases, a parallelepiped can


have one or two pairs of rectangular faces:
The net of a
parallelepiped

q
a a

A parallelepiped with one pair of rectangular faces A parallelepiped with two pairs of rectangular faces

Property Space Diagonals of a Parallelepiped, Center of a Parallelepiped


The space diagonals of any parallelepiped all intersect at the same point and bisect one
another. The intersection point is called the center of the parallelepiped. Note that the space
diagonals are of different lengths.
D¢ C¢ In parallelepipeds, each pair of parallel faces can be called
A¢ B¢ the bases of the prism, and a parallelepiped has three sets of four
P congruent edges. For example, in the figure opposite,
AB  ABCD  CD
D C AD  AD  BC  BC
A B AA  BB  CC  DD

16 Solid Geometry
EXAMPLE 8 What is the height of the adjacent
parallelepiped if its lateral edge is 10 m? l
10 m 45°

6m
13 m

Solution The dimensions of the rectangular base are


not important in this question. The lateral l h
h
edge is 10 m and it makes a 45° angle with the 10 m 45° 45°
base.
6m
So h = l · sin 45° 13 m
2
= 10 . =5 2 m.
2

6ñ3 cm

EXAMPLE 9 Find the length of the space diagonal of the parallelepiped


6 cm
6ñ2 cm

shown in the figure, using the information given.

30°

Solution In order to find the red space diagonal AC in the figure, we first need to find the length of
the green face diagonal BC. This is because ACand BC are part of a right triangle.

If we extend the side BC to make BT, we get height TC. C¢


6ñ3 cm B¢
6ñ2 cm
By trigonometry in triangle CTC, we have TC= 3 cm A¢
6 cm
and TC = 3ñ3 cm. B
T
Hence BT = 6ñ3 + 3ñ3 = 9ñ3 cm. C 30°
30°

Applying the Pythagorean Theorem in right triangle BTC, D A

we get BC2 = BT2 + TC2 = (9ñ3)2 + 32

and so BC2 = 252.

We know AB = 6ñ2 cm. Applying the Pythagorean Theorem in right triangle ABC gives us

AC2 = AB2 + BC2 = (6ñ2)2 + 252 = 324, i.e. AC=18 cm.

This is the length of the space diagonal.

Polyhedrons 17
Now we will consider three special types of parallelepiped: a cuboid (six rectangular faces),
a cube (six square faces), and a rhombohedron (six rhomboid faces).

2. Rectangular Solid (Cuboid)

Definition Cuboid, Rectangular Solid

A cuboid (also called a rectangular solid) is a special


type of parallelepiped. It is a parallelepiped whose
faces are all rectangles. In a cuboid, all the faces join
c
each other at right angles. Four edges which meet
at a common vertex of a cuboid form the dimensions b
a
(a, b and c) of the cuboid.

A net of a cuboid We can calculate the lengths of the face diagonals of a


cuboid using the Pythagorean Theorem, since each diagonal dac
dbc
is the hypotenuse of a corresponding right triangle. For c
dab
example, in the figure, b
a
dab = a2 + b2 dac = a 2 + c 2 d bc = b 2 + c 2 .

EXAMPLE 10 Find the lengths of the face diagonals of a cuboid with dimensions 3 m × 4 m × 6 m.
6m
Solution Applying the formula given above, we have d1 3m

d1  32  62  3 5 m, d2
d3
4m
d2  32  42  5 m,
3m
d3  62  42  2 13 m.
6m

Property Space Diagonals of a Cuboid


C¢ B¢
In a cuboid, the four space diagonals have the same
lengths and they bisect each other at the center of the
polyhedron. In the figure opposite, C P
B
AC  BD  CA  DB and P is the center of the cuboid. D¢ A¢

This property is a special case of the property of the space


diagonals of a parallelepiped.
D A

18 Solid Geometry
Theorem Length of the space diagonal of a cuboid
In a cuboid, the length of any space diagonal is the square root of
the sum of the squares of the dimensions of the cuboid. d
c

d = a2 + b 2 + c2
abc b
a

Proof In the adjacent cuboid, AG is a space diagonal and AC is a H G


face diagonal of base ABCD. The edge GC is perpendicular to
c
AC because if a line is perpendicular to a plane then it is E
D F C
perpendicular to any line on that plane.
b
In order to find the length of AG, which is the hypotenuse of
A a B
the right triangle ACG, we first need to find AC.
AC is the hypotenuse of the right triangle ABC, so AC2 = a2 + b2.

In right triangle ACG we have AG2  AC 2  CG2 and d2  a2  b2  c2 ,

so d  a2  b2  c2 , as required.

EXAMPLE 11 A rectangular box has dimensions 3 cm, 5 cm and 6 cm. What is the length of its space
diagonal?

Solution The space diagonal length will be d  a2  b2  c2 .

d  32  52  62  9  25  36  70 cm

EXAMPLE 12 In the figure, the bird wants to fly from corner D of G


F E H

the room to its nest at F. Find the shortest distance


the bird needs to fly if the room dimensions are 9m
B A
9 m × 12 m × 20 m. 20 m

C 12 m D

Solution The shortest distance from the bird to its


F
nest is the space diagonal DF.

DF  a2  b2  c2  92  122  202

 25 m
D

Polyhedrons 19
EXAMPLE 13 Musa wants to travel by plane with his
spear which is 120 cm long. The airline
will only carry items which fit into a
30 cm × 60 cm × 100 cm rectangular box.
Can Musa fit his spear in the box?
(Do not consider the thickness of the box
sides.)

Solution The longest possible line in a rectangular solid is its space diagonal. The dimensions of the
box in this problem are 0.3 m × 0.6 m × 1 m. The body diagonal of this box is

d  a2  b2  c2  0.32  0.6 2  12  1.45 m.

Musa’s spear measures 1.2 m  1.44 m. Since 1.44  1.45 ,

Musa can fit his spear in the box.

Definition Square Box, Square Cuboid

A square box (also called a square cuboid) is a special


cuboid with a pair of square faces.

Right square box Oblique square box

EXAMPLE 14 A square cuboid room has floor area 169 m2 and height 5 m. What is the sum of all the edge
lengths of the corresponding cuboid?

Solution If the base area is 169 m2 then the basal edge will be a = ó169 = 13 m.
Look at the figure bellow. There are 8 basal edges a and four heights h.
The sum of the edge lengths is therefore
Sumedges = 8a + 4h = 8 · 13 + 4 · 5 = 124 m. 13 m

13 m

5m
h=5 m
13 m
a=13 m

20 Solid Geometry
EXAMPLE 15 In a square box, the edge length of the square base is 7 cm and the height is 12 cm.
a. Find the length of the base diagonal of this box.
b. Find the length of the space diagonal.

Solution a. The base diagonal is the diagonal of the square base. So dbase  2a2  2  7 2  7 2 cm.
b. We can calculate the length of the space diagonal of a square box in the same way as for
a rectangular box:

dsolid  2a2  h2  2  72  12 2  11 2 cm.

Euler Bricks
A Euler brick is a special type of solid which is named after the
e
famous Swiss mathematician Leonhard Euler. A Euler brick is a cuboid a
d
with the following special property: all its edge lengths and face diagonals
f
are integers. In algebraic terms, a Euler brick is a solution to the set of
b
equations c
a2 + b2 = d2 , b2 + c2 = e2 , a2 + c2 = f 2

in which a, b, c, d, e and f must all be integer.


This interesting structure in math has attracted many people who are trying to find which dimensions
make Euler bricks. The smallest Euler brick, discovered by Paul Halcke in 1719, has edge lengths
240, 117 and 44 units and face diagonals of 267, 244 and 125 units. Some other possible integer
values for the dimensions a, b and c of a Euler brick are (275, 252, 240), (85, 132, 720) and (693,
480, 140).
We can see that there are an infinite number of Euler bricks, since we can multiply the dimensions
of any Euler brick by the same integer to get a bigger one. A primitive Euler brick is a Euler brick
whose dimensions have no common factor (i.e. they are relatively prime). Euler bricks also have
the property that if (a, b, c) are the dimensions of a brick, the dimensions (bc, ac, ab) make another
Euler brick.
A perfect cuboid (also called a perfect box) is a special type of Euler brick with the property that
the space diagonal of the brick is also an integer. We can add this property to the system of equations
above as the equation a2 + b2 + c2 = g2, where g is integer.
Interestingly, nobody has yet found some possible dimensions of a perfect cuboid, but mathematicians
cannot prove that a perfect cuboid does not exist. This has created an unsolved problem in mathematics,
known as the brick problem, the diagonals problem or the perfect box problem. Research using
computer calculations has shown that one of the edges of a perfect cuboid must measure at least
4.3 billion units.

Some solutions to the set of perfect cuboid equations have been found for six integers and one
non-integer. One such set has the dimensions (104, 153, 672): for this set, all diagonals except one
face diagonal are integer lengths. Another set of dimensions is (18720, 211773121 , 7800).
This set has one non-integer edge length but all the diagonals are integers.

Polyhedrons 21
3. Cube

Definition Cube, Regular Hexahedron

A cube (also called a regular hexahedron) is a parallelepiped


whose faces are all congruent squares.
We can also say that a cube is a right square prism with congruent
edges, congruent faces and congruent space diagonals.

There are 11 possible nets for a cube. Four of them are shown
at the left. As a challenge, can you find the other seven nets?

EXAMPLE 16 The caterpillar in the figure wants to reach the grape


by moving only along the edges of the cube. The edge length
of the cube is 10 cm. Given that the caterpillar cannot travel
along the same edge twice, find
a) the shortest possible distance to the grape.
b) the longest possible distance to the grape.

Solution

a) Look at the figure above. The caterpillar b) To reach the same point along the
can reach the grape by moving along only longest possible path, the caterpillar
three edges. must move along seven edges.
This distance is 3  10 = 30 cm. This is 7  10 = 70 cm.
There are several variants of this path. There are several variants of this path.
Can you find two more? Can you find two more?

In a cube with edge length a units, any face diagonal is añ2 units. Can you explain why
this is so?

22 Solid Geometry
EXAMPLE 17 Find the area of triangle ACD in the adjacent cube if the D' C'
edge length is 6ñ2 cm.

Solution As we can see in the figure, ACD is an equilateral triangle A' B'
and we have
|AC| = |CD| = |AD| = 6ñ2 · ñ2 = 12 cm. D
C
Remember!
The area of this triangle is therefore
The area of equilateral
triangle with side
| AC |2 . 3 12 2 . 3 A B
a2 3 A(ABC) = = = 36 3 cm 2 . 6ñ2
length a is . 4 4
4

EXAMPLE 18 The cube from Example 16 is changed to a box with edge length
10 cm. A spider can move anywhere on the surface of the cube.
What is the shortest possible route from the spider to the grape?

Solution The shortest way for the spider to reach the grape is the direct path PQ shown on the net below.
We can calculate distance PQ with P
opening out
the net P
the Pythagorean Theorem: 10
10 10 Q
PQ  10 2  20 2  10 5  22.4cm.
Q

Theorem Space Diagonal Length in a Cube

The space diagonal length ds of a cube with a side length a is ds = añ3.

Proof Since a cube is a special type of cuboid, we can apply the space G F

diagonal formula d = a 2 + b 2 + c 2 to a cube. E


d
However, in a cube all sidelengths are identical: b = a and c = a. a
C B

Therefore we have d = a2 + a2 + a2 = 3a2 = a 3 . a


D a A

Conclusion

For a cube with edge length a units, the face diagonal length is añ2 and the space diagonal
length is añ3 units.

Polyhedrons 23
EXAMPLE 19 Find the greatest possible distance between any two points on a cube with edge length 6 m.
A
Solution The furthest points from each other on a cube are two non-adjacent
vertices of the cube.
The distance between these points is the length of the space diagonal
of the cube. By the previous theorem, this is
d = añ3 = 6ñ3 m. B

8 cm
K

EXAMPLE 20 In the adjacent cube, what is the length AK?

Solution Let us define a midpoint R like K.


To find AK, we must first find AR in right triangle ADR: A

K
AR2 = AD2 + DR2 = 82 + 42 = 80.
Now in right triangle ARK,
8 cm
B C AK2 = AR2 + RK2 = 80 + 82 = 144
R
AK = 12 cm.
A 8 cm D

21
G
EXAMPLE The solid shown opposite is an oblique parallelepiped. All side lengths
are 6 cm and the inclination angle is 60°.
A
Find the length of the space diagonal AG. 60°

B 6 cm C

Solution In order to find AG in right triangle ARG, we must first find AR E


6 cm
and GR. For this, we will work with three right triangles: RGC, F
G
ARD and ARG.
Since the inclination angle is 60°, angle RGC in triangle RGC is 30°.
30° 6 cm
D
1 1 A
In this triangle, RC = GC . =6 . = 3 cm. 60° R
2 2
B 6 cm C
Since R is the midpoint on DC, we have DR = RC = 3 cm.

On the other hand, GR = RC . 3= 3 3 cm.


In right triangle ARD, AR2 = AD2 + DR2 = 62 + 32 = 45.
Finally, in right triangle ARG we have AG2 = AR2 + RG2 = 45 + (3ñ3)2 = 72.
Therefore AG = ò72 = 6ñ2 cm.

24 Solid Geometry
4. Rhombohedron

Definition Rhombohedron

A rhombohedron is a three-dimensional figure like a


cube, except that its faces are not squares but rhombi.
Alternatively, we can say that a rhombohedron is a
parallelepiped whose edges are all the same length.

The three figures below show the orthogonal projections of a rhombohedron.

The open net of a


rhombohedron

Front view Top view Right view

Check Yourself 2 5m

1. In the adjacent figure, what is the height of the 6m


man from the ground? 60°
10 m
2. The perimeter of the base of an equilateral triangular oblique prism is 21 cm, and the
perimeter of a lateral face is 36 cm. What is the sum of all the edge lengths of this prism?
3. Find the sum of the edge lengths of a cuboid if it has dimensions 5 cm × 9 cm × 16 cm.
4. What are the lengths of all the face and space diagonals of a box with dimensions
3 m × 6 m × 8 m?
5. A cuboid has length 2ñ5 m, width 2ñ2 m and space diagonal 8 m. What is the height of
this cuboid?
6. The area of one face of a cube is 25 m2. Find the lengths of its face diagonal and space
diagonal.
7. The determined caterpillar, still in search of the shortest
distance along the edges of a cube to a grape, finds a space
diagonal connected directly to the grape.
Find the distance, to the nearest centimeter, that the caterpillar
must now travel if the edge length of the cube is 10 cm.
Answers
1. 3ñ3 m 2. 75 cm 3. 120 cm
4. face diagonals: 3ñ5 m, ò73 m, 10 m; space diagonal: 109  10.44 m 5. 6 m
6. face diagonal: 5ñ2 m, space diagonal: 5ñ3 m 7. 10ñ3  17 cm

Polyhedrons 25
Fun with Cubes
Some of the world's most popular games and puzzles are based on cubes and blocks. Here are some examples. Have you tried
them all?
Rubik’s Cube
Rubik's Cube is a mechanical cube puzzle which was invented in 1974. It takes its name from its inventor,
Erno Rubik, who is a Hungarian professor. The original Rubik's Cube has six colored faces with nine
colored stickers on each face. Each face can turn independently, meaning that the colors can be mixed
up. To solve the puzzle, each face must be returned to a single color.
Rubik's Cube, originally called the Magic Cube, began to be sold internationally in 1980. It quickly became very popular, and
in 1980 it received the German Game of the Year award for Best Puzzle. Over the next twenty years, 350 million cubes were
sold, making it the world's top-selling puzzle game of that period.
Rubik's cube is still popular with puzzle solvers around the world, and many other similar puzzles have been designed,
although not all by Erno Rubik. There are also international Rubik puzzle-solving competitions. The shortest single time for
solving the 3 x 3 x 3 cube is currently 5.66 seconds, recorded in Australia in 2011.

Picture Blocks
Puzzles like this are popular with young children. They are made from different
numbers of cubes (usually wooden), depending on the level of difficulty. The
child must put the cubes together so that all the upper faces make a picture.
As a cube has six faces, there are six puzzle pictures in each set of blocks.

The Soma Cube


The Soma cube has seven blocks, which are made from combinations of unit cubes. In the
original puzzle, all the pieces must be put together to make a 3 x 3 x 3 cube. There are also
other challenges to make different shapes.

The pieces of the Soma cube are all the combinations of four or fewer unit cubes which make convex
shapes (i.e. pieces with one inside corner). This means that there are six four-block pieces and one
three-block piece. The puzzle has 240 different solutions.

An Interesting Number Cube


You can make an interesting cube yourself by following these instructions: Find a blank cube, or make one from cardboard.
Write the number 1 on one face, and write 2 on the face opposite 1. Write 3 on a third face, and 6 on the face opposite this.
Write the numbers 9 and 18 on the remaining faces. Now look at the edges of your cube. For each edge, write the sum of
the numbers on the two faces that make the edge. Finally, for every vertex, write the sum of the numbers on the faces at
that vertex. Look at the numbers you have written. Do you have all the numbers from 1 to 26?

26 Solid Geometry
D. SURFACE AREA OF A PRISM
Definition Surface Area of a Prism, Lateral Surface Area
The surface area of a prism is the total surface area of its faces. Since a prism has two
congruent bases and several lateral faces, the surface area of a prism is As = Abases + Alateral.
The lateral surface of a prism is the surface created by all its lateral faces. The lateral surface
area (or lateral area) is the area of this surface.
Look at the following prisms together with their nets:

As we can see, each prism has two congruent bases. If we want to calculate the total surface
area, we must find the area of a base, and the total area of the lateral surface. Then we can
use the formula As = 2  Abase + Alateral.
Let us begin by looking at the surface area of an oblique prism.
1. Surface Area of an Oblique Prism
Look at the following oblique prism and its right section KLMNO.
E' A'
A' E'
B' D'
D' C' A'
B'
C'
l K
K O Lateral surface
Remember! L L
l M
The right section of a N N
M O K
prism is an intersection E
A A
area of a prism and a plane
which is perpendicular
D E
to the sides of the prism. B D A
B
C C

In the net of this pentagonal oblique prism, each lateral face is a parallelogram. The lateral
edges have identical lengths, say . Remember that the formula for the area of a parallelogram is
A = base × height. The lateral surface is the sum of the areas of these parallelograms:
Alateral = A(ABBA) + A(BCCB) + A(CDDC) + A(DEED) + A(EAAE)
=   KL +   LM +   MN +   NO +   OK
=   (KL + LM + MN + NO + OK) =   Pright section
Therefore the lateral surface area of an oblique prism with lateral edge  is Alateral= Pright section
As a result, the total surface area of an oblique prism is As = 2  Abase +  Pright section

Polyhedrons 27
EXAMPLE 22 The adjacent figure shows an oblique triangular C' F C
prism. The right section of the prism is an
equilateral triangle with sidelength 5 cm. If the
length of a lateral edge (AA) of the prism is B' E B
10 cm, find its lateral area.
A' D A
Solution It is understood from the question that the right section DEF is an equilateral triangle, so
DE = EF = FD = 5 cm. Also, we know that lateral edge AA = BB = CC = 10 cm.
The lateral area is the product of the perimeter of the right section and the length of a lateral
edge. So A1 = PDEF · AA = 3 · 5 · 10 = 150 cm2.

2. Surface Area of a Right Prism


We can apply the area formula we have just seen to right prisms. For a right prism, using the
formula is easy because the right section of any right prism is
a polygon congruent to the base.
h
A lateral edge of a right prism is congruent to the height of the
prism ( = h). d c
Look at the adjacent right pentagonal prism.
e b
In this right prism there are two congruent bases and the a
lateral surface includes five rectangles. Pentagonal right prism

Alateral = A1 + A2 + A3 + A4 + A5
= (h  a) + (h  b) + (h  c) + (h  d) + (h  e)
= h  (a + b + c + d + e)
= h  Pbase.
Therefore for a right prism, Alateral = h . Pbase and the total surface area is As=2  Abase + h  Pbase.

EXAMPLE 23 Find the lateral area of a right pentagonal prism with height 13 cm and basal edges 3 cm,
5 cm, 6 cm, 4 cm and 7 cm.
Solution By the formula we have just seen, the lateral area is
Alateral = Pbase · h = (3 + 5 + 6 + 4 + 7) · 13 = 325 cm2.

Conclusion
 The total surface area of any prism is the sum of the areas of its bases and the
lateral area:
A = 2 · Abase + Alateral.
 The surface area of a right prism is A = 2  Abase + h  Pbase.
 For oblique prisms the surface area is A = 2  Abase + l  Pright section.

28 Solid Geometry
EXAMPLE 24 The length of a basal edge of a regular hexagonal right prism is 5 cm and the height is
11 cm. Find the total surface area of the prism. F' E'

A' D'
Solution The lateral area of the prism is
B' C'
Alateral = PABCDEF · h = 6 · AB · 11 = 330 cm2.
11
The area of a base is

AB2 3 52 3 75 3 A D
Abase  AABCDEF  6  6  cm 2. 5 5
4 4 2
B 5 C

75 3
Therefore the total surface area is AS  2  Abase  Alateral  2  +330=15(22+5 3) cm 2.
2

4m

6m
EXAMPLE 25 What is the total surface area of the parallelepiped in the
figure?
60°
9m
Solution Look at the figure below. This parallelepiped is an oblique prism and has only one pair of
parallelogram faces. For the solution we will use the face PQRS as one of the bases.
The area formula for an oblique prism is As = 2  Abase+ l  PRight Section.
Here we need to find the perimeter of the right section of the prism.
The right section is the rectangle ABCD and BC = 4 m. D R

In order to find the other side AB, we apply trigonometry: A S


3
AB = l  sin60°, i.e. AB = 6  = 3 3 m. l=6m
2 C
Substituting the values in As = 2  Abase + l  Pright section 60°
Q
B 4m
P
gives us As =2  24 + 9  (8 +6 3 ) = 120 + 54 3 m 2.

EXAMPLE 26 The chocolate bar shown opposite has a box in the shape of a
triangular prism. CHO
COL
The base is an equilateral triangle with sidelength 6 cm. The ATE
length of the box is 30 cm. What is the total surface area of the
box, to the nearest square centimeter?

Solution The total surface area is given by the formula Aprism=2 . Abase + Alateral.
The lateral surface area is determined by the perimeter of the base:
Alateral = h . Pbase = 30 . 18 = 540 cm2.

62 3
So Aprism = 2 . Abase + Alateral = 2 . + 540 = 18 3 + 540  571 cm 2.
4
Polyhedrons 29
EXAMPLE 27 Find the total surface area of an isosceles trapezoidal prism with height 12 cm if the parallel
edges of the base are 2 cm and 8 cm and the legs of the base are 5 cm each.
2 cm
Solution The perimeter of the base is Pbase = 2 + 8 + 5 + 5 = 20 cm.
5 cm
Since the base is an isosceles trapezoid, h' = 4 cm and so
1 1
Abase  h  ( b1  b2 )  4  (2  8)  20 cm 2 .
2 2
5 cm hý 12 cm
Therefore the total area is
AS = 2 · Abase + Pbase · h = 2 · 20 + 20 · 12 = 280 cm2. 8 cm

3. Surface Area of a Rectangular Solid


EXAMPLE 28 Find a formula for the surface area of a rectangular solid with dimensions a, b and c.

Solution Let the base of the box be a rectangle with sides a and b,
then the height will be c.
Apply the formula AS = 2  Abase + Pbase  h: c
AS = 2  (ab) + (2a + 2b)  c = 2ab + 2ac + 2bc.
b
As we can see, the total surface area of the solid is two a
times the sum of the areas of each rectangular face. Can you see why?

Conclusion

For a rectangular solid with dimensions a, b and c, the total surface area is
AS = 2(a  b + b  c + a  c).

EXAMPLE 29 What is the total surface area of a closed rectangular box with dimensions 3.5 cm, 6 cm and
8 cm?

Solution The total surface area of the box is


AS = 2 · (ab + bc + ac) = 2 · (3.5 · 6 + 6 · 8 + 3,5 · 8) = 194 cm2.

EXAMPLE 30 The sidelengths of a rectangular prism have the ratio 1 : 2 : 3.


If the total surface area is 198 cm2, find the dimensions of the prism. k

Solution We can write the sidelengths as a = k, b = 2k and c = 3k. Then 2k


2 2 3k
As = 2(ab + bc + ac) = 2(k  2k + 2k  3k + k  3k) = 22k = 198 cm .
So k2 = 9 and so k = 3 cm. Therefore the sidelengths are a = 3 cm, b = 6 cm and c = 9 cm.

30 Solid Geometry
EXAMPLE 31 Find the total surface area of a rectangular prism if its dimensions are directly proportional
to 1, 2, 5 and the length of its space diagonal is ò60 cm.

Solution Let the dimensions be a, b and c, then we can write a = t, b = 2t and c = 5t. The space
diagonal is ò60 cm, so

d= a + b + c = t +(2t) +(5t) = 30t = 60 , which gives us t = ñ2.


2 2 2 2 2 2 2

So a = ñ2 cm, b = 2ñ2 cm and c = 5ñ2 cm.


When the area formula AS = 2 · (ab + bc + ac) is applied, we have
A = 2(ñ2  2ñ2 + 2ñ2  5ñ2 + ñ2  5ñ2) = 68 cm2.

For Experts
This rectangular box is made from bamboo sticks.
Find the total length of bamboo needed if the diagonal
shown is 5 m and we need exactly 24 m2 of paper to cover
the box.

4. Surface Area of a Cube


EXAMPLE 32 Find a formula for the surface area of a cube with side length a.

Solution This time we have a rectangular solid with congruent edges.


Use the formula for the surface area of a rectangular solid:
AS = 2(a  b + b  c + a  c) a

= 2(a  a + a  a + a  a) a
a
= 2  3a2.

For a cube with edge length a, the total surface area is AS = 6  a2.

EXAMPLE 33 The edge length of a cube is 15 cm. What is its total surface area?

Solution The surface area is AS = 6  a2 = 6 · 152 = 6 · 225 = 1350 cm2.

EXAMPLE 34 A student breaks a long stick into 12 equal parts. He uses the sticks to make a cube, then
covers the cube with exactly 294 cm2 of paper. What is the initial length of the stick?

Solution This problem is left as an exercise for you.

Polyhedrons 31
EXAMPLE 35 What is the space diagonal length of a cube if its total area is 216 m2?

Solution For a cube, AS = 6a2, and if AS = 216 m2 we get a = 6 m. Therefore the space diagonal length
is 6ñ3 m.

8 cm
EXAMPLE 36 A student removes a cube from the middle of a
rectangular brick to get the solid shown opposite. Find
the total surface area of the solid.
14 cm
Solution This problem is left as an exercise for you.
25 cm
For Experts

Find the area of the fabric which covers this mosquito


net for babies, given that the length is 10 dm, the
40° 40°
width is 6 dm and the triangular parts are isosceles 50°
50°
triangles. 6 dm
10 dm

Check Yourself 3
1. What is the lateral area of a regular octagonal prism if all its edges are 5 cm?
2. What is the total surface area of a rectangular solid with dimensions 2 cm × 5 cm × 7 cm?
3. Find the total surface area of a regular hexagonal prism with basal edges 4 m and height 7 m.

4. If we remove a cube from the vertex of a rectangular brick as shown


in the figure, what happens to the surface area of the brick?

5. Find the total surface area of a cube if one of its face diagonals is 5ñ2 cm.
6. The length, width and altitude of a rectangular prism are in the proportion 3 : 4 : 5. If a space
diagonal of the prism is ó200 cm, find the total surface area.
7. If the areas of three different lateral faces of a right rectangular prism are 12 cm2, 18 cm2,
and 24 cm2, find the shortest edge of the prism.
8. What is the total surface area of the right triangular prism in
5 cm
the figure?
2 cm
Answers 12 cm

1. 200 cm2 2. 118 cm2 3. 24(7 + 2ñ3) m2 4. It does not change.


2 2
5. 150 cm 6. 376 cm 7. 3 cm 8. 120 cm2

32 Solid Geometry
E. VOLUME OF A PRISM
We have seen how to find the surface area of different types of prism. Now we will look at
calculating the volume of a prism. We will begin with the volume of a right prism.

1. Volume of a Right Prism


How many blocks have been used to make this solid?

Instead of counting all the blocks one by one, we can look at a slice of the solid.

Look at the pattern of blocks at the right. This pattern is


repeated from the bottom to the top of the solid.

There are nine blocks in this slice. Looking at the figure we can see that there are six
identical slices in the solid. So there are 9 . 6 = 54 blocks in the solid.
How many apartments
are there in this building This example suggests a formula for the volume of a right prism.
if there are three
apartments on each
floor?

Theorem Volume of a Right Prism

The volume of a right prism is the product of the


base area and the height of the prism. Height Height

Volumeright prism = Areabase × Height


Base Base

Proof As we have seen in the example above, we can divide any prism into congruent layers (slices)
of 1 unit thickness.
Since the thickness of
1 unit each layer is 1 unit,
h we can say that the
The base area has the same magnitude as volume of a layer has the
1 unit same magnitude as the
the volume of a single layer. 1 unit base area.
1 unit

As a result, the total volume of a right prism is


Volumeright prism = Areabase × Height.

Polyhedrons 33
EXAMPLE 37 What is the volume of this rectangular solid?
3m
2m
5m

Solution

Layer 3 1m

Layer 2 1m

2m Layer 1 1m 2m 10 m3 1m
5m 5m

The measurements are in meters, so we can take each layer with a height of 1 meter. Let us
begin with the base layer (Layer 1).
Abase = 10 m2  Volumelayer1 = 10 m3
Therefore Volumesolid = Abase  h = 10  3 = 30 m3.
Notice that the volume of the solid is three times the volume of a single layer, which we find
from the base area.

EXAMPLE 38 What is the volume of this solid if each cube


is one cubic unit?

Solution The base consists of 4 cubes. Since the volume of the


base layer is 4 unit3, the volume of the solid is
x3
Volumesolid = Areabase × h = 4 . 3 = 12 unit3.

EXAMPLE 39 What is the volume of this solid if each cube is 1 cm3?

height

Solution This prism is shown horizontally, with its base facing us. The volume
Base
of of the prism is
10 cm2
Volumeprism = Areabase × h = 10 . 3 = 30 cm3.

34 Solid Geometry
EXAMPLE 40 The green star at the right forms the base of a right prism and has an area
of 23 cm2. What is the volume of the prism if its height is 8 cm?

Solution We know all the parameters in the formula Vprism = Abase  h.


So we get Vprism = 23  8 = 184 cm3.

EXAMPLE 41 Our classroom is in the shape of a rectangular prism with height 2.5 m and volume 100 m3.
Find the area of its ceiling.

Solution The classroom is a rectangular prism and its volume is given.


V = Areabase · h = 100 m3, therefore Areabase · 2.5 = 100. So the ceiling is 40 m2.

EXAMPLE 42 In a right square prism, the basal edge is 4 cm and the space diagonal is
9 cm. Find the volume of the prism.
h 9

Solution The space diagonal length for a square prism with base a × a and height h
is d  a  a  h  4  4  h  9 cm.
2 2 2 2 2 2

4
So h = 7 cm, and therefore the volume is Vprism=Abase× h=42.7 = 112 cm3. 4

8 cm

EXAMPLE 43 The prism shown opposite has a right trapezoidal base and a height of 6 cm

14 cm. What is its volume?


14

Solution The prism has a right trapezoidal base. We can calculate the area of this
3 cm
base using the area formula for a trapezoid:
base1  base 2 83
Area base  htrapezoid   6.  33 cm 2.
2 2

So the volume of the prism is Volumeprism = Areabase . h = 33 . 14 = 462 cm3.

EXAMPLE 44 Find the volume of the triangular right prism in the figure.
5m

6m
Solution This problem is left as an exercise for you.
8m
(Hint: Use Heron's formula for the area of a triangle.)
Answer: 48ñ6 m3 7m

Polyhedrons 35
EXAMPLE 45 The origami box in the adjacent picture is a regular hexagonal
prism with basal edge 5 cm.

If the box is 8 cm high, find its volume to the nearest cubic


centimeter.

Solution The box is a regular hexagonal prism and its volume is

3a2 3
Volume prism  Area base  h   h.
2
The basal edge is 5 cm and the height is 8 cm, so

3  52 3 .
Volume prism  8  300 3 cm 3 .
2

6m

EXAMPLE 46 3 cm

10 cm
5 cm

13 cm

The steel construction beam above is 6 m long.


If one dm3 of steel weighs 7.75 kg, what is the weight of this beam?

Solution This beam is a right prism with height 600 cm.


3 cm
Using the given dimensions of the beam, the area of its cross
3 cm section is 108 cm2.
5 cm 5 cm
The volume of the beam is therefore
10 cm
Volumeprism = Areabase × h = 108 . 600 = 64800 cm3.

13 cm

The weight formula is Weightobject = Densitymaterial × Volumeobject.


Applying the formula for the beam with the volume converted to dm3, we have
Weightbeam = Densitysteel × Volumebeam = 7.75 × 64.8 = 502.2 kg.

36 Solid Geometry
EXAMPLE 47 The jug in the adjacent picture is in the shape of a regular
octagonal prism. Its octagonal base has radius 8 cm.
If the depth of the water in the jug is 15 cm, find the volume of
the water in the jug, to the nearest cubic centimeter.

Solution A regular octagon consists of 8 identical isosceles triangles with apex 360°  45°.
8
We can find the area of a triangle from two sides and an included angle using trigonometry:

1
Atriangle = ab  sin  a
2
a
b

So the area of the octagonal base is r


1 45°
Aoctagon = 8  Aisosceles = 8  r 2sin 45°  181 cm 2. r
2

This gives us the answer

Vwater = Aoctagon  h  2715 cm 3.

2. Volume of a Rectangular Solid

Theorem Volume of a Rectangular Solid


If the dimensions of a rectangular solid are a,
b and c then the volume of the solid is the
product of the length of these three edges: c

V = a  b  c.
b
a

Proof Just like a right prism, a rectangular box can be


separated into layers. If the basal edges are a and b
c
then the height is c. We know

Volumeright prism = Areabase × Height.


1 unit b
The base area here is a . b square units. a

Therefore the formula becomes Volumerectangular solid = a . b . c.

Polyhedrons 37
EXAMPLE 48 What is the volume of the adjacent box if the dimensions of
the box are 11 cm × 22 cm × 33 cm?

Solution The box is a rectangular prism, so the volume is


Vbox = a · b · c = 11 · 22 · 33 = 7986 cm3.

EXAMPLE 49 An auditorium in the shape of a cuboid has volume 288 m3, length 4 m and width 8 m. What
is its height?

Solution The volume 288 m3 is the result of the product of the three dimensions of the auditorium.
Vbox = length · width · height = 4 · 8 · h = 288 m3. So the height is h = 9 m.

EXAMPLE 50 Find the volume of water required to fill this swimming pool
if its length is 50 m, width is 25 m and depth is 2 m.

Solution The pool is in the shape of a rectangular solid.

Vpool =  · w · d = 50 · 25 · 2 = 2500 m3. So we need 2500 m3 of water to fill the pool.

EXAMPLE 51 If the length, width and height of a rectangular solid are all doubled, what happens to its
surface area? What happens to its volume?

Solution For a rectangular solid, AS = 2(a · b + b · c + a · c). If the dimensions are doubled, the new
area will be ASnew = 2(2a · 2b + 2b · 2c + 2a · 2c) = 4 · 2(a · b + b · c + a · c). So the
surface area increases by a factor of 4.

The second part of the question is left as an exercise for you.

This question is related to the topic of similar solids, which we will study later in this book.

EXAMPLE 52 The base perimeter of a cuboid is 8 cm. If the height is 3 cm, draw the graph of its volume
V versus its length x. y
12
Solution The base perimeter is P = 2(w + x) = 8 where w is width.
So w = 4 – x, and since V = w . x . h we
have V = (4 – x) . x . 3  V = 3x(4 – x).
x
This is a quadratic equation with the parabolic graph shown 0 4
opposite.
38 Solid Geometry
3. Volume of a Cube

Theorem Volume of a Cube

For a cube with edge length a, the volume is V = a3.

a
a
Proof A cube is a right prism. Apply the formula for the volume of a right prism:
Volumeright prism = Areabase × Height.
In a cube with edge length a units, since the base area is a2 and the height is also a, we have
Volumecube = a2  a = a3.

EXAMPLE 53 What is the volume of the adjacent cubic gift box if one edge is 22 cm?

Solution The gift box is a cube.


Vbox = a3 = 223 = 10648 cm3

EXAMPLE 54 Find the surface area of a cube if its volume is 343 mm3.

Solution The volume of a cube is Vcube = a3, and if this is 343 mm3 we have a = 7 mm.
Applying the surface area formula, Acube = 6a2 = 6 · 72 = 294 mm2.

EXAMPLE 55 The stack of paper notes in the picture makes a cube with edge length
8 cm. If there are 750 pieces of paper in the stack, find the volume of a
single paper note.

Solution The volume of the whole paper cube block is


Vblock = a3 = 83 = 512 cm3.
This is the volume of whole paper block, namely 750 pieces of
paper. The volume of one piece is obtained by division:

V 512
V1paper  block   0.683 cm 3.
750 750

As an extension, try finding the volume of a single page of this Solid Geometry book.

Polyhedrons 39
EXAMPLE 56 Find a formula for the volume of a cube in terms of its space diagonal length d.

Solution We know if the side length is a, the space diagonal d of the cube is d = añ3.

d 3
If we extract a, then a  . Applying the volume formula for a cube, we get the volume
3
3
d 3 d3 3
Vcube  a3     
 3  9

EXAMPLE 57 Using cuboid blocks with dimensions 4 cm x 5 cm x 5 cm we want to build the smallest
possible cube. How many blocks are needed?

Solution A side length of the cube must be divisible by 4, 5 and 6.


Since it must be the smallest possible cube, the side length a
must be the least common multiple of these numbers:
a = LCM (4,5,6) = 60 cm.
So the required number of blocks for each dimension will be 15,
12 and 10.
Therefore the answer is 15 x 12 x 10 = 1800 blocks.

EXAMPLE 58 The surface area of a cube is A cm2 and its volume is V cm3. If A = V then find the length
of the diagonal of the cube.

Solution This problem is left as an exercise for you.

EXAMPLE 59 The length, width and height of a cuboid are in the proportion 1 : 2 : 4. Write a ratio to
compare the surface area of this cuboid and the surface area of a cube with the same volume
as this cuboid.
Solution Let the dimensions of the cuboid be l = k, w = 2k and h = 4k.
Then the volume of the cuboid is 8k3. Since the cube has the same volume, the edge length
of the cube must be 2k.
The surface area of the cuboid is 28k2 while the surface area of the cube is 24k2.

So
Acuboid 28 k2 7 .
= =
Acube 24 k2 6

Conclusion
Two prisms with the same volume may not necessarily have the same surface area.

40 Solid Geometry
The Miracle of the Honeycomb
The Miracle of the Honeycomb
Honey is one of life's special treats. We share it with bees. Bees store their honey in honeycombs,
and if you have ever bought natural honey, you have probably seen the beauty of a honeycomb. Did
you know that a honeycomb has some interesting geometric properties?
Bees make their honeycombs from wax, which they also produce themselves. Bees use the cells
of a honeycomb for storing honey and pollen (a sweet powder they take from flowers), and for
raising new bees. If you ever see the miracle of bees creating a honeycomb, you will see lots of activity and you might think
that the result would be a big mess! In fact, the resulting honeycomb is very well organized, and shows the efficiency and
beauty of a particular geometric design which is based on hexagonal prisms.

Bees begin making a honeycomb at different points, independently of each


other, but the cells they create all have approximately the same size (between
5.2 and 5.4 mm wide) and the same hexagonal shape. Each internal angle of
the cell wall is 120°, and the walls are the same thickness, meaning that the
cells of a honeycomb fit next to each other perfectly. Each honeycomb also has
two sides, meaning that hundreds of cells on one side share a common base
with cells on the other side of the comb. To help the cells fit tightly at their
bases, bees make a base from three equilateral rhombi, instead of a flat base.
Why Hexagons?
Nature always uses the most efficient shapes and structures possible for each purpose, so that nothing is wasted. In the case
of a honeycomb, producing wax takes a lot of effort and time. Bees must consume up to 10 kg of honey to produce each kilogram
of wax, and so they must be economical when they use the wax: they need to make the greatest number of cells with the
least amount of wax possible. Studies have shown that bees can make a 20 x 38 cm comb, with 7000 cells able to hold nearly
two kilograms of honey, using just 36 g of wax!
If bees used a shape such as a pentagon, octagon or circle as the shape of the cells, there would be gaps between the cells.
This would be inefficient, because the cells would not be able to share walls and so wax would be wasted. Instead, it makes
sense to use shapes that fit together exactly, such as triangles, squares or hexagons.
The table opposite shows the data for triangular, square and hexagonal cells.
Among these three shapes, the hexagon has the smallest perimeter for the
same area. This means that if a triangular, square and hexagonal cell all have
equal volumes (Vprism), the surface area of the walls (Alateral) is smallest for a
hexagonal cell. In other words, hexagonal cells have the highest ratio of vol-
ume to lateral area, i.e. volume to wax usage. Can you believe that this fact is
used by tiny insects whose brains occupy only 0.74 cubic millimeters? It is cer-
tainly a miracle!
You can find hexagon shapes which use the same honeycomb structure in some common household items, where things need
to fit together tightly without leaving any empty space.

Hexagonal cake tins Hexagonal ice cubes Hexagonal dishes

Polyhedrons 41
Check Yourself 4
1. The adjacent prism has a base consisting of whole and
half unit squares. What is the volume of the prism?
3 cm
2. What is the volume of a nonagonal prism if its base area 11 cm
is 10 cm2 and height is 7.5 cm?

3. An equilateral triangular right prism has base edge 8 cm


and height 12 cm. What is its volume?

4. How many liters of water are needed to fill a rectangular


shaped aquarium with dimensions 25 cm×42 cm×72 cm?

5. Find the volume of a regular hexagonal prism with height


20 cm and lateral area 480 cm2.

6. In the cross section of one of the adjacent iron beams, the


outer rectangle the dimensions 10 cm x 6 cm and the
inner rectangle has dimensions 9 cm x 5 cm. If one beam
is 1.5 m long and the density of the iron is 7.8 kg/dm3,
find the weight of the beam.

7. What is the volume of this cube-shaped candle if its edge


length is 7 cm?

8. If a face diagonal of a cube is k units, what is the volume


of the cube in terms of k?
4 cm
10 cm 10 cm
9. The net of a right prism is shown at the right. Find the

11 cm
volume of the prism.
20 cm

10.Find the volume of the adjacent regular octagonal right


a
prism if the circumradius of its base is 6 cm and the
height is 10 cm.
a is the circumradius
of this hexagon.

Answers
1. 60.5 cm3 2. 75 cm3 3. 192ñ3 cm3 4. 75.6 L 5. 480ñ3 cm3

k3 2
6. 17.55 kg 7. 343 cm3 8. cm 3 9. 792 cm3 10. 720ñ2 cm3
4

42 Solid Geometry
4. Cavalieri's Principle

Definition Cavalieri's Principle


If two solids have the same height and the same cross-sectional area at every corresponding
level then they have the same volume.

Look at the solids below. The solids have equal heights but different cross sections. However,
if the areas of their cross sections are equal at every level, then by Cavalieri’s principle the
solids have equal volumes.

Look at the identical stacks of paper below. All three paper stacks have the same number of
sheets of paper.

Since the cross sections are the same at every level (one sheet of paper) and the stacks have
the same heights, the volumes of the stacks are the same.

Polyhedrons 43
5. Volume of an Oblique Prism
We can use Cavalieri’s principle to calculate the volume of an oblique prism. If we slant a
right prism, it will be an oblique prism with the same base.

What changes?
1. The height decreases.
2. The right section gets smaller.
3. The volume decreases.
l H
l
h What does not change?
l

q Base q 1. the base area


Base
2. the lateral edge

As we can see, changing the angle of inclination of a prism affects two things: its height and
the area of its right section.
Now we will look at two formulas for the volume of an oblique prism depending on these
parameters.

Theorem Volume of an Oblique Prism (1)


The volume of an oblique prism is equal to the product of the
area of the base of the prism and its height.
l h
l
V = Abase × h
Base q

Proof Suppose that the oblique prism is obtained by inclining an initial prism with height H by the
angle . Although the base of the prism is the same, now the new prism does not have the
same volume as the original one since its height has decreased.

l H
l
h
l

q Base q
Base
Base

By Cavalieri’s principle, the oblique prism above has the same volume as the green prism,
which has the same base and height:
V = Abase × h.
44 Solid Geometry
This formula is the basic formula for the volume of an oblique prism. However, we can
develop another volume formula for an oblique prism depending on the other changing
parameter: its right section.

Theorem Volume of an Oblique Prism (2)


F E

The volume of an oblique prism is equal to the area of


A T D
its right section and the length of its lateral edge. B C
S
l P
V  Aright section  
R
Q
l

F E

Proof Let the angle of inclination be . A D


T
q
Then the height of the prism, depending on the lateral B C
S
l
edge , will be h = sin  · . P

ADR relates the right section to the base. Q


R
l
Since the right section APQRST is the projection of the q

base ABCDEF on the right section plane, we can say

Aright section
Aright section = Abase × sin  , i.e. Abase = .
sin 

As we have already seen from the previous theorem, V = Abase × h.


Let A be the area of the
ý Substituting Abase and h = sin  · into this theorem gives us
projection of an area
A on the plane having  Aright section
degree angle, then V= . sin  .  = Aright section .  = AAPQRST .  . This completes the proof.
Aý = A.cos .
sin 
Remember! We now have two formulas for the volume of an oblique prism. We can see that they are
cos  = sin (90° – )
equivalent as follows: in the oblique prism, if we remove the section between the base and a
right section and append it to the other base, we see the oblique prism transformed into a
right prism:

The base of the new right prism is the right section of the old prism, and its height equals a
slant edge. This corresponds to the formula given above.
Polyhedrons 45
We now know two formulas for the volume of an oblique prism. When we are solving a problem,
we simply choose the best formula for the problem.

EXAMPLE 60 What is the volume of a pentagonal oblique prism with


base area 32 cm2 and height 9 cm?
h
Solution Voblique prism = Abase × h = 32 . 9 = 288 cm3

EXAMPLE 61 Let us find the volume of the oblique prism given in Example 22.

Solution The right section of the prism is an equilateral triangle with side length 5 cm, and the lateral
edge is 10 cm. We have C' F C

a2 3 52 3 25 3
Aright section    cm 2 .
4 4 4
So the volume will be B' E B

25 3 125 3 A' D A
Vprism  Aright section  h  10  cm 3 .
4 2

EXAMPLE 62 The figure shows a rectangular oblique prism with an inclination


13
angle of 65°. The base dimensions of the prism are 7 cm × 10 cm
and the lateral edge is 13 cm. What is the volume of this prism?
(Use sin 65° = 0.9.) 7 65°
10

We can use either of the volume formulas for an oblique prism.


Solution 1 Voblique prism = Abase  h
Now we have to calculate h, because it is not directly given: h l 13

h = l  sin 65° = 13  0.9 = 11.7 cm and


65°
Vprism = 7  10  h = 70  11.7 = 819 cm3. 7 65°
10
Solution 2 Vprism = Aright section  l We must find the height or
the area of right section using Trigonometry.
The right section is the projection of the base onto the right section plane. So
Aright section = Abase  sin 65° = 70  0.9 = 63 cm2 and
Vprism = Aright section  l = 63  13 = 819 cm3.

Conclusion
As long as the base area and height of a prism do not change, the volume does not change.

46 Solid Geometry
6. Volume of a Parallelepiped

Theorem Volume of a Parallelepiped


H b
The volume of a parallelepiped with edge lengths a, b and c is G
c
V = a  b  c  sin   sin, c
E b
F
where b and c represent the edges of the base,  is the b

included angle between b and c, and  is the angle between a


a
a and the plane of the base.
D
C
a Base
A c
b
B

Proof Since a parallelepiped is also an oblique prism, H b G


we can use the same volume formula. However, c
c
note that a parallelepiped is a hexahedron with E
b g
F
b
parallelogram faces. The base is any of the six
h
parallelograms that form the parallelepiped. a
a h
Let us choose parallelogram ABCD as the base to
D
work out our formula, so the height is EO.
a C
b
The volume of the parallelepiped is simply A
O Base c
b
V = Abase  h = A(ABCD)  h. B

However, we need to find this formula in terms


of the side lengths a, b and c. Here the angle  is the angle included by the sides of the base,
and the angle  is the angle between the lateral edge and plane of the base.

By the area formula for a parallelogram, A(ABCD) = b  c  sin , and in right triangle AOE
we have h = a  sin .

This gives us the final formula for a parallelepiped volume as

V = a  b  c  sin  cos.

Another variation of the same formula is h = a  cos , where is the angle at G shown in the
figure.

Then the volume becomes V = a  b  c  sin   cos . Since  and  are complementary angles,
the two formulas are essentially the same.

Polyhedrons 47
63
d
H G
EXAMPLE In the parallelepiped shown in the figure, line d is
30° F 6m
E
perpendicular to the base. 30°
D
Find the volume of the parallelepiped. C
7m
A 10 m B

Solution We know the formula Vparallellepiped = Abase  h = a  b  c  sin   sin Here the angle  is the
angle at the vertex of the base which is 30°. Angle  is the angle of inclination of side GC with
the base, so  = 60°.

1 3
So the volume is Vparallelepiped =10  7  6  sin30° sin60° = 420   =105 3m 3 .
2 2

If the coordinates of the vertices of a parallelepiped are given, we can also find its volume
using vectors and determinants.

Suppose that a vertex of a parallelepiped is located at the origin in the 3D coordinate system,
and that the edges containing that vertex are determined by the vectors V1, V2 and V3. This
means that the whole parallelepiped is defined by these vectors. The volume of the paral-
lelepiped is the absolute value of the determinant of the three-dimensional matrix of vec-
tors V1, V2 and V3:

V3
V  x y1 z1 
 1  1 
Vparallelepiped = det  V2  = det  x2 y2 z2  V2
   
x z3 
 V3   3 y3

V1

EXAMPLE 64 What is the volume of the parallelepiped in the figure?

Solution The parallelepiped is initiated by the vectors OÂ P , OÂ Q and OÂ R. So

 z
 OP  6 8 3 
  
Vparallelepiped = det OQ  = det 4 3 5  = – 56 = 56 unit 3 . Q(4, 3, 5)

  
 OR  2 5 2  R(2, 5, 2) P(6, 8, 3)
y
O

48 Solid Geometry
Check Yourself 5
1. What is the volume of an oblique heptagonal prism with base area 45 cm2 and height
21 cm?

2. Find the volume of an oblique triangular prism if its base area is 10 cm2, the lateral edge
is 15 cm and the inclination angle is 45°.

3. A pentagonal oblique prism has a right section with area 24 mm2 and its lateral edge is
15 cm. Find the volume of the prism.
80°
20 m
4. What is the area of the right section shaded in red in the
adjacent parallelepiped? The parallelepiped has only one
pair of parallelogram faces. 8m
25 m

5. A square prism is inclined at 54°. The basal edge of the prism is 6 cm


and the lateral edge is 20 cm. Find the volume of the prism to the nearest
20 cm
cubic centimeter by using two distinct formulas. (Use sin 54° = 0.8.)

6. The base of an oblique prism is a right triangle with legs 5 cm and


54°
8 cm. If the 12 cm lateral edge of the prism makes a 60° angle with the 6 cm

base, what is the volume of the prism? 12 cm

7. Find the volume of the prism formed by the net at the 9 cm

right.
30°
6 cm
8. Can we find the angle of inclination of an equilateral
triangular oblique prism given that the base edge is 4 cm,
the lateral edge is 10 cm and the volume is 60 cm3? If so, find it.

9. What is the volume of the parallelepiped shown in the figure?

10.Find the volume of a parallelepiped with one vertex at the


5ñ2 cm
origin and three adjacent vertices at points 45°
60°
(–1, 2, 1), (4, 2, 2) and (2, 1, 5). 10ñ3 cm
5 cm

Answers
1. 945 cm3 2. 75ñ2 cm3 3. 3.6 cm3 4. 160.cos10°
3 3 3
5. 576 cm 6. 120ñ3 cm 7. 324 cm 8. yes, 60°
3
9. 375 cm 10. 40 cubic units

Polyhedrons 49
EXERCISES 1 .1
A. Basic Concepts E
11 . How many face and space diagonals does a
A D pentagonal prism have?
1. Name the upper base and a B C
lateral face of the prism in J 12 . If a prism has 20 vertices, how many face diagonals
F
the figure. I
does it have?
G H

2. This hexagonal table is 110 cm 13 . If a prism has 96 diagonals, which polygon does it
tall. What is the total length of its have as its base?
legs? 14 . A heptagonal regular prism is made from wooden
3. Sagir uses clay balls for vertices and straws for sticks in the same lenght. Their total length is
edges to make a prism. If he used totally 14 balls, 105 cm. What is the base perimeter of the prism?
how many straws did he use? 15 . Find the sum of the lengths of all diagonals of an
B. Types of Prism equilateral triangular right prism with height
8 cm and base perimeter 18 cm.
4. B is the midpoint of a A
side of this triangular 10 m 16 . In a prism with 18 identical 7 cm edges, what is
prism. What is length 10 m
12 m the length of the longest diagonal?
AB? 15 m 17 . Find the difference between the long and short
B
space diagonals of a right prism with a regular
5. How many rectangular faces does a heptagonal
hexagonal base, if its basal edge is 5 cm and
right prism have?
height is ò69 cm.
6. What is the dihedral angle between any two adjacent C. Some Special Prisms
lateral faces of a regular pentagonal right prism?
18 . Draw the front, side and
4m 3m
7. Draw an accurate net for the top views of the figure
5m formed by this net.
prism in the figure.
6m

19 . What is the difference in shape between an

8. oblique rectangular box and a right prism with a


What is the altitude of a regular pentagonal prism
parallelogram base?
if a base edge is 10 cm and the area of one lateral
face is 75 cm2? 20 . Find the height h of the
8 cm
adjacent parallelepiped. h
9. What is the height of an oblique prism if a lateral 72°
(Use sin 72° = 0.95.) 6 cm
edge is 16 cm and the angle of inclination is 30°?
10 cm
10 . How many edges and vertices does a nonagonal 21 . Find the length of the shortest face diagonal of
oblique prism have? the solid in question 20.

50 Solid Geometry
22 . A 20 cm x 12 cm x 14 cm cuboid 30 . Osman stretched a triangular
gift box is tied with ribbon as 14 piece of fabric between three
shown. The bow uses 33 cm of corners of a cubic room, as
12 20
ribbon. What is the total length of shown opposite. If the edge
ribbon used for the gift box. length of the room is 3 m, find
the area of the fabric used.
23 . Find the lengths of the face diagonals of a rectangu-
lar solid with dimensions 5 m x 9 m x 12 m. 31 . What is a face area of a cube in terms of f, if f is
a face diagonal?
24 . The cuboid below is cut into halves as shown.
Find the area of the cross section. 32 . If we remove a portion from a
vertex of a cube as shown in the
6 cm
figure, what is the area of the
new yellow face? 1
5 cm
4

8 cm

33 . What is the length of the space diagonal of a cube


25 . An ant wants to get from B with edge length 25 cm?
point A to point B in the 3m
rectangular room in the 34 . If we slant a cube, do we get a rhombohedron?
figure. Find the shortest
2m
possible distance it must 35 . If we make a rhombohedron-shaped dice, will it
A 4m
cover. be fair? Explain your answer.

26 . Find the space diagonal length of a cuboid with D. Surface Area of a Prism
dimensions 3 cm x 7 cm x ò23 cm.
36 . The oblique prism in the
figure has an irregular
27 . The height of a cuboid is three times its length pentagon base and a regular
and its length is two times its width. If the space pentagon right section
12 cm

diagonal is ó123 cm, find the area of the largest face. with side length 3 cm. Find the area of the lateral
surface.
28. If the face diagonals of a rectangular box are 6 cm,

ò43 cm and 7 cm, find the length of a space 37 . We have an oblique cuboid
diagonal. with an inclination angle
6 cm
of 76°. Using the given
29 . Find the length of the space diagonal in a square dimensions, find the total 76°
4 cm 9 cm
prism if the base area is 16 cm2 and the height is surface area of the solid
8 cm. to the nearest square centimeter.

Polyhedrons 51
38 . What is the lateral surface area of a heptagonal 46 . 4 cm, 5 cm, and 7 cm are the lengths of the basal
right prism whose side lengths are 3, 5, 7, 9, 11, edges of a triangular right prism whose height is
13, 15 cm, and height is 8 cm? 1 m. Find its total area. (Hint: Use Heron's formula)

39 . What is the base area of a heptagonal right prism 47 . If the base perimeter of a square based prism is
if its total area is 300 m2 and lateral area is 200 m2? 20 cm and the height is 8 cm, what is its total
surface area?

40 . Find the area of a single base of a decagonal right


48 . Find the lateral surface area of a regular dodecagonal
prism if the base edges have the lengths of
right prism with base edge 4 cm and height 7 cm.
successive even numbers from 2 cm to 20 cm, the
height is 5 cm, and the surface area of the whole
49 . Find the total surface area of a rectangular solid
prism is 1000 cm2.
which is 17 m long, 12 m wide and 8 m high.

41 . What is the total surface area of a right triangular 50 . The adjacent box has a
right prism if the legs of a base are 9 cm and 12 cm surface area of 278 cm2. 2
and the height is 10 cm? Two of the faces have the cm
4
46 cm2 2
areas shown. What is the
42 . Find the total surface area 10 cm area of the top face?
8 cm
of the prism in the figure.
14 cm 51 . The surface area of a rectangular prism is 774
12 cm square inches. The height and the width are
18 cm
respectively 8 and 12.5 inches. Find the length of
this prism.
43 . How much canvas is
used to make this
5m 52 . A bus shelter will
tent, including the 1m y
6m be constructed as
floor? 12 m
shown in the
x
figure. Find the
44 . Find the interior area of this octagonal total area of the x
water jug if its basal edge is 4 cm and surface of the shelter in terms of the given
height is 20 cm. dimensions.

53 . A rectangular solid is made from sticks. The


length is twice the width and the height is twice
45 . Find the height of a regular hexagonal right prism the length. If the total length of sticks used is 84 cm,
if the total surface area is 24ñ3 cm2 and a basal find the minimum area of the paper we must use
edge is 2 cm. to cover the solid.

52 Solid Geometry
54 . To make this open rectangular E. Volume of a Prism
5 cm
box, its net will be cut from 62 . Find the volume of the solid formations shown if
a single piece of rectangular 15 cm
each small block is 1 cm3.
40 cm
cardboard. Find the area of
a. b.
the smallest possible piece of cardboard which
can be used.

55 . If we cut a rectangular solid-shaped bar of soap with


dimensions 9 cm x 4 cm x 8 cm in half to get two
square prisms, by how much does the total surface 63 . Find the volume of this prism if
area of the soap change? the base area is 48 cm2 and the
height is 11 cm.

56 . A swimming pool with a 25 m x 15 m rectangular


base and 2 m depth will be covered with 20 cm x 64 . 156 tons of water were used to
20 cm square tiles. At least how many tiles are fill this pentagonal shaped
required? pool to a depth of 1.2 m. Find
the surface area of the water.
57 . The three different faces of a rectangular box (1 m3 of water weighs 1 ton.)
have areas 45 cm2, 60 cm2 and 75 cm2. Find the 65 . Find the volume of the 4 cm
edge lengths of this box. prism in the figure. 6 cm

5 cm

58 . The lateral area of a 5 cm high rectangular box is 6 cm 13 cm


60 cm2. Draw the graph of width y versus the
66 . The figure below shows the foundations of a
length x.
building. The grey parts will be filled with concrete
to a depth of 6 m. Find the total volume of
59 . What is the total surface area of a cube with 7 cm concrete required.
edge length?
6m 8m
4m
10 m
60 . The area of a cube is 1350 m2. Find the perimeter 9m
8m
of one face. 4.5 m

15.5 m
61 . From a cube, a square prism is 2
extracted as shown in the figure. 67 . What is the volume of this square
1 based, 14 cm deep box if the base
Find the surface area of the 1
2

remaining block. 1
edge is 30 cm?

Polyhedrons 53
68 . The total surface area of a square prism is 42 times 74 . The net of a prism is
its base area. One basal edge is 7 cm. What is the shown opposite. If the
volume of this prism? perimeter of the triangular
50 cm2 130 cm2 120 cm2
base is 60 cm, find the
69 . Birol the farmer volume of the prism.
discovered an
underground right
triangular prism- 75 . One side of the base of a regular right hexagonal
75 m 50 m prism is 3 cm. Find its total surface area if its
shaped water cavity
as shown in the 85 m volume is 54ñ3 cm3.
figure. Find the volume of water in the cavity.
76 . A 10 cm high cuboid has total surface area 160 cm2

70 . Find the volume of the and lateral area 140 cm2. Find its width and
length.
dumper in the picture 2m
according to the given
0.8 m 77 . This cuboid-shaped metallic flower
dimensions, given that
the dumper is 1.5 m 0.4 m planter has dimensions
wide. 5 dm x 2.5 dm x 12 dm. Find its
volume in liters.

71 . Find the volume of the


78 . The locker unit in the figure is 2 m
adjacent rhomboid right 45°
15 cm high, 1 m long and 0.5 m wide. If the
prism.
unit has four lockers, find the volume
10 cm of one locker.

72 . Býdýk the dog has the wooden 79 . The depth of this bread box is
kennel shown in the figure. 24 cm and the volume is 10.8 L.
The hut is 0.8 m long. Find Find the area of the lid of the box.
75 cm
its volume in cubic meters. 1m

(Consider the walls vertical.)


80 . The cost of a wooden crate increases with its surface
60 cm
area. Of these two wooden crates, which one will
be cheaper?

73 . The adjacent 8 cm deep


2 feet 3 feet
hexagonal box has a regular
base with edge length 12 cm. 4 feet 4 feet
Find its volume. 6 feet 4 feet

54 Solid Geometry
81 . The water trough in the 86. d

adjacent figure has inner base
dimensions 0.7 m x 2.4 m. If
it can hold 1 ton of water, find
the minimum possible depth, to the nearest
centimeter. (1 m3 of water weighs 1 ton.)
From each corner of a square sheet of cardboard
82. A double-glazed PVC window with side length d, a square with side length 5 cm
has two layers of glass to provide is cut out and the remaining figure is folded to form
heat and noise isolation. The an open box. If the volume of the box is 320 cm3,
double-glazed window in the find the area of the original cardboard sheet.
figure is made from two glass
panes with dimensions 40 cm x 90 cm. Find the
volume of air between the panes if they are 1.7 87. The surface areas of the three different faces of a

cm apart. rectangular prism are in the ratio 3 : 4 : 5. If the
volume of the prism is 450 cm3, find the length of
83. Ahmed bought a fridge whose its shortest side.
capacity was labeled as 250 L.
To check this capacity, he 90 cm
measured the internal height 88. The dimensions a, b, c of a rectangular prism have

and length as 90 cm and 70 70 cm
the property 1 + 1 + 1 = 1 . Find the volume of
cm respectively. What is the minimum expected
a b c 2
internal width of the fridge, to the nearest
the prism if its total surface area is 288 cm2.
centimeter?

84. The drawer unit shown at the 89. This hydraulic garbage truck can compress
right has a square base and two garbage by 50% on average. The dimensions of
square drawers. The top drawer the right trapezoidal truck are shown in the figure.
has height 15 cm and the bottom If a single garbage container has dimensions
drawer has height 25 cm. Find 100 cm x 100 cm x 120 cm, how many full
the ratio of the volumes of the two drawers. containers of garbage can the lorry collect?

85. This cuboid aquarium’s front 2m


4m
and side faces are similar
rectangles. The width and 2m
height of the aquarium are
32 cm and 48 cm respectively. 5m
Find the amount of glass used to make the
aquarium if five of the faces are glass.

Polyhedrons 55
90. What is the volume of a cube with side length 98 . Find the volume of the
8 cm? oblique square prism
15
shown in the figure.
91. Find the surface area of a cube if its volume is 60°
3
729 cm3.

99 . An equilateral triangular oblique prism with all


92. Japanese cubic watermelons are
edges 4 cm is constructed. Its slant angle is 60°.
famous all around the world.
Find its volume.
They are grown in a special
cubic container to create their
100 .What is the volume of the 8m
shape. The one in the figure has
prism in the figure?
22 cm side length. Find its average volume, to the 12 m
nearest tenth of a liter.
5m 7m 6m
2
93. To make a cube, we used 96 cm of cardboard.
Find the volume of the cube. 101 .The right section of
this oblique pentagonal
94. The surface area and the volume of a cube have prism is 24 cm2. Find
the same magnitude. Find the volume of the cube. its volume if a lateral
edge is 15 cm.
95. At least how many right rectangular prisms with
dimensions 4 cm x 6 cm x 10 cm do we need to 102 .The adjacent octagonal
make a cube? oblique prism has a 60° 12 cm
slant. Find its volume if
its base area is 40 cm2.
96. The charity box in the figure is
mounted on a prism-shaped
stand. The prism has a curved 103 .
lateral edge, and a square base 45°

with 2 dm side length. The he- 8 dm


4 2 cm
60°
ight of the stand is 8 dm. Find
the volume of the stand. 2 dm 5 3 cm 5 cm

Find the volume of the parallelepiped in the figure.


97 . Find the volume of the
oblique prism shown in 104 .OA, OB and OC are three edges of a parallelepiped
14 cm
the figure. joining at a vertex. The corresponding coordinates
30°
are A(4, 1, 2), B(2, 3, 2) and C(3, 2, 4). Find the
3 cm
10 cm volume of the parallelepiped.

56 Solid Geometry
A. BASIC CONCEPTS
Definition Pyramidal Surface
Consider a fixed polygon and a fixed point A A
Apex
which is not in the plane of the polygon. As a
moving line k passing through A moves along
the sides of the polygon, it generates a surface
called a pyramidal surface.
The moving line is called the generatrix and
k
the polygon is called the directrix of the
surface. The fixed point is called the apex.

Definition Pyramid
A pyramid is a polyhedron that is bounded by an apex, a polygon and the pyramidal surface
that they generate.
We can see that a pyramid is a polyhedron with one polygon face (called the base). All the
other faces are triangles which meet at the apex.

A
1. Elements of a Pyramid Apex
(vertex)
The altitude of a pyramid is the line segment which joins
the apex and the plane of the base, perpendicular to the Lateral Lateral
edge face

Altitude
base. The length of the altitude is called the height of the
pyramid. Sometimes the altitude can lie outside the
pyramid. The point where the altitude meets the plane of E D
the base is called the foot of the altitude. Basal
B C edge
Foot
of altitude
Apex

Height

Base
Foot
of altitude
Polyhedrons 57
In a pyramid, the lateral faces are triangles and all these triangles have surface altitudes. We
call these altitudes slant heights. Note that the altitude, slant height and lateral edge of a
pyramid are three different things, as shown in the pyramid below. A Apex

We can name a pyramid using the point of the apex

Slant
BC
Lateral
followed by the other vertices.

t
igh
Edge

Heig C
Altitude
He
For example, the pyramid opposite is pyramid ABCDE.

ht D
nt
Sla
E
B
A

C D
The lateral edges of a pyramid do not
E have to be identical. This means that in
D
some pyramids the slant heights may not
be the same. Compare the slant heights
with the lateral edges and altitude of the
B C
adjacent pyramid.
Pyramids are named according to their bases. For instance, a pyramid with a hexagonal base
is called a hexagonal pyramid. A pyramid with a triangular base (triangular pyramid) has a
special name: a tetrahedron. We will study tetrahedrons and square pyramids later in this book.

EXAMPLE 65 How many faces, edges, vertices and face diagonals do the following pyramids have?

A B C D
Solution Completing this table is left as an exercise for you.
Face Space
Base Faces Edges Vertices
Diagonals Diagonal
A Triangle 4 6 4 0
B
None
C
D
It is easy to check that for a pyramid with n-gon base, there are (n + 1) faces, (n + 1)
vertices and 2n edges. Can you see why this is true? What can you say about the number of
face and space diagonals in a pyramid? Give a reason for your answer.

58 Solid Geometry
Remark Sections Parallel to the Base of a Pyramid
P
If a plane parallel to the base of a pyramid cuts all the lateral edges of
the pyramid, then
 this plane divides the altitude and lateral edges proportionally.


 the section formed is similar to the base of the pyramid. S
For example, in the figure opposite,
Q T
PQ PT B
 and polygon B  polygon S.
PQ PT 

B. TYPES OF PYRAMID
We can classify pyramids according to the position of their apex with respect to the centroid
of their base.

Definition Centroid
The centroid of a polygon is its geometric center. It is the average sum of the displacements of all
the points in the polygon from a relative origin. The centroid of a plane figure is also its center of
gravity. There are different methods which we can use to find the centroids of different shapes.

The centroid of a triangle is the The centroid of a parallelogram is The centroid of a regular
intersection point of its medians. the intersection point of its diagonals. hexagon is the center of its incircle.
An irregular shape requires further calculations for locating the centroid.

1. Right and Oblique Pyramids


Definition Right Pyramid, Oblique Pyramid
A right pyramid is a pyramid for which the line joining the centroid of the base and the apex
is perpendicular to the base. In right pyramids, the foot of the altitude is at the centroid of
the base. If a pyramid is not right, it is an oblique pyramid.

h h h

C C C

Right pyramid Oblique pyramid Oblique pyramid

In oblique pyramids, the foot of the altitude does not lie at the centroid of the pyramid.

Polyhedrons 59
EXAMPLE 66 A right pyramid has a rectangular base with sides 6 m and 10 m. The length of a lateral edge
is 13 m.
a. Find the sum of the edge lengths of the pyramid.
b. Find the slant heights of the pyramid.

Solution a. The pyramid has four basal and four lateral edges. A
Sumedges = Perimeterbase + 4  Lateral Edge
= 32 + 4  13 = 84 m 13

b. The lateral edges are all congruent because the 13


6 E
pyramid is a right pyramid. As the base is rectangular, B
2
there are two different slant heights. K O 10
5
Look at the figure. On side BC, K is the midpoint and C 3 L 3 D
KC = 5 m.
In right triangle AKC,
AC2 = AK2 + KC2  132 = AK2 + 52, so slant height AK = 12 m.
Similarly, in right triangle ACL, AC2 = AL2 + CL2  132 = AL2 + 32.
So slant height AL = 4ò10 m.
O

EXAMPLE 67 In the adjacent rectangular right pyramid, the base


D
11

dimensions are 8 cm and 10 cm and the lateral edge A


is 11 cm. What is the height of this pyramid? 8

B 10 C
Solution Look at the figure below. At the base ABCD, point K is the projection of apex O, and points P
and R are the projections of point K on the sides and they are midpoints. This means
AP = 4 cm and BR = 5 cm. Our strategy will be to find the altitude using the right triangles OAP
and OPK. In right triangle OAP, O

OA2 = AP2 + OP2 11


2 2 2 11
11 = 4 + OP 11
h
2
OP = 105. A
D
4 K
In right triangle OPK, PK = BR and P 8
2 2 2 2 2 2 4
OP = PK + OK  105 = 5 + h  h = 80.
B 5 R 5 C
Therefore the height of the pyramid is h = 4ñ5 cm.

60 Solid Geometry
V
EXAMPLE 68 The figure shows an oblique triangular prism with
isosceles base MKL. 123

Triangle VKL is equilateral with side length 6 cm, h


6
M
VM = ò123 cm and MK = ML = 9 cm. K

9
Find the altitude of the pyramid that projects
L
outside the base.

Solution Look at the figure. Finding VT and TM will help V

us to find the altitude h.


123
Since VKL is equilateral, MH passes through the h
6
midpoint T so that KT = TL = 3 cm. MKL is M
K
3
isosceles, so MTL is a right triangle. T 9
x 3
H
L
By the Pythagorean Theorem,

ML2 = TM2 + TL2  92 = TM2 + 32  TM = 6ñ2 cm.

On the other hand, VT is the is the height of triangle VKL and VT = 3ñ3 cm.

We will use the triangular section VTM with the side lengths.

Is this information sufficient to find the altitude VH?

Yes, it is. Here is the explanation: V

Here we have two right triangles VHT and VHM. h 123 cm


3ñ3 cm
In right triangle VHT, we have
H x T 6ñ2 cm M
2 2 2 2 2
VH + HT = VT  h + x = 27.

In right triangle VHM, we have VH2 + HM2 = VM2  h2 + (x + 6ñ2)2 = ò123 2.

So h2 + x2 + 12ñ2x + (6ñ2)2 = 123.

Recall that h2 + x2 = 27 , hence 27 + 12ñ2x + 72 = 123  12ñ2x = 24 and x = ñ2.

Finally, using h2 + x2 = 27, we get h = 5 cm.

Polyhedrons 61
2. Regular Pyramids
Definition Regular Pyramid

If the base of a right pyramid is a regular polygon, the pyramid is called a regular pyramid.
In some math books, a regular pyramid can be oblique. In this book, when we talk about a
regular pyramid, we mean a regular right pyramid.

Properties of a regular pyramid


1. The lateral edges of a regular pyramid are congruent.
The base of a regular pyramid is a regular polygon whose centroid is equidistant to the
vertices. The altitude together with the line segments joining the centroid and the vertices
form right triangles.

By SAS congruence, all these right triangles are congruent, so their


hypotenuses are all congruent.
So the lateral edges of a regular pyramid are congruent.

2. The lateral faces of a regular pyramid are identical isosceles triangles.


By the first property, it is obvious that the lateral faces will be congruent isosceles triangles.

3. The slant heights of a regular pyramid are all equal. This measure is called the apothem
of the pyramid.
By the second property, the congruent lateral faces all have congruent heights.
A
In the figure, [AQ] is the apothem of the pyramid, or simply the
apothem, and [HQ] is the apothem of the base.
Note that only a regular pyramid can have an apothem.
In a pyramid which one is longer, the apothem or the altitude?
a
4. If a is a basal edge of a regular n-ggonal pyramid, then the base H
perimeter will be n  a. a Q

62 Solid Geometry
EXAMPLE 69 A square right pyramid has base edge 6 cm and height 11 cm. What is the apothem of the
pyramid? O

Solution The pyramid has a square base. Let its centroid be H. Length
HK is half of a basal edge, and the apothem of the
pyramid with height OH and the base apothem form the right
11
triangle OHK. C
OK2 = OH2 + HK2 = 112 + 32 = 121 + 9 = 130 H
3
K
Therefore the apothem length is OK = ò130 cm. 6 B

EXAMPLE 70 The apothem of a regular pentagonal pyramid is 10 cm, and a lateral edge is 12 cm. Find the
perimeter of the base.

Solution Let us name the pyramid as KLMNOP, and let the midpoint of basal edge MN be R. Here,
KRN is a right triangle with known sides KR = 10 cm and KN = 12 cm. The third side RN
of this triangle can be found with the Pythagorean Theorem:

RN 2  KN 2  KR2  RN  122  10 2  2 11 cm.

Remember that point R is the midpoint of a basal edge. So a basal edge will be
a = 2RN = 4ò11 cm. So the perimeter of the regular pentagonal base is 5a = 20ò11 cm.
The figure for this solution has been omitted. Drawing this figure is left as an exercise for
you.

3. Regular Tetrahedron
Can you make four equilateral triangles using 6 matchsticks? At first this problem looks
difficult to solve without cheating and breaking the sticks. However, in the world of polyhedrons,
the answer is not very far away.

Definition Regular Tetrahedron


A regular tetrahedron is a triangular pyramid whose edges are all
congruent. All the faces of a regular tetrahedron are equilateral
triangles.

The regular tetrahedron is a very special figure. It is the only


polyhedron with four faces and six edges. No other polyhedron can
A net of a
be built with fewer faces or edges.
regular tetrahedron

Polyhedrons 63
An altitude of a regular tetrahedron is called a median of the tetrahedron. Notice that a
tetrahedron has four congruent altitudes (medians). The medians of a tetrahedron all meet
at the same point, called the centroid of the tetrahedron.

Theorem Centroid of a Tetrahedron


The centroid of a regular tetrahedron is a quarter height away V

from the centroid of each base. In the figure,


OH 1 OK 1 OG 1 OF 1 . K
   
VH 4 AK 4 BG 4 CF 4 G F
C O B

H
Proof In tetrahedron VABC at the right, we have the medians VH, AK,
A
BG and CF. Note that CM = VM because CM and VM are the
V
congruent heights of two congruent faces. These heights form the
isosceles triangle VMC. In this isosceles triangle, we have the a a
congruent altitudes VH and CF.
O F
C B
Note that VH and CF are also two altitudes of the tetrahedron, a
H 2
and they intersect at point O. M
a
Let the length of an edge of this regular tetrahedron be a. A
2

a 3 .
Since each face is an equilateral triangle, we have CM  VM 
2
On the other hand, point F is the centroid of a face, so
2 2 a 3 a 3 1 1 a 3 a 3
VF   VM    and FM  HM   VM    .
3 3 2 3 3 3 2 6
Now in the right triangle VHM, leg VH, which is also the median of the tetrahedron, is
2 2
a 3 a 3 2a2 a 6 by the Pythagorean
VM  VH  HM  VH  
2 2 2

2
    VH 
 2   6  3 3
   
Theorem. By the AAA similarity theorem, VOF  VMH. V

a 3
VO VF VO a 6
So  and  3  VO  .
VM VH a 3 a 6 4
2 3 a 6
F
O
VO 3
Our aim is to compare VO and VH:  4  .
VH a 6 4 C M
H
3
VO AO BO CO 3 .
Therefore, by analogy we can say that    
VH AK BG CF 4

64 Solid Geometry
Conclusion

If the length of one edge of a regular tetrahedron is a, then


a 3
 its slant height is ha  .
2
a 6
 its altitude (median) is h  .
3

EXAMPLE 71 Find the slant height and median height of a regular tetrahedron with edge length 10 cm.

a 3 10 3
Solution The slant height is ha    5 3 cm and the median height (the height of the
2 2

a 6 10 6
tetrahedron) is h   cm.
3 3

EXAMPLE 72 What is the distance from the centroid to the base of a regular tetrahedron with edge length
12 cm?

Solution The distance d from the centroid to the base of a regular tetrahedron is one quarter of a median
1
length: d   h.
4

1 a 6 1 12 6
d     6 cm.
4 3 4 3

EXAMPLE 73 What is the measure of the dihedral angle between any two faces in a regular tetrahedron?

Solution Let the regular tetrahedron be ABCD with a side a. AK is the slant height of the face ABC. If
we draw the altitude (median) AH of the tetrahedron, we now have a right triangle AKH.

a 3 A
a 6
In this right triangle, AK  and AH 
2 3 .
a a
a 6
AH 2 2 D
 3 
B
Then by trigonometry, sin K   0.943
AK a 3 3 H a
K
2
C
and m( K )  arcsin0.943  70.5° .

Polyhedrons 65
Tetrahedrons
A tetrahedron is the only polyhedron with four faces. We can also call a tetrahedron a triangular pyramid. If
all the edge lengths of a tetrahedron are congruent, the faces are all equilateral triangles and the tetrahedron
is called a regular tetrahedron.

A water melon grown in The net of a tetrahedron with right triangle lateral faces Regular tetrahedron
tetrahedral shape. out of a single square sheet of paper. No cutting, just fold!
An isosceles tetrahedron has three pairs of congruent edges and its faces are all isosceles triangles. Another special type of tetrahedron
is the trirectangular tetrahedron, which has three right angles at its vertex.

Isosceles tetrahedron Trirectangular tetrahedron


Tetrahedrons appear in nature, in particular in the molecular structure of certain elements and compounds.

In these structures, the angles between the


different nodes are all 109.5°.

Structure of a methane Structure of an ammonium Structure of a


molecule (CH3) molecule (NH3) diamond crystal

Tetrahedrons can also be used to make some interesting kites. A tetrahedral kite is
made up of smaller tetrahedron-shaped cells. Often the kite itself has a tetrahedron
shape. This type of kite was invented by Alexander Graham Bell, who also invented
the telephone. Although tetrahedral kites are not very easy to make, they are very
strong and easier to fly than traditional kites. If they are designed correctly, they can
fly well even in strong wind. Some Tetrahedral Kites

Perhaps the most useful man-made application of the tetrahedron is for packaging. In 1944, a young Swedish engineer called Erik
Wallenberg developed a tetrahedron-shaped carton which could hold milk and other drinks, using a tube of paper. His classic design
gave its name to a new company, Tetra Pak, which still produces food and drinks cartons today. Wallenberg's design, the Tetra
Classic, is used in many countries for milk and juice packaging.

Erik Wallenberg at work, and some samples of the Tetra Pak Classic (Photos courtesy of Tetra Pak)

A caltrop is probably the most unpleasant type of tetrahedron. A caltrop was a medieval weapon with four sharp
points. When it is dropped on the ground, one point always points straight up. Caltrops were used on battlefields
to injure horses and stop enemy soldiers.

66 Solid Geometry
4. Square Pyramid
Definition Square Pyramid
A square pyramid is a pyramid with a square base and
triangular sides.
The lateral faces of a regular square pyramid are isosceles
triangles.

The square pyramid is a special type of pyramid. Throughout history, its engaging shape has
attracted the attention of many scientists, architects and mathematicians, and square pyramids
have many applications in the world around us.

EXAMPLE 74 How many vertices, faces and edges does a square pyramid have?

Solution A square pyramid has a square base, so it has (4 + 1 = 5) vertices,


5 faces and 2 . 5 = 10 edges.

EXAMPLE 75 What is the height of a regular square pyramid whose edges are all congruent and
10 cm?
A
Solution Look at the figure. The base diagonal is 10ñ2 cm by the the
Pythagorean Theorem. When we draw the altitude AO, in
the resulting right triangle AOB we get AB2 = AO2 + OB2 10 h
and 102 = h2 + (5ñ2)2, h = 5ñ2 cm. D
E 5ñ2
As an alternative solution, since the triangles OAB and OCB 5ñ2 O
5ñ2

are congruent triangles, AO = OC = 5ñ2 cm. B 10 C

a 2
The height of a square pyramid with all edge lengths equal to a is h = .
2

Polyhedrons 67
EXAMPLE 76 Can we find the lateral face area of a regular square based pyramid given that its base perimeter
is 32 m and the lateral edge is 5 m? If so, find it. O

Solution If the base area and a lateral edge are given, we can find the
lateral surface area. For this, we need to find the apothem. 5

Look at the figure. Since the base perimeter is 32 m, one base


edge will be 8 m.
P 4 T 4 Q

Hence we have TQ = 4 m.

In the right triangle OTQ, if we apply the Pythagorean Theorem we get OT = 3 m.

A lateral face is a triangle with the base 8 m, and the apothem is 3 m, so

1 1
A( OPQ )   PQ  OT   8  3  12 m 2 .
2 2

EXAMPLE 77 The base area of a right square pyramid is 36 cm2 and the lateral edge is 15 cm.
What is the measure of the dihedral angle between a lateral face and a base, to the
nearest degree?
T
Solution The dihedral angle between the base and a lateral face is shown by angle
OHT in the figure.

Our strategy is to use the cosine of this angle in triangle OHT.

For this, we must find the apothem TH of the pyramid.

Since the base area is 36 cm2, one base edge is 6 cm. H


O
K
H is the midpoint of a side, and so TH  KH and KH = 3 cm.

In right triangle THK, TK2 = HK2 + TH2  152 = 32 + TH2.

So TH = 6ñ6 cm.
OH 3
Finally, in right triangle OHT, OH = 3 cm and cos( OHT )    0.204.
TH 6 6

From this we get the measure of OHT as approximately 78°.

68 Solid Geometry
The Pyramids of Egypt
The Egyptian Pyramids are amazing structures which have captured the imaginations of historians, architects, tourists, and of course
mathematicians throughout history. There are over 130 pyramids, located mostly in Saqqara and Giza (two areas close to Cairo), but
of course some pyramids are better known than others. Part of the attraction of the Pyramids is the many secrets that surround their
design and location.
The history of the Pyramids dates back to around 2600 BC and the reign of the Egyptian king Djoser.
Before the time of King Djoser, Egyptian kings were buried in large rectangular tombs called
mastabas. According to legend, when King Djoser was thinking about the design of his own mastaba,
his chief architect, Imhotep, suggested constructing a layered or stepped mastaba. Each layer of
such a mastaba would get smaller as the structure rose. This interesting structure, the first Egyptian
Pyramid, was built on the plateau of Saqqara, where it can still be visited today. It was 62 m high and
The stepped Pyramid of Djoser
was originally covered in white limestone.
Following King Djoser's example, Egyptian kings began ordering the construction of their pyramids before they died. One of the most
fascinating examples today is the Pyramid of Khufu, also known as the Great Pyramid of Giza. It is the only one of the Seven Wonders
of the Ancient World that can still be visited today.
The architects of the Great Pyramid oriented the structure to point exactly north. The pyramid is
made of around 2.3 million stone blocks, which each weigh 2.5 tonnes on average. Some of the
internal blocks weigh around 80 tons. The exact methods used for construction are still a mystery
today. Archaeologists do not know for sure how many people worked on the Great Pyramid and how
long it took, but some estimates are for 100 000 workers and 14 years.
As a geometric structure, the Great Pyramid sits on a square base with sidelength 230.4 m, and
has an angle of elevation of 51.5°. It was most probably 146.6 m tall when built, but now its peak
The Khufu (Cheops) Pyramid
is 138.8 m above its base. Its dimensions are astonishingly accurate: the base edges have a difference
of only 58 mm between them, and there is a difference of only 1.5 cm along the horizontal. The pyra-
mid was the tallest structure in the world for over 3800 years after its construction.

Some basic calculations with the dimensions of the Great Pyramid show us some interesting results
with irrational numbers:
BD  AH
The base diagonal - height ratio gives . The ratio of the lateral and base apothems is = .
h 2 . HK

Another famous pyramid is the Pyramid of Khafre. Khafre was the son of Khufu. At first this pyramid
looks bigger than the Great Pyramid, but this is because it sits on a raised platform. The original
pyramid was 143.5 m tall, but today it is 136.4 m tall. Its angle of elevation is 53.1°, which is slightly
steeper than the angle of the Great Pyramid. Originally all the Pyramids were covered in white
polished limestone, so that they shone from a distance, but unfortunately only the apex of the
Pyramid of Khafre is still covered in limestone today.
The Pyramid of Khafre
The ancient Egyptians kept many written records that have helped us
understand some important things about the Pyramids. They wrote
using characters called hieroglyphics. The illustration opposite shows
the title of this book (Solid Geometry) written using hieroglyphics.

Polyhedrons 69
5. Regular Octahedron

Definition Regular Octahedron


A regular octahedron is a solid composed of eight
equilateral triangular faces. At each vertex, four of
these faces meet.
We can think of a regular octahedron as a base-to-base
combination of two congruent square pyramids with
equilateral faces.

A net of a
regular octahedron

EXAMPLE 78 How many diagonals does a regular octahedron have?

Solution A regular octahedron has three space diagonals and


no face diagonals. Notice that all the space diagonals
are the same length.

EXAMPLE 79 Try to construct the net of a regular octahedron with an edge length of 4 cm.

Solution This problem is left as an exercise for you. (Hint: Use the figure shown next to the definition
above.)

EXAMPLE 80 What is the total surface area of a regular octahedron if its edge length is 7 cm?

a2 3
Solution Each face is an equilateral triangle and has area Aface  , where a is an edge.
4
An octahedron has eight congruent faces, so its total surface area is
a2 3
Atotal  8  Aface  8   2a2 3. Using a = 7 gives us
4

Atotal = 98ñ3 cm2.

70 Solid Geometry
Check Yourself 6
1. How many vertices, faces and edges does a nonagonal pyramid have?
A

?
2. In the adjacent pyramid, triangle BCD is a cross section 5 cm
D
formed by the intersection of a plane parallel to the base B
4 cm 12 cm
FGH. Find AD and BF. ? C

H
F
3. For a rectangular right pyramid, the base dimensions are 6 cm 12 cm

and 12 cm and the lateral edge is 10 cm. What is the height of G

this pyramid?

4. The apothem of a hexagonal regular pyramid is 11 m and the height of the pyramid is
9 m. What is the apothem of the base?

5. The base perimeter of a regular octagonal pyramid is 64 cm and a lateral edge is 9 cm.
Find the length of an apothem.

6. What is the length of the median of a regular tetrahedron if one edge is 9 cm?

7. The base edge of a square pyramid is 10 units and its lateral edge is 13 units. What is the
measure of the dihedral angle between two non-adjacent lateral faces, to the nearest
degree? (Hint: Use the law of cosines.)

8. The Great Pyramid of Giza is a right square pyramid with base edge 230.4 m, and lateral
edge 219.1 m. Find the apothems and height of this pyramid. Notice the ratio of the
lateral apothem to the base apothem. Is this ratio familiar?

9. Find the total surface area of a regular octahedron if every edge is 5 cm. V

10.What is the length of the diagonal of this regular octahedron in terms of c


c, where c is an edge of the octahedron?

Answers

1. V: 10, F: 10, E: 18 2. AD = 6 cm , BF = 10 cm 3. ò55 cm 4. 2ò10 cm 5. ò65 cm


6. 3ñ6 cm 7. 49°
8. lateral apothem: 186.4 m, base apothem: 115.2 m, altitude: 146.5 m, golden ratio
9. 50ñ3 cm2 10. cñ2 cm

Polyhedrons 71
C. FRUSTUM OF A PYRAMID
Definition Frustum of a Pyramid
A frustum of a pyramid is the lower portion V

of the pyramid that remains when the top of


the pyramid is removed along a plane parallel
to the base.
The bases of the frustum are the base of the D¢


pyramid and the section made by the parallel H¢

plane. A¢ B¢
D
C
In the figure at the right, polygons ABCDE and E H
ABCDE are the two bases of the frustum.
A B
Notice that the plane that creates a frustum
is parallel to the base. By the result we have seen previously, both bases of a frustum are the-
refore similar polygons.
A perpendicular segment between the bases is called an altitude of the frustum. In the figure
above, segment HH is the altitude of the frustum while the segment VH is the altitude of the
pyramid. The length of an altitude is the height of the frustum.
The open net of a square frustum is shown below. A slant height is shown by the red segment.

The lateral faces of a pyramidal frustum are trapezoids. a'


a' a'
If the frustum is from a regular pyramid, it is called a regular frustum.
A frustum is a right frustum if its axis is perpendicular to both ba-
ses, and oblique otherwise.
hs hs
We can easily verify the following properties of a regular frustum.
1. The lateral faces are congruent isosceles trapezoids.
a a
2. The altitudes of the lateral faces are all equal. This measure is a
the slant height of the frustum.
If the frustum is not a regular frustum, the lateral face trapezoids may have different
side lengths and slant heights.
3. The upper and lower bases are similar regular polygons.

72 Solid Geometry
EXAMPLE 81 How many face diagonals and space diagonals does a rectangular frustum have?

Solution Like a rectangular solid, a rectangular frustum has 12 face diagonals and 4 space diagonals.

EXAMPLE 82 What is the slant height of the adjacent regular frustum if the
lateral edge is 9 cm and the base edges are 7 cm and 13 cm?

3 cm 7 cm 3 cm
Solution Look at the figure. For this trapezoidal
lateral face we have 92 = hs2 + 32 by
hs the Pythagorean Theorem.
9 cm 6ñ2 cm
This gives hs2 = 72 and so
hs = 6ñ2 cm.

5
6
EXAMPLE 83 The adjacent frustum is from a regular square pyramid.
What is the height of this frustum? 6

Solution Look at the figure. First let us drop perpendiculars from points A and D. Notice that segment
LQ creates an isosceles trapezoid ADLQ. B A
5
In this trapezoid, 5
6
C
KH is congruent to AD and KH = 5 cm. D h
B¢ Q A¢
Also, LK is congruent to HQ, and H
2
2
5 R
9
LQ  AD 9  5 5
LK    2 cm. K
2 2 2 P
2
C¢ L D¢
By a similar argument, KP = 2 also, and if we
apply the Pythagorean Theorem in square PKLD', the diagonal KD' = 2ñ2 cm.
Note that in the right triangle DKD', DK is the height of the frustum and again
by the Pythagorean Theorem we have DD' 2 = DK2 + KD' 2  62 = h2 + 8. So h = 2ñ7 cm.

Polyhedrons 73
12
EXAMPLE 84 In the adjacent rectangular right frustum with the given edge
16
lengths, one of the slant heights is 5 cm. Find the altitude of
5
the main pyramid and the other slant height of the frustum. ha

hb 24

18

Solution To find the altitude of main pyramid of the frustum, V


we must complete the frustum to the main pyramid.

Look at the figure. Here, VH is the altitude of the


pyramid and since the points H and H' are the centroids
of the respective bases, we have HP = QB = 9 cm and D¢ 12 C¢
H'P' = 6 cm .
16 P¢
H¢ 6
Since HK  H'P' = 6 cm, we get KP = 3 cm and by C
D
the Pythagorean Theorem in right triangle KPP',
A¢ 5
we get the altitude of the frustum as KP' = 4 cm. Q B

Here we have two similar triangles: VHP  P'KP, so H K 3 P


hb
24
VH HP h 9
   . So the altitude of the main
PK KP 4 3
pyramid is h = 12 cm.
A Q 9 B
We can find the other slant height by working with right triangle VQH. However, is it necessary
to work on the main pyramid in order to find the slant height QQ'? Think about it. Finding
the other slant height is left as an exercise for you.

For Experts
6 cm
4 cm
The bases of this solid are parallel but not similar.
Is this solid a frustum by definition?
Can we always extend such a solid to get a pyramid?
What kind of problems could occur?
10 cm
5 cm

74 Solid Geometry
Check Yourself 7
1. What kind of polyhedron is a hexagonal frustum?
2. How many faces does a pentagonal frustum have? How many edges does it have?

3. For the adjacent square right frustum, find the slant height 6 cm

of the darker lateral face.


13 cm

16 cm

4. The base edges of a square frustum are 1 cm and 9 cm. If a lateral edge is also 9 cm, find
the height of the frustum.

5. The base perimeter of a square frustum lampshade is


128 cm and the dihedral angle between this base and a
lateral face is 60°. If the slant height is 20 cm, what is the
perimeter of the upper base of the frustum?

2
6. The slant height of the rectangular frustum in the figure is
4
5 cm. What is the other slant height of the frustum and
height of the main pyramid?

5
12

7. Try to construct a square frustum from cardboard by forming its net, given that the upper
and lower base edges are 2 cm and 8 cm and the lateral edge is 5 cm.

Answers
1. an octahedron 2. F: 7, E: 15 3. 12 cm

4. 7 cm 5. 48 cm 6. hslant = ò13 cm, hpyramid = 4.5 cm

Polyhedrons 75
D. SURFACE AREA OF A PYRAMID
We have seen that pyramid consists of a base and three or more triangular lateral faces. The
surface area of a pyramid is the sum of the area of its base and its lateral area. As an
example, look at the net of the rectangular pyramid below.

Triangle1

Remember!
Triangle2 Base Triangle4
The lateral area of a
solid is the sum of the Base Triangle3
areas of its lateral
faces.

Areapyramid = Areabase + Arealateral


The lateral area is the sum of the areas of the triangular lateral faces. Therefore the lateral
area formula for this pyramid is Arealateral = AreaT1 + AreaT2 + AreaT3 + AreaT4.
As we can see above, calculation of the lateral area depends on the dimensions of the triangular
faces. There is no standard area formula for a pyramid. Sometimes each lateral face area
must be calculated separately, and enough data must be provided for this.

EXAMPLE 85 Calculate the total surface area of the pyramid shown in the figure.
6
The pyramid has an equilateral triangular base and one edge
perpendicular to this base.

Solution If we open out the net, we can see that the lateral faces include two 8
identical right triangles and an isosceles triangle.
V 10 C

6 10
6 8

A 8
V
6 8
8 10
V 10 B
The pyramid has an equilateral base with side length 8 cm.
82 3
So the base area is Area base   16 3 cm 2. Also,
4
1 1
A(VBC)= BC .hBC = .8 .2 21 = 8 21 cm 2 (height of triangle VBC to BC is 2ò21 cm by the
2 2 Pythagorean Theorem)

As a result the lateral area is Area lateral  2  A( VAC )  A( VBC )  2  24  8 21 cm 2,

and so the total surface area is Area total  Area base  Area lateral 16 3  48  8 21 cm 2 .

76 Solid Geometry
1. Surface Area of a Regular Pyramid
If a pyramid is regular, calculating its lateral area is easier because the lateral faces are congruent
isosceles triangles. We can see this with the following pentagonal pyramid.
Lateral Surface

hs

hs a
a a a a a
a a a a
Base
a

a a

Here, a is the basal edge and hs is the slant height.


The lateral area is the sum of the areas of all the congruent isosceles triangles :
 a  hs  n . a . hs
Area lateral = n  Area triangle = n .   .
 2  2
1
Notice that n . a is the base perimeter. Therefore Area lateral = Pbase hS .
2
Therefore we can write a general formula for an n-gonal regular pyramid as
1
Arearegular pyramid = Areabase + Arealateral  Area regular pyramid = Areabase + Pbase hS.
2

EXAMPLE 86 Find the total surface area of a right square pyramid with altitude 16 cm and base edge 24 cm.

Solution A right square pyramid is a regular pyramid.


Look at the figure. VO is the altitude, and V V

by applying the Pythagorean Theorem in right triangle VOM,


hs2 = VO2 + OM2.
16 cm

hs 16 cm hs
So the slant height hs = 20 cm.
D
C
Applying the formula 12 cm
O M O 12 cm M
1
Area regular pyramid  Areabase  Pbase hs
2 A 24 cm B

1. .
for this pyramid, Area square pyramid = 24 2 + 4 24 . 20 =1536 cm 2.
2
Polyhedrons 77
EXAMPLE 87 What is the lateral surface area of a regular hexagonal pyramid-shaped teepee with height
8 feet, if the teepee base occupies an area of 24ñ3 feet2? V

Solution The teepee base is a regular hexagon. A hexagon has area


3a2 3
Ab ase  for base edge a.
2 h
l

3a2 3
Since Abase   24 3, we get a = 4 feet. B
2 O p K
To find the slant height l, we can use right triangle VOK. a
a A
The base consists of 6 equilateral triangles and the apothem p of the base is

a 3
p  2 3 feet. In VOK we have l2  p2  h2  l  (2 3)2  82  2 19 feet.
2
A tepee is a native 1 1
American tent made with Finally, the lateral area is Alateral  Pbase l and Ala teral   6  4  2 19  24 19 feet .
2

poles and animal skins. 2 2

EXAMPLE 88 A regular octagonal pyramid with apex T and base ABCDEFGH is given. The midpoints of CD
and GH are K and M respectively. The perimeter of triangle TKM and the perimeter of the
base are both 72 cm. Find the total area of the pyramid.
T

Solution This example shows us that it is possible to solve a geometry


problem without always using the same formula.

In the open net of the pyramid, we can see that the whole
G E
figure is the composition of 8 identical kites (like OCTD M
O D
H K
and OGTH). Remember the area formula for a kite: A C
B
( OK  KT )  CD
Area total  8  A( OCTD)  8 
2
 4 ( OK  KT )  CD.
T
Here, OK + KT is the diagonal of kite OCTD, and half the
perimeter of the initial triangle TKM. G

P(TKM )
Hence, in the open net: OK  KT   36 cm. H
O
2
On the other hand, we can find a side length of the base K D
P 72
as CD  base   9 cm. C
8 8
Since the diagonals of a single kite are OK + KT = 36 cm and
T
CD = 9 cm, we have Areatotal = 4  36  9 = 1296 cm2.

78 Solid Geometry
EXAMPLE 89 Find the surface area of a regular tetrahedron with edge length 11 cm.

Solution A regular tetrahedron is a formation


of 4 equilateral triangles.

So the area formula is


a2 3
Area total  4  Area equilateral face  4   a2 3.
4
For the tetrahedron in this question, the area is

Areatotal = a2ñ3 = 112ñ3 = 121ñ3 cm2.

Conclusion

 The surface area of a regular tetrahedron with edge length a is

S = a2ñ3.

 The surface area of a regular octahedron with edge length a is

a2 3
S=8   S = 2a2ñ3.
4

Check Yourself 8

1. What is the total surface area of the adjacent regular square 7m

pyramid?
·

12 m

2. The adjacent regular octagonal skylight window has basal edge


42 cm, lateral edge 75 cm and isosceles triangular lateral
faces. Find the total area of glass in the window.

Polyhedrons 79
3. Calculate the total surface area of the adjacent pyramid, given
that the pyramid has an isosceles right triangular base and an ò17
edge perpendicular to the base.

4. A cube with edge length 10 inches is cut as shown in the figure.


What is the surface area of the pyramid created?

5. What is the total surface area of a rectangular right pyramid with base dimensions
14 cm x 16 cm and lateral edge 17 cm?

6. Find the total surface area of a regular hexagonal pyramid with base perimeter 60 mm and
lateral edge 13 mm.

7. Find a formula for the surface area of a right square pyramid in terms of the base length
a and height h.

8. A regular pentagonal pyramid has base length 6 m and slant height 8 m. What is the
lateral surface area of this pyramid?

9. What is the total surface area of a regular octahedron with edge length 20 cm?

86 ft

10.The adjacent house is symmetrical.


20 ft
What is the surface area of its roof? 20 ft
100 ft

30 ft

Answers

1. 312 m2 2. 12096 cm2 3. 32 + 28ñ2 + 8ò17 cm2 4. 150 + 50ñ3 in2

5. 464 + 56ò15 cm2 6. 150ñ3 + 360 mm2 7. Area  a2  a a2  4h2 unit 2 8. 120 m2

9. 800ñ3 cm2 10. 5064 ft2

80 Solid Geometry
2. Surface Area of a Pyramidal Frustum
The general surface area formula for the frustum of a pyramid is the sum of the base areas
and the lateral area:
Areafrustum of pyramid = Areabaselower + Areabaseupper + Arealateral

Studying the net of a solid can be very useful in solid geometry. We can use this technique to
understand the surface area of a frustum of a pyramid. Let us open out the net of the
following triangular frustum:

A¢ Upper

c¢ b¢
c¢ B¢ a¢ Base C¢ A¢
C
C¢ A¢ B¢ C¢ b¢
B¢ a¢ Trapezoid2
A hc Trapezoid1 ha Trapezoid3
b hb
c
C A c B a C b A
a Lower
B Base

As we have already seen, the lateral faces of a frustum are trapezoids:


Arealateral = Areatrp + Areatrp + Areatrp .
1 2 3

ha h h
For a frustum like the one above, Area lateral = ( a + a )+ b ( b + b )+ c ( c+ c  ).
2 2 2
4 cm C
A
6 cm
4 cm
6 cm
B 12 cm F

90
D
·

EXAMPLE What is the total surface area of the adjacent triangular frustum?

12 cm

E
Solution The frustum is not regular. We need to apply the area formula for a trapezoid for the lateral
faces, and Heron's formula for the areas of the triangular bases.
Note that lateral edge AD is perpendicular to the bases, and the bases are isosceles triangles
with AB = AC. This gives us two congruent right trapezoid lateral faces (ABED and ACFD).
On the other hand, since a frustum has similar bases, using the similarity of the upper and
C
lower bases gives us EF = 18 cm. A 4 cm
6 cm
4 cm
6 cm
4 cm 6 cm
B F
·
10 cm 10 cm
10 cm 6 cm D 8 cm
10 cm
· 6 cm 6 cm 6 cm
8 cm 4 cm 18 cm 18 cm
12 cm

E
Polyhedrons 81
By the Pythagorean Theorem, we can find BE = CF = 10 cm. Therefore the height of
trapezoid BEFC is 8 cm.
Now we have the total lateral face area:
6  8
Area lateral =2×Area ABED +Area BEFC =2   (12+4) + (18 +6) = 192 cm 2.
2  2

Since the bases are two similar triangles for which we know only the side lengths, we apply

Heron's formula A= u (u – a) (u – b) (u – c) for P = 2u:

Area ABC = 7(7 – 4)(7 – 4)(7 – 6)=3 7 cm 2 and

Area DEF = 21(21–12) (21–12)(21 –18) = 21 9 9 3 = 27 7 cm 2 .

Now the total surface area of the frustum is

Area total =Area lateral +Area ABC +Area DEF =192 +3 7+27 7 = 192+30 7 cm 2 .

Is it possible to find the areas of the bases in another way? Give a reason for your answer.

For the frustum of an n-gonal regular pyramid, we can find an elegant formula for the
surface area. The lateral faces of such a pyramid are congruent isosceles trapezoids. Each
trapezoid has equal slant height hs, congruent lower base a, and congruent upper base a. The
lateral area is n times the area of a single trapezoidal face:
a'
a' a'
Area lateral = n .Area trapezoid face

hs
= n. ( a+ a  ) hs
2 hs

hs
= ( n .a+ n .a )
2 a a
a
Note that n . a is the perimeter of the lower base and n . a is the perimeter of the upper base.
hs
Area lateral = ( Pb ase 1 + Pb ase 2 )
2

Conclusion
a' a'
a'
The surface area formula for the frustum of regular pyramid is
hs
Area regular frustum =Areabase 1 +Area base 2 + ( Pbase 1 + Pbase 2 ). hs
2
a a
a
82 Solid Geometry
EXAMPLE 91 A regular frustum has equilateral triangle bases with 8 m and 12 m basal edges. If the slant
height is 6 m, find the total surface area of the frustum.

Solution Since the frustum is a regular frustum, the surface area is

hs
Area fr ustum  Area base1  Area base2  ( Pbase1  Pbase2 )
2 8 cm

82 3 12 2 3 6
   (3  8  3  12)
4 4 2 6 cm

Area fr ustum  180  52 3 cm . 2

12 cm

4 dm

EXAMPLE 92 What is the total area of tin required to make the square frustum-
shaped garbage can shown in the figure? 5 dm

2 dm

Solution We need to change the formula we used above slightly, since the garbage can has no upper
base:
hs
Area frustum  Area base1  ( Pbase1  Pbase2 ).
2
In this question, the slant height hs is not directly given. 1 dm 2 dm 1 dm
We must find it by using an isosceles trapezoid face.

Look at the figure. By the Pythagorean Theorem, hs = 2ñ6 dm. hs 5 dm

Now we can find the total area:

hs 2 dm
Area frustum  Area base1  ( Pbase1  Pbase2 )
2

2 6
 22  (4  2  4  4).
2
So we need 4 + 24ñ6 dm2 of tin to produce the garbage can.

Polyhedrons 83
Check Yourself 9

6ñ2 m
1. The adjacent right frustum has isosceles right triangular
bases with the hypotenuse lengths shown. If the height
of the frustum is 3 m, find the total surface area of the
frustum.
10ñ2 m

2. A regular octagonal frustum-shaped metal bowl has base


perimeters 40 cm and 64 cm. If the slant height of the
bowl is 10 cm, what is the total lateral surface area of the
outside of the bowl?

3. A regular square frustum with 7 cm and 11 cm basal edges has slant height 25 cm. What
is its total surface area?

4. The adjacent regular rectangular frustum-shaped lampshade


has bases 6 cm x 10 cm around the top and 16 cm x 20 cm
around the bottom. The lateral edge for all faces is 13 cm. Find
the area of green required required to cover the lateral surface.

5. The inside of a regular hexagonal frustum-shaped swimming


pool will be painted with waterproof paint. The top edge is
5 m and the bottom edge is 2 m. If the slant height is 3 m,
find the total area to be painted.

Answers

1. 116 + 8ò34 m2 2. 520 cm2 3. 1070 cm2 4. 624 cm2 5. 63 + 6ñ3 m2

84 Solid Geometry
E. VOLUME OF A PYRAMID
The two pyramids below have equal altitudes and the same base area. It can be shown that
any two sections of the pyramids taken parallel to and equidistant from the bases also have
the same area.

A Aý
h

B Bý

If AreaB = AreaBý then AreaA = AreaAý.

This property leads us to some properties of the volumes of pyramids using Cavalieri’s Principle.

Theorem Equal Volume Pyramids


If two pyramids have the same base areas and
the same heights then they have the same
volumes.

This theorem is an application of Cavalieri’s Principle, which we have already studied for
solids in general. A separate proof is not given here.

Conclusion

If the bases of two pyramids are the same polygon and


V W
their vertices are in the same plane parallel to their
b
bases, then their volumes are equal.

In the figure, VVABCDE = VWABCDE .

D
E C
a A B

Polyhedrons 85
Theorem Volume of a Pyramid
The volume of any pyramid is one-third the volume of a prism

with the same base and same height.


The formula for the 1
volume of a pyramid Vpyramid   h  Vprism
can be used for both 3
right and oblique h
pyramids. The volume of any pyramid is equal to one-third the product of
the area of its base and its height.
1
Vpyramid   h  Abase
3

Proof Calculating the volume of a pyramid was an interesting


challenge for early mathematicians. Probably the early
Egyptians knew the formula, and they also had an idea of a
proof. However, one of the first concrete proofs was made by
Eudoxus (408-305 BC). Eudoxus reasoned that a pyramid can
be approximated to a collection of n layers of square prisms of
reducing size.

Eudoxus’ proof requires calculus, and here we will show a more geometrical and easier proof,
which was given by Euclid. Euclid used the fact that any pyramid can be divided into multiple
Euclid (325-265 BC)
smaller triangular pyramids:
Z
Euclid of Alexandria is Z Z
He first showed that the volume of Y
one of the most famous Y
ancient Greek any triangular pyramid is one-third V
Mathematician. He is the volume of the prism that contains it, V V
the author of Elements, a V
set of books which as follows. If we complete the original
described the geometry pyramid VABC at the right to its prism
we now call Euclidean
with the same base and height h, we
geometry. Elements C C
included chapters on must add two more pyramids, namely C B
B B
perspective, plane and BVZC and BVYZ. These pyramids are
space geometry. B
A
separated by the diagonals of the
It is reported that when
faces of the prism. A
King Ptolemy asked
whether there was a
shorter way to learn
Since VC is a face diagonal, it separates the parallelogram face into two equal triangles VAC
geometry than reading and VZC. Considering the two pyramids raising on these bases ending up with the vertex B,
Elements, Euclid replied,
“There is no royal road
we can say our original pyramid has the same volume as the pyramid BVZC.
to geometry.” On the other hand, using the congruent triangles ABC and VYZ as bases, we can see that the
two pyramids VABC and BVYZ have the same volume since they have a common height
between their parallel congruent bases. (We can also say that these two pyramids with
congruent bases VAB and VBY have their vertices C and Z on the same line parallel to the
bases.)
86 Solid Geometry
We can conclude that the volumes of all three pyramids VABC, VBZC and VBYZ are equal.

Therefore the volume of our original pyramid VABC is one-third the volume of its prism ABCVYZ:
1 1
Vpyramid VABC   Vprism ABCVYZ , and so Vpyramid VABC   AABC  h .
3 3

This proves the proposition for a triangular pyramid. Now let us take any pyramid with altitude
h, like the pentagonal pyramid below, and draw diagonals AD and BD of the base from a
vertex like D.
S

S S

E A D

A D A D
B C B
D

B C

If we cut the pyramid with two planes through apex S and the diagonals AD and BD, we can
see that the pyramid has been cut into three smaller triangular pyramids with the same
height. The volume of the original pyramid is equal to the sum of the volumes of the new
small pyramids:
Vpyramid  VSDEA  VSDAB  VSDBC .
By using the result obtained above, we can write
1  1  1  1
Vpyramid    ADEA  h     ADAB  h     ADBC  h    h  ( ADEA  ADAB  ADBC )
 3   3   3  3
1
Vpyramid   h  Abase .
3
We can divide any pyramid into triangular pyramids like, this and the result will be the same.
This concludes the proof.

Conclusion
The volume of any pyramid is one-third the volume of the prism
with the same base and height.

1
Vpyramid   Vprism
3 h

Polyhedrons 87
V
EXAMPLE 93 What is the volume of this pyramid?

7 cm C

Solution The pyramid has a right triangle base whose area is 90  2 = 45 cm . 2 10 cm


A
1
By the formula Vpyramid   Abase  h , 9 cm
3
B
1
the volume is Vpyramid   45  7  105 cm 3 .
3
A B

K
O
EXAMPLE 94 Find the volume of the pyramid shown in pink in the figure if the
volume of the cube is 64 mm3.
C

Solution We will need to find an edge length of the cube.


E D
Since Vcube = a3 = 64 mm3, we have a = 4 mm.
We can describe this pyramid in two ways: as a pyramid with the right triangle base OKD and
height OB; or as a pyramid with equilateral base BKD and respective height. The second one is
comparatively more difficult to work with in this problem.
1 1 a2 1 42 32
Therefore we will use the first pyramid: Vpyramid   AOKD  OB    a    4  mm 3 .
3 3 2 3 2 3

EXAMPLE 95 A heptagonal pyramid-shaped cup with depth 21 cm can hold 1610 cm3 of
water.
The cup has a lid that fits the top exactly. What is the area of the lid?
Solution For any pyramid we have
1 3V 3  1610
Vpyramid   Atop  depthpyramid , so Atop  pyramid   230 cm 2 .
3 depth 21

EXAMPLE 96 In a square regular pyramid, all the edge lengths are a. Find the volume of the pyramid
in terms of a.
3a2
Solution In the right triangle KMC we have l2  by the Pythagorean K
4
Theorem, and in the right triangle formed by the height we have
2
a 3a2 a2 a2 a 2 h
a
h  l   
2 2
  , i.e. h  . l
2 4 4 2 2 a
2
Applying the volume formula for a pyramid, we have a
a C
M
1 1 a 2 a3 2 2
Vpyramid   Abase  h   a 2   unit 3 .
3 3 2 6

88 Solid Geometry
EXAMPLE 97 The adjacent oblique pyramid has a volume of 72 cm3.
h
Its pentagon base has area 36 cm2.
What is the altitude of this pyramid?

Solution Recall that the volume formula we have seen can also be used for oblique pyramids.

1 3V 216
Vpyramid   Abase  h, so the height of the pyramid is h  pyramid   6 cm.
3 Abase 36

86 ft

EXAMPLE 98 The house in the figure is symmetrical. In Check


20 ft
Yourself 8 you found the surface area of its roof.
20 ft
What is the volume of the house?

100 ft
30 ft

Solution The body of the house is a cuboid. The roof of the house is not a prism and is not a pyramid either.
However, we can separate it into three parts, which are a prism and two identical pyramids.
In the isosceles trapezoid VADT V
86 ft
T
we have BC = VT = 86 ft, so
AB = CD = 7 ft.
A 7 ft 20 ft E
B
By the Pythagorean Theorem 86 ft
M
in right triangle TDM, we have C
7 ft 15 ft
D
TD = 25 ft. V
86 ft T
Again by the theorem in triangle T
G
TCD, we get TC = 24 ft. h 24 ft h F
24 ft F E
We can apply the Pythagorean B
N
N
Theorem for the third time to 86 ft 15 ft
f C
30 ft
C 7 ft D
find the height h of the pyramid.

In right triangle TCN, h2 = TC2 – CN2 = 242 – 152 = 351, i.e. h = 3ò39 cm.
Remember that we have two pyramids like this.
Now let us find the volume of prism VBGCFT in the roof:
30  3 39
VVBGCFT  Abase  h  ACFT  BC   86  3870 39 cm 3 .
2
ln summary, the total volume of the house will be
Vhouse = Vbody + Vroof = Vcuboid + ( VVBGCFT + 2Vpyramid)

= 100  30  20 + (3870ò39 + 2  210ò39) = 60000 + 4290ò39 ft3.


Polyhedrons 89
EXAMPLE 99 Find the volume of a regular octagonal pyramid with base diameter 18 cm and height 11 cm.
P Q
Solution We need to find the base area of the pyramid.
Look at the figure. The radius is r = 18 ÷ 2 = 9 cm, r r
and the included angle is 45° since a whole octagon is 360°. a

Now let us find the area of the isosceles triangle POQ.


O
From the sine formula for the area of a triangle we have
1 2 1 2 81 2
APOQ   r  sin    92   cm 2 .
2 2 2 4
81 2
The base includes 8 of these triangles, so Abase  8  APOQ  8   162 2 cm 2 .
4
1 1
So we can find the volume of the pyramid as Vpyramid   Abase  h   162 2 11  594 2 cm 3 .
3 3

EXAMPLE 100 Draw the graph of the volume y of a square pyramid with height 6 depending on its base edge x.
Volume
y = 2x2
Solution The base of the pyramid is a square with area x2.

Therefore its volume is Vpyramid  1  Abase  h  1  x2  6  2x2 .


8

3 3
If we graph the volume as y, the corresponding graph will be
y = 2x2.
2
This graph is half a parabola. Notice that the origin is excluded Base edge x

from the graph. 1 2

EXAMPLE 101 This Tetra Pak milk carton has a regular tetrahedron shape.
If an edge length is 13 cm, find out if we can store 250 ml of milk in the
carton.
Solution First let us develop a general formula for the volume of a regular tetrahedron: V

a 3
Look at the figure. In right triangle VCN, l  VC  CN , so l 
2 2 2
. a
2
a 6. h l
In adjacent right triangle VON, h2 = VN2 – ON2, so h  C
3
a
AN A 2
(Note that O is the centroid of the base, so ON  )
3 O N
1 1 a2 3 a 6 a3 2 a a
Finally, Vtetrahedron  . Abase  h  .   . 2
3 3 4 3 12
B
a3 2 133 2
Now, considering Vcarton    259 cm 3  259 ml, we can say yes, the carton can
12 12
hold 250 ml of milk.
90 Solid Geometry
Conclusion Parallel Sections of a Prism

If a is an edge length then


a
 the volume of a regular tetrahedron is
a3  2 a a
V= .
12
a
 the volume of a square pyramid with congruent edges is

a3  2 a
V= .
6 a

 the volume of a regular octahedron is a

a3  2 a
V= .
3 a

Pyramids in a Cube

Any polyhedron can be divided into pyramids. As an activity, you can try making your own cube from three pyramids. The net
given below left is the net of a single pyramid. If you make three copies, you will get identical pyramids of equal volumes. Then
arrange them in the way shown below to make a cube.

ñ2

This confirms the formula Vc u b e  3  Vp y r a m i d .


Note that the square base of each pyramid is one face of the cube.

Polyhedrons 91
Check Yourself 10
1. A pyramid and a prism have congruent decagonal bases with area 22.3 m2. Their heights
are also identical and equal to 8.2 m. Give the ratio of their volumes.
2. A pyramid has a rectangular base with dimensions 12 m x 16 m and height 15 m. Find
the volume of the pyramid.
5m
3. What is the volume of this pyramid?

7m
4m

4. The volume of a regular hexagonal oblique pyramid is 20ñ3 m3 and its height is 10 m.
What is its base length?
5. The total surface area of a regular square pyramid is 144 cm2 and the length of one side
of its base is 8 cm. Find the volume.
6. A building contractor will construct a square right pyramid with base edge 150ñ2 m and
lateral edge 250 m. Find the volume of concrete needed, if the pyramid is solid.

7. What is the total volume of the house


5m
shown at the right?
3m

6m
6m

8. What is the volume of a regular tetrahedron-shaped Tetra Pak juice


carton, if one face is 36ñ3 cm2?

9. The pyramids in the figure have


similar heptagonal bases.
Pyramid I and pyramid II have the
same height, but the base edge of the
first one is half that of the second one.
Pyramid II and pyramid III have the
same base, but the height of pyramid I II III
III is half the height of pyramid I.
Order the volumes of the three pyramids.

Answers
1 70 3
1. 2. 960 m3 3. m 4. 2 m 5. 64 cm3 6. 3,000,000 m3
3 3
7. 108 + 12ñ7 m3 8. 144ñ2 cm3 9. VII > VIII > VI

92 Solid Geometry
F. VOLUME OF A PYRAMIDAL FRUSTUM
In 1892, a Russian Egyptologist named Vladimir Golenishchev bought an ancient papyrus
which was from the ancient city of Thebes in Egypt. The papyrus, which was written some
time between 1700 and 1850 BC, was a set of 25 math problems with their brief solutions.
Problem 14 on this papyrus is an interesting geometry problem. It is about the volume of a
frustum. The way the Egyptians solved the problem shows that they knew the formula for the
volume of a frustum, however we do not know how they found it.

Theorem Volume of a Frustum of a Pyramid

If the base areas of a frustum of a pyramid are A1 and A2 A2

and the height of the frustum is h, then the volume of


h
the frustum is
1
V = h( A1 + A2 + A1A2 ). A1
3

Proof As far as we know, this formula was proved many centuries later by the Greek mathematician
and engineer Heron of Alexandria (10-70 AD) and the 12th-century Arabic mathematicians.
The proof is as follows:
Since a frustum is a portion of a pyramid, to prove the
formula, we complete the frustum to its original pyramid.

Here, h is the height of the smaller pyramid.
Let us try to express h in terms of h, because we will
A2 h + h¢
need it.
The two pyramids are similar. As we know, there is a
h
relation between the heights and areas of these two
pyramids:
height 1 h  h A h  h A1 A1
  k  1  k2 , i.e.  .
height 2 h A2 h A2

h A1 h A1 A1  A2 A2
So 1     1  , which means h  h  .
h A2 h A2 A2 A1  A2

A1A2  A2
Finally, we get h  h 
A1  A2

Polyhedrons 93
The volume of the frustum is the difference between the volumes of the larger pyramid and
the smaller pyramid:
 h  h   h  h h h
Vfrustum  V1  V2    A1     A2    A1   A1   A2 ,
 3  3  3 3 3

h h
so VFrustum   A1   ( A1  A2 ) .
3 3
If we substitute our previous expression for h into this formula, we get

h h A1 A2  A2 h
Vfrustum   A1    ( A1  A2 )   ( A1  A1 A2  A2 ).
3 3 A1  A2 3

This concludes the proof.

We can use Problem 14 on the Moscow Papyrus as our


first example of using this formula. The problem on the
papyrus reads:
"If you are told: a truncated pyramid of 6 for the vertical
height by 4 on the base by 2 on the top:
You are to square the 4; result 16. You are to double 4;
result 8. You are to square this 2; result 4.
You are to add the 16 and the 8 and the 4; result 28.
You are to take a third of 6; result 2.
Some part from the problem-14 script of
You are to take 28 twice; result 56. Moscow Papyrus
See, it is 56. You will find this correct." 2

The problem calculates the volume of the adjacent frustum


in the following way: 6
6
 (42  2  4  2 2 )  2  (16  8  4)  2  28  56.
3
If we look carefully, we can see that the Egyptians used the
4
volume formula for a square frustum to solve the problem.

The Egyptians were more successful than the Babylonians in many areas of math. The
Babylonians also had good skills in geometry, but they calculated the volume of a frustum
incorrectly as the product of the height and half the sum of the base areas. Can you see why
they might have thought this was true?

94 Solid Geometry
EXAMPLE 102 Find the volume of a frustum with the base areas 12 cm 2
and 27 cm2 if the height is 7 cm.

Solution The volume of the frustum is 12 cm2

h 7 7 cm
Vfrustum   ( A1  A1 A2  A2 )   (27  27 12 12)
3 3
7 27 cm2
  57  133 cm 3 .
3

EXAMPLE 103 A bowl has a pentagonal frustum shape. Its base area is 180 cm
2
2
and
the top area is 320 cm .
If the bowl can contain 2 liters of water, find the depth of the bowl, to
the nearest centimeter.
h
Solution In the volume formula for a frustum VFrustum   ( A1  A1 A2  A2 ) , let us substitute the given
3
values:
h h
2000   (180  180 320  320 )  2000  740.
3 3
3  2000
So h   8 cm is the approximate depth of the bowl.
740

104 What is the volume of the regular hexagonal right frustum at the right?
3cm
EXAMPLE
The lower and upper basal edges are 7 and 3 cm respectively, and the 8cm

lateral edge is 8 cm.

7cm
Solution For a hexagonal base, the radius will be the same as the edge.
Q 3
Let O and Q be the centers of bases. 3 S
If we drop the perpendiculars QO and SP in right triangle PRS,
8
we have PR = 4 cm. Then by the Pythagorean Theorem we get
O 3
h = 4 3 cm. 4
7 P
147 3 27 3 7 R
Note that A1 = cm 2 and A2 = cm 2 .
2 2
Now let us apply the volume formula for a frustum:

h 4 3 .  147 3 63 3 27 3 
Vfrustum = .(A1 + A1A 2 +A 2 )=  + + 
3 3  2 2 2 
4 3 . 237 3
= =474cm 3.
3 2

Polyhedrons 95
EXAMPLE 105 This figure is a frustum of a right pyramid with 6m
two square bases. The upper base edge is 6 m, the
lower base edge is 12 m and the slant height is 5 m. 5m
a. Find the height of the original pyramid.
b. Find the volume of the frustum.
12 m

Solution In this problem, the height is not given directly. We need to find the height of the frustum
and the height of the pyramid.

a. Look at the figure. We have extended the


V
frustum to its pyramid.

We can see that the extension of the frustum


forms a new pyramid VABCD above it. Since

the two bases are parallel, the triangles VGK and
D¢ C¢
VGK must be similar: G¢ 3m


GK VK 3m 3m B¢ 5m
VGK  VG K   =  VK = 5 m. h
GK  VK  D C
G 6m
Triangle VGK is a right triangle, so the height K
of the small pyramid is h = 4 cm by the
Pythagorean Theorem. A 6m 6m B

On the other hand, the same similarity gives us

VK  VG  5 h
VGK  VG K   =  = , which means that h = h = 4 m.
K K G G 5 h

So the height of the pyramid is h + h' = 8 m.

1
b. Applying the frustum volume formula V  h( A1  A2  A1  A2 ) gives us
3
1. 4
V 4(144  36  144 36 )   252  336 m 3.
3 3

Note that if we only needed to find the height of the frustum, we could find it using an

easier way. However, the first question asked us about the main pyramid.

96 Solid Geometry
3m
EXAMPLE 106 This rectangular frustum-shaped water storage 12 m
tank is symmetrical and will be filled with water.
9m
How much water can it contain, to the nearest
liter? 5m
20 m

Solution We have rectangular bases with areas K 3m


A1 = 100 m2 and A2 = 36 m2. All we need 12 m
to find is the depth of the tank.
l1
l2 h
Since the tank is symmetrical, the lateral 9
faces are isosceles trapezoids. O
16 m M
This gives us MP = 1 m and NP = 4 m. N 4 m P1 m
Now, through one of the slant heights, we have to find the height h.
Let us chose l1.
Since the lateral edge is 9 m, in right triangle KMP we have
l12 = KP2 – MP2  l12 = 92 – 12 = 80 by the Pythagorean Theorem.
On the other hand, in right triangle KOM we have OM = NP = 4 m and
h2 = l12 – OM2,
h2 = 80 – 42  h = 8 m.
In summary, A1 = 100 m2, A2 = 36 m2 and h = 8 m.
Now we can use the volume formula:
h 8
Vfrustum  ( A1  A1 A2  A2 )  (100  100 36 36).
3 3
8 1568
Finally we find Vfrustum   196   523 m 3 = 523,000 L as the capacity of the tank.
3 3

For Experts

The basal edges of this regular right decagonal frustum are


4 cm and 8 cm, and its lateral edge is 10 cm. Find its volume,
to the nearest cubic centimeter.

Polyhedrons 97
Check Yourself 11

1. What is the volume of this pentagonal frustum if its height is 6 cm


and the base areas are 20 cm2 and 45 cm2?

2. The adjacent octagonal cake tin has base


areas 125 cm2 and 320 cm2. We use
1935 cm3 of cake mixture to fill it. What is
the depth of the tin?

3. Using Moscow Papyrus problem 14, write a formula for the volume of a square frustum
with base edges a and b and height h.

4. Using the formula obtained in question 3, find the top water surface area of a square
oblique frustum-shaped pool if its bottom base edge is 3 m, the depth is 6 m and the
volume of water needed to fill the pool is 98 m3.

5. Murat likes making aquariums. He builds for his


turtles a rectangular prism-shaped aquarium
with a pyramid inside. The aquarium has the 4 dm
same height as the pyramid (4 dm), but Murat
fills the aquarium to the mid height so that the
top of the pyramid is an island for the turtles.
3 dm
Find the volume of the portion of the pyramid
6 dm
left under the water.

6. Mirzabek has a rectangular frustum-shaped gold ingot with


dimensions 10 cm by 15 cm on the top and 14 cm by 21 cm at
the bottom. If a lateral edge measures 9 cm and the density of
gold is 19.6 g/cm3, find the mass of this ingot to the nearest
kilogram.

7. What is the height of the main pyramid of this


rectangular right frustum? 40 cm
30 cm
65 cm
80 cm

60 cm

Answers
h
1. 190 cm3 2. 9 cm 3. Vsquare frustum   ( a2  ab  b 2 ) 4. 25 m2
3
5. 21 dm3 6. 35 kg 7. 120 cm

98 Solid Geometry
EXERCISES 1 .2
A. Basic Concepts 13 . Find the height of a regular square pyramid if the
1. How many faces, edges and vertices does a hexagonal base perimeter is 48 cm and a lateral edge is 10 cm.
pyramid have? 14. The distance between V
2. Order the slant height, lateral edge and height of  AC and BV in the adjacent
a regular right pyramid. square pyramid is 1 unit.
Find the measures of D
3. Can we make a regular hexagonal pyramid with C
AVB and AVC. 2
all equal edge lengths? Explain your answer. A 2 B
B. Types of Pyramid
15. Find the dihedral angle between the base and a
4. The base perimeter of a regular hexagonal pyramid  lateral face of a square right pyramid, to the nearest
is 48 m, and the height is 6 cm. Find the slant height.
degree, given that the base area is 256 cm2, and the
5. A regular pentagonal pyramid with 4 cm base edge
lateral edge is 4ò11 cm.
and 6 cm slant height is opened along its lateral
edges to make a star-shaped net. What is the 16 . Evaluate the length of a space diagonal in a regular
perimeter of this net? octahedron with edge length 20 cm.
6. The open net of a x
rectangular pyramid
is shown opposite. 12 cm 15 cm C. Frustum of a Pyramid
One slant height is 24 cm 17 . How many faces does the frustum of a pentagonal
15 cm. Find the other pyramid have?
slant height.
18 . Find the area of a lateral face of a regular hexagonal
7. A 12 cm high right pyramid has a rectangular
base with sides 6 cm and 8 cm. Find a lateral edge. frustum with lateral edge 10 cm, given that the
base edges are 4 cm and 16 cm.
8. The rectangular base of a right pyramid has
dimensions 10 cm and 14 cm. Find the height of 19 . Find the height of the square
the pyramid if its lateral edge is 9ñ3 cm. 6 cm 8 cm
right frustum in the figure,
9. In the figure, the pink section is
6 cm and find the height of its
produced by a plane parallel to
main pyramid. 14 cm
the base intersecting the pyramid. b
Find a and b. 6 cm a 4 cm
20 . Find the altitude of the adjacent 8 cm
9 cm 12 cm rectangular right frustum.
9 cm
10 . The perimeter of the base of a regular heptagonal 20 cm
pyramid is 112 mm and the lateral edge is 28 mm.
10 cm
Find the length of the apothem.
11 . What are the slant height and median height of D. Surface Area of a Pyramid
a regular tetrahedron if an edge length is 12 cm? 21 . What is the lateral area of a regular nonagonal
12 . Evaluate the distance between the centroid and a pyramid with base perimeter 60 cm and slant
vertex of a regular tetrahedron with edge length height 13 cm?
16 m.

Polyhedrons 99
22 . What is the total surface V 30 . Is this figure a frustum? 6m
14 m
area of this rectangular If it is, find its lateral area.
8 cm
pyramid, given that AV
is perpendicular to the 14 m
A
base? 9m
15 cm
6 cm
31 . A square right pyramid with 20 m base edge and
24 m height is cut by a plane parallel to the base
23 . Find the lateral area of a pentagonal regular pyramid
with basal edge 11 cm and slant height 24 cm. halfway along the altitude. Find the area of a
lateral face of the frustum formed.
24 . What is the total surface area of a square right 4m
pyramid with basal edge 6 cm and height 2ò10 cm? 32 . The adjacent frustum has 6m
two similar isosceles triangle 3m
4m
25 . Originally, as the other 8m
bases. Find its total surface
pyramids, the Khafer
area.
Pyramid was covered 8m
with plates of bright
white limestone. Today 33 . Evaluate the total lateral area of a regular pentagonal
only part of this covering remains. Find the total frustum with basal edges 7 cm and 11 cm, given
area of the initial limestone covering to the nearest that the slant height is 16 cm.
cubic meter. The pyramid is a right pyramid with
base edge 215.5 m and a height of 143.5 m. 21 m
45 m
34 . Evaluate the total surface
26 . Find the total surface area of this rectangular right 25 m
area of this rectangular 25 cm 75 m
frustum.
right pyramid. The 35 m

foot of the altitude is at


35. The figure shows a 26 cm-deep
the center of the  metal flowerpot in the shape of a
14 cm 30 cm
base. regular hexagonal frustum.
27 . What is the surface area of a regular tetrahedron- Evaluate the area of metal in the
flowerpot if the base edges are
shaped Tetra Pak milk carton if one edge length is 6 cm and 18 cm.
10 cm?
28. Find the total surface area of this
 regular hexagonal pyramid to the b

nearest square centimeter if a base E. Volume of a Pyramid


edge of pyramid is 4 cm and the 36 . Find the volume of a pentagonal pyramid with base
angle shown is 45°. area 22 cm2 and height 15 cm.

29 . The adjacent net belongs 37 . The volume of a decagonal pyramid is


to a square right pyramid 160 m3 and the height is 10 m. Find the
with a 36 cm2 base and base area.
48 cm2 lateral area. Find
the perimeter of the net.

100 Solid Geometry


38 . Find the volume of 47 . The total surface area of a right square pyramid is
this equilateral triangle- 6 cm 144 cm2 and a basal edge is 8 cm. Find the volume.
based oblique pyramid. 48 . Find the height and the volume of a regular
4 cm
hexagonal pyramid with lateral edges 10 ft and
39 . A tent is to be made in the form of a right square base edges 6 ft.
pyramid. If the altitude is fixed at 12 ft, what 49 . What is the volume of a regular dodecagonal
must be the minimum side length of the base in oblique pyramid with base radius 4 cm and height
order to ensure that the tent encloses at least 400 9 cm?
cubic feet of space? 50 . What is the volume of a regular tetrahedron with
13 cm edge length 4 m?
12 cm
40 . Evaluate the volume of 51 . The distance between a vertex and the center of
7 cm
the solid formed by the a regular octahedron is 3ñ2 mm. Find its volume.
net shown opposite.

F. Volume of a Pyramidal Frustum


52 . A popcorn carton has a square
41 . Aziz wants to make a square right pyramid using frustum shape as in the figure.
an 80 cm bamboo pole. If the base edge must be
8 cm, what will the volume of the resulting
The base edges are 8 cm and POPCORN
12 cm. If the box is 15 cm deep,
pyramid be?
what is the volume of the box?
42 . Find the volume of this square
pyramid watermelon to the 53 . Find the volume of the
4 cm
8 cm
nearest liter if all the edges are adjacent rectangular right
approximately 20 cm.
frustum. 9 cm
20 cm
43 . A square right pyramid has a 2 cm base edge. If
the dihedral angle between the base and a lateral 10 cm
face is 30°, find the volume of the pyramid. 54. A right pyramid with a base of 120 m2 is 15 m

A B high. If the upper 6 m (measured along the
44 . Find the ratio between the K altitude) is removed, find the volume of the
O
volume of pyramid OBKD remaining frustum.
and the cube.
C 55. The flowerpot in the figure is 18 cm

the same 26 cm deep regular
E D
hexagonal metallic flowerpot
45 . The three sides of the base of a triangular pyramid
are 7 cm, 10 cm and 13 cm. Find the volume of from Question 35. Evaluate
the pyramid if its height is 60 cm. the volume of the flowerpot. 6 cm

46 . Joshua plans to 56. In a regular triangular frustum, the upper face


5m
construct the roof 
has side length 2ñ3 cm, the lower face has side
in the figure for 6m
length 6ñ3 cm and the lateral edge is 5 cm. Find
his house. Find 4m 8m 4m
the volume.
the volume of the roof.

Polyhedrons 101
CHAPTER SUMMARY
 A prism is a portion of a prismatic surface which is bounded  For a cuboid with sides a, b and c, A = 2(ab + bc + ac).

by two parallel planes.


 For a cube with edge length a, the surface area is 6a2.
 The two polygons created by the parallel planes and the
 The lateral surface area of an oblique prism with lateral
prismatic surface are the bases of the prism.
edge l is Alateral= l  Pright section.
 A right section of a prism is a section made by a plane
 For all prisms, the volume is V = Abase . Height .
which intersects all the lateral edges and is perpendicular
 For a cuboid with sides a, b and c, V = a . b . c .
to those edges.

 A prism can be right or oblique.  For a cube with edge length a, V = a3 .

 The altitude of a prism is the distance between the bases.  Two prisms with the same volume may not necessarily

have the same surface area.


 An n-gonal prism has n+2 faces, 2n vertices and 3n edges.

 Cavalieri's principle: If two solids have the same height


 A parallelepiped is a prism with six faces which are all
and the same cross-sectional area at every corresponding
parallelograms.
level, then they have the same volume.
 A cuboid is a parallelepiped whose faces are all rectangles.

In a cuboid, all the faces intersect at right angles.  As long as the base area and height of a prism do not

change, the volume of the prism does not change.


 The space diagonal of a cuboid with side lengths a, b and c

has length
2
a +b +c
2 2
.  The volume of an oblique prism is equal to the product of

the area of the base of the prism and its height.


 A square box is a prism with a square base.

 The volume of a parallelepiped with sides a, b and c is


 A cube (also called a regular hexahedron) is a parallelepiped
whose faces are all congruent squares. V = a  b  c  sin  sin where b and c represent the

edges of the base,  is the included angle between b and


 In a cube with edge length a, the face diagonal is añ2 and
c, and  is the angle between a and the plane of the base.
the body diagonal is añ3.

 A rhombohedron is a special parallelepiped which has  A pyramid is a polyhedron that is bounded by an apex, a

identical rhombic faces. polygon and the pyramidal surface that they generate.

 For all prisms, the surface area is A = 2Abase + Alateral .  In a pyramid, the vertex opposite the base is called the

apex of the pyramid.


 For a right prism, A = 2Abase + h . Abase .

102 Solid Geometry


 A n-gonal pyramid has n + 1 faces, n + 1 vertices and 2n  If two pyramids have the same base areas and the same

edges. heights then they have the same volumes.

 In a regular pyramid, the lateral edges are all congruent,


3
a 2
 The volume of a regular tetrahedron is V= .
the lateral faces are identical isosceles triangles and the
12
slant heights are all equal.
 The volume of a square pyramid with conguent edge
 The slant height of a regular pyramid is called the
3
a 2
lengths a is V= .
apothem of the pyramid.
6
 A regular tetrahedron is a triangular pyramid whose
3
a 2
edges are all congruent.  The volume of a regular octahedron is V  .
3
 The centroid of a regular tetrahedron is a quarter height
 If a frustum of a pyramid has base areas A1 and A2 and
above the centroid of each base.
a 3 h
 The slant height of a regular tetrahedron is . height h, then its volume is V = ( A1+A 2+ A1A 2 ) .
2 3

a 6
 The height of a regular tetrahedron is .
3
 A square pyramid is a pyramid with a square base and

triangular sides.
a 2
 The altitude of a right square pyramid is .
2
 A frustum of a pyramid is the lower portion of the pyramid

that remains when the top of the pyramid is removed

along a plane parallel to the base.


1
 The surface area of a regular pyramid is A=Ab a s e + Pb a s e .
2
 The surface area of a regular tetrahedron is A = a2 ñ3 .

 The surface area of a regular octahedron is A = 2a2 ñ3 .

 There is no single surface area formula for all frustums.

 The volume of a pyramid is one-third the volume of a

prism with the same base and the height.

Polyhedrons 103
19. If a prism is regular, what does this mean?
Concept Check
1. Does a solid figure have one unique net? 20. What is a parallelepiped?

2. How can we find the area of a prism? 21. A parallelepiped has no right angles. What can you say
about the shape of its faces?
3. State the formula for the surface area of a regular
tetrahedron. 22. What is a cuboid? How is it different from a cube?

4. Describe the relation between the surface areas of a 23. A cube has edge length 2 units. What are the lengths of

regular tetrahedron and a regular octahedron if they its face diagonal and space diagonal?

have congruent edge lengths. 24. What is a rhombohedron?

5. Which two things do we need to know to find the height 25. State a general formula for the surface area of a prism.
of an oblique prism?
26. State a general formula for the volume of a right prism.
6. Compare the longest side and the diagonals of a cuboid.
27. State two formulas for the volume of an oblique prism.
7. What is the difference between a cube and a rhombohedron?
28. Mustafa makes a prism from six unit cubes and another
8. What is Cavalieri’s principle? prism from 12 unit cubes. What can you say about the

9. Are all parallelepipeds rhombohedrons? Explain your volumes of these two prisms? What can you say about

answer. their surface area?

10. For a pyramid, compare the number of faces with the 29. State five important elements of a pyramid.

number of vertices. 30. How many space diagonals are there in a square pyramid?

11. What is the apothem of a pyramid, and an apothem of 31. The foot of the altitude of a pyramid is at the centroid of
the base? its base. What can you say about this pyramid?

12. Where is the centroid located in a regular tetrahedron? 32. What can you say about the shape of the lateral faces of

13. Why do you think the kings of ancient Egypt had their a regular pyramid?

tombs made in the shape of pyramids? 33. Give an example of a common frustum-shaped object in

14. Why is calculating the surface areas of pyramids and the world around us.

frustums easier if the solids are regular? 34. Sema cuts the top off different pyramids at different

15. Name the seven main elements of a prism. angles. Are the resulting solids frustums?

16. What is the difference between a face diagonal and a 35. State a general formula for the surface area of a pyramid.

space diagonal of a prism? 36. A prism has volume 15 cm3, and a pyramid has the same

17. A prism has octagonal bases. How many edges and height and base area. What is the volume of the pyramid?

vertices does it have? 37.State the formula for the volume of a frustum in terms of

18. How many space diagonals are there in a triangular its base areas and height.

prism?

104 Solid Geometry


CHAPTER REVIEW TEST 1
1. Which shape below cannot be made from the 6. In an octahedron, what are the possible vertex
intersection of a cube and a plane? angles included by two edges meeting at that
EEE
vertex?
A) triangle B) rectangle C) square CCC
D) trapezoid E) circle A) 30° , 60° B) 45° , 60° C) 60°, 90°
D) 75° , 90° E) 60° , 120°

2. Which number below can be the number of


vertices of a prism?
7. What is the total surface area of a regular
DDD octahedron if its edge length is 5 cm?
A) 4 B) 5 C) 9 D) 14 E) 19 BBB
A) 100ñ3 cm2 B) 50ñ3 cm2 C) 25ñ3 cm2

D) 75 cm2 E) 200 cm2

3. What is the difference in length between the


longest and the shortest diagonal of a regular
8. What is the length of a space diagonal of a cuboid
octahedron with edge length 12 cm?
AAA with dimensions 12 cm x 15 cm x 16 cm?
A) 0 cm B) 1 cm C) ñ2 cm D) ñ3 cm E) 2 cm CCC
A) 18 cm B) 24 cm C) 25 cm
D) 27 cm E) 21 cm

4. Melih has 3 identical cubes. Each cube has total


surface area 9 cm2. Melih joins the cubes in a
9. Many unit cubes are combined in a row to make
row to make a prism. Find the total surface area
a square right prism with surface area A square
of the prism.
DDD units. What is a possible value of A?
A) 12 cm2 B) 14 cm2 C) 18 cm2 DDD
A) 2011 B) 2012 C) 2013 D) 2014 E) 2015
D) 21 cm2 E) 27 cm2

10. If we increase each edge length of a cube by 1 cm,


5. Find the volume of a cube if its space diagonal is its total surface area will increase by 12 cm2.
6 cm. What is the new edge length?
AAA CCC
A) 24ñ3 cm3 B) 24ñ2 cm3 C) 72 cm3 A) 2.5 cm B) 2 cm C) 1.5 cm
D) 6ñ2 cm3 E) 12ñ3 cm3 D) 1 cm E) 0.5 cm

Chapter Review Test 1 105


11. One diagonal of the parallelogram base of a pyramid 15. I. height II. a base diagonal
is 12 cm. The pyramid is 8 cm high. Which one of III. apothem IV. a basal edge
the following is definitely a lateral edge if the foot V. half of a base diagonal
of the altitude is at the center of the base of the
In a right regular pyramid, which of the above
pyramid?
items is / are always shorter than the lateral edge?
CCC EEE
A) 14 cm B) 12 cm C) 10 cm A) only III B) I and V C) only I
D) 9 cm E) 8 cm D) II and IV E) I, III and V

12. The height of a regular tetrahedron is h. What is 16. The hexagonal frustum-shaped trophy
its surface area? at the right has 4 cm upper and 2 cm
AAA lower edge lengths. Find the volume
3h2 3 5h2 3 h2 3 of the trophy if it is 5ñ3 cm deep.
A) B) C)
2 3 4
CCC
h2
2 A) 150 cm3 B) 180 cm3 C) 210 cm3
D) h 3
2
E)
3
D) 220 cm3 E) 240 cm3

17. I. a tetrahedron with isosceles faces


13. The edge length of a single
II. a hexagonal pyramid with all equal edges
cell in the adjacent ice cube
III. an octahedron with all right triangle faces
tray is 2 cm. If the cells are 4
IV. a frustum with all trapezoid faces
cm deep, find the total
V. a cuboid with all similar faces
weight of the water needed
to fill the tray, to the nearest gram. Which of the above solids is / are impossible to
(1 cm3 of water weighs 1 g.) make?
DDD BBB
A) 525 g B) 540 g C) 564 g
A) I and III B) II and V C) IV and V
D) 582 g E) 605 g D) II and III E) only V

18. A standart international freight


10 m container is 40 feet long by 8
14. The adjacent block is a right 20 m
feet wide by 8.5 feet high
frustum with 25 m lateral
(12.2 by 2.4 by 2.6 meters).
edge. Find its total surface 25 m
Find the volume of a container
area. 50 m
to the nearest cubic meter, ignoring the thickness
BBB 40 m
of the metal.
A) 4200 m2 B) 4600 m2 C) 5200 m2 AAA
A) 76 m3 B) 52 m3 C) 152 m3
D) 3600 m2 E) 5600 m2 D) 104 m3 E) 84 m3

106 Chapter Review Test 1


CHAPTER 2

SOLIDS OF
REVOLUTION
INTRODUCTION
In this chapter we will look at a different group of solids. Unlike the solids in the previous
chapter, these solids have curved surfaces. This group includes three basic types of solid:
cylinders, cones and spheres.

A top is an example of
a solid of revolution
Cylinder Cone Sphere

When these solids are regular, they have perfect symmetry around a central axis. This property
means that we can generate these solids by rotating some planar figures around an axis.

A cylinder can be generated by any We can generate a cone by rotating A sphere can be generated
rectangle rotated around its side. a right triangle around one leg. The by any semicircle rotated
One side of the rectangle creates the leg forms the axis of the cone, and around its diameter.
axis of the cylinder, and the other the other leg is the radius.
side is the radius.

A D A
Making pottery on a A
spinning potter’s wheel

B C B C
A Axis D C
B Axis
A
A D
r

Axis Axis
B C
A cone created by
r B C rotation around
B C A cylinder created by r axis BC. The radius
rotation around axis is AB.
A cylinder created by AD. The radius is AB. A cone created by A sphere created by
rotation around axis rotation around rotation
AB. The radius is AD. axis AB. The radius
Shaping wood on a is BC.
lathe
Because of this property, these solids are called solids of revolution. There are several solids
of revolution, but in this book, we will study only the three types of solid shown above.

108 Solid Geometry


A. BASIC CONCEPTS
Definition Cylindrical Surface
Consider a fixed plane curve C and a line d which is not parallel to the plane of the curve.
If a moving line k always intersects the curve and remains parallel to d, then it generates a
surface called a cylindrical surface.
d k

generatrix

directrix

A generated cylindrical surface with its elements


The moving line is called the generatrix of the curve. The curve itself is called the directrix.
The generatrix in any of its positions is called an element of the surface.
If the directrix of a cylindrical surface is a closed curve then the surface is called a closed
cylindrical surface.

Definition Cylinder
A cylinder is a solid formed by the intersection of a
closed cylindrical surface and two parallel planes.
The two planes create two parallel shapes
which are the bases of the cylinder. The curved
surface joining the two bases is called the lateral
surface of the cylinder.

Elements of a cylinder
Base (upper)

Element
A line segment on the cylindrical
surface joining two corresponding points on the
Altitude
circumferences of the bases.
A line segment between the two
bases which is perpendicular
to the bases.
Lateral surface
The cylindrical surface included
Base (lower) between the planes of the bases.

Solids of Revolution 109


The length of any altitude is called the height of the cylinder. The height is the perpendicular
distance between two bases.
In the previous chapter we studied prisms, which we defined as solids with polygonal bases.
From one point of view, we can think of a cylinder as a special type of prism whose polygonal
base has infinitely many edges.

n=6 n = 12 n = 24 n=

As we can see, as n approaches to infinity, the solid gets closer to the shape of a cylinder.

Conclusion

 The bases of a cylinder are always congruent.


A C
 The sections of a cylinder made by two parallel planes
B
intersectiong all the elements are congruent. X
A'
 All right sections of a cylinder are congruent. C'
Y
B'
 The elements of a cylinder are all equal.

Two parallel sections of


a cylinder are congruent.

Definition Tangent Plane of a Cylinder

A plane that contains only one element of a cylinder is called a tangent plane

tangent plane of the cylinder.

Obviously, a cylinder has infinitely many tangent planes, but


through a given element, only one tangent plane can be drawn.

an element
of the cylinder

110 Solid Geometry


Theorem Section of a Cylinder Produced by Elements

Every section of a cylinder created by a plane which passes through exactly two elements of
the cylinder is a parallelogram.
D C
Proof Let AD be an element of the cylinder.
If a plane passes through AD and intersects the bases along AB and DC,
then AB // DC. (The bases are parallel and if two parallel planes are cut
by another plane, then the lines of intersection are parallel.)
Through C if we draw a line parallel to AD, it will intersect line AB. A B
Because A, B, C are coplanar, the line passing through C and parallel to the element AD is
another element, say BC.
Now we have AB // DC and AD // CB, and so ABCD is a parallelogram. This completes the
proof. We can see that in a right cylinder, a plane that cuts the cylinder forms rectangles.

B. TYPES OF CYLINDER
Like a prism, a cylinder can be either right or oblique.

A cylinder, depending on its base


may also be elliptical.
Right cylinder Oblique cylinder
Cylinders can also be classified by the shape of their bases.

Definition Circular Cylinder


A circular cylinder is a cylinder with circular bases.

h h
2pr
r
right circular oblique circular
cylinder cylinder
In this book, unless otherwise indicated, the word “cylinder” will mean a circular right
cylinder. When we talk about the radius or diameter of a cylinder, we mean the radius and
diameter of its base.

Solids of Revolution 111


In a circular cylinder, the line segment joining the
Axis:
Axial section The line segment two centers of the bases is the axis of the cylinder.
joining the two
centers An axial section of a circular cylinder is a plane
section which contains the axis.
Base
Radius

C
L
D
The axis of a cylinder is congruent and parallel to its elements.
As we saw in the previous section, ABCD opposite is a parallelogram.
Since AK = DL, AKLD is also a parallelogram, i.e.
KL // AD and KL = AD.
B
K
A

Definition Elliptical Cylinder


An elliptical cylinder is a cylinder with elliptical bases.

h h

Conclusion

 The axis of a right cylinder is one of its altitudes.


 The axis of a cylinder passes through the centers of all the sections parallel to its bases.

EXAMPLE 1 In the adjacent circular right cylinder the height is 8 cm and the perimeter
of a base is 11 cm. What is the area of the shaded triangle?

P 11
Solution Given the perimeter, the diameter can be calculated as d = = =11 cm, so the area of
 
d  h 11  8
the triangle is Atriangle = = = 44 cm 2.
2 2

112 Solid Geometry


Definition Slant Height

The slant height of an oblique cylinder is the length


of any element.
Slant height Height
Notice that the slant height is not equal to the
l l h
height of an oblique cylinder.

EXAMPLE 2 A circular oblique cylinder has an inclination of 60°. The radius of the
base is 2 m and the height of the cylinder is 6 m. What is the area of
the rectangle drawn in red in the figure?
60°
Solution In the figure, the blue parallelogram is in the inclination plane.
A
ABCD is perpendicular to that plane, and ABCD contains O1
Q
the axis, so it makes a 60° slope with the base.
D
To find the area of rectangle ABCD, we need to know the
height of the rectangle, which is O1O2. Here, l
l h
PQ = O1O2 = l and we can find l using trigonometry.
h 6
In triangle QPS, sin60  = and l = = 4 3 m.
l 3 B
60° 60°
2 P
S O2
AD is the diameter of the bases and AD = 2  2 = 4 cm. C

So the area of the rectangle ABCD is A(ABCD) = AD  l = 4  4ñ3 = 16ñ3 cm2.

EXAMPLE 3 A circular right cylinder with radius 10 cm and height P


D C
20 cm is cut by a plane P perpendicular to the base of the
A
cylinder, as shown in the figure. If the plane is 2 cm from B

the highest point of the circumference, what is the area of


the rectangle ABCD?

Solution If plane P is perpendicular to the base, it is parallel to the axis


T
of the right cylinder. The figure shows a cross section of the 2 6
A 6 D
cylinder. K
8
By the properties of a circle, if KT is the distance from AD to the 10 10

circumference then OT  AD, which gives us AK = KD. O


In right triangle OKD, we have KD = 6 cm. So AD = 12 cm.
Finally, the area of the rectangle is
A(ABCD) = AB  AD = 20  12 = 240 cm2.

Solids of Revolution 113


Check Yourself 1
1. A right circular cylinder has height 5 cm and radius 6 cm. A point K is marked on the
circumference of a base. If the farthest point from K on the cylinder is point P, what is
the length of segment KP?

2. The cannons shown in the picture are Shahi cannons,


which were used during the conquest of Constantinople.
Given that the inner perimeter of the cylindrical barrel
is 240 cm, what is the maximum possible radius of a
Shahi cannonball?

M
3. In the adjacent circular right cylinder, the height is 8 cm and the area
of a base is 9 cm2. What is the perimeter of the shaded triangle?

A D
4. The figure shows a circular oblique cylinder with slant height 8 m
and radius 5 m. Find the area of parallelogram ABCD.

C
45°
B

5. In the adjacent figure, a square prism is inscribed in a circular cylinder.


Find the area of the prism if the height of the cylinder is 9 cm and the
diameter is 8 cm.

Answers
120
1. 13 cm 2. cm 3. 24 cm 4. 40ñ2 m2 5. 64 + 144ñ2 cm2

The Tsar Cannon is one of the largest cannons ever made. It is 5.34
meters long, weighs an impressive 40 tons, and has a caliber
(internal diameter) of 89 cm.
It has never been fired in a war, but is on display in Moscow as a
fine example of Russian workmanship from 1586.

114 Solid Geometry


C. SURFACE AREA OF A CYLINDER
Similar to the net of a prism, the net of a cylinder consists of three parts: two congruent bases
and a lateral surface. The area of a cylinder is simply the sum of the areas of these surfaces.

A( )= + 2×

Theorem Lateral Area of a Cylinder

The lateral area of a cylinder is the area of its lateral surface. It


is equal to the product of the perimeter of a right section and
the length of an element of the cylinder. ri
l sectght
Alateral = Pright section  l ion
P

Proof As we did when proving the lateral area of a prism, we

open the net of the cylinder. Suppose that the lateral

area is separated into parallelograms, as shown in the h1


h2
blue net opposite. h3
l
If h is the height of a parallelogram then each small P

parallelogram has the area

Aparallelogram = l  hk, so A1 = l  h1 , A2 = l  h2 , ...

The sum of the areas of these parallelograms will give

us the area of the lateral surface.

ALateral = l  h1 + l  h2 + l  h3 = (h1 + h2 + h3)  l l


l R.S
.
P We can conclude ALateral = P  l. P

Increasing the number of parallelograms will make

them fit the curved surface better.

Solids of Revolution 115


EXAMPLE 4 Find the lateral area of an oblique cylinder if one element is 9 cm and
the circumference of its right section is P = 7 cm.

Solution By the theorem for the lateral area of a cylinder, l


Alateral = Pright section  l = 7  9 = 63 cm . 2

EXAMPLE 5 For an oblique circular cylinder, the inclination angle is 60° and the height is 12 cm. If the
perimeter of a right section is 5ñ3 cm, find the lateral area of the cylinder.

Solution The turquoise region in the figure is the right section of the G

cylinder. Now we need to find l = BG. In BCG:


CG CG 12
sin B =  BG = l = =  l = 8 3 cm. h
BG sin60  3
2
Now we know both necessary components, and so 60°
A C
O B
Alateral = Pright section  l = 5ñ3  8ñ3 = 120 cm . 2

If a cylinder is a right cylinder as shown in the figure below, the base will be congruent to a
right section (R.S.) and they will have the same circumferences, so the lateral surface will be
a rectangle with one side equal to the perimeter of the base.
P B2
P=L
B2
B1 // R.S. // B2 L
and
B1 @ R.S. @ B2
R.S.
h h
R.S.

B1

B1

In the case of a right circular cylinder,


we can obtain more precise results:

h h

r
L = Pbase = 2pr

116 Solid Geometry


Conclusion

In a circular right cylinder with radius r and height h,


 the lateral area is AL = 2rh.
h
Explanation: The circumference length (perimeter) of the base is
r
C = Pbase = 2r, so Alateral = Arectangle = Pbase  h = 2rh.

 The total surface area is Acylinder = 2r(r + h).


Explanation: The total area of the cylinder has two parts.
h
Acylinder = 2Abase + Alateral. The base is a circle with area r2 so
r
Acylinder = 2r2 + 2rh = 2r(r + h)

EXAMPLE 6 The bug shown in the adjacent figure moves around the lateral A
surface of an elliptical right cylinder on a helical path from A to B,
to reach some food. The height of the cylinder is 10 cm and the
base perimeter is 24 cm A and B are on the same vertical element.
Find the distance the bug must travel.
B

Solution If we open the net of this cylinder, we will obtain a rectangle as shown below, in which the
path AB shows the path of the bug.

A A

B B' B

In right triangle ABB, by the Pythagorean Theorem, AB = AB + BB = 10 + 24 = 26 cm.


2 2 2 2

EXAMPLE 7 Find the lateral area of a cylinder with radius 4 cm and height 6 cm.

Solution The lateral area of the cylinder will be Alateral = 2rh = 2  4  6 = 48 cm2.

EXAMPLE 8 Furkan rolls a square sheet of tin with area 100 cm2 to make a cylinder. What is the diameter
of the cylinder?
Solution The side length of the square is 10 cm, which is also the perimeter of the circular base.
P = 2r = 10 gives us the diameter as D = 2r = 10/  3.2 cm.

Solids of Revolution 117


EXAMPLE 9 What is the lateral area of a circular right cylinder if its base area is 25 m2 and its height is
10 m?

Solution To find the lateral area, we need to know the base perimeter. We can get this from
the radius: Abase = r2 = 25 m2.
So r = 5 m and Alateral = 2rh = 2  5  10 = 100 m2.

EXAMPLE 10 Find the total surface area of a cylinder if its radius is 4 cm and height is 5 cm.
Solution The total area of this cylinder is ACylinder = 2r(r + h) = 2  4(4 + 5) = 72 cm2.

EXAMPLE 11 Find the total surface area of a cylinder if the diameter of its base and the height are both 4ñ7 m.
d 4 7
Solution The radius is r = = = 2 7 m and the surface area will be
2 2
Acylinder = 2r(r + h) = 2  2ñ7(2ñ7 + 4ñ7) = 4ñ7  6ñ7 = 168 m2.

EXAMPLE 12 A 4 cm x 6 cm rectangle is rotated around one of its sides and then the other. Compare the
lateral areas and total surface areas of the two generated cylinders.
6 cm h=6
Solution The two possible cylinders are shown opposite.
The scale factor of the lateral areas is 4 cm 2
AL1 2 r1h1 6  4
   1.
AL2 2 r2 h2 4  6 r=6 r=4

This means they have equal lateral areas.


1
h=4

Comparing the total surface areas is left as an


exercise for you.

EXAMPLE 13 Murat rolls up a rectangular sheet of paper with dimensions 7 cm × 10 cm so that it
becomes a circular cylinder with height 7 cm. Then the bases are covered. Find the greatest
possible total surface area of the obtained cylinder.

ALateral = 7  10 = 70 cm2 and the height is 7 cm. r


Solution
10p cm
This means that the rectangle is rolled around the
side of 10 cm. This will be the base perimeter.
7 cm 7 cm
By the formula Pbase = 2r, we get the radius r = 5 cm.
So the total area is 10p cm

Acylinder = 2Abase + Alateral = 2  25+ 70 = 120 cm2.

118 Solid Geometry


EXAMPLE 14 A rectangle with perimeter 28 cm is rotated around one of its sides so that it forms a circular
cylinder. If the perimeter of a base is 5 cm, find the total surface area of the cylinder.

Solution The perimeter of the rectangle is PRectangle = 2  (r + h) = 28 cm. Hence r + h = 14 cm.


The perimeter of a base is PCircle = 2r = 5 cm.
The total surface area of the cylinder is ACylinder = 2r2 + 2rh = 2r(r + h). If we substitute,
we get the total surface area of the cylinder as: ACylinder = 5  14 = 70 cm2.

Check Yourself 2
1. Find the lateral area of an oblique cylinder if its base perimeter is 24 m and the length of
an element is 15 m.
2. Find the total area of a circular oblique cylinder with 30° inclination if its base perimeter is
8 cm and the height is 6 cm.

3. How many square meters of metal are needed to make the outer
surfaces of a cylindrical metal water tank with diameter 5 m and
height 8 m?

4. What is the base area of a cylinder with lateral area 90 cm2 and height 5 cm?

D
C
5. In the figure, A and D are the centers of the bases of the cylinder,
and B and C lie on the same element of the cylinder. What is the
ratio of the lateral area of the cylinder to the area of the rectangle?

A
B
6. The longer side of a rectangle with dimensions 10 m × 20 m is rolled so that it becomes
a circular cylinder. Then the bases are covered. Find the maximum possible total surface
area of the cylinder obtained.

7. In the picture, the road roller's barrel is 2 m wide and 1.2 m


high. Find the area of asphalt, to the nearest square meter, that
can be rolled with 20 rotations of the barrel.

Answers
1. 360 m2 2. 128 cm2 3. 40 m2 4. 81 cm2
 1
5. 26. 200 1   m 2 7. 48  150 m2
 

Solids of Revolution 119


D. VOLUME OF A CYLINDER
Since prisms and cylinders are related solids, we can use the same approach to find their
volume.
C r

Let us develop a formula for the volume of a cylinder from the given
parameters r (radius) and h (height). h

A cylinder can be separated into layers, just like a prism. If the radius is r and the height is
h then there are h unit layers. So the
C r C r
total volume will be h times the volume
of one layer:
Volumecylinder = Volume1 layer  h. h h layers

Since each layer has 1 unit thickness,


the volume of each layer has the same 1 unit
magnitude as the area of the base.
Therefore the formula becomes
Volumecylinder = Areabase  height.

Theorem Volume of a Cylinder

The volume of any cylinder is the product of its base area and its height.
Volumecylinder = Areabase  height
height

Base

Proof We can use the relation of cylinders and prisms in the following
way.
Consider a right prism inscribed in a right cylinder with the
same height, as shown in the figure. The base of the prism is B' h
and the base of the cylinder is B. For the prism, the number of
sides is n.
Therefore the volume of the prism is
B
Vprism = AB  h.

120 Solid Geometry


Obviously there is a difference between the bases of both figures. As we increase the number
of sides, the prism will get closer and closer to a cylinder, but the volume formula for the
prism will remain unchanged.

n=5 n=8 n = 15 n = 24

As the number of sides of the base approaches infinity, the base of the prism will get closer
and closer to a base of the cylinder: B  B.
This means that our prism will become the cylinder circumscribing it. We can now apply the
same formula for the volume of cylinder: Vprism = Vcylinder  Vcylinder = AB  h.
With the help of Cavalieri's principle, we can generalize this formula for all the other cylinders.
This completes an intuitive proof of the theorem.

Remark
One way to find the area of a circle is to divide the
circle into sectors and rearrange the sectors to
h
make a shape close to a rectangle. h
S S

S S

The cylinder above is divided into 8 equal sections. If we rearrange the sections, which
figure will the cylinder approximate?
Notice that increasing the number of sections results in a smoother rectangular prism than
above. Using the length, width and the height of the new solid, try to develop an alternative
volume formula for a circular cylinder in terms of its radius and height.

Solids of Revolution 121


EXAMPLE 15 What is the volume of the adjacent cylinder if its
base area is 45 m2? 11 m

Solution Vcylinder = Abase  h = 45  11 = 495 m3.

Remark
By the definition of a cylinder, the following solids are also cylinders and their volumes can
be calculated using the same formula.

Conclusion

 In a right circular cylinder with radius r and height h, the


volume is h
Base
Vcircular cylinder = r2  h.

 In an oblique cylinder with slant height l and inclina-


l
tion angle , the volume is a
Base
Voblique cylinder = Areabase  l  sin 

Explanation: By trigonometry, h = l  sin  .

EXAMPLE 16 Find the volume of a glass beaker with 5 cm radius and 10 cm height.

Solution The volume of the beaker is V = r2h = 52  10 = 250 cm3.

EXAMPLE 17 Find the height of a cylinder with volume 24 m3 and radius 4 m.

24 3
Solution By the formula V = r2h we get 24 = 42  h, so h = = m.
16 2

122 Solid Geometry


EXAMPLE 18 An oblique circular cylinder has 12 m base perimeter and axis length 9 m. If the angle of
inclination is 60°, find its volume.

Solution The base perimeter is P = 2r = 12 m, so the radius is r = 6 m.


6m
Note that the axis of an oblique cylinder is not the same as its
height. In this question, the height is found by trigonometry.
h 9 3 9m h
sin60  = , so h = m.
laxis 2
60°
By the volume formula V = r2h we get
9 3
V = 6 2 =162  3 cm 3.
2

EXAMPLE 19 Find the volume of a circular right cylinder made of tin, if 50 cm2 of tin is used for the bases
and 90 cm2 of tin is used for the lateral surface.
5

Solution 2
The base area is 25 cm , so the radius r = 5 cm.
The lateral area formula for the right circular cylinder is
AL = 2rh = 2  5h = 90 and so the height is 9 cm.
So Volumecylinder = r2h = 52  9 = 225 cm3.

EXAMPLE 20 How many cubic meters of iron are used to make a 12 m long iron pipe with external
diameter 60 cm and internal diameter 56 cm?

Solution The base of the pipe is a circular ring.


The area of this ring is the difference of the outer and inner
circles. The radii of the outer and the inner circles are 30 cm and
28 cm respectively.
Therefore the area of this ring is
r2 A
Aring = Aouter circle – Ainner circle = 302 – 282 = 116 cm2 = 0.0116 m2. O
r1
So the volume of the pipe is
V = Abase  h = 0.0116  12 = 0.1392  0.14 m3.

Notice that we can also find the same result by taking the difference of the volumes of two
cylinders.

Solids of Revolution 123


EXAMPLE 21 The cylindrical cake shown at the right has 32 cm diameter
and 9 cm height. If the slice taken out has a 50° central angle,
find the volume of the slice.

Solution By the definition of a cylinder, the slice can also be considered as a cylinder. Therefore we
can apply the volume formula of the cylinder to the slice.
 50  320
First we find its base area: Aslice base = Acircle =   16 2  =  cm 2 .
360 0 360  9
320
So the volume of the slice will be Vslice = Aslice base  h =  9 = 320  cm 3 .
9
Another way to obtain the same result is to find the volume of the whole cake, then use
proportion to find the volume of the slice.

EXAMPLE 22 A cylindrical candle is made from paraffin wax consumes 10 cm3 of paraffin in 1 hour. If its
diameter is 5 cm and the height is 12 cm, find how long approximately the candle can stay
alight.

Solution The required time is proportional to the volume.


2
5
The volume of the candle is V = r 2 h =     12 = 75  cm 3 .
2
This volume is approximately 236 cm3.
Since 10 cm3 paraffin is burnt each hour, the candle can stay alight
for nearly one day.

For Experts

Uncle Halil is making soup in a cylindrical pan with diameter


24 cm.
The spoon he is using is 26 cm long and has volume 32 cm2.
It accidentally falls and sinks into the soup. Calculate the
minimum volume of soup that must be in the pan if the
soup completely hides the spoon.

124 Solid Geometry


EXAMPLE 23 A cylindrical bottle containing some water is lying 10

horizontally on the ground. If the radius of the base is


6 cm, the length of the bottle is 10 cm and the surface
of the water is 3 cm from the ground, find the volume 3
of water in the bottle.

Solution The cross section of the bottle is shown in the figure.


We have OP = OR = OB = 6 cm and AB = 3 cm.
So OA = OB – AB = 6 – 3 = 3 cm. O
6 60° 60° 6
30°
Since the surface of water is parallel to the ground, P A R
3
OA 3 1
AOR is a right triangle and   . B
OR 6 2
We can easily conclude that triangle AOR is a 30° – 60° – 90° triangle.
It is clear that m(AOR) = 60° and m(POR) = 120° .
Therefore the area of the segment formed by the water is

Awater  Asec tor POR – Atriangle POR

POR 1
    OR2 –  OP  OR  sin POR
360  2

120  1 3
    62 –  6  6 
360  2 2

 (12  – 9 3) cm 2.

The volume of water is equal to the product of the base area (i.e. the area of the segment we
have just found) and the length of the bottle. Therefore it is
V = A  h = (12 – 9ñ3)  10 = (120 – 90ñ3) cm3.

For Experts

The circular cylindrical tank shown in the figure is


partly filled with water. The diameter of the tank is 1 m
and its length is 2 m. The depth of the water at its
deepest point is 75 cm.
Find the volume of water in the tank, to the nearest
liter.

Solids of Revolution 125


Check Yourself 3
1. The cylindrical beaker in the picture has radius 3.5 cm and height 10 cm.
Find the volume of the beaker, to the nearest cubic centimeter.

2. The slant height of this circular oblique cylinder is 12 cm. If its angle of
inclination is 60° and the base diameter is 4 cm, find the volume of the
cylinder.

3. Murat has to fill a cylindrical barrel with diameter 56 cm and


height 85 cm with water from a small cylindrical bucket with 14
cm radius and 34 cm height. At least how many times must Murat
fill the bucket?

4. These rooms made from pieces of pipe are rooms at a designer


hotel in Austria. Their inner length is 3.2 m and the volume
inside is 10 m3. Find the approximate inner diameter of one pipe
room.

5. The adjacent can is 12 cm high and contains 170  cm2 of tin. Find
the volume of the can.

Answers
245
1.   385 cm3 2. 24ñ3 cm3 3. 10 times 4. 2 m 5. 300 cm3
2

A pizza is a cylindrical object. Consider the pizza below with radius z and height a:
The volume of the pizza will be:

z APizza = Abase . h = . r2 . h = pi . z . z . a

126 Solid Geometry


EXERCISES 2 .1
A. Basic Concepts 7. What is the length of the longest piece of spaghetti
which can fit in a cylindrical spaghetti jar with
1. Which objects below are solids of revolution?
internal radius 4 cm and height 15 cm?
a. b. c.
8. The diameter of a right circular cylinder is 8, and
the diagonal of the largest rectangle made by its
vertical cross section is 16. Find its altitude.

d. e. f. 9. Omar wanted to make a D C



cylinder by rolling up rectangle x x

ABCD shown at the right and E F

joining AB with CD. Then he


g. h. i.
realized that the cylinder
would be too big. Shortening A B
ABCD by x, he rolled rectangle ABFE and joined
AB with EF. What is the difference between the
2. Which everyday object radius of this cylinder and the radius of the first
do we get by rotating the one?
adjacent red curve around
10. In the adjacent circular right
axis t?
cylinder, the area of a base is
B. Types of Cylinder Concepts 36 cm2 and the area of the
3. What is the diameter of the cylinder obtained by triangle is 24 cm2. What is
rotating a rectangle of 20 cm perimeter around its the height of the cylinder?
3 cm side?
11. What is the area of the axial section of a right
4. What is the base area of a cone if it is obtained by circular cylinder with 5 m height and 6 m base
rotating a right triangle with 10 cm hypotenuse perimeter?
around its 7 cm leg?
12. A student chooses six elements on the lateral
5. What is the loci of points in space which are surface of a cylinder. The distance between each
equidistant to a line? pair of adjacent elements is equal to the radius of
the cylinder. If the elements were the lateral edges
6. For the cylinders below, draw the open nets. of a prism, which type of prism would they form?
a. b.
13. A cylinder with 7 cm radius is intersected by a
plane perpendicular to the bases, creating a
square section. The plane is 3 cm from the axis of
the cylinder. Find the area of the square section.

Solids of Revolution 127


14 . The endpoints of two elements of a cylinder on 21 . The height and the diameter of this
one base form the endpoints of an arc of 90° on cylindrical drinks can are respectively
the circumference of that base. If the length of 12 cm and 5 cm.
this arc is 9 cm, what is the distance between Muaz removed the label covering the
the elements? whole lateral surface. What was the
shape and minimum area of the label?
15 . The radius of this 30°
oblique circular cylinder is
22 . Find the lateral area of a cylinder if its radius is
3 units. Find the area of its
4 cm and the height is 5 cm.
axial section.
16. This staircase with 1.2 m 23 . Jamo rolls a sheet of A4 paper and joins the short
 radius makes one full rotation edges to make a cylinder. What is the diameter of
and climbs 3 m. the cylinder? (A4 paper is 21 cm x 29.7 cm.)
Find the approximate distance
covered by a boy walking up the 24 . Mamuka wants to cut
staircase, assuming that he these sections with dashed
walks along the middle of the borders from a piece of
stairs. rectangular paper, and then
he wants to make a cylinder with them. What is
17. The staircase around this 10 m
 the area of the whole sheet of paper?
high grain silo runs along a 120°
section of the curved silo wall. 25 . What is the total surface area of a cylinder if its
Find the length of the staircase radius is 7 m and the height is 13 m?
if it is 4.5 m from the central axis of the silo.
26 . The fish can in the picture has
diameter 10 cm and height 4 cm.
C. Surface Area of a Cylinder Find the total area of material used
18 . If an oblique cylinder has a circular base, is its to make the can.
right section a circle?
27 . Complete the table with the missing information
19 . What is the lateral surface area of an oblique
for four cylinders.
cylinder if its element is 8 inches long and the
perimeter of its right section is 24 inches? Lateral One base
Radius Height Total area
area area
20 . The cylinder in the picture has
a base area of 7.5 m2 and a Cylinder1 7m 10 m
base perimeter of 12.5 m. The Cylinder2 12 cm 25 cm2
height is 6 m.
Cylinder3 3 mm 66 mm2
a. Find its lateral area.
Cylinder4 60 cm2 220 cm2
b. Find its total surface area.

128 Solid Geometry


28 . A cylinder-shaped bar of soap is cut through its 37 . The base of this can
axial section into two halves. Find the total consists of two semicircles
surface area of one half in terms of its radius r combined with a rectangle
and height h. in between. The length of
the can is 20 cm, the width is 8 cm and the depth
29 . The surface area of a circular cylinder is 96 cm2,
is 5 cm. Find the external surface area of the can.
and the diameter is equal to the height. Find the
radius.
38 . Verify the total area formula
30 . A square with side length 11 cm is rotated around A = 2(R + r)(R – r + h) for
one of its sides. Find the total surface area of the a pipe with external radius R,
generated cylinder. internal radius r, and height h.

31 . If one base area is equal to the lateral area of a 39 . The external and internal
circular cylinder, what is the relation between h diameters of this iron pipe
and r? are 14 cm and 12 cm. Use
32 . If one base area is equal to the area of an axial the formula in the previous
section of a cylinder, express the total surface question to find its total
area in terms of r. surface area if the length of the pipe is 4 m.

33 . Find the total surface area of a cylinder to the 40. The cylinder in the figure is

nearest square meter if its radius is 2ñ5 m and its surrounded by three congruent
height is 8 m. square tangent panels with the
34 . Can we roll a 55 cm long same height as the cylinder. If
strip of a paper 25 times the total area of the panels is
around a pencil with 243 cm2, find the lateral area
7 mm diameter? Give a reason for your answer. of the cylinder.

35 . The paint roller in the 41 . A 90° wedge is removed O

picture has 3 cm radius from the cylinder in the


5 cm
and 18 cm length. figure. Find the surface
Find the area of the area of the remaining
surface it covers if it solid. 3 cm
rotates 20 times.
42. A 3 m long cylindrical
36 . The lateral surface of a cylindrical 
log with 12 cm radius is
metallic garbage bin is perforated 6 cm
floating in water. A
by 800 identical small circular
section of height 6 cm
holes with 5 mm radius. If the
above the water
radius of the bin is 20 cm and the height is 50 cm,
remains dry. Find the dry area of the log.
find the area of metal in the lateral surface.

Solids of Revolution 129


D. Volume of a Cylinder 50 . The water reservoir of this
Turkish-style cylindrical tea
43 . Find the volume of this
urn has diameter 20 cm and
cylindrical lampshade with
height 32 cm. Find its
height 1.5 dm and base area
capacity, to the nearest liter.
6 dm2.

44 . The volume of the cylindrical 51 . The biggest piece in this percussion


tabletop shown in the picture set is a drum with 40 cm diameter
is 6 dm3. If it is 5 cm thick, and 40 cm depth. Find its volume
find the area of the table top. in liters.

45 . The volume of the bucket


52 . This glass demijohn has a
of this excavator is 1.1 m3.
capacity of 19 liters. Find its
The width of the bucket is
internal diameter, to the
2.2 m. What is the base 40 cm
nearest centimeter.
area of the bucket?

46 . What is the volume of 53 . Find the ratio of the volumes of two different
this oblique prism cylinders with the same height if the radius of the
which has height 5 feet first cylinder is one-quarter the radius of the second.
and a radius of 3 feet?
54 .
3.25 cm
47 . Find the volume of an oblique cylinder if its base
area is 32 cm2 and the length of an element which
makes a 45° angle with the base is 14 cm. What is the volume of this cylindrical can, to the
nearest cubic centimeter, if these 3 balls fit snugly in?

48 . Find the volume of this


55 . Find the volume of a 0.5 mm pencil lead if its
oblique circular cylinder
standard length is 12 cm.
with radius 10 mm.

56 . The diameter of a 1 Turkish Lira coin


49 . The pans in the picture
is 26.15 mm. If the thickness is 1.95
are special copper pans
mm, find the volume of metal used in
for community cooking in
one coin.
Turkey. They have
cylindrical shapes with 57 . A standard metal drinks can holds
different sizes. Find the volume of a pan with 330 ml of liquid. Find one and verify
70 cm diameter and 20 cm depth, to the nearest its volume by measuring the dimensions.
liter.

130 Solid Geometry


58 . The cylindrical box at the right 65 . A contractor company used
has a base perimeter of 26 cm. nearly 450 m2 of metal for the
Find its volume if its total surface construction of the lateral
area is 572 cm2. surfaces of these 3 identical
silos. They are working on the
59 . This beaker holds 2 liters of fourth silo now. If the height of
water. If it is 16 cm high, find its the silos is 12 m, find the total capacity of the four
silos, to the nearest cubic meter.
diameter.

66 . This 12 ml syringe has an


60 . A cylindrical tin with diameter 6 cm and height internal diameter of 14 mm.
2 cm is full of water. Can a box with dimensions Find approximately how many
7 cm x 2 cm x 4 cm contain all the water in the millimeters the plunger must be
cylinder? pulled to take in 4.6 ml of
medicine.
61 . This tent is a shelter mounted
67 . A slice taken out of this
on a sledge in Antarctica. It is
cylindrical cheese has a
in the shape of a half cylinder.
central angle of 80°. The
The height is 3.4 m and the
radius and the height of the
length is 5 m. Find its volume.
wheel are both 12 cm.
a. Find the area of the slice.
62 . The area of the largest right section of a cylinder
is 30 cm2 and the base area is 9 cm2. Find the b. Find the volume of the slice, to the nearest
cubic centimeter.
volume of the cylinder.
68 . In a particular country, it is
63 . Find the ratio of the volumes of two different illegal to put anything longer
cylinders obtained by rotating the same rectangle than 12 m on a trailer. If
around its two different sides of 3 cm and 5 cm. Michael wants to order
construction of a trailer
64 . Oil and natural gas are with a cylindrical tanker of 50 m3 volume, what
usually transferred long must its minimum diameter be? Calculate to the
distances through pipelines. nearest centimeter.
The pipeline in the figure 69 . Each cylinder of a powerful
has an internal diameter of 6-cylinder car engine is
32 cm. If the oil constantly moves through it at 14.3 cm long and has
0.6 km/h, find the amount of oil carried in one diameter 7.2 cm. Find the
day, to the nearest cubic meter. total engine capacity, to the
nearest tenth of a liter.

Solids of Revolution 131


70 . The hydraulic car lift 75 . For a pipe with external
in the figure has a radius R, internal radius r,
narrow and a wide and length h, the volume is
cylinder pump. It the difference of the volumes
operates when you of its external and internal
2 2
apply force to a liquid (oil) in the narrow cylinder. cylinders: V =  R h –  r h .
The pressure is transmitted to the wide cylinder, This formula can be rewritten as V = 2 rh t ,
which raises the car off the ground. In this where r is the average radius, and t is the
hydraulic car lift system, the diameter of the wide thickness of the pipe. Verify this theorem
piston is 5 times that of the narrow one. Find how
a. algebraically. b. geometrically.
far the car rises if the man pushes the first piston
0.5 m down. 76 . Find the volume of the cylindrical
metallic pipe segment shown at the
71 . By what percentage does the volume of a cylinder right. The segment has external and
change if its radius is increased by 20% but the internal diameters of 14 cm and
height is decreased by 25%? 10 cm and height 20 cm.

72 . What is the length of 4 mm 77 . The aluminum ring in the picture


copper wire in a coil that has a thickness of 1.2 cm, and its
weighs 5 kg? Find to the external and internal diameters are
nearest meter, given that the respectively 18 and 24 cm. Find
copper has density 8.96 g/cm3. the weight of the ring if the den-
sity of the metal is 2.7 g/cm3.
73 . One of the best ways to find the volume of irregular
78. Show that the volume of a
solids is by putting them in water. A stone put into 
pipe can be formulated as
water increases the water level by 2.3 cm in a 2
V =  c h , where c is the
cylinder with 6 cm internal diameter. What is the
half length of the segment
volume of the stone?
that is a chord of the external
circle and tangent to the
74. A cylindrical metal rod is placed
 internal one, and h is the length of the pipe.
in a cylindrical glass container
which is partially full of water, as in 79 . ABC in the figure is a
the figure. The diameter of the
quarter circle from which
rod and the container are 8 cm
a semicircle has been C
and 18 cm respectively. 12 cm of
removed. The remaining
the rod is submerged. Find the 7 cm
part forms the base of a
change in the water level when the rod is A 4 cm B
cylinder with length 7 cm.
removed.
Find the volume of this cylinder.

132 Solid Geometry


A. BASIC CONCEPTS
Definition Conical Surface
Consider a fixed plane curve and a point V which is not in the plane of the curve. As a
moving line g passing through V moves along the curve, it generates a surface which is called
a conical surface.

Conical
surface

Vertex

V Upper
V nappe
Directrix
C

C Lower
nappe

Generatrix
g
g Element

The moving line is called the generatrix of the conical surface.


Curve C serves to direct the path of the generatrix, hence it is called the directrix of the
surface. The moving line in any of its positions is called an element of the surface.
Since the generatrix is a line with indefinite length, pivoting around a point, it creates two
different conical surfaces. The pivot point is called the apex of the surfaces. Each of the
surfaces is called a nappe.

Definition Cone

A solid formed by a conical surface and a


plane which intersects all the elements of the
surface is called a cone.

Solids of Revolution 133


Elements of a cone

V Apex

Lateral surface

Altitude
Element

Base A salakot is a conical hat


made of straw, which is worn in
H
Far Eastern countries.

An element of a cone is a line segment on the conical surface which connects its apex with
a point on the circumference of the base. V

altitude
The altitude of the cone is the line segment that joins the
height
apex and the plane containing the base of the cone,
perpendicular to the base. The length of the altitude is
called the height of the cone. B

Notice that the altitude of a cone sometimes falls outside the cone, as shown in the figure.

Remark
A section of a cone made by a plane parallel to the base is a closed curve which is similar to
the base.

All similar cross sections All similar cross sections

134 Solid Geometry


B. TYPES OF CONE
We can classify cones according to their
bases. The most common types of cone are
circular and elliptical cones.
Cones with irregular bases are called
irregular cones.

Definition Elliptical Cone


An elliptical cone is a cone with an ellipse as its base.

Definition Circular Cone

A circular cone is a cone with a circular base.

The nets of some cones are shown below. Notice that the lateral surface of some cones may
have quite a surprising shape.
V

Elliptical oblique cone Circular right cone Circular oblique cone

Solids of Revolution 135


In this book we will study mostly circular right cones. Unless it is stated otherwise, we use
the term cone to mean a circular right cone.

Front view Top view Bottom view

Orthographic views of a right circular cone apex

In a circular cone, the line segment joining the center of the


base to the apex is called the axis of the cone.
axis
An axial section of a circular cone is a plane section which
altitude
contains the axis and an element. element

The radius of a circular cone is the radius of its base.


Note that any element of the cone shares a unique plane
with the axis.
A sea shell r
If the axis of a circular cone is perpendicular to the base, the C
with the shape cone is called a right cone, otherwise it is an oblique cone.
of a circular cone Base radius

Is it possible to talk about an elliptical right cone? Explain your answer.

Conclusion

In a circular right cone,


 the axis is perpendicular to the base.
 the axis is the altitude.
 all the elements are congruent. The length of an element is called the slant height of the
cone.

Conclusion

 A section of any circular cone made by a plane parallel to the base is a circle.
V
 The axis of a circular cone passes through the center of •
every section taken parallel to the base.
h2
 A section parallel to the base forms the base of another
A'
O'• •
cone which is similar to the original cone, and it divides h1

the altitude and elements proportionally:


VO VA h2 A • •
= = . O
VO VA h1

136 Solid Geometry


Can we make a circular right cone whose radius is equal to its slant height?
Is the slant height always greater than the height of a circular right cone? Share and explain
your ideas.
V

EXAMPLE 24 Find the slant height of the adjacent circular right cone with
base diameter AB = 12 cm and height h = 8 cm. h

Solution AB is the diameter of the circular base. A


O r B

So AB = 2r = 12 cm and the radius is r = 6 cm.


In right triangle VOB, VB is the hypotenuse and also the slant height of the cone.
By the Pythagorean Theorem, VB2 = OB2 + OV2. So l2 = 62 + 82 and l = 10 cm.

EXAMPLE 25 In a right circular cone, the height is 12 cm and the slant height is 13 cm.
Find the circumference of the base.

r O
Solution Look at the figure. h and r are the legs of a right triangle with hypotenuse l.
Of this triple, the missing length is r and we can find it using the
Pythagorean Theorem: r = 5 cm. h
l
The base is a circle and its circumference is
Pbase = 2r = 10 cm.

EXAMPLE 26 A right triangle with hypotenuse 17 cm is rotated around its 15 cm leg. Find the area of the
base.

Solution This problem is left as an exercise for you.

Remark Axial Section of Cone, Angle of a Cone


All the axial sections of a right circular cone are congruent. These
Angle of
sections are isosceles triangles. cone
Slant
The angle at the vertex of one of these triangles is called the angle of height
the cone.

Solids of Revolution 137


EXAMPLE 27 The area of the base of a right circular cone is 12 m2. Find the measurement of the angle of
the cone, if its height is 6 m.

V
Solution The base area is 12 m2. V

Since the base is a circle, the radius is 2ñ3 m.


30°
h
In right triangle VOA, VO = 6 m and OA = 2ñ3 m. 6m

2 3 3
So the tangent of angle OVA is tan OVA   ,
K A 6 3
r O A
which means that angle OVA measures 30°. 2ñ3 m

Finally, the angle of the cone is twice the measure of angle OVA. So the angle of the cone
is 60°.

For Experts

A circular right cone has angle 120° and slant height 14 cm. What is the area of the
shape created by the cone and a plane which passes through its vertex and one of
its base diameters?

Check Yourself 4
1. The slant height of a right circular cone is 8 m and the height is 5 m. Find its base area.
R
2. Compare the slant height and diameter of the cone obtained by
rotating the adjacent right triangle around side PQ.
30°
P Q
3. The angle of a cone is 30° and its slant height is 6 m.
What is the base area of this cone? V

4. In the adjacent right circular cone, the radius is 15 cm and the height is
20 cm. A plane passing through the apex cuts the base of the cone 12 cm
from the center of the base. What is the perimeter of the generated triangle?
A B

5. The base circumference of the adjacent oblique circular cone with apex V
V is 20 units. VQR is a vertical section passing through the axis of the
cone, VQ = 10ñ7 units and VR = 10 units. What is the height of this
cone?
Q R

Answers
1. 39 m2 2. They are the same. 3. (18 – 9ñ3) m2 4. 68 cm 5. 5ñ3 units

138 Solid Geometry


C. CONICAL FRUSTUMS
Definition Frustum of a Cone (Conical Frustum)

A frustum of a cone is the portion of a main cone which is contained between two planes
parallel to the base of the cone. The lower plane may or may not contain the base of the cone.
However, by convention the lower plane is usually the plane of the base of the cone.

Elements of a frustum
Axis
Base (upper)
An element of a frustum is a segment of a
corresponding element of the main cone between O'
Lateral A'
the two bases. surface
Altitude
An altitude of a frustum is a perpendicular line
segment between the bases.

O Element
H

Base (lower)

The length of any altitude is called the height of the frustum. If the frustum is from a
circular right cone, it is called a circular right frustum. It has two different circular bases, and
any element of the frustum ia as long as its slant height. The open net of a circular frustum
is shown below.
l
O2
r2
O2

h l

O1 r1
O1
h: height r1: lower radius
l: slant height r2: upper radius

Solids of Revolution 139


Note that any cross section of a frustum which is perpendicular to the
bases and cuts both bases has the shape of an isosceles trapezoid.

EXAMPLE 28 Find the slant height of the frustum of a right cone if the base diameters
are 28 mm and 20 mm and the height is 12 mm.

Solution Look at the figure. Quadrilateral PQRS is the axial section of the frustum. It is an isosceles
trapezoid whose base lengths are the diameters of the frustum. The radii are respectively
14 mm and 10 mm.
O2 r2 R O2 r 2 Q
R Q

l
h l h h l

S r1 P
O1 P
S O1 T
r1

In right triangle QTP, by the Pythagorean Theorem,


QP 2 = QT 2 + TP 2  l 2 = h2 + (r1 – r2)2  l 2 = 122 + (14 – 10)2 = 160.
So the slant height is l = 4ò10 mm.

Check Yourself 5
1. A cone-shaped bar of soap has diameter 12 cm and height 9 cm. The top portion is
removed by cutting parallel to the base so that the remaining part forms a frustum. If the
frustum height is 6 cm, what is the radius of its upper base?
2. The adjacent plastic cup has top diameter and slant height both equal to
8 cm. The base diameter is 4 cm. How tall is the cup?

3. r1
The radii of the frustum of a cone are r1 and r2. If the
slant height is r1 + r2, what is the height of the frustum?
l

r2

Answers
1. 4 cm 2. 2ò15 cm 3. 2 r1 r2

140 Solid Geometry


D. SURFACE AREA OF A CONE
A cone has two surfaces: its lateral surface and the base. The surface area of a cone is the
sum of the areas of these surfaces.
Acone = Alateral + Abase

A circular cone

In general, the areas of oblique cones and non-circular cones are not easy to calculate. In
this section we will study the area of only circular right cones.
In order to understand how to derive the surface area formula for a cone, we first need to
understand the relationship between the base perimeter and the slant height of the cone.

Remark Circumference of the Base of a Cone

The circumference of the base of a cone corresponds


to the arc of the lateral surface of the cone.

C = PïQ
l

Remark Lateral Surface of a Circular Cone


The lateral surface of a circular right cone
V
is a sector of a greater circle.

l
l V

r
r

Solids of Revolution 141


Theorem Lateral Area of a Circular Cone

The lateral area of a right circular cone is l


equal to half the product of the slant
height and the perimeter of the base.
1
l AL = — . P . l
1 2
AL =  P  l
2
This formula can also be written as r
O
AL  r l, P O r
where r is the radius of the base.
P

Proof To prove this theorem, we can use the relationship between pyramids and cones.
Let the lateral area of a cone be AL, let its circumference be P and let the slant height be l.
Consider a regular pyramid inscribed in the cone.
Let its base perimeter be P and its lateral area be AL.
Note that the pyramid has the same slant height as the cone.
Remember that by the formula for the lateral area of a pyramid,
1
AL =  P  l.
2
We can see that the base of the pyramid has a shorter perimeter than the cone’s base, and so
the lateral area of the pyramid is less than the lateral area of the cone. However as we
increase the number of sides, the pyramid gets closer and closer to a cone, but the
lateral area formula for the pyramid will remain unchanged.

n=5 n = 12 n = 20 n =¥

As the number of sides in the base approaches infinity, the base of the pyramid will meet the
base of the cone (P'  P), and the areas of the lateral surfaces will coincide too (AL'  AL).
This means that our pyramid will turn out to be the cone which circumscribes it. Therefore
we can use the same formula for the area of the lateral surface of the cone:
1
AL  AL  AL =  P  l.
2

142 Solid Geometry


Alternative Another way to prove this theorem is by relating the lateral surface of a cone to its base.
Proof Look at the following cone with its open net:
B

l
l

V
V r
O
P

Arc AïB has the same length as the circumference of the base: |AïB| = P.
Remember that the lateral surface is a sector of a circle.
If we complete the lateral surface to its full circle, it will look like this:

B
l
Here, point V is the center O r
V
and l is the radius of the larger circle.
P

Now, by considering the big circle, we can say that the area of the sector is proportional to
the length of arc AïB:
|
| AB Area sector Psmall circle AL
=  = .
Perimeterbig circle Area big circle Pbig circle Abig circle

P A 1
So = L2 , and finally after simplification we get AL =  P  l , as required.
2l l 2

EXAMPLE 29 Find the area of the lateral surface of a circular right cone if the perimeter of the base is
15 cm and the slant height is 8 cm.
1 1
Solution By applying the formula AL   P  l , we get AL   15  8  60 cm 2 .
2 2

EXAMPLE 30 Find the area of the lateral surface of a circular right cone if the radius is 6 m and the slant
height is 11 m.

Solution By the formula AL = rl, the lateral area will be AL =   6  11 = 66m2 .

Solids of Revolution 143


Conclusion
l
In a right circular cone with radius r and slant height l,
 the lateral area is AL = rl.
Explanation: The circumference length of a circle is P = 2r.
r
Substituting this into
1
AL   P  l gives us AL = rl .
2

 the total surface area is Acone = r(r + l). l

Explanation: The total area of the cone has two parts:


Acone = Abase + Alateral .
r
The base is a circle with area r2, so
Acone = r2 + rl which gives us Acone = r(r + l).

EXAMPLE 31 How much tin must we use in total to make the adjacent right circular
cone and its lid, if the diameter must be 10 cm and the slant height
must be 8 cm?
Find the answer to the nearest square centimeter.

Solution Here the lid is the base of the cone and its diameter is 10 cm. So the radius is 5 cm.
To find the total area of tin required, let us use the formula.
Acone = r(r + l)  Acone =   5  (5 + 8), so Acone = 65 cm2 .
Substituting   3.14 gives us Acone = 204.1 cm2. Therefore we need approximately 204 cm2
of tin.

EXAMPLE 32 What is the external surface area of an ice-cream cone if it is 12 cm


long and the diameter at its top is 6 cm?

Solution The top of an ice-cream cone is open. Therefore we only need to find the lateral area.
r O The radius is half the diameter and its measure is 3 cm.
Let us find the slant height in the right triangle formed by h, l, r.
l2 = h2 + r2 = 122 + 32, so l = 3ò17 cm.
h
l Finally, by the formula AL = rl, the surface area of the cone is
Acone =   3  3ò17 = 9ò17  116.6 cm2 .

144 Solid Geometry


EXAMPLE 33 What is the base area of a right circular cone with lateral surface 72 m2 and angle 60°?

Solution Look at the figure. If the angle of the cone is 60°, we have
m(OVK) = 30°. By trigonometry,
O V
KO r 1 h 30°
we get the ratio sin 30   i.e. sin 30    and so l = 2r.
KV l 2 r
l
AL = rl  72 =   r  2r, so r = 6 m.
K
The base area is now AB = r2 =   62 = 36 m2.

EXAMPLE 34 A cone is obtained by rotating this right triangle around side KM. K

a. Find the lateral area of the resulting cone. 5 cm


b. Find its total surface area.
L M
12 cm
Solution The resulting cone is shown in the figure.
K To solve part a, we need to find the slant height KL.
2 2
l
h By the Pythagorean Theorem, KL = l = r + h and l =13 cm.
a. AL = rl  AL =   12  13 and AL = 156 cm2.
L r M
b. Finding the total surface area is left as an exercise for you.

EXAMPLE 35 A 120° sector taken from a circle with radius 18 cm is rolled up to make the lateral surface
of a cone. What is the base area of the cone?

Solution
l = radius = 18 cm

l
120°

l
To avoid confusion, let l be the radius of the sector.

So the length of the arc of this sector will be 2l .
360  l

Remember that when the sector is rolled, this arc will create the
circumference of the circular base of the cone:
r
 120 0
Larc  Pbase  2 l  2 r and 2  18   2  r .
360 0 360 0
Therefore the base radius is r = 6 cm and the base area is ABase = r2 = 36 cm2.

Solids of Revolution 145


EXAMPLE 36 The slant height of a cone is 3 cm. Sketch a graph that shows the total surface area of the cone
as a function of its radius.

Solution The surface area of a right circular cone is ACone = r(r + l). Using l = 3 we can define the
function A(r) as follows: A(r)
ACone = A(r) = r  (r + 3) = r2 + 3r.
This is a quadratic function (parabola) of the radius r. 18p

The roots of the equation are r1 = 0 and r2 = –3.


Although these values are not valid radius lengths, they will
help us to sketch the graph. In fact, the domain of r will be
between 0 and 3 (can you see why?). r
Depending on the radius, the area will range between 0 and –3 3

18 cm2: 0 < r < 3  0 < A(r) < 18 cm2 .

Check Yourself 6
1. The diameter of the straw salakot hat in the picture is 36 cm and its slant
height is 24 cm. Find its approximate external surface area, to the nearest
square centimeter.
2. Find the total area of circular right cone if its base perimeter is 12 cm and the slant
height is 10 cm.

3. If the slant height of a cone is 5 cm, its total surface area is always smaller than a
number a. What is the minimum value of a?
4. The lateral area of a cone is 60% of its total surface area. Find the ratio of the radius and
the slant height.

5. The adjacent figure shows the net of a cone. What is the radius of r 72°
the base? 5 cm

A
6. The sector given in the figure is rolled up to make a right cone.
If the altitude of the cone is 16ñ5 cm, find the lateral area of the
O 40°
cone.
B

7. Ismail wants to make a conical tent with base diameter 3 m and height 2 m. What is the
total area of canvas required, to the nearest square meter, if the base will also be covered
in canvas?

Answers
2
1. 1357 cm2 2. 96 cm2 3. 50 cm2 4. 5. 4 cm 6. 144 cm2 7. 6  19 m2
3
146 Solid Geometry
E. SURFACE AREA OF A CONICAL FRUSTUM
Theorem Lateral Area of a Frustum
The lateral area of a right circular frustum is half the product of the slant height and the sum
of the perimeters of the bases.
1 P2
P2
AL   l  ( P1  P2 ) l
2
O2

l
1 . (P + P ) . l
AL = — 1 2
2

O1

P1

P1

Proof Look at the figure below. If we extend the elements of the frustum, they meet at the top,
forming two similar cones, one with radius r1 and the other one with radius r2. The
difference of their lateral surfaces gives us the lateral surface of the frustum.
V
AL  AL  AL
frustum bigger cone smaller cone

1 1
 l1P1  l2 P2
2 2 l2
1 1 l1
 ( l  l2 )P1  l2 P2
2 2
r1
O2 B
1 1 1
 lP1  l2 P1  l2 P2 P2
2 2 2 l

1 1 r2
 lP1  l2 ( P1  P2 ) O1 A
2 2
Note that l1 = l + l2 . P1

l1 r1 l P
Also, triangles VO1A and VO2B are similar:  , which gives us 1  1 .
l2 r2 l2 P2
l  l2 P1 P2l
Since l1 = l + l2 , we have  . Solving this equation for l2, we get l2  .
l2 P2 P1  P2
Substituting this expression in the formula above, we get
1 1 P2 l 1
AL  lP1  ( P1  P2 ) and so AL  l( P1  P2 ) . This completes the proof.
frustum 2 2 ( P1  P2 ) frustum 2

Solids of Revolution 147


EXAMPLE 37 Find the area of the lateral surface of the frustum of a right cone if the lower base perimeter
is 4 m, the upper base perimeter is 3 m and the slant height is 6 m.

1
Solution Using the formula AL  l( P1  P2 ) , the lateral area of the frustum will be
2
1
AL  6(4  3)  21 m 2 .
2

Conclusion

In a frustum of a right circular cone with radii r1 and r2, and slant height l,
 the lateral area is AL = l(r1 + r2).
Explanation:
The perimeter of a circular base is P = 2r.
1 1
Substituting this into AL  l( P1  P2 )  l(2 r1  2 r2 ) ,
2 2
we get AL = l(r1 + r2).

 the total surface area is Afrustum   l( r1  r2 )  ( r1  r2 ) .


2 2

Explanation:
The total area of the frustum has two parts:
Afrustum = Alateral + Abases .
The bases are circles, so AFrustum = l(r1 + r2) + (r1 2 + r22 ) which gives us
Afrustum = [l(r1 + r2) + (r12 + r12 )].

EXAMPLE 38 What is the lateral area of the frustum of a right cone if the radius of the upper base is 1 cm,
the radius of the lower base is 3 cm and the slant height is 5 cm?

Solution By the formula AL = l(r1 + r2), the lateral area of the frustum is
A =   5  (3 + 1) = 20 cm2.

EXAMPLE 39 Find the surface area of the frustum of a right cone if the radii of the bases are 3 cm and
6 cm and the slant height is 7 cm.

Solution Using the formula Afrustum = [l(r1 + r2) + (r12 + r12 )], the total surface area of the frustum
will be A = [7(6 + 3) + (62 + 32)] = 108 cm2.

148 Solid Geometry


EXAMPLE 40 Find the surface area of a conical frustum-shaped plastic cup, if the diameter
of the base is 5 cm, the top diameter is 7 cm and the slant height is 8 cm.

Solution The surface area is AL + AB, which is l(r1 + r2) + r12 . So


2
5 7 5  217 
A  8        170.4 cm 2 .
2 2 2  4

EXAMPLE 41 The height of a right circular frustum is 8 m and its base diameters are 6 m and 18 m.
Find the area of this frustum.
O2 r2
Solution As in Example 28, we can use the isosceles trapezoid section of D C

the frustum to solve the problem.


In right triangle BCK, BC 2 = CK 2 + KB 2 and h l
2 2 2 2 2
l = h + (r1 – r2) = 8 + (9 – 3) = 100, i.e. l = 10 m.
Now we know the slant height and we can use it in the formula: K
A B
O1 r1
AFrustum = [l(r1 + r2) + (r + r )] = [10(3 + 9) + (3 + 9 )] ,
2
1
2
1
2 2

which gives us Afrustum = 210 cm2. r1 = 9 m and r2 = 3 m

Check Yourself 7
1. The lampshade in the picture is a frustum of a circular cone with base radii
7 cm and 18 cm. What is the area of fabric covering the lateral surface if the
slant height is 40 cm?

2. Find the area of the lateral surface of the frustum of a right cone if the lower base
perimeter is 7 dm, the upper base perimeter is 2 dm and the slant height is 8 dm.
3. Megaphones were used in the past as a practical way to make a
voice louder. A megaphone has the shape of a right circular
frustum which is open at both ends.
The megaphone in the photograph opposite has base diameters
2 dm and 12 dm, and length 12 dm. What is the lateral surface
area of this megaphone, in square decimeters?

4. The radii of the bases of the frustum of a right cone are 3 m and 8 m and the slant height
is 5 m. Find the total surface area.
Answers
1. 1000 cm2 2. 36 dm2 3. 91  286 dm2 4. 128 m2

Solids of Revolution 149


F. VOLUME OF A CONE
The relationship between the volume of a cone and the volume of its corresponding cylinder
is the same as the relationship between a pyramid and its corresponding prism.

Theorem Volume of a Cone


The volume of a cone is equal to one-third the
V V
product of the area of its base and its height.
1
Vcone  Abase  h
3
In other words, the volume of a cone is one- h h

third the volume of a cylinder with the same


base and height.
Base Base
1
Vcone   Vcylinder
3
Proof To prove this theorem, we will again use the relationship between pyramids and cones.
Let the volume of the cone be V, the base area be S
and the height be h.
Consider a pyramid inscribed in the cone. Its base is
l l
a polygon with a countable number of sides, its
h
volume is V and its base area is A . Note that the
pyramid has the same height as the cone.
1
The volume of the pyramid is V'   A'  h by
3
the formula for the volume of a pyramid.
As we can see, the base area and volume of the pyramid are less than the base area and
volume of the cone. However, as we increase the number of sides indefinitely, the pyramid
will get closer and closer to a cone.

n=5 n = 12 n = 20 n =¥
Obviously the volume formula for the pyramid will remain the same.
Finally, as the number of sides in the base approaches infinity, the base of the pyramid will
meet the base of the cone: A  A . Similarly, for the volumes we have V  V.

150 Solid Geometry


Now our initial pyramid has become the cone which circumscribes it. Therefore we can apply
the same volume formula for the cone:
1
V = V  Vcone   Abase  h .
3

EXAMPLE 42 The adjacent cone has an elliptical base with area 27 cm2.
Find the volume of the cone if its height is 10 cm.
h
Solution The volume of a cone is independent of the shape of its base. We
only need to know the area of the base.
1 1
Vcone   Abase  h gives us Vcone   27 10  90 cm 3 .
3 3

Conclusion

 The volume of any circular cone is one-


third the volume of the cylinder with the
same base and height.
1
Vcone  Vcylinder
3

In other words, we can say that any =3x


cylinder is three times the volume of its
cone, or conversely that any cone fills only
one-third of its corresponding cylinder.
 For a circular cone with radius r and height h, the volume
h
1 2
is V  r h.
3
r r
Explanation: Substituting the base area formula Abase = r2
into the formula for the volume of a circular cone,
1 1
we get V  Ab h  r 2 h .
3 3

EXAMPLE 43 Find the volume of a circular cone with radius 6 cm and height 7 cm.

1 2 1
Solution Applying the volume formula V  r h , we get the volume as   62  7  84  cm 3.
3 3

Solids of Revolution 151


EXAMPLE 44 An ice cream cone has diameter 6 cm and slant height 10 cm. Find its volume, to the nearest
cubic centimeter.

Solution The radius of the cone is 3 cm.


As shown in the figure, because the radius, height and
the slant height of the cone form a right triangle the
r
cone is circular and right.

So h  l 2  r 2  10 2  9  91 cm. h l

Therefore the volume of the ice cream cone will be


1 2 1
V r h    32  91  3 91  90 cm 3 .
3 3

EXAMPLE 45 Find the height of a cone with 24 m3 volume and 4 m radius.

1 2
Solution In this question, the height h is the unknown parameter in the equation V  r h .
3
1
We can write 24    42 h , which gives us h = 4.5 m.
3

EXAMPLE 46 The adjacent figures are a cone and a cylinder with congruent
radii. The height of the cone is twice the height of the cylinder.
Compare their volumes.

Solution Which solid do you think has a bigger volume: the cylinder or the cone? Make a guess, then
check:
Vcylinder r 2 h 3
  .
Vcone 1 2 2
r (2h)
3
Was your guess correct?

EXAMPLE 47 Yunus has a cone made of cork. The height of the cone is three times its
radius. If the volume of this cone is 64 cm3, find its surface area.

152 Solid Geometry


Solution The height h is given as 3r, so
1 2 1
V r h  r 2 (3r )  r 3  64  .
3 3
l
Hence the radius r = 4 cm and h = 12 cm. 3r

The slant height can be found by applying the Pythagorean Theorem


in the right triangle formed by h, l, r:
r
2 2 2 2 2
l = h + r = 12 + 4 , so l = 4ò10 cm.
Now, by the surface area formula Acone = r(r + l) for a cone,
Acone = 4(4 + 4ò10) = 16 + 16ò10 cm2.

EXAMPLE 48 A 3-4-5 cm triangle is rotated around its longer leg to generate a cone. Find the volume of
the cone.

Solution This problem is left as an exercise for you.

EXAMPLE 49 A 72° sector is removed from a circular right cone with slant height 3ó101 cm and height
30 cm. Find the volume of this piece of the cone.

r
Solution Given h = 30 cm and l = 3ó101 cm, l
h
applying the Pythagorean Theorem in the
right triangle gives the radius of the base as
r = 3 cm.
When a piece of 72° is removed, its volume
will be proportional to its base area:
Asector Vpiece
 . (Can you see why?)
Abase Vcone

The base of the piece is a sector of the circular base and its area is Asector  Abase  .
360 

Abase 
After substituting the expressions, the proportion becomes 360  Vpiece .
Abase 1 2
r h
3

1 2  1 72 
Finally, we get Vpiece  r h    32  30   18  cm 3 .
3 360  3 360 

Solids of Revolution 153


EXAMPLE 50 A circular oblique cone has base perimeter 10 cm and axis length 11 cm. If the axis of the
cone is inclined at 60° to the base, what is the volume of the cone?
V
Solution As we have already seen, the slant height of a cone is not
the same as its height.
Look at the figure. In triangle VCK, h = a  sin 60° and the
base area becomes 25 cm2 since the base perimeter is
a h
10 cm. So the volume of the cone is
1
Vcone   Abase  h
3
60°
1 1 11 3 275 3 K
  Abase  a  sin60    25    cm 3. C
3 3 2 6

EXAMPLE 51 A cone with diameter 12 cm and height 16 cm is cut parallel to its base at the midpoint of
its altitude. What is the volume of the new small cone that is created?

Solution Look at the figure. The cut generates two similar right triangles, since it is parallel to the base.
VO1 VO2
So VO1A  VO2B and  . V
AO1 BO2
V
h1 h2
This gives us  , and note that h2 = 8 cm. h2
r1 r2
r2 B
16 8 O2
  r2  3 cm.
6 r2 h1

So the volume of the small cone is


1 2 1
Vcone  r2 h2    32  8  24  cm 3. r1
3 3 A
O1
As an exercise, find the volume of the frustum
created by the cut. Note that r1 = 6 cm

For Experts

A funnel in the shape of a circular cone with radius 5 cm and height 12 cm


is filled with water. There is a hole at the vertex of the cone and water
begins to drip from it at a constant rate. At the end of the first minute the
water level is already 1 cm below the top of the funnel. How long will it
take for all the water to drain from the funnel?

154 Solid Geometry


Check Yourself 8
1. The adjacent conical goblet has 6 cm diameter and 14 cm depth. How much
water can it hold, in cubic centimeters?

2. Find the volume of a cone that has a 25 m2 base area and 13 m slant height.
3. The radius of a cone is doubled and its height is decreased by half. By what proportion
does the volume change?
4. A solid metal cylinder with radius 6 cm and height 18 cm is melted down and recast as a
solid cone with radius 9 cm. Find the height of the new cone.
5. The angle of a cone is 60° and its slant height is 6 cm. What is the volume of this cone?
C
6. A circular sector with radius 9ñ2 m and angle 120° is rolled to make 120°
a right cone. What is the volume of the cone?

7. A circular oblique cone has diameter 10 m. Its 6 m axis makes an angle of 30° with the
base. What is the volume of the cone?
8. A cylindrical pencil with radius 1 cm is sharpened as shown in the figure. The length of
the pencil does not change but the shaved part is 2 cm long. Find the volume of wood
removed from the pencil.

Answers
1. 42 cm3 2. 100 cm3 3. It doubles. 4. 8 cm
4
5. 9ñ3 cm3 6. 24 cm3 7. 25 m3 8. cm 3
3

Some Other Cones

We can generate some other types of cone by rotating some

familiar shapes. If we rotate a whole parabola around its axis,

we get a paraboloid. If we rotate a segment from a parabola we

get a parabolic cone. A parabolic cone has a sharp nose and is

one of the best aerodynamic figures. A paraboloid and a

parabola both have circular bases but slightly different volume Paraboloid Parabolic Cone
formulas. r 2 h 8r 2 h
V= V=
2 15

Solids of Revolution 155


G. VOLUME OF A CONICAL FRUSTUM
Example 51, at the end of the previous section, gives us a clue for a
method to find the volume of a conical frustum. We know that a conical
frustum is a portion of a cone that remains after we remove a smaller
cone from its top. This information is helpful when we want to prove the
following theorem.

Theorem Volume of a Frustum of a Cone

r2
The volume of a frustum of a right cone where r1 and r2 are
the radii of the frustum and h is its height, is
1
Vfrustum  h( r12  r22  r1 r2 ). h
3
r1

Proof In the adjacent figure, the upper conical portion is removed V


from the circular right cone to create a frustum. The
volume of the remaining frustum is the difference of the
volumes of these cones:
Afrustum = Vcone1 – Vcone2 . h2
Obviously these cones are similar, and this means that h1
triangles VO1A and VO2B are also similar.
r2
O2 B
Here r1, r2 and h1, h2 are the radii and the heights of the
larger and smaller cones respectively. h

Note that the height of the larger cone is h1 = h + h2. r1


A
Since the triangles VO1A and VO2B are similar, we can write O1

h1 r1 h  h2 r1
 , which gives us  .
h2 r2 h2 r2
hr2
Solving this equation for h2, we get h2  .
r1  r2

Remember the volume of the frustum is Afrustum = Vcone1 – Vcone2 , and so


1 2 1
Vfrustum   r1 h1   r22 h2
3 3
1 1 1
   r12 ( h  h2 )  r22 h2    r12 h  r12 h2  r22 h2    r12 h  (r12  r22 ) h2  .
3 3 3

156 Solid Geometry


hr2  hr2 
Substituting h2  , we get Vfrustum  1   r12 h  ( r12  r22 )  . So
r1  r2 3  r1  r2 

1  2 hr2  1
Vfrustum    r1 h  ( r1  r2 )( r1  r2 )  and Vfrustum    r12 h  ( r1  r2 )hr2 .
3  r1  r2  3

1
Finally, the volume of the frustum is found as V  h( r12  r22  r1r2 ) .
3

EXAMPLE 52 The base radii of a frustum-shaped dessert are 5 cm and


3 cm. If the height of this dessert is 6 cm, find its volume.
1
Solution Applying the formula Vfrustum  h( r12  r22  r1 r2 ) ,
3

1
we have Vdessert    6  (52  32  5  3) , and so the volume of the dessert is 98 cm3.
3

EXAMPLE 53 Find how many liters of water will fill a frustum-shaped bucket if the
bucket is 20 cm wide at the bottom, 30 cm wide at the top and 40 cm
high. Round your answer to the nearest integer.

Solution 1
By the formula we have just seen, the volume of the bucket is Vbucket  h( r12  r22  r1 r2 ) .
3
The given widths are the diameters of the bases, so the radii are r1 = 10 cm and r2 = 15 cm.
1 19000
So the volume of the bucket is Vbucket    40(15 2  10 2  15 10)   cm 3.
3 3
Therefore the volume of the bucket is nearly 20 L.

Solids of Revolution 157


EXAMPLE 54 The slant height of a large right circular frustum-shaped marquee is 10 m and its base
diameters are 6 m and 22 m. Find the volume of this marquee.
r2 O2
C
Solution We can solve this problem by considering a section of the
circular right frustum.
l h
In right triangle ABC, AB = r1 – r2 .
O1
Applying the Pythagorean Theorem gives us A
B r1
l2 = h2 + (r1 – r2)2 = 102 – (11 – 3)2 i.e. h = 6 m. r1

So the volume of the frustum is r1=11 m and r2=3 m

1 1
Vfrustum  h( r12  r22  r1 r2 )    6  (112  32  11  3)  326   1024 m 3.
3 3

EXAMPLE 55 In a circular right frustum, the sum of the diameter lengths is 16 cm and the product of the
diameter lengths is 40 cm. If the height of the frustum is 7 cm, find its volume.

Solution This question requires some skill in manipulating polynomial identities. Let the diameters be
d1 and d2. So d1 + d2 = 16 and d1 . d2 = 40.
This means r + r = 8 and r . r = 10.
1 2 1 2

By using a perfect square, we have (r1 + r2)2 = r12 + r22 + 2r1r2 .


In the volume formula we need r12 + r22 + r1r2 , which can be written as
(r1 + r2)2 – r1r2 .
Substituting the expression in the formula, we get

1 1 1
VFrustum  h( r12  r22  r1 r2 )  h ( r1  r2 )2  r1 r2     7 8 2 10   126  cm 3 .
3 3 3

7 cm
EXAMPLE 56 The right trapezoid in the figure is rotated around its side AB. A

Find the volume of the generated solid. 4 cm

Solution This problem is left as an exercise for you. B


10 cm

EXAMPLE 57 Verify that the volume formula V 


1
3
h( A1  A2  A1  A1 ) for the frustum of a pyramid is also

valid for the frustum of a cone.

Solution The proof is left as an exercise for you.

158 Solid Geometry


Check Yourself 9
1. This small bucket is 10 cm wide at the bottom and 16 cm wide at the top.
The depth of the bucket is 12 cm. How much water can it hold?

2. Find the volume of a circular frustum if its base diameters are 2 m and 4 m
and its height is 3 m.

3. A circular frustum has 4 cm and 10 cm base perimeters. The slant height makes a 45°
angle with the base. What is the volume of this frustum?

4. Lampshades are often in the shape of a frustum. The lateral area of the
frustum of this lampshade is 400 cm2, the slant height is 25 cm, and
the upper radius is 10 cm shorter than the radius of the lower base.
What is the volume contained by the lampshade?

5. For a circular right frustum, the sum of the base areas is 42 cm2. If the product of the
radii is 6 cm and the height is 11 cm, find the volume of the frustum.

Answers
1. 516 cm3  1.62 L 2. 7  22 m3 3. 39 cm3 4. 3250 3 cm 3 5. 176 cm3
3

The Volume of a Log


Nearly 30% of the Earth’s surface is covered with forests, and one of the benefits we get from
forests is wood. Each type of tree has a different type of wood, but the trees usually have a similar
stem shape. Calculating the volume of a log is important for people who deal with wood, because
logs are traded by volume more than by weight.
The body of a tree is not a perfect Euclidean figure, however it can be approximated to a frustum. An engineer has to
select the Euclidean shape that most closely approximates the natural shape of the log to work out the volume using the
appropriate formula.
Several formulas have been developed for the calculation of the volume of a log. The formulas are usually derived from
the formulas for the figures mentioned above. Today there are three most common formulas:
Assume that A and a are the respective base areas for the base diameters D and d, õA is the mid-sectional area for the
mid-sectional diameter õD, and h is the height or length of the log. Then the volume V can be given by

( A  a)
I. V  h  (Smalian's formula)
2
A
2
h  D A
II. V  h  A  (Huber's formula) a
4
h
h 2 2
III. V   ( D  4 D  d ) (Newton's formula).
24

All these formulas have slight differences, but sometimes they give the same volume for certain log shapes.

Solids of Revolution 159


H. CONIC SECTIONS
Definition Tangent Plane of a Cone
A plane that contains only one element of a cone is called a a V

tangent plane of the cone.


Obviously, through a given element, only one tangent plane
can be drawn. The intersection of this plane and the plane b

containing the base of the cone forms a line. This line is


A
tangent to the base of the cone.
Definition Conic Section
The intersection of the conical surface of a circular right cone and a plane is called a conic
section.
The plane generates different types of conic section depending on its position relative to the cone:

Circle: If the plane is parallel to the base and


Circle
intersects all elements of the cone, the section is
a circle.

Ellipse: If the plane is oblique to the base and


intersects all elements of the cone, the section is
Ellipse
an ellipse.

Parabola: If the plane is parallel to one and only one


element of the cone, the section is a parabola.
Element

Parabola

Axis

Hyperbola: If the plane is parallel to the axis of the cone,


it intersects both nappes of the conical surface.
The conic section is a composition of two different curves
which is called a hyperbola. Hyperbola

160 Solid Geometry


Hyperbola

Circle

Circle

Ellipse
Parabola Ellipse

Parabola

Hyperbola

It is incredible that these unique curves can all be generated using a cone. We can find
examples of conic sections in nature and in some other applications that are the product of
highly skilled engineering.

Mostar Bridge Parabolic path

Parabolic Antenna Hyperbolic roof

Solids of Revolution 161


Planet

Sun
Focus Focus

Elliptical orbit of a planet Parabolic arcs in Istanbul’s Hyperbolic projection of


spice market light on a wall

Conic sections are used in many different academic fields, including architecture, math and
physics. Historically, the first definition of a conic section came from the Greek geometer
Menaechmus, who died in 320 BC. His studies were continued by Apollonius of Perga, who
died in 190 BC. Conic sections also helped famous physicists such as Kepler and Newton to
create their theories of the planets and space.

You can learn more about conic sections in a different book of this high school math series.

Check Yourself 10
B
1. In the circular right cone at the right, AE is perpendicular to AC
and AC is perpendicular to the base. B and E are on the same
element and BE // AD. A plane cuts the cone and creates a shape.
What is the shape of the intersection if the cut lies A E
a. along AB? b. along AC? c. along AD? d. along AE?

C D

2. A pencil is first sharpened and then always held in the same


position while writing. Which shape will the surface of the used tip
form if
a. the pencil is held at a constant angle?
b. it is held vertically?

Answers
1. a. an ellipse b. a hyperbola c. a parabola d. a circle
2. a. an ellipse b. a circle

162 Solid Geometry


EXERCISES 2 .2
B. Types of Cone 8. Find the base area of A

1. How many surfaces and edges does a cone have? the cone generated
15 m
by rotating the 5 m
adjacent right
2. What is the minimum slant height for a cone, in triangle around AB. B C
terms of its radius?

3. In this circular oblique


9. Find the base perimeter of the cone generated by
cone, two slant heights are 8 cm
6 cm rotating an equilateral triangle with 14 cm side
given. If the radius is 5 cm,
length around its altitude.
find the axis length.

10 . VA and VB are two elements of V


4. Find the height of the
12 cm a cone as shown in the figure.
adjacent oblique cone.
The length of an element is
30° 15 cm, and the radius of the
cone is 9 cm. If the segment
AB is 5 cm from the center,
find the area of triangle VAB. A B
V
5. In this right circular cone the
diameter is 12 cm and the
height is 10 cm. Find the 10
slant height. 11 . Find the angle of a cone if its base area is 18 cm2
A B and the slant height is 6 cm.
O

6. Find the base perimeter of a right circular cone


12 . The angle of a cone is 30°, and a slant height is
with height 12 cm and slant height 15 cm.
8 cm. Find the area of an axial section of this cone.

7. This conical construction is made from


13 . A room in the shape of a cone has base area
12 m long metallic bars on a circular
36 feet2 and altitude 20 feet. We want to make a
base with diameter 4 m. Find its
second floor with base area 9 feet2 which is m feet
height.
above the ground. Find m.

Solids of Revolution 163


C. Conical Frustums 21 . Find the lateral surface area of a cone if the
14 . How many surfaces does a frustum of a cone have? radius is 3 cm and the height is 4 cm.

15 . The radii of the bases of a frustum of cone are 22 . This conical teepee is sitting
15 cm and 9 cm. If the angle between an element on a circular plot of 2.25 m2.
and the base of the cone is 45°, find the height of What is the area of canvas
the cone. used for the teepee, not
including the base, if the
16 . Find the slant height of 5 cm height of the teepee is 2 m?
the frustum in the figure. l
8cm

11 cm 23. What is the lateral surface area of a cone if its



base perimeter is 20 m and a slant height is 9 m?

17 . This trapezoid is rotated B 3cm


around side AB. What is 24 . The Melbourne Central Cone is a
2cm
the sum of the edge lengths glass roof in the shape of a cone.
of the frustum? A 5cm Find the area of glass used in the
cone if we suppose that the diameter
and height of the cone are both 50 m.
18. A right circular frustum with 15 cm

and 20 cm base radii is cut by a
plane perpendicular to the bases.
The plane is 12 cm from the axis, 25 . Mount Fuji (Fujiyama) is a
which is 25 cm long. Find the area famous volcano in Japan with
of the trapezoid of intersection. a shape very close to a circular
cone. Its slant height makes a
30° angle with the ground and
its radius is 6 km. Find the lateral area of the
D. Surface Area of a Cone mountain.
19 . Find the lateral surface area of a cone if the
radius is 2 cm and the slant height is 5 cm.
26 . The slant height of a cone is 7 cm. What is an
upper limit for the total surface area of the cone?
20 . The radius of this party hat is 7 cm
and its slant height is 20 cm. What is
27 . The lateral area of a cone is equal to its base area.
its lateral surface area?
What can you say about the relation between its
slant height and its radius?

164 Solid Geometry


28 . What is the total surface area of a cone if the 36 . The lateral area of a cone is three times its base
radius is 3 cm and the slant height is 7 cm? area. If the slant height is 12 cm, what is the
base area?

29 . Find the total surface area of a cone if the slant


37 . A student used the
height is 10 m and the diameter is 8 m.
adjacent net to make a
90° r
cone. What is the slant 6 cm
height of the cone?
30 . Find the total surface area of a cone if its radius
is 6 cm and its height is 8 cm.

38 . Construct a cone with 3 cm radius and 4 cm


31 . Find the total surface area of this height using a compass, ruler and paper.
wooden cone, to the nearest square
centimeter, if the base perimeter is 8
cm and the slant height is 14 cm. E. Surface Area of a Conical Frustum
39 . Find the lateral area of this
frustum with 2 cm upper and
32 . A cone is generated by rotating a right triangle 5 cm lower base radii, and slant
with 17 cm hypotenuse around its 15 cm leg. height 8 cm.
What is its total surface area?

33 . A 72° sector of a circle with 15 cm radius is rolled 40 . Find the surface area of the
up to make the lateral surface of a cone. What is frustum-shaped segment of
the base area of this cone? this pipe. The wide opening
has 30 cm radius and the
narrower opening has 20 cm
34. The circular sector shown in the radius. The slant height is 20 cm.

figure is rolled up to form a right
cone. If the altitude of the cone is
21 m, find the total surface area of 41 . Conical pet collars like the one
the cone. in the figure are often given to
animals after an operation. This
one has a 10 cm diameter at the
35 . The net of the lateral surface of a cone is a neck and 20 cm diameter at the face. Its slant
semicircle. What is the relation between the height is 14 cm. Find its lateral area, to the
radius r of the cone and its slant height l? nearest square centimeter.

Solids of Revolution 165


42 . 47 . This bucket is made of tin. Its
base diameters are 20 cm and
30 cm and the slant height is
32 cm. Find the area of tin used
to make this bucket.
This conical frustum-shaped segment is a piece
of NASA’s Ares-9 rocket. It has base diameters of
18 feet and 12 feet and height 10 feet. Find its
lateral area, to the nearest square foot. F. Volume of a Cone
48 . Find the volume of a cone with base area 40 cm2
and height 18 cm.
43 . Find the height of this lampshade
with 26 cm top and 34 cm bottom
diameters, given that it has a 49 . Find the volume of the
120ò10  cm2 lateral surface area. adjacent elliptical cone
if its base area is 20 cm2. 6 cm

44 . Find the total surface area of the


adjacent frustum with base radii
3 cm and 7 cm and slant height 50 . What is the volume of a circular oblique cone
9 cm. with base radius 7 cm and height 9 cm?

45 . Find the total surface area of a circular frustum if 51 . What is the volume of a circular right cone if its
its base radii are 4 cm and 10 cm and the height base radius is 6 cm and its height is 11 cm?
is 8 cm.

52 . This ice cream bowl has 12 cm


46 . This conical frustum-shaped glass
diameter and 7 cm depth. Find
has internal base radii 2 cm at the
its volume, to the nearest cubic
bottom and 4 cm at the top. The
centimeter.
glass is filled with water to half its
depth.
a. Find the top surface area of the water. Is it the
53 . What is the weight of this gold cone
arithmetic mean of the base areas?
if its base radius is 7 cm and the
b. Find the area of glass that is in contact with
height is 12 cm? Find the answer to
the water if the inner slant height of the glass
the nearest kilogram, given that the
is 12 cm.
density of gold is 19.6 g/cm3.

166 Solid Geometry


54 . Find the volume of a cone if its base perimeter is 61. A sandglass with two identical

6ñ5 cm and the slant height is 7 cm. conical parts measures
10 minutes at a time with a
completely full cone of sand. 12 cm

55 . The adjacent figures are two The height of the sandglass is


circular cones. The first one 24 cm and its diameter is
has twice the height but half
I 12 cm. 6 cm
II
the radius of the second cone. a. Find the rate of the flow of the sand in cm3/sec.
Compare their volumes. b. 8 m 45 s have passed since the sandglass was
inverted. What is the radius of the remaining
sand in the upper half?

56 . A cone has a base area of 48 m2 and its slant


height makes a 30° angle with the base. What is
the volume of the cone?
62. A cone with identical r

radius and height is
r
inscribed in a cylinder d
57 . A right triangle with 13 cm hypotenuse is rotated with the same base and r
around its 12 cm leg. Find the volume of the height. Then the space between the cone and
generated cone. cylinder is gradually filled with water. The depth
of the water at any instant is d.
a. Find the area of the surface of the water for any
instant depth d.
58 . This conical basket was used by
the Klamath Indians of Oregon b. Find the volume of water for any instant depth d.
State in the USA. The basket has
diameter 40 cm and capacity
21 liters. Find its depth, to the nearest centimeter.
63 . Figure I opposite shows
h1 r h2
a solid made from the A B
combination of two I
59 . A 120° sector of a paper circle with 18 cm radius cones with a common
is rolled to make a cone. Find its volume. base. The solid in r
h
A
Figure II is a cone with B
II
identical radius and axis
60 . A cube-shaped lead block with 6 cm edge length AB congruent to the first one.
is melted and recast as a cone with 6 cm base a. Find a formula for the volume of the first solid.
diameter. Find the height of this cone, to the b. Compare the formula with the volume formula
nearest centimeter. of the second cone.

Solids of Revolution 167


64 . A machine part with 70 . Find the volume of the P 7 cm
2
the shape shown in frustum obtained by 6 cm
5 10
the figure will be made rotating this trapezoid
from titanium. Find the around side PQ. Q 11cm

required approximate weight of the part if the


density of the metal is 4.5 g/cm3.

71 . This conical tank with 12 cm base


diameter and 10 cm slant height is full
65. The longest element of an oblique circular cone is
 of water to half of its height. Find the
6ñ3 cm and its shortest element is 6 cm. These
volume of the water.
two elements meet at an angle of 30° at the apex.
Find the volume of the cone.

72 . For a particular circular right frustum, the sum of


G. Volume of a Conical Frustum the base areas is 75 cm2. If the product of the
66 . Each of these recyclable paper radii is 9 cm and the height is 7 cm, find the
cups has a volume of 74 cm3. The volume of the frustum.
base radii are 3 cm and 4 cm. Find
the height of one cup.

73 . A circular frustum has base areas 75 m2 and


12 m2. The slant height makes a 150° angle with
67 . Some cheeses are produced in the the smaller base. Find the volume of the frustum.
shape of a frustum. This one has base
perimeters 16 cm and 20 cm. Find
the volume of the cheese if its height is 10 cm.
74 . The base radii of this glass are 2 cm and
8 cm and the slant height is 12 cm. It is
full of water to half its height. How much
68 . This bucket has internal base radii
more water is needed to fill the glass?
8 cm and 15 cm. It has a slant height
of 25 cm. Find the volume of the
bucket.
75 . A conical vessel with
height 10 ft and diameter
69 . The adjacent figure shows the net 13 m
10 ft is partially full of
of a frustum of a circular cone. water to a depth of 2 ft. Then it is inverted, as
The radii are 2 m and 7 m. Find shown in the figure. Find the new depth of the
the volume of the frustum. water.

168 Solid Geometry


A. BASIC CONCEPTS
A sphere is often thought of as a symbol of perfection.

Definition Spherical Surface

In space, the set of all points which are equidistant from a fixed
point is called a spherical surface. r
The fixed point is called the center and the constant distance is r
O
called the radius of the spherical surface.

The center is O and the radius is r.

A spherical surface separates space into three subspaces: the space outside the surface, the
surface itself, and the space inside the surface.

Definition Sphere

A sphere is the solid figure generated by a complete spherical surface.

Spherical surface
O

Interior space

As we can see, there is a strong relation between a circle and a sphere. We can say that a
sphere is a three-dimensional translation of a circle. In other words, if we rotate a circle (or
a semicircle) around its diameter, it will generate a sphere. This relation results in similarities
between the definitions of elements of the circle and the sphere.
It is impossible to make a true net of a sphere. This is because a sheet of paper cannot be
curved in two directions at the same time. Therefore any sphere made from paper or card will
be an approximation, and not a perfect sphere. Probably the best way to approximate a sphere
with a net is to make a polyhedron with a large number of sides, similar to a leather football.

Solids of Revolution 169


Another way to make a net of a sphere is with a row of pointed ellipses, as shown below.
Globes can be made in this way, with the edges of the net running along the longitudes.
The perfect shape of
the sphere has
fascinated scientists
and philosophers
since early times. At
their tiniest, spheres
are the theoretical
shape of atoms and
molecules. Many
people also think of
stars and planets as
spherical bodies
(although physically
they are not exact
spheres). Stars and
planets are rounded
because their center
of gravity pulls their
mass towards their
center, and the sphere
is the most economic
shape which gathers
the mass around a
single point.

Elements of a sphere

Chord
A line segment whose Diameter
endpoints are on the Any chord that contains
surface of the sphere. the center of the sphere.
Q
P Radius
Secant Any line segment that
B connects the center of the
A line that intersects the sphere
at more than one point. O sphere to any point on
r the sphere.
A

Tangent
A line that intersects the T
sphere at one point.
Here, the point T is called
the point of tangency.

Notice that any chord of a sphere is the intersection of a secant and the sphere.
Balloon Fish The diameter can also be said to be the longest chord in the sphere. Its length is twice the
radius of the sphere. A sphere with center O and radius r can be written as S(O, r).

170 Solid Geometry


If we consider the relative positions of a point C and sphere with center O and radius r, we
can say:
C is inside the sphere if OC < r,
C is on the sphere if OC = r, and
C is outside the sphere if OC > r.

Remark Determination of a Sphere


In space, any four distinct points determine one and only one spherical surface. In other
words, we must know at least four surface points to define a sphere.
Compare this with the minimum number of points we need to define a circle or a plane.

B. RELATIVE POSITIONS OF A SPHERE AND A PLANE


Consider a sphere S(O, r) and a plane P. A line segment drawn from the center O to the plane
P intersects the plane perpendicularly at point F. This point F is called the foot of line
segment OF. Let the length of this segment OF be d. Comparing d with the radius r gives us
three possible cases:

1. The sphere and the plane are distinct (d > r):


In this situation, the sphere and the plane have no F
common points. r

d
The plane and the sphere are disjoint. O P

The distance between the sphere and the plane is d – r.

2. The plane is tangent to the sphere (d = r):


r
In this situation, the sphere and the plane intersect at
O
one point.
d
The plane is tangent to the sphere.
There is no distance between the sphere and the plane. P
F

If a plane is tangent to a sphere, the radius to the foot is perpendicular to the plane.

3. The plane intersects the sphere (d < r): F P

In this situation, the plane intersects the sphere at d


r
infinitely many points. The set of points is a circle. O

Solids of Revolution 171


EXAMPLE 58 A sphere is tangent to two parallel planes. The distance between the planes is 28 cm. Find
the radius of the sphere.

Solution The distance between the parallel planes is the length


of the diameter of the sphere, so the radius is 14 cm. K d

EXAMPLE 59 The distance between a plane and the center of a sphere is 10 cm. If the radius of the sphere
is 12 cm, find the area of the circle of intersection.

Solution Lok at the figure. The distance between the center C and the plane
is shorter than the radius r, which means that plane P cuts the
sphere. Let the radius of the circle of intersection be r1.

C
d K In the right triangle KCN, we can use the Pythagorean Theorem to
r1 get r12 = r2 – d2 , i.e.
r
r12 = 122 – 102 = 44 and r1 = 2ò11 cm.
N P
So the area of the circle of intersection is r12 = 44cm2.

For Experts

Ten identical spherical cannon balls with radius 6 cm are to be put


together in an equilateral triangular prism so that each sphere
remains tangent to its neighbour. Find the volume of the smallest
possible prism, to the nearest cubic centimeter.

The intersection of two spheres produces a circular section.

EXAMPLE 60 The radii of two spheres are 10 cm and 8 cm. If the distance between the centers is 12 cm,
find the area of the region produced by their intersection.

172 Solid Geometry


Solution Look at the figure. O1 and O2 are the centers of the
spheres, and O is the center of circle of intersection.
Since O1O2 is 12 cm, if we denote O2O as x
then O1O = 12 – x.
For right triangles TO1O and TO2O we have the
equations

2 2 2 2 2 2
OO1 + OT = TO1  (12 – x ) + r = r 1  144 – 24 x+ x 2 + r 2 =100 and
2 2 2 2 2 2 2 2
OO2 + OT = TO2  x +r = r 2  x + r = 64.

9
If we subtract these two expressions side by side, we get 144 – 24x = 36, i.e. x = cm.
2
Substituting this value in x2 + r2 = 64, we obtain

2 175 2 175 2
r = , hence Acircle = r =  cm .
4 4

Check Yourself 11
1. A plane is 20 m away from a sphere. If the radius of the sphere is 3 m, how far is the
farthest point of the sphere from the plane?
2. A sphere is intersected by a plane. The radius of the circle of intersection is equal to the
radius of the sphere. Find the distance from the center of the sphere to the plane.
3. Two parallel planes intersect a sphere and one of them passes through its center. The areas
of the circles of intersections are 81m2 and 9m2. Find the distance between the planes.
4. If the distance between the centers of two externally tangent spheres is 13 cm and the
radius of the smaller sphere is 5 cm, find the diameter of the other sphere.
5. Two identical spheres with 10 cm radii intersect and the area of the intersection region is
36 cm2. What is the distance between the centers of the spheres?

Answers:
1. 26 m 2. 0 3. 6ñ2 m 4. 16 cm 5. 16 cm

This is an image of one of the most accurate


man-made spheres, as it refracts the image of
Scientists throughout
Einstein in the background. This sphere differs
history have competed with
in shape from a perfect sphere by no more than
each other to make the most
40 atoms of thickness. It is thought that only
perfect solid sphere
neutron stars are smoother. (Photo courtesy of
NASA)

Solids of Revolution 173


C. SECTIONS OF A SPHERE
In this section we will study some special sections of a sphere.
Some of these sections are generated by the intersection of a sphere with
a plane, while some others are obtained by rotating a segment in the
sphere.

1. Spherical Segment of One Base

Definition Spherical Segment of One Base

A portion of a sphere cut off by a plane is called Major spherical


segment of one base
a spherical segment of one base.
O Minor spherical
segment of one base

h Spherical cap

Naturally, every plane which cuts a sphere separates that sphere into two spherical segments
of one common base.
The spherical surface of a segment produced by a plane is called a spherical cap or
spherical shell. The altitude of this cap is shown as h in this figure.

EXAMPLE 61 In a sphere of 10 cm radius, a plane 8 cm away from the center forms a segment of a base.
What is area of the base?

r1
Solution The sphere is cut by the plane, generating a C B
segment of a base.
r =10 cm
Let C be the center of this circle and let r1 be its O
radius. In the figure, the center of the sphere is
denoted by O, and radius of the sphere by r.

Since the points O, C and B form a right triangle,


we have r2 = OC2 + r12.

Hence r12 = 102 – 82 = 36, and since the area formula for the circle is A = r12,
the area of the circle with the center C is A = 36 cm2.

174 Solid Geometry


Definition Hemisphere
A hemisphere is a half sphere formed by a plane that cuts
through the center of a sphere.

We can see that a hemisphere is a special type of segment of one base.


If a plane forms a hemisphere, the circular intersection of the plane and the sphere is called
a great circle of the sphere. The circumference of a great circle is called an equator.

Hemisphere

Hemisphere

Great circle

2. Spherical Segment of Two Bases

Definition Spherical Segment of Two Bases, Spherical Zone

A portion of a sphere between two parallel Spherical


segment of two
planes which cut the sphere is called a spherical bases Base
segment of two bases.

O1

Spherical O2
zone Altitude
Spherical Zone
Base

The circular sections generated by the planes are called the bases of the segment. In the
figure above O1 and O2 are the centers of the two circular bases.

The distance between the bases of the segment is called the altitude of the segment.

The curved lateral surface of the segment is called a spherical zone.

Solids of Revolution 175


There are two possible cases for a segment with two particular bases: the bases can be either
in the same hemisphere or in different hemispheres.

O1 O1 In the examples opposite, the bases of the


O2
two segments are respectively congruent but
C the segments are different. This situation
C
O2 must be taken into consideration when
solving problems.

Segment without the center of Segment including the center


the sphere (minor segment) of the sphere (major segment)

EXAMPLE 62 A spherical segment of two bases is removed from a sphere. The base radii of the segment are
15 cm and 7 cm. The radius of the sphere is 25 cm. What is the altitude of the segment?

Solution As we have seen above, there are two possible cases for the segment:

I. II.
r1
O1
C
r1 C
O1
r2 r2
O2 O2

Let the radius of the sphere be R, the radii of the bases of the segment be r1 and r2, and
the height of the segment be h. Note that h is the distance between the centers of bases, i.e.
O1 and O2.
I. II.
r1
O1
C R
R C
r1 O1 h
R R
h
r2 r2
O2 O2

In both cases, by applying the Pythagorean Theorem, we can find the respective distances
between the center of any base and the center of the sphere as:

CO1
2
 R2  r12 CO2
2
 R2  r22
CO1
2
 252  15 2 and CO2
2
 25 2  7 2
CO1  20 cm CO2  24 cm
In Case I, the bases are in the same hemisphere, so the height of the segment is
h = CO2 – CO1 = 24 – 20 = 4 cm.
In Case II, the bases are in the opposite hemispheres, so the height of the segment is
h = CO2 + CO1 = 24 + 20 = 44 cm.

176 Solid Geometry


3. Spherical Sector
We know that one full rotation of a semicircle around its diameter as an axis generates a
sphere.

Definition Spherical Sector

If a sector of a semicircle is rotated around the diameter of the semicircle, the generated
portion is called a spherical sector.

Spherical sector

Spherical zone

Note that the surface of the spherical sector is a spherical zone, and the vertex (apex) of the
sector is the center of the sphere.
We can also describe a spherical sector as a combination of two cones with the same vertex
and the spherical zone between them.

EXAMPLE 63 We have a semicircle with radius 10 cm. A spherical sector is generated by rotating a sector
of the semicircle around the diameter. The spherical sector has base radii 5 cm and 6 cm.
What is the angle measure of the initial circular sector?
Solution Look at the figure.
We need to find angle  which is the difference between the
B r1 C
angles COA and COB:  = m(COA) – m(COB)
A r2 r
r q r2 r
Since sin COA   0.6 and sin COB  1  0.5
O r r
we have m(COA) = arcsin 0.6 = 37° and
m(COB) = arcsin 0.5 = 30°. Therefore the angle of the sector
is  = m(COA) – m(COB) = 37° – 30° = 7°.

Solids of Revolution 177


A spherical cone is a special type of spherical sector.
Definition Spherical Cone
If a sector of a semicircle has a side on its diameter, the solid formed when the sector is
rotated around the diameter is called a spherical zone.

Spherical cone

4. Spherical Wedge

Definition Spherical Wedge, Lune


A
A portion of a sphere between two great semicircles of the
Lune
sphere is called a spherical wedge. r
The spherical wedge contains a part of the spherical surface.
This part is called a lune. O
a
C B D

Spherical
B wedge

The dihedral angle formed by the planes of the semicircles is called the angle of the wedge
or angle of the lune. In the figure above,  is the angle of the wedge.

EXAMPLE 64 How many wedges of 72° angle can we cut a sphere into?
Solution Since one whole sphere is 360°, we can cut a sphere
360
into  5 wedges of 72°.
72

178 Solid Geometry


Check Yourself 12
1. A sphere is cut by a plane. Which kinds of geometric shape can be generated by this
intersection?
2. The spherical room in the picture has an inner diameter of 3 m. The
center of the room is 0.6 m above the ground. Find the area of the
circular base of the room.

3. Two swords are fixed together parallel to each other, with a space in between. If we cut a
spherical water melon with this tool, which sections do we obtain?
4. A spherical segment of two bases and a frustum of a cone have the same bases and the
same altitude. Which solid has the biggest volume?
5. A spherical segment of two bases in a sphere has a height of 24 cm. The radius of the sphere
is 20 cm. One base area of the segment is 256 cm2. What is the area of its other base?
6. If we rotate a particular sector of a circle around one of its sides, the diameter of the
spherical cone it forms is the same as the radius of the main sphere. What was the angle
measurement of the initial sector?
7. A wedge is removed from a sphere with radius 25 cm. Find the total length of the edges
of the wedge.
Answers
1. only a circle 2. 1.44 m2 3. two spherical segments of one base, and a spherical segment of two bases
4. the spherical segment of two bases 5. 256 cm2 6. 30° 7. 50( + 1) cm

Spherical Geometry
You have studied the laws of Euclidean plane geometry in previous courses. However, if we
try to apply these rules on the surface of a sphere (for example, in large-scale calculations on
the surface of the Earth), we will find that they are not all true. As an example, the angles of the
triangle shown in the picture have a sum greater than 180°. The special properties of lines and
figures drawn on a sphere are the subject of a type of geometry called spherical geometry.
Spherical geometry has many practical applications in navigation and astronomy. In spherical
geometry, a line is defined as any great circle of the sphere. A practical application of such lines
is the system of longitude and latitude that we use to locate points on earth.
Menelaus of Alexandria seems to be the first mathematician who studied spherical
geometry. He wrote a book on the topic called Sphaerica. Of the Islamic mathematicians, Abu
al-Wafa (940-998 CE) devoted the first seven chapters of his book Kitab al-Majisti to spherical
geometry, while The Book of Unknown Arcs of a Sphere, written by the mathematician
al-Jayyani (989-1079 CE), is the first concise book written on the topic. Al-Jayyani's work studied right triangles and trigonometry on
a sphere. Following this work, the great scholar Nasir al-Din Tusi (1201-1274) was the first mathematician to list the six distinct cases
for a right triangle on a sphere.
European works on spherical geometry came later, and built on the work of the Islamic mathematicians. One example is the book
On Triangles, written by the German mathematician Regiomontanus in around 1463.
In summary, in Medieval times spherical geometry was much more developed in the Islamic world than in the West. One reason
for this could be that Islamic mathematicians used spherical geometry to calculate important distances and directions on earth.

Solids of Revolution 179


D. SURFACE AREA OF A SPHERE
The surface area of a sphere depends on a single parameter: its radius.
Theorem Surface Area of a Sphere
The surface area of a sphere with radius r is 4 r 2.
Asphere = 4 r 2
For the previous solids in this book, we have calculated surface area
using a net. However, it is difficult to derive the surface area of a
sphere by means of its net. This is because it is not possible to make a
net of a sphere, as we have seen. Therefore we will try to develop a
geometric proof.

Proof We can approximate a sphere to a pile of very thin frustum layers which gradually increase
and then decrease in volume. This is the approach we will use in this proof.
We will divide the proof into two steps.
Step one: We will show that the lateral area of a conical frustum obtained by rotating a
segment BC around an axis VO is AL=2r'h. Here the height of the frustum is h, and r' is the
perpendicular from the midpoint K to the axis VO. Note that in this frustum, the slant height
is BC = l, and the middle radius is KM = m.

B V
a
l b m
K M h
a

b
C T
Q
r'

As we have already seen, the lateral area of the frustum will be A L = l(r 1 + r 2) = l (BV + CT).
Since m is the middle radius, BV + CT = 2m and so AL =  l2m = 2 lm.
On the other hand, since KO is perpendicular to BC, we have two similar triangles:
BC BQ l h
BCQ  KOM. So    , which means lm = r'h.
KO KM r m
If we substitute this in the formula above, we have A L = 2 l m = 2 r ' h .
Step two: In this part of our proof, we consider a sphere as a pile of frustums. On the circle
below we develop from BC a half regular polygon ABCDE… with an indefinite number of
equal sides and center O. In this polygon, all the side lengths are l, so the perpendiculars
from their midpoints to the axis VO will again be r'.

180 Solid Geometry


A
B h1

h2
C r'
r'
h3
r'
D O
r' h4

E
r'

hn

If we rotate this half regular polygon ABCDE… around the axis VO again, we will have a
compound solid made of multiple frustum layers. The solid is not a sphere, but it is very close
to a sphere. Let us name the surface area of this solid A', and let the surface area of its
circumscribed sphere be A.
Let An be the lateral area of the nth frustum obtained by the rotation, then
A1 = 2h1 , A2 = 2r'h2 , A3 = 2r'h3 , ... , An = 2r'hn .
Let A' be the total lateral areas of all frustums produced.

A  A1  A2  A3  ...  An  2 r h1  2 r h2  2 r h3  ...  2 r hn  2 r ( h1  h2  h3  ...  h n )


  
2r
A  2 r 2r  A   4  r r

Now, to make this sphere-like solid even more like a sphere, we must increase the number of
frustum layers. This will result in a shorter height h and a longer r' for each frustum. If we make
infinite layers, the polyhedron will become the sphere containing it, the segment r' will be as
long as radius r, and the area A' of the polyhedron will be the area A of the sphere:
r  r and A  A
A  4rr 
infinite sections
 A  4r  r  4r 2 .

This completes the proof.

EXAMPLE 65 The radius of this marble sphere is 3 cm. Find its surface area.

Solution The surface area of a sphere for radius r = 3 is


A = 4r2 = 432 = 36 cm2.

Solids of Revolution 181


EXAMPLE 66 The diameter of the adjacent fun beach ball is 1 m. Find its approximate
surface area, in square meters.
d 1 1
Solution The radius of the sphere is r   m , so A = 4 . ( )2 =   3 m2.
2 2 2
Notice that if the diameter of a sphere is D, we can write the surface area of the sphere as D2.

EXAMPLE 67 The surface area of a sphere is 48cm2. What is the diameter of this sphere?

Solution Given that the surface area of the sphere is 48cm2, we have
A = 48= 4r2  12 = r2  r = 2ñ3 cm.
So the diameter of the sphere is d = 2r = 4ñ3 cm.

EXAMPLE 68 A great circle of the adjacent spherical fountain has circumference


10 dm. What is the surface area of the fountain?

Solution For a great circle, P = 2r and P = 10= 2r, so r = 5 dm.


So the surface area of the sphere is A = 4r2 = 452 = 100 dm2.

EXAMPLE 69 Show that the surface area of a hemisphere with radius r is 3r2.

Solution This problem is left as an exercise for you.

Check Yourself 13
1. The radius of the adjacent granite marble sphere is 30 cm.
What is its surface area?
2. We want to paint the surface of this wooden sphere.
If the radius is 5 cm, find the surface area of the sphere to the
nearest square centimeter.

3. The Earth is an approximate sphere and rEarth = 6380 km. What is the approximate
surface area of the Earth, in square kilometers?
4. The surface area of a sphere is 144 cm2. What is the circumference of its great circle?
5. For which radius will a sphere have the circumference of its great circle k m and surface
area k m2? Find k.

Answers
1. 3600 cm2 2. 314 cm2 3. 162 817 600 km2 4. 12 cm 5. r = 0.5 m, k = 

182 Solid Geometry


E. SURFACE AREAS OF SPHERICAL SECTIONS
1. Surface Area of a Spherical Segment of One Base
To find the surface area of a spherical segment of one base, we first need to know the area
formula for a spherical cap.

Theorem Surface Area of a Spherical Cap

The surface area of a spherical cap with altitude h is


r1 h
O1
Acap= 2 rh , where r is the radius of the original a
sphere.
O

EXAMPLE 70 Find the outer surface area of this spherical cap-shaped metal bowl
if it is 12 cm deep and its main sphere has a radius of 16 cm.

Solution Since the complete sphere would have a radius of 16 cm,


Acap = 2rh
Abowl = 2 . 16 . 12 = 384 cm2.

EXAMPLE 71 The domes of buildings were traditionally covered with lead panels
for protection. The dome of Selimiye Mosque in Edirne is one of
the largest leaded domes in Turkey. The dome's outer diameter is
33 m and the portion covered with lead has a height of 10 m.
Find the total area of the lead used for covering the dome, to the
nearest square meter.

The dome of Selimiye Mosque

Solution The leaded part of the dome is a spherical cap.


The radius of the dome is 16.5 m.
Therefore the area of the portion of dome covered with
lead is
A = 2rh  A = 2  16.5  10 = 330  1037 m2.

Solids of Revolution 183


EXAMPLE 72 The outer surface of this spherical bowl is glazed. The diameter of its
mouth is 12 cm, and its depth is 18 cm. Find the area of the glazed surface.

Solution A r1=6 O1 In this question, the main radius is not


given directly. However, we can find it by
r 18 –r
setting up the right triangle OO1A:
O 18 cm OO12 + AO12 = AO2  (18 – r)2 + 62 = r2 and
r 324 – 36r + r2 + 62 = r2  36r = 360, i.e. r = 10 cm.
So the area of the glazed surface is
Abowl = 2rh = 2 . 10 . 18 = 360 cm2.

EXAMPLE 73 A saj pan is a special type of metal plate in the shape


of a spherical cap which is used for cooking. The
pictures at the right show a saj pan cooking a popular
Turkish-style savoury pancake called gözleme. This
saj has an 80 cm circumference and 8 cm height.
What is the total surface area on top of the pan?

Solution This problem is left as an exercise for you.

Theorem Area of a Spherical Segment of One Base


The surface area of a spherical segment of one base is the sum of the area of its spherical cap
and the area of the circular base of the cap.
Asegment = Acap + Acircle = 2rh + r12  Asegment = (2rh + r12)
Note that r1 is the radius of the circular base of the segment, not the radius of sphere.

EXAMPLE 74 A sphere of radius 5 cm is cut by a plane 3 cm from the center of the sphere, forming two
spherical segments of one base. Find the total surface area of the smaller segment.

Solution Look at the figure. The plane  separates the sphere into A
two segments. r1
O1 B
In the smaller segment, OO1 = 3 cm and AO1 = h = 2 cm a
r = 5cm
since the radius of the sphere AO is 5 cm. On the other O
hand, r12 = r2 – OO12 = 52 – 32  r1 = 4 cm.
Therefore the total area of the segment, including the
circular base, will be
Asegment = (2rh + r12) = (2 . 5 . 2 + 42) = 36cm2.

184 Solid Geometry


In the questions below, you will need to use spherical geometry to discover some astronomical
facts. You may need to use the following numbers in your calculations.
RadiusEarth  6380 km, RadiusSun 700 000 km, RadiusMoon 1737 km,
DistanceMoon to Earth 384 000 km, DistanceSun to Earth 149 000 000 km

For Experts

Since the Sun is a spherical light source bigger than Earth, we


can assume that it illuminates more than half of the Earth. Find
what percentage of Earth is illuminated by the Sun at any time.

For Experts

During a full solar eclipse, the Earth, Moon and Sun are
aligned so that the Moon blocks sunlight and therefore
causes a shadow on Earth. This region is called the umbra.
Find the area of the umbra, to the nearest square kilometer,
in a full solar eclipse.

2. Surface Area of a Spherical Segment of Two Bases


As we have seen, a spherical segment of two bases is made up of a spherical zone and two
bases. Before we can find its total surface area, we need to know the surface area formula for
a spherical zone.

Theorem Surface Area of a Spherical Zone


The area formula for a spherical zone with altitude h and radius
of the containing sphere r is
Aspherical zone = 2rh.
h r

Notice that the area of a spherical zone depends only on the radius of the corresponding
sphere, and is independent of the radii of its bases. In other words, a spherical zone moving
over the surface of a particular sphere always has the same area, assuming the zone keeps
the same height.

Solids of Revolution 185


EXAMPLE 75 There are several ways to slice an orange. Aliyu cuts a big
spherical orange with diameter 10 cm using parallel cuts of
his knife.
Each slice is 2 cm thick. Find the area of orange peel in the
third slice, to the nearest square centimeter.

Solution The area of peel on a slice with thickness (height) h is determined by the formula
Aspherical zone = 2rh.
As we have seen, the area of a spherical zone does not depend on its position on the sphere.
Therefore, no matter which slice Aliyu chooses, the area of its peel will be
A = 2rh = 2 . 5 . 2 = 20  63 cm2.
Now consider the surface area of the first slice, regarding it as a spherical cap. Compare this
area with the answer above.

Theorem Surface Area of a Spherical Segment of Two Bases

The surface area formula for a spherical segment of two bases is


the sum of the area of its spherical zone and the area of its r
circular bases.
Asegment = Aspherical zone + Abase + Abase
1 2
r2
= 2rh + r12 + r22 h
r3

EXAMPLE 76 A sphere with radius 5 cm is cut by two parallel planes which are 1 cm apart from each other.
If the plane closest to the center is 3 cm away from the center, find the total surface area of
the generated segment with two bases.

Solution The closer plane passes 3 cm away from the center. We can
conclude that the farther plane passes 4 cm away from the
center, giving us a segment of height 1 cm.
O
r
OO1 = 3 cm and OO2 = 4 cm. r1
O1
r
Since the radius r of the sphere is 5 cm, by the Pythagorean r2
h
O2
Theorem we have r1 = 4 cm and r2 = 3 cm.
Therefore the total surface area of the segment is
Asegment = 2rh + r12 + r22 as Asegment = 2 . 5 . 1 + 42 + 32 = 35cm2.

186 Solid Geometry


3. Surface Area of a Spherical Sector
For a spherical sector, if we study the whole surface area, we will see that it consists of three
parts:

r1 Lateral surface of an inner cone


O1 with radius r1
r h
r2
O2 A spherical zone
r
O

Lateral surface of an outer cone


with radius r2

Therefore the surface area of the spherical sector will be the sum of the areas of these three
surfaces:
Aspherical sector = Aspherical zone + Ainner cone + Aouter cone = 2rh + r1r + r2r.
In conclusion, Aspherical sector = r(2h + r1 + r2).

EXAMPLE 77 A 15° sector of a circle with radius of 10 m is rotated around a diameter which makes an
angle of 30° with the closest side of the sector. Find the total surface area of the spherical
sector generated by the revolution.

Solution The solid of revolution is shown in the adjacent figure. This solid is O1
r1
a spherical sector. It has two cones with bases of radii r1 and r2. O2
r2
Since the area formula for the solid is Aspherical sector = r(2h + r1 + r2),

°
30
15°
we need to find r1, r2 and h. For this, we will apply trigonometry. O r

3
OO1 = r . cos 30° = 10 . = 5ñ3 m and
2
2
OO2 = r . cos 45° = 10 . = 5ñ2 m.
2
Since h = OO1 – OO2 , we have h = 5(ñ3 – ñ2) m.
1 2
On the other hand r1 = r . sin 30° = 10 . = 5 m and r2 = r . sin 45° = 10 . = 5ñ2 m.
2 2
Substituting these values in the formula for the surface area of a spherical sector, we have

Aspherical sector  10  2  5( 3  2 )  5  5 2   10  10 3  5 2  5 


   
Aspherical sector  50 (2 3  2  1) m 2.

Solids of Revolution 187


k
EXAMPLE 78 The shaded sector in the figure is a 60° sector of a circle with center
O and radius 6 cm. If it is rotated around the axis k as in the figure,
what is the total surface area of the generated solid? 6 cm O
60°
Solution This problem is left as an exercise for you.

h
As a special case, if we have a spherical cone as shown in the O1 r1
adjacent figure, its total surface area will be the sum of the r
areas of its spherical cap and the lateral area of the cone:
O
Aspherical cone  Aspherical cap  Acone  2  r h  r1 r.
This gives us Aspherical cone  r(2 h  r1 ).

EXAMPLE 79 A circular sector of 45° is rotated around one of its radii. If the sector was itself 2 cm , find 2

the total area of the figure generated by the revolution.



Solution The area formula for a circular sector of angle  is Acircular sector  r 2 for a circle with
360 
radius r.

Let us find the radius of the circular sector.


45
Since the area of the sector is 2 cm2, Acircular sector  2  r 2 and so r = 4 m.
360 
The solid formed by the revolution is a spherical cone, as shown below.

A A

r r
r1
45° O 45° O
B B
C
h

By trigonometry in right triangle AOC, we get r1 = OC = 2ñ2 cm.


On the other hand, BC = BO – CO, which means h = r – CO = 4 – 2ñ2 cm.
The total surface area of the spherical cone is Aspherical cone = r(2h + r1).

Therefore Aspherical cone    4 2(4  2 2 )  2 2     4 8  2 2   8   (4  2 ) cm 2.


   

188 Solid Geometry


4. Surface Area of a Spherical Wedge
As we have seen, a wedge intercepts a part of the spherical surface called a lune. The area of
a lune is proportional to the measure of the angle it includes.

Theorem Area of a Lune


A
The area of a lune with a central angle  on a sphere with radius r is Lune
r  lune
2
r
Alune  .
90
O a
C B D
Proof If we consider the whole sphere as a lune of 360°, we can easily r

find the surface area formula for a lune: Spherical


B wedge
2
Alune anglelune angle lune  r  lune
  Alune  Asphere  4  r 2 lune  A lune  .
Asphere 360  360  360  90 

EXAMPLE 80 Find the area of a lune if the angle of the lune is 40° and the radius of its sphere is 60 cm.
Solution The area of a lune is proportional to the measure of the angle it includes.
r 2  lune 60 240 
For this lune the area will be Alune    1600  cm 2 .
90 90 

EXAMPLE 81 The adjacent spherical ball has five equal red sections. What is the total area
of the red sections?
Solution This problem is left as an exercise for you.

EXAMPLE 82 Find the surface area of the remaining shell of a spherical watermelon,
if a slice in the shape of a wedge of 66° is removed. The radius of the
watermelon is 15 cm.

Solution This problem is left as an exercise for you.

A wedge of a sphere is not only a lune. It also contains two great semicircles.
Theorem Surface Area of a Wedge
A
The surface area of a wedge with a central angle , taken from a
  r
sphere with a radius r, is Awedge  r 2  lune  1.
 90  O a

Proof The surface area is a lune and two semicircles. The two semicircles C B D
r
will make a whole great circle of the sphere.
B
r 2  lune  
Awedge  Alune  2 Asemicircle   r 2  Awedge  r 2  lune  1
90  90  

Solids of Revolution 189


EXAMPLE 83 Find the total area of a wedge cut from a sphere of 12 cm radius, if its angle is 25°.
2  25   23
Solution The total area of the wedge will be Awedge  12   1  144    184  cm 2 .
 90  18

Check Yourself 14
1. This cap is a part of a sphere with 20 cm radius. The height of the
cap is 7 cm. Find the area of the cap.

2. This bowl is a part of a sphere with radius 10 cm. The open mouth of
the bowl is 12 cm wide. What is the area of the outer surface of the O

bowl, to the nearest square centimeter?

3. A sphere has a diameter of 26 m. What is the total surface area of the spherical segment
created by two planes which are 10 cm apart from each other, if the two planes are
equidistant from the center of the sphere? Pole
60°
4. The latitudes range from 0° to 90° between the Equator and
the poles on Earth. 0°(Equator)

Find the area of the zone between the latitudes 0° and 60°.
(Consider the Earth as a sphere and use rEarth = 6380 km.)
t
5. The shaded sector shown at the right is a 48° sector of a
circle with center O and radius 8 cm. If it is rotated around
the axis t as in the figure, what is the total surface area of the 66°
48° O
generated solid? m
8c
(Use sin 24°  0.4 and cos 24°  0.91.)

6. What is the total surface area of the spherical cone in the


adjacent figure?
O
cm 60°
20

7. From a spherical apple, a wedge slice with 40° angle is removed. The diameter of the apple
is 6 cm. What is the area of skin on the apple slice, to the nearest square centimeter?

Answers
1. 280 cm2 2. 360  1131 cm2 3. 548 m2 4. 40 704 400ñ3km2
5. 687 cm2 6. 200(5 – 2ñ3) cm2 7. 4 cm2

190 Solid Geometry


F. VOLUME OF A SPHERE
A sphere is the most perfect and balanced solid known, and the challenge of calculating its
volume attracted the attention of many people in early times.

Theorem Volume of a Sphere

4 3
The volume of a sphere with radius r is Vsphere  r .
3

We will look at two elegant proofs of the volume of a sphere.

Proof 1 This proof was probably developed by Archimedes. It uses the relationship between a
cylinder and a sphere circumscribed by that cylinder. First let us take a hemisphere and a
cylinder, as shown in the figure. Assume that the cylinder has the same base and altitude as
the hemisphere. Now, from the cylinder, let us remove a cone that has the same base and
altitude as the cylinder.

The volume of the hemisphere is equal to the volume of the cylinder that remains after we
remove the cone.
Vhemisphere  Vcylinder  Vcone
Is this hard to believe? Let us look at an explanation. Cavalieri’s principle tells us that if
corresponding planes parallel to the bases generate sections of equal areas on two solids, we
can conclude that the two solids have equal volumes.

r' r a
r a a
a

Therefore for any plane parallel to the bases, let us show that the sections generated by the
plane have equal areas in both solids in the figure.
The radius of the sphere and the radius of the cylinder are both r. Let the parallel plane be
a units above the base, so it will generate a circle in the sphere, and a ring in the cylinder.
The radius of the circle is r' and the ring has two radii, r and a.

Solids of Revolution 191


For any a, the area of the circle generated in the circle will be ACircle = r ' 2 , and it is easy to
obtain the equality r ' 2 = r2 – a2 by the Pythagorean Theorem, which gives us Acircle = (r2 – a2).
Again for the same a, we will have the area of the ring generated in the cylinder as
Aring = r2 – a2, i.e. Aring = (r2 – a2).
As we can see, for any parallel cut, the sections on both figures have an equal area, which
means that the volumes are equal.
1 2
Now, Vhemisphere  Vcylinder  Vcone   r 2h   r h.
3
1 2
Since h  r, we have Vhemisphere   r 2 r   r 2 r  r 3
3 3
4
The volume of a sphere is twice the volume of a hemisphere, and so Vsphere  r 3 . This is
3
the formula for the volume of a sphere.
We will use some of Archimedes' important observations regarding the volume of the sphere
at the end of this section.

Proof 2 This proof uses approximation, like the proof of the surface area
formula for a sphere. The adjacent polyhedron is similar to a sphere
and it has been divided into many pyramids which have common
heights and common vertices at the center of the polyhedron.
The height of each pyramid is slightly less than the radius of the
polyhedron. The volume of each pyramid is determined by its base area
1
and this common height, i.e. Vpyramid  Abase  h
3
The volume of the polyhedron is the sum of the volumes of these pyramids:

Vpolyhedron  Vpyramid 1  Vpyramid 2  ...  Vpyramid n


1 1 1 h
 Abase1 h  Abase 2 h  ...  Abase n h  ( Abase 1  Abase 2  ...  Abase n )
3 3 3 3

Since the sum of the base areas of all the pyramids is the surface
h
area of the polyhedron, we can write Vpolyhedron  Apolyhedron .
3
As the number of pyramid sections increases to infinity, the figure
will become a sphere while the common height h becomes the
radius r of that sphere:
h polyhedron  Sphere and h  r r r
Vpolyhedron  Apolyhedron 
infinite pyramidal sections
 Vsphere  Asphere  4r2 .
3 3 3
4 3
Therefore the volume formula for the sphere is VSphere  r .
3

192 Solid Geometry


EXAMPLE 84 The radius of this exercise ball is 3 dm. Find the volume of the ball.
4 3 4 3
Solution The volume of the spherical ball is Vsphere  r  3  36  dm 3.
3 3

EXAMPLE 85 This type of coiled basket is used by the Pima Indians of Arizona.
This particular basket is in the shape of a hemisphere with diameter 3 dm.
Find its volume, to the nearest liter.
1 2
Solution The volume formula for the hemisphere is Vhemisphere  Vsphere  r 3 and the radius is
2 3
3
3 2 2 3 9
r dm, so Vhemisphere  r 3        7.06858... dm 3.
2 3 3 2 4
This approximates to 7 liters.

EXAMPLE 86 A coconut contains coconut flesh inside its shell, as shown in the
picture. The inner diameter of a spherical coconut shell is 11 cm, and
this is also the outer diameter of the flesh. If the coconut flesh is 1 cm
thick, find the volume of the flesh, to the nearest cubic centimeter.

Solution The flesh forms a sphere that has a spherical volume of air inside. The volume of the flesh
is the difference between these two spherical volumes. Let the outer radius of the flesh be R
11 9
and the inner radius be r. This gives us R  cm and r  cm , so
2 2
Vcoconut flesh  Vinner volume  Vinner aerial sphere
3 3
4 4 4  11  4  9  r
 R3  r 3        
3 3 3  2  3 2  R
4  1331  729  602 3
    6   315 cm .
3  8 

EXAMPLE 87 The Sun is nearly one million times bigger than the Earth in volume. Compare their diameters.

Solution Let us name the radii of Sun and Earth respectively as R and r,
and the diameters D and d.
VSun
Then we can set up their volume ratio as  1 000 000 .
VEarth
4 3
R 3
Vsun R3  R  R D
 3  3     1000000, so  100 and  100.
Vearth 4 r r r d
r 3
3
The diameter of the Sun is therefore one hundred times longer than that of the Earth. Considering
that Earth’s diameter is nearly 12 750 km, you can now estimate the diameter of the Sun.

Solids of Revolution 193


EXAMPLE 88 3 m3 of air is used to inflate an empty spherical balloon. Find how much
more air should be blown into the balloon to double its diameter.

Solution The answer is not 6 m3. The balloon begins with 3 m3 of air, and the new
diameter must be double the current diameter.
This means the new radius will be 2r, if the current radius is r.
4 4 4
So the new volume will be Vnew  (2 r )3    8 r 3  8   r 3  8  Vcurrent .
3 3 3
This means the new balloon will contain 8 times more air, namely 24 m3.
So it will take 21 m3 more air to double the diameter of the balloon.

EXAMPLE 89 The surface area of a ball is 616 cm2. Find the volume of the ball, to the nearest cubic
centimeter, using the radius rounded to the nearest centimeter and  = 3.14.

Solution The surface area formula is Aball = 4r2.


Aball  4r 2  4  3.14  r 2  616 cm 2  r  7 cm, so
4 3 4
Vball  r   3.14  7 3  1436 cm 3.
3 3

EXAMPLE 90 The adjacent sphere is inscribed in a cube and tangent to the cube. If the
surface area of the cube is 96 cm2, find the volume of the space between
the cube and the sphere.

Let the side length of the cube be a and the surface area of the cube be Acube  6a  96 cm ,
2 2
Solution

a
then a = 4 cm. a is also the diameter of the sphere, hence r  . So the volume between
2
4 a 4 a3 32
the cube and the sphere is Vcube  Vsphere  a3  ( )3  a3    64   cm 3 .
3 2 3 8 3

EXAMPLE 91 A right circular cone has the same radius as a sphere. If the volumes are also the same,
compare the height of the cone to its radius.

1 2 4 1
Solution Vcone = Vsphere gives us the equalityr h  r 3 . If we simplify both sides by r 2 , we will
3 3 3
get the relation h = 4r. Therefore the height of the cone must be four times its radius.

194 Solid Geometry


EXAMPLE 92 Three scoops of ice cream are put on an ice-cream cone as in the figure. The
diameter of each scoop is 4 cm. The diameter of the cone is 6 cm and the depth
of the cone is 11 cm. Assuming that the scoops are all spheres and all the ice
cream melts into the cone, will the ice cream fill the cone when it is left to melt?

Solution This problem is left as an exercise for you.

EXAMPLE 93 If a sphere is inscribed in a cylinder, find the ratio of their volumes.


Solution This problem is left as an exercise for you.

Archimedes and the Sphere


Archimedes (287-212 BC), who lived a generation after Euclid, is one of the best known
mathematicians of the ancient world. He made discoveries in many areas, but his greatest success
was in the study of the round solids. In his two books he stated 44 propositions related to the area
and volume of these solids. Here are two of them:

Archimedes found that the volume of a sphere is


exactly half the volume of the cylinder inscribing
the sphere. Archimedes believed that this theorem
was his greatest discovery.

He also found that the volume of a sphere is four


times the volume of the cone inscribed in the
sphere if the base of the cone is a great circle of the
sphere.
According to legend, Archimedes requested that a sphere inscribed by a cylinder be engraved
upon his tomb. After defending Syracuse with his brilliant strategies for three years against the
Romans, he was killed when the town was finally taken. His tomb was eventually forgotten over
time, and was only rediscovered 175 years after his death. At that time, the Roman officer Cicero
was appointed to Sicily. He had heard about the tomb of Archimedes and wanted to find it.
Although none of the local people could tell him where it was, he began a search and eventually
found it. This discovery is shown in the picture at the right. Cicero cleared the tomb and was able
to read the inscriptions on it. Today the exact location of Archimedes' tomb is unknown.

Solids of Revolution 195


Check Yourself 15

1. What is the volume of this granite sphere if its radius is 6 dm?

2. The adjacent spherical tank is used for gas storage in Japan. If the tank
has a diameter of 14 m, find its volume to the nearest cubic meter.

3. An igloo is a small hemispherical room made of ice blocks.


Igloos were traditionally used by Eskimos, who live in very
cold areas near the Arctic Circle.
The igloo in the figure has an inner diameter of 3 meters.
Find the volume of the hemispherical part, ignoring the
entrance.

4. If the volume of this metal hemisphere is 144 cm3, what is its


total surface area?

5. An official NBA basketball ball has a circumference of 74.9 cm. Find


the inner volume of the ball, to the nearest cubic centimeter, if the
rubber is 0.5 cm thick.

6. A shackle was a device that was once used to prevent prisoners


from escaping. This adjacent shackle has an iron ball with a
diameter of 16 cm. Given that the density of iron is 7.8 g/cm3,
find the weight of only the ball, to the nearest kilogram.

196 Solid Geometry


7. A sphere is inscribed in a cube, as shown in the figure.
a. Find the ratio of their surface areas. b. Find the ratio of their volumes.

8. The surface area and volume of this marble sphere are respectively
k cm2 and k cm3. What must the radius be?

9. The average radius of the Earth is 6380 km, and the radius of the
Moon is 1740 km. How many times is Earth bigger than the Moon
in volume?

Earth and Moon


from Galileo probe-NASA

10. A single bunch of grapes contains around 100 grapes.


If each single grape is a sphere with 3 cm diameter, evaluate the
total volume of grapes in a single bunch, in cubic centimeters.

Answers
9
1. 288 dm3 2. 1436 m3 3.   7 m3 4. 108 cm2
4
 
5. 6240 cm3 6. 17 kg 7. area ratio: , volume ratio:
6 6
8. 3 cm 9. nearly 49 times 10. 450  1414 cm3

Solids of Revolution 197


G. VOLUMES OF SPHERICAL SECTIONS
In this section, we will give some important formulas for the volumes of spherical sections
without proof, since they mostly require higher-level mathematics.

1. Volume of a Spherical Segment of One Base

Theorem Volume of a Spherical Segment of One Base


A
The volume of a spherical segment of one base is given by the formula
h
B
1
Vsegment  h2 (3r  h), r
3 O
where the segment has altitude h and r is the radius of the sphere.

EXAMPLE 94 Find the volume of the spherical cap-shaped metal bowl from
Example 70. It is 12 cm deep and its main sphere would have a
radius of 16 cm.

Solution The cap is a spherical segment of one base.


1 2
Its volume can be calculated by the formula Vsegment  h (3r  h), which gives us
3
1
Vsegment    12 2 (3  16  12)  1728  cm 3 .
3

EXAMPLE 95 The adjacent spherical fishbowl has a radius of 13 cm. If it is filled


to a depth of 21 cm, find the volume of the part filled, to the nearest
liter.
Solution The mouth of the bowl is not part of the spherical shape. However,
this does not affect our volume calculation since the problem is
related to only the filled segment of one base, which has the volume
1 2 1
Vsegment  h (3r  h)    212(3 13  21)  2646  cm 3  8.3 L.
3 3

EXAMPLE 96 A lampshade is produced by removing a 4 cm cap from a sphere which has radius 24 cm.
Find the volume of the lampshade, to the nearest liter.

Solution The lampshade is in the shape of a spherical segment of one base.


Since the height of the removed cap is 4 cm, the height of the lampshade
is h = diameter – 4 = 48 – 4 = 44 cm.

198 Solid Geometry


Now we can calculate the volume as
1 2 1 54208
Vsegment  h (3r  h)    44 2(3  24  44)   cm 3  57 L.
3 3 3

EXAMPLE 97 The pan in the picture is a type of saj pan used in Turkish
cuisine. This pan is in the form of a spherical cap and its
inner surface is used for cooking. This saj pan has a
diameter of 36 cm and depth 6 cm. What is its volume?

Solution Look at the figure. The depth of the saj is 6 cm, so in the right
triangle OO1P we have OO1 = r – 6 cm and r1 = 18 cm.
Now let us apply the Pythagorean Theorem:
O
OP2 = OO12 + O1P2  r2 = (r – 6)2 + 182,
r hence 12r = 360 and r = 30 cm.
r– 6
O1 r1
So the volume of the saj is
6cm P
1 2 1
Vsaj  h (3r  h)    6 2(3  30  6)  1008  cm 3.
3 3

EXAMPLE 98 This bowl is a part of a sphere. The mouth of the bowl is 10 cm wide. If
the bowl is 25 cm deep, what is the volume of the bowl?

Solution This problem is left as an exercise for you.

For Experts

Verify that an alternative volume formula for a spherical cap

h 2 2
with height h and base radius r1 is V =  (3r1 +h ) .
6

Solids of Revolution 199


2. Volume of a Spherical Segment of Two Bases
Theorem Volume of a Spherical Segment of Two Bases
The volume of a spherical segment of two bases is
r1 O1
1
Vsegment  h( h2  3r12  3r22 ),
6 O
h
where r1 and r2 are the radii of the circular bases of the segment and
r2
h is the altitude of the segment. Notice that the radius r of the sphere O2
is not a parameter of the formula.

EXAMPLE 99 The adjacent metal soup bowl is a spherical segment of two bases with radii 8 cm and 11 cm.
The bowl is 9 cm deep. What is the volume of the bowl?

Solution The volume can be calculated with the formula


1
Vsegment  h( h2  3r12  3r22 ), so
6
1
Vbowl    9(92  3  8 2  3  112 )  954  cm 3  3 L.
6

EXAMPLE 100 A sphere with radius 5 cm is cut by two parallel planes. The planes are 3 cm and 4 cm from
the center. Find the volume of the spherical segment contained between them.

Solution Let the center of the sphere be O. The distance between the planes is not given in the
question, so there are two possible cases.
In the first case, the planes intersect the sphere in the same
hemisphere. This case means OO1 = 3 cm and OO2 = 4 cm, so O
r
O1O2 = h = 1 cm. O1
r1
r
h
Considering that the radius r of the sphere is 5 cm, applying the O2
r2

Pythagorean Theorem gives us r1 = 4 cm and r2 = 3 cm. So the


1 1 38
volume for the first case is Vsegment  h( h2  3r12  3r22 )   1(12  3  4 2  3  3 2 )   cm 3.
6 6 3
In the second case, the planes intersect the sphere in
distinct hemispheres. r1
O1
Now OO1 = 3 cm and OO2 = 4 cm, so O1O2 = h = 7 cm. O
r
h
This distinction will not affect the radii r1 and r2. Now the volume is r
r2
O2
1 1 434
Vsegment  h( h2  3r12  3r22 )    7(7 2  3  4 2  3  3 2 )   cm 3.
6 6 3
In this question, we can see that if the base moves to the other hemisphere, the volume
increases nearly 12 times.

200 Solid Geometry


EXAMPLE 101 Aholemetal sphere with radius 30 mm is drilled along its diameter. If the radius of the drilled
is 18 mm, find the volume of the remaining portion of the sphere.

Solution The drill will remove from the sphere two caps and the cylinder between the caps. Therefore
the remaining portion is a segment of two bases with a cylindrical hole inside. The remaining
portion will have the volume

Vportion  Vsphere  2Vcap  Vcylinder h


hc

However, first we need to find the height hc of the cylinder. r1 R=30 mm


Let the radius of the sphere be R, then by the Pythagorean
O1 O
Theorem,

OO1  R2  r12  302  18 2  24 mm  hc  48 mm. hc

Therefore the height of the segment of one base is


h = r – OO1 = 30 – 24 = 6 mm.
Now we can find the volume of the portion as

4 3 1
Vportion  Vsphere  2Vcap  Vcylinder   r  2   h 2 (3r  h)   r1 2hc
3 3
4 1
   30 3  2    6 2(3  30  6)   18 2  48
3 3
 36000   2016  15552  18432  mm 3.

3. Volume of a Spherical Sector

Theorem Volume of a Spherical Sector


The volume of a spherical sector or a spherical h h
cone is
r r
2 O O
Vspherical sector  r 2 h,
3
where r is the radius of the sphere and h is the
altitude of the spherical zone or cap. Spherical cone Spherical sector

EXAMPLE 102 Derive the volume formula for a spherical cone using the formulas for the volume of a
spherical cap and the volume of an ordinary cone.

Solution This problem is left as an exercise for you.

Solids of Revolution 201


EXAMPLE 103 A circular sector of 60° and 3 m radius is rotated about one of its sides. Find the volume of
the generated spherical cone.

Solution The solid of revolution is a spherical cone. h


O1 r1
The volume formula requires the height of the cap, and for this
60° 60° r
we will use trigonometry:
O
1 3
h  r  OO1  r  r  cos60   3  3   m.
2 2
2 2 2 3
So the volume is Vspherical cone  r h  32  9  m 3.
3 3 2
y
EXAMPLE 104 In the figure, the shaded 30° sector with center O has a radius of
5 cm and makes a 30° angle with the y-axis. If it is rotated around
the x-axis as shown in the figure, what is the volume of the O
x
30
generated solid? °

30
r

°
Solution The solid obtained by the revolution will be a spherical sector.
In order to find its volume, we need to find the height of its zone: y

3 1 5
h  r  cos 30   r  cos60   5   5   ( 3 1) cm.
2 2 2
O h
So the volume of the sector is 30° x
30
°
2 2 5 125 

30
Vspherical sector  5  ( 3  1)  ( 3 1) cm 3. r

°
3 2 3

4. Volume of a Spherical Wedge

Theorem Volume of a Spherical Wedge

For a spherical wedge with central angle , A


Vspherical wedge  r 3 , where r is the radius of the main sphere. r
270
O
a
We can obtain this volume formula by using proportion.
C B D
If we regard the whole sphere as a wedge of 360°, then the r

  4 3
volume of the wedge is Vspherical wedge   Vsphere   r . B
360  360  3

After simplification we get Vspherical wedge  r 3 .
270

202 Solid Geometry


EXAMPLE 105 Find the volume of a wedge-shaped slice cut out of a spherical
watermelon with radius 15 cm, given that the angle of the slice is 36°.

Solution The volume of the slice is


 36
Vspherical wedge  r 3    153  450  cm 3.
270  270 

For Experts

One way to produce optical lenses is to press melted glass between two
spheres, as shown in the figure. A spherical cap and a sphere are used as a
mould to shape the lens. To produce a particular lens, the radius of the cap
r and the radius of the sphere R are adjusted accordingly.
Find the volume of the lens if r = 12 cm, R = 15 cm and the diameter of the
lens is 6 cm.

Check Yourself 16
1. This balance has two pans, each in the shape of a spherical segment of
one base. The depth of pan is 6 cm, and the main sphere would have
radius 25 cm. What is the volume of a single pan?

2. The interesting house in the picture is called an


archipod. It has the shape of a spherical segment of single base. The
inner diameter of the pod is nearly 3 m and the height of the pod is
2.5 m. Find the internal volume of the archipod.

3. A spherical fish bowl is filled with water to a depth of 24 cm. The water
surface also has diameter 24 cm. What is the volume of the water
together with the fish?
4. This is a steel teapot set with a small and a large
teapot. The teapots are spherical segments of two bases.
The larger pot has height 12 cm and its lower base and upper base
have diameters 14 cm and 10 cm respectively. What is the capacity
of the larger pot?

5. The adjacent picture shows a vase for flowers. It is a part of a sphere with
O
radius 25 cm. The base of the vase has diameter 30 cm, and the mouth
has diameter 14 cm. Find the volume of the vase.

Solids of Revolution 203


6. A 30° sector of a circle with 2 m radius is rotated about one of its sides. Find the volume
of the generated figure.
t

7. O is the center of a circle with radius 12 cm. Axis t bisects a 90°


circular sector of this circle.
O
x
When the sector is rotated about the axis x, find the volume of the
generated solid.

8. What is the volume of an orange slice in the shape of a wedge, if the whole orange was a
sphere with diameter 9 cm and the wedge angle is 24°?

9. The candle in the picture is in the shape of a sphere with height 12 cm.
We know that this candle will last for 8 hours. If we light this candle at
21:00, by what time will the top 3 cm of the candle have burnt?

Answers

1. 828 cm3 2. 25   13 m 3 3. 4032 cm3 4. 732 cm3  2.3 L


6
60676 8
5.  cm 3 6.  (2  3) m 3 7. 1152ñ2 cm3 8. 8.1 cm3 9. 22:15
3 3

Measure and See


A man who thought he was very wise came to Nasreddin Hodja
and asked,
“Can you tell me where the exact center of the Earth is?”
“Yes, I can!” replied the Hodja. “It is just under my donkey’s back
left foot.”
The man was surprised. “How can you prove it?” he asked.
“If you doubt my word, just measure and see,” said the Hodja.

204 Solid Geometry


EXERCISES 2 .3
A. Basic Concepts C. Sections of a Sphere
1. AB is one of the longest possible chords in a 11 . If one plane cuts a sphere, which section of a
sphere. What is the loci of all the chords with the sphere do we obtain?
same length which are perpendicular to chord AB?
12 . A plane intersects a sphere such that the
2. The center of a sphere with 24 m diameter is circumference of the intersection is 24 cm. If
40 m above the ground. How high is the base of the plane is 4 cm from the center of the sphere,
the sphere from the ground? find the area of the great circle of the sphere.

3. A sphere with radius 7 cm is tangent to a plane. 13 . The bowl in the picture has its
What is the distance from the plane to the farthest mouth 8 cm above the center of the
point of the sphere? sphere. Find the area of the mouth
if the radius of the sphere is 12 cm.
4. A sphere is inscribed in a cube with 150 cm2
surface area. What is the radius of the sphere? 14 . The base perimeter of a spherical cap is 10 cm.
Find the height of the cap if the radius of the
5. The distance between a plane and the center of a main sphere is 13 cm.
sphere is 7 cm. If the radius of the sphere is 9 cm,
find the area of the shape formed by the intersection. 15 . In a sphere of 12 cm radius, a spherical segment
with two bases has one base area 108 cm2 and
6. A bowl in the shape of a hemisphere with 50 cm height 14 cm. Find the area of the other base of
diameter contains some water to a depth of 10 cm. the segment.
What is the area of the water's surface?
16 . Two parallel planes 14 cm apart from each other
7. What is the distance between the centers of two
cut a sphere on either side of the center of the
internally tangent spheres if their diameters are
sphere between them. The circles of intersection
4 cm and 14 cm?
have radii 10ñ2 cm and 12 cm. Find the radius of
8. The radii of two spheres are 4ñ5 cm and 5 cm. If the sphere.
the distance between their centers is 11 cm, find
the perimeter of their intersection. 17 . The base areas of a spherical segment with two
bases are 225 cm2 and 64 cm2. Find the
9. Three identical spheres with possible heights of the segment if the radius of
12 cm radius are placed the sphere is 17 cm.
tangent to each other between
two parallel blocks as shown 18 . The shaded sector of
in the figure. Find the semicircle shown in
distance between the blocks. the picture is rotated °
15
around the diameter of
10. The radius of Earth is approximately 6380 km. 30°
 the semicircle. What 9 cm
What is the spherical distance between two
are the diameters of
points on Earth which are located on the 40th the generated spherical sector?
East longitude and 32nd West longitude, if both
points are on the 60th North latitude? Find the 19 . How many wedges of 24° can we cut a sphere into?
answer to the nearest kilometer.

Solids of Revolution 205


20 . From a sphere of 18 cm radius a wedge of 90° is 30 . A sphere is inscribed in a cube. Find the ratio of
cut. What is the sum of the lengths of its edges? the surface area of the sphere to the surface area
of the cube.
D. Surface Area of a Sphere
31 . Electroplating is the process of
21 . A great circle of a sphere has area k. What is the covering one metal with another,
surface area of the sphere, in terms of k? especially a valuable one. Florin
22 . Find the surface area of a sphere if it has radius took his three identical iron balls
10 cm. to be electroplated with gold. If the
radius of each ball is 3 cm, and the
23 . A large hemispherical room is to be painted. If 5 cost of electroplating is 20 cent/cm2, will Florin be
tins of paint are used to paint the floor, how many able to remain under his $70 budget?
tins of paint will be used to paint the hemisphere?
32 . What is the area of the Moon, to the nearest
square kilometer? (The radius of the Moon is
24 . Musa wants to paint this rusty iron
1737 km.)
ball which has diameter 14 cm.
Find its surface area to the nearest 33 . Earth is 70% covered with water. Find the
square centimeter. approximate area of all the land on Earth, then
find what percentage of this area is used by your
25 . The hemispherical lampshade country. (The radius of Earth is 6380 km.)
shown opposite has a diameter
34 . The atmospheric pressure on
of 40 cm. Find its exterior
Earth is nearly 10332 kg/m2 at
surface area.
sea level. What is the approximate
weight of the atmosphere, in
26 . If the surface area of a sphere is 900 mm2, what tons? (The radius of Earth is
is the circumference of its great circle? 6380 km.)
27 . Find the total surface area of a hemisphere with
radius 8 m. E. Surface Areas of Spherical Sections

28 . The surface of this spherical 35 . What is the surface area of a spherical cap with
experiment chamber is made height 3 cm taken from a sphere with radius 8 cm?
from 12 congruent segments. 36 . Verify the formula for the surface area of a sphere,
Each segment has an area of regarding its whole surface as a cap.
0.27 m2. What is the diameter
of the sphere? 37 . A sphere is cut by a plane that intersects a radius
perpendicularly at its midpoint. Find the ratio of
29 . The standart NBA basketball
the surface areas of the minor and major caps.
size is 75 cm. This is the
circumference of its great 38 . What is the outer surface
circle. Find the surface area of area of this bowl? It is
a basketball complying with only 3 cm lower than a
these standards, to the nearest hemisphere with radius
square centimeter. 11 cm.

206 Solid Geometry


39 . 46 .

Find the approximate area of the Earth's surface


which is bounded by the tropics of Capricorn and
A panel with a circular hole of diameter 24 cm is Cancer if they have the latitudes 23.5° S and 23.5°
put on a rusty iron sphere with 15 cm radius. N respectively. (The radius of Earth is 6380 km.)
Then the part of the sphere which fits through
the hole is spray painted. Find the area of the 47 . A spherical segment of two bases has a height of
surface painted. 3 cm. If the segment belongs to a sphere with radius
7 cm, find the area of the zone of the segment.
40 . The perimeter of the cap of this
spherical teapot is 2ò51 cm. 48 . Two planes 2 m apart from each other cut a sphere
Find the surface area of the cap if with radius 10 m. The center of the sphere is the
the radius of the sphere is 10 cm. center of the spherical segment intercepted
between them. Find the total area of the segment.
41 . What is the area of the section of the Earth’s 49 . In a sphere of 12 cm radius, a base area of a
surface which lies above the 30th North parallel? spherical segment with two bases is 108 cm2 and
(The radius of Earth is 6380 km.) the altitude of the segment is 14 cm. Find the
42. The depth of this spherical cap total area of the segment. y

is 2 cm and its edge perimeter
is 16 cm. Find the total area of the cap. 50 . Find the surface area of the 10 c 30°
30°

m O
43. The radius of a sphere is 14 cm. How many solid generated by a complete
 revolution of the green circular
centimeters away must we put a spotlight source
sector around the y-axis.
from this sphere so that a quarter of the sphere’s
surface is illuminated? 51 . Find the surface area of the spherical cone
44 . Find the area of a spherical zone if it is 5 cm high generated by rotating a 60° sector of a circle with
and the radius of the main sphere is 11 cm. 8 cm radius around one of its sides.

45 . 52 . A 36° wedge is cut out of a sphere with radius


5 m. Find the total surface area of the wedge.

F. Volume of a Sphere

53 . Find the volume of a sphere with radius 3 cm.


The figures above show a spherical zone with
54 . Find the volume of a sphere with diameter 4ñ3 cm.
altitude r and a hemisphere of the same sphere
with radius r. Compare their areas. 55 . Find the volume of a hemisphere with radius 8 cm.

Solids of Revolution 207


56 . A double cone is inscribed in a 65 . The average diameter
cylinder as shown. Find the volume of a billiard ball is
of the space inside the cylinder 2r 57 mm. Find the
but outside the double cone, in volume of fifteen
r
terms of r. balls, to the nearest
cubic centimeter.
57 . Compare the volumes of a hemisphere and a cone
if they have the same base area and height. 66 . A steel trader has to fill
an order for 100 000
58 . What can you say about the sum of the volumes steel ball bearings
of a hemisphere and a cone, if the cone has the with diameter 12 mm.
same base area and height as the hemisphere? What is the total
weight of the metal required to fill this order, if
59 . A right circular cylinder has the same radius as a steel weighs 7.8 g/cm3?
sphere. If the volumes are also the same, compare
67 . The surface area of this
the height of the cylinder to its radius.
wooden sphere is 100 cm2.
What is its volume?
60 . We have a hemispherical
teacup with diameter 9 cm.
How much tea can it hold, to
68 . The volume of a sphere is 36 m3. Find its surface
the nearest cubic centimeter?
area.

61 . The adjacent quarter circle with A


69 . An Eskimo used 4.5 m2 of
6 cm radius is rotated around
ice to build his hemispheric
side AB through 90°. Find the
igloo on the snow. This is
volume of the generated solid. B C the outer surface of the
62 . The watermelon in the igloo. Find the volume of
adjacent picture has a the igloo to the nearest
spherical shape. What is the cubic meter if the thickness
volume of the watermelon, of the ice walls is 20 cm. (Ignore the entrance.)
to the nearest liter, if its
70 . The radius of a sphere is increased by 26%. By
diameter is 18 cm?
what approximate factor does the volume change?
63 . The diameter of a gold atom is 2.8 x 10–7 mm.
Find the volume of a single atom, in cubic 71 . An ice cream company sells
millimeters. boxes of ice cream with
dimensions 25 cm x 10 cm x
64 . A granite Shahi cannonball has 16 cm, with a scoop included
diameter 88 cm and the density to make ice cream balls with
of granite is approximately diameter 4 cm. The company claims that the box
2.7 g/cm3. Find the approximate contains nearly 120 scoops of ice cream. Is this
weight of a Shahi cannonball. true?

208 Solid Geometry


72. The volume and the area of a sphere are equal to 79 . A solid cone with radius 4 cm and height 54 cm

each other in magnitude. A point P is taken 5 is melted down and recast as a solid metal ball.
units away from the center. If a tangent is drawn What is the radius of the ball?
from P to the sphere so that the point of tangency
is Q, find PQ.
73 . G. Volume of Spherical Sections

80 . What is the volume of this


spherical cap with height
3 cm if it is taken from a
A cylindrical can with diameter 20 cm is partially sphere with radius 7 cm?
full of water. Then three identical spheres with 3 cm
radius are placed in the water. If the water covers 81 . Find the volume of this spherical
the balls, by how much does the water level rise?
cap-shaped large bowl, to the
74 . Remzi wants to calculate the nearest liter. The top of the
diameter of an iron sphere. He
bowl is 48 cm wide and the
submerges the sphere in the water
main sphere has diameter 60 cm.
in this cylindrical container with
radius 8 cm. If the water level in
the cylinder rises 4.5 cm, find the 82 . Find the volume of the water in
radius of the sphere. this fishbowl in the shape of a
spherical segment of one base.
75 .
The water is 15 cm deep and
the radius of the bowl is 12 cm.
Round your answer to the
nearest liter.
A copper sphere with radius 6 cm is melted and
recast as cylindrical disks of 8 cm diameter and
6 mm thickness. How many disks can be produced. 83 . What is the volume of this spherical
cap-shaped bowl, if the mouth of
76 . A cube is inscribed in a sphere. Find the volume of
the bowl is 12 cm wide and the
the sphere if the volume of the cube is 216 cm3.
main sphere has 10 cm radius.
77 . The internal and external radii of a metal spherical
shell are 9 cm and 6 cm. Find the volume of metal
84 . Find the volume of this spherical
in the shell.
cap-shaped vase, to the nearest cubic
78 . A metal ball 10 cm in diameter weighs 4 kg. Find
centimeter. The 24 cm wide mouth is
the weight of a shell which is 5 cm thick and has
5 cm above the center of the main
external diameter 50 cm, if it is made from the
sphere.
same metal.

Solids of Revolution 209


85 . The Golden Arena is a sports 91 . A fishbowl in the shape of a
compound in Stockholm built spherical segment of two
in the shape of a spherical cap. bases is full of water to a
What is its volume, to the height of 9 cm. A small fish
nearest cubic meter, if the 4 cm below the water surface
is at the center of the main sphere. Find the
diameter of the sphere is 114 m and the center of
volume of the water if the fish is at most 20 cm
the sphere is 32 m above the base of the building?
from the glass of the bowl.

86 . The arc length around the cap 8pcm 92 . A 60° sector of a circle with 6 cm radius is rotated
shown opposite is 8 cm. Find around one of its sides. Find the volume of the
generated spherical cone.
its volume.
93 . A 30° sector of a circle with ñ3 radius is rotated
12 cm
around one of its sides. Find the volume of the
generated spherical cone.

94 . The shaded sector in the


y
87 . This bowl is 9 cm deep and its figure is a 36° sector of a
circle with center O and
volume is 162 cm3. Find the
radius 10 cm. If it is rotated
perimeter of the mouth. 36° O
around the y-axis as in the
78°
figure, what is the volume
88 . The two radii of a spherical segment of two bases of the generated solid?
(sin12°  0.2, sin24°  0.4,
are 4 cm and 8 cm. If the height of the segment
cos12° 0.98, cos24° 0.91)
is 6 cm, find its volume.
95 . A spherical wedge has a 40° central angle. Its
89 . Find the volumes of the possible spherical segments sphere has radius 15 cm. Find the volume of the
(major and minor) bounded by two parallel wedge.

planes if one plane is 1 cm and the other is 3 cm 96 . The total area of a spherical wedge taken from a
from the center of the sphere, given that the sphere with radius t is 1.5t2. Find the volume of
radius of the sphere is 5 cm. the wedge in terms of t.
97. We name the set of points in a cube with its

90 . The depth of the dipper in the internal body as C, and a sphere as S. The cube
adjacent picture is 6 cm. The has edge length a and the radius of the sphere is r.
dipper is a segment from a The sphere and the cube are concentric and they

sphere and its lower and upper a a 2


satisfy the relation <r .
bases are respectively 10 cm2 and 20 cm2. Find 2 2
the volume of the dipper. Identify the set R if it is defined as R = S – C.

210 Solid Geometry


CHAPTER SUMMARY 2
 If we rotate a planar figure around an axis in space, we  The general volume formula for all frustums of cones and
get a solid. A solid generated in this way is called a solid 1
of revolution. pyramids is Vf r u s t u m = h( A1 + A2 + A1A2 ) .
3
 A generatrix together with a directrix produce a cylindrical
 If we cut a cone with a plane it produces one of several
surface.
significant sections: circle, ellipse, parabola and hyperbola.
 A segment of the generatrix bounded by two planes is
 The sphere is the most perfect known figure.
called an element of the cylinder.
 Every point on a sphere is equidistant from its center.
 A cylinder is the union of a cylindrical surface and two bases.
 This distance is called the radius.
 In a circular cylinder, the axis is the line segment joining
 The intersection of a sphere with a plane or another
the centers of the bases.
sphere is a circle.
 The axial section for a right cylinder is the plane
 If we cut a sphere with a plane, two different segments of
containing the axis.
a common base are formed. The surface of either segment
 A section of a cylinder is perpendicular to all its elements.
is called a spherical cap.
 The surface area of any cylinder is
 If we cut a sphere with two parallel planes, then between
A = 2Abase + Pright section . lelement .
them a spherical segment of two bases is formed. The
 The surface area of a circular right cylinder is surface of the sphere belonging to this segment is called
A = 2r(r + h) . a spherical zone.
 The volume of any cylinder is V = Abase . h .  A hemisphere is a half sphere formed by a plane that cuts
 The volume of a circular right cylinder is V = r2h . through the center of a sphere.
 The volume of an oblique cylinder is V = Abase . l . sin a.  A spherical cone is a solid of revolution generated by a
 The vertex of a cone is called its apex. sector of a circle rotated around one of its sides.
 Cones can be classified according to their base shape and  A spherical wedge is a portion of a sphere separated by
inclination. two great semicircles.
1  A lune is the spherical surface of a spherical wedge.
 The lateral area of a circular cone is A = PB a s e . l and  The surface area of a sphere is A = 4r2.
L
AL = rl. 2  The surface area of a spherical cap and a spherical zone
 The total surface area of a circular cone is A = 2r(r + 1). is A = 2rh.
 A frustum of a cone is the portion of a main cone which  The surface area of a spherical sector is A = 2r2(2r + r1).
is contained between two planes parallel to the base of
the cone. 2   
 The surface area of a spherical wedge is A =  r  +1.
 The lateral area of a right circular frustum is 90° 
1 4
A = l ( P P ) .  The volume of a sphere is V =  r 3 .
L 1 2
2 3
 The total surface area of a circular frustum is  The volume of a spherical segment of one base is
A = l( r1 + r2 ) + ( r1 + r2 ) .
2 2
1
  2
 h ( 3 r – h) .
3
 The volume of a circular cone is one-third of the volume of
 The volume of a spherical segment of two bases is
a cylinder with the same base and height:
1 2 2 2
1  h( h + 3r1  3r2 ) .
Vc o n e = Vc y l i n d e r . 3
3
2 2
 The volume of a spherical sector is r h .
1 2
 The volume of a circular cone is Vc o n e =  r h . 3
3
a 3
1  The volume of a spherical wedge is r .
2 2
 The volume of a frustum of a cone is  h( r1 + r1 r2  r2 ) . 270°
3

Solids of Revolution 211


14. Which two things together create a conic section?
Concept Check
15. Can a triangle be a conic section? What about a rectangle?
1. What is a solid of revolution? 16. How can we generate a sphere from a circle?
2. Which two things are needed to generate a cylindrical 17. What happens to the circle of intersection as the
surface? intersecting plane of a circle gets closer to its center?
3. Name four different types of cylinder.
18. Describe the possible relative positions of a sphere and a
4. The slant height of a cylinder is greater than its height.
plane.
What can you say about this cylinder?
19. The two spherical caps created when a plane cuts
5. Which two-dimensional shape can help us to prove the
through a sphere are congruent. What can you say about
formula for the lateral area of a cylinder?
the circle of intersection?
6. The axis and an altitude of a circular cone are congruent.
20. What is the difference between a spherical cap and a
What can you say about this cone? What about its ele-
spherical segment?
ments?
7. How can we find the volume of a cone using the volume 21. Give examples of things with the following shapes in the
of a cylinder? world around us: a sphere, a spherical segment of two
bases, a wedge.
8. Which shapes are in the net of a circular right cone?
9. How many faces does a circular frustum have? 22. How many times is the surface area of a sphere greater
than the area of its great circle?
10. Which shapes are in the net of a circular frustum?
11. How can we find the volume of a cone using the volume 23. State the volume formulas for a sphere, cone and cylinder.
of a pyramid? 24. Which two kinds of solid can help us to prove the formula
12. How many frustums can we cut from a cone? for the volume of a sphere?
13. What is the relation between the volume of a cone and the 25. Which things do we need to know to calculate the area of
volume of a cylinder with the same base area and height? a spherical zone?

Other Curved Solids


An ellipsoid is a solid shape whose planar cross-sections are all ellipses. Its three
axes intersect at the center of the ellipsoid. Each axis is perpendicular to the other two
axes, and the ellipsoid is symmetrical around all three axes. The longest axis is called
major axis, the other ones are medium axis and minor axis. The respective axis lengths
in the adjacent figure are 2a, 2b, and 2c.
Because of these different axis lengths, a triaxial ellipsoid can not be a solid of
revolution. Beach pebbles are usually ellipsoid solids.

A spheroid is a special case of an ellipsoid if it has two axes of equal length.


Therefore, a spheroid is a solid of revolution generated by an ellipse. If the longest
axis is the different axis, it is called a prolate spheroid, like a rugby ball. If the
shortest axis remains distinct, it is an oblate spheroid. Many planets, including the
Earth, are oblate spheroids.
Examples of prolate and oblate spheroids

An ovoid (from the Latin word "ovum" meaning egg) is a solid figure which is
generated by rotating a convex oval curve about one of its axes of symmetry. An oval
curve has no certain definition but it is like a flattened circle at one or two points, but
not like an ellipse. An ovoid has at least one axis of symmetry.
An egg An ovoid candle Another ovoid
A simit bread ring
A torus is a solid of revolution which is generated by rotating a closed has the shape of a
curve (usually a circle) around an axis outside the curve. A polyhedral torus.
torus has polygonal faces.
A torus

212 Solid Geometry


CHAPTER REVIEW TEST 2
1. Find the volume of a cylinder with radius ñ2 cm 5. If we make four cones from
 the sectors of the circle at the A
and height cm. B
2 98° 92°
BB right, which one will be the
A) 4 cm3 B) 2 cm3 C) 27 cm3 88° 82°
tallest?
D C
D) 64 cm3 E) 4 cm3 CC
A) A B) B C) C D) D
E) They will all be the same.
2. A road roller's barrel is 1.6 m
wide and 1 m high. Find the
area of asphalt that can be rolled 6. What is the approximate ratio of the lateral area
with 30 rotations of the barrel. of a cylinder to the area of its axial section?
CC
BB
A) 8 B) 12 C) 15 D) 18 E) 24 A) 2.75 B) 2.89 C) 3.14 D) 3.65 E) 6.28

3. A manufacturer wants to make two cylindrical 7. The 1 m x 4.5 m x 2 m


containers. The radius of the first container is rectangular dumper of
twice the radius of the second, but the height of the truck shown above
the second container is twice the height of the was half full of sand.
first. Compare their volumes. When it dumped its load,
AA the sand made a conical pile on the ground with
A) The first container is larger.
diameter 3 m. Find the height of the pile.
B) The second container is larger. DD
3 4 6 9
C) They are the same. A) 1 m B) m C) m D) m E) m
   
D) The radii must be given.
E) The heights must be given.

8. What is the total 2


volume of this
4. The outer diameter of a standard
composition of two
CD is 12 cm, and the diameter of 9
cones?
the central hole is 8ñ3 mm. If the
thickness of the CD is 1.25 mm,
4
what is its volume?
BB
BB
A) 3.32 cm3 B) 4.44 cm3 C) 3 cm3 A) 40 cm3 B) 36 cm3 C) 33 cm3
D) 2ñ3 cm3 E) 2.24 cm3 D) 27cm3 E) 24cm3

Chapter Review Test 2 213


9. Revolving doors are often 13. An ice cream manufacturer
installed in buildings to sells ice cream in conical
reduce heating costs. This frustum cups. The radii of the
four-panel revolving door has bases are 5 cm and 6 cm. Find
2 m radius and 2 m height. the height of a cup if it holds 182 cm3 of ice
Find the volume of air enclosed in two sections.
cream.
CC
BB
A) 2 m3 B) 3 m3 C) 4 m3 D) 6 m3 E) 8 m3 A) 8 cm B) 6 cm C) 5 cm D) 4 cm E) 3 cm

10. A conical tank with the


dimensions shown at the 14. A sphere and a cylinder have the same radius r. If
right is being filled with 3 dm their volumes are equal, what is the relationship
water. Express the volume between the height h of the cylinder and r?
of water in the tank as a r EE
9 dm 1 3
function of the depth h. A) h = 4r B) h=  2r C) h=  2r
h 2 2

EE 3r 4r
D) h= E) h=
1 4 3
A) f(h) = h3 B) f (h)= h3 C) f(h) = 9h3
3
1
D) f(h) = 3h3 E) f (h) = h3
27

15. Find the surface area of a cone whose base


perimeter is 30 m and volume is 600 m3.
EE
11. Which solid below does not have an edge?
DD A) 320 m2 B) 360 m2 C) 400 m2
A) cone B) frustum of a cone C) cylinder D) 420m2 E) 480m2
D) sphere E) hemisphere

12. The radius of a planet is 16. A spherical segment of two bases has a height of
9000 km. Find the volume of 17 cm. The radius of the sphere is 13 cm. One
this planet, in cubic kilometers. base radius of the segment is 12 cm. What is the
BB radius of its other base?
AA
A) 945  109 B) 972  109 C) 945  106 A) 5 cm B) 6 cm C) 8 cm
D) 972  10 6
E) 981  10 12
D) 9 cm E) 10 cm

214 Solid Geometry


CHAPTER 3

ANALYTIC ANALYSIS
OF LINES
1 COORDINATE SYSTEM AND ANALYTIC PLANE
French philosopher René Descartes (1596 - 1650) can be accepted as the establisher of
analytic geometry. He saw in his dream that he had been given a magical key which opens
the gate of the treasures of universe enlightening the fundamentals of sciences. He
understood that algebra can be applied to the geometry and he left many works about ana-
lytic geometry.
Analytic geometry can be defined as algebraic analysis to demonstrate the geometrical
concepts and figures, or illustration of algebra by the help o geometrical concepts and figures.
Analytic geometry has many applications in different branches of science and makes many
easiness in solving problems.

A. ANALYTIC ANALYSIS OF POINT


1. Basic Concepts
Definition Analytic Plane
Two number lines which are perpendicular to each other at the origin (point zero) is called
rectangular coordinate system, and the plane used for this is called the analytic plane.

On analytic plane, the horizontal line y - axis


is called x - axis and the vertical line 2nd quadrant 1st quadrant
is called y - axis. 6 (+) direction
5
These axes divide analytic plane into 4
4 quadrants (regions) as shown in the 3
figure 1. 2
(–) direction 1 (+) direction
The point O(0, 0) is called the origin O(0,0)
x - axis
–6 –5 –4 –3 –2 –1 1 2 3 4 5 6
of the analytic plane. –1
–2
On analytic plane, we can specify –3
exact place of any point by using its –4
(–) direction –5
x and y - coordinates.
–6
3rd quadrant 4th quadrant

Figure 1

216 Analytic Analysis of Lines and Circles


Let’s take a point P in any quadrant of the y
analytic plane and draw perpendicular line P(a, b)
b
segments from P to x and y - axes which meet
the coordinate axes at a and b.
x
O(0,0) a
The real number a on the x - axis in called the
x - coordinate or abscissa of point P.

The real number b on the y – axis is called the


Figure 2
y - coordinate or ordinate of point P.

The ordered pair (a, b) is called the coordinates of the point P and is denoted by P(a, b).

EXAMPLE 1 7
Plot the points A(1, –2), B(–3, –4), C(–2, 1), D(3, 2) and E( − , 4) on the analytic plane.
2

Solution
y
E(– 7 , 4)
2
4
3
2 D(3, 2)
C(–2, 1)
1
–4 1 2
x
–3 –2 –1 3
–1
–2 A(1, –2)
–3
–4
B(–3, –4)

The ordinate of every point on the x - axis is y


(0, y)
zero, the abscissa of every point on the y - axis (0, 5)
is zero, and the coordinates of the origin are (0, 4)
(x, 0) (0, 3)
(0, 0). (0, 2)
(0, 1)
(3 )
(4 0)
(5 0)
)
(2 )
,0

,0
,0

,
,
(1

x
O
(– 0)
(– 0)
(– , 0)
0)
(– 0)

(0, –1)
4,
3,

1,
5,

2

(0, –2)
(0, –3)
(0, –4)
(0, –4)

Figure 3

Coordinate System and Analytic Plane 217


EXAMPLE 2 Plot the points K(4, 0), L(0, 5), M(–3, 0) and N(0, –2) on the analytic plane.

y
Solution
5 L(0, 5)
4
3
2
M(–3, 0) 1 K(4, 0)
x
–3 –2 –1 1 2 3 4
–1
– 2 N(0, –2)

Given a point P(a, b) in the coordinate plane y

I. a > 0, b > 0 when P is in the 1st quadrant.

II. a < 0, b > 0 when P is in the 2nt quadrant. P(–, +) P(+, +)


rd
III. a < 0, b < 0 when P is in the 3 quadrant.
x
O
IV. a > 0, b < 0 when P is in the 4th quadrant.
P(–, –) P(+, –)

EXAMPLE 3 If the point A(a, a ⋅ b) is in the 2nd quadrant of the analytic plane, in which quadrant is the
a
point B( a2 , ).
b

Solution If (x, y) is in the 2nd quadrant, then x < 0, y > 0


A(a, a ⋅ b) is in the 2nd quadrant so a < 0 and a ⋅ b > 0 then b < 0 also.
a a
For the point B( a2 , ), a2 > 0 and > 0 .
b b
st
Hence B is in the 1 quadrant.

EXAMPLE 4 If the points A(a, a – 2b) and B(–3, b) are coincident find their coordinates and the quadrant
in which they are.

Solution A(a, a – 2b) = B(–3, b)


a = –3 and a – 2b = b so b = –1 then A(–3, 1) is in the 3rd quadrant.

218 Analytic Analysis of Lines and Circles


EXAMPLE 5 The sides of the rectangle are parallel to the coordinate y
axes, and its length is 4 times its width. If the perimeter
A G D(8, 2)
of the rectangle is 50 units and D(8, 2).
Find the coordinates of the points A, B, C. H F
x
O

Solution The sides of the rectangle are parallel to x and


y - axes, so they are perpendicular to the axes at points B E C

E, F, G, H.
Here, AD = 4 ⋅ DC and Figure 4

P = 2 ⋅ (AD + DC) = 2 ⋅ (4 ⋅ DC + DC) = 10 ⋅ DC = 50 units


So DC = 5 units.
DC = DF + FC and DF = 2 units
so FC = DC – DF = 5 – 2 = 3 units
AD = 4 ⋅ DC = 4 ⋅ 5 = 20 units
similarly AG = AD – GD = 20 – 8 = 12 units
then the coordinates are A(–12, 1), B(–12, –3) and C(8, –3).

Check Yourself 1
5 1 7
1. Plot the points A(–3, 0), B( , 2), C(0, − ), D( −2, − ) and E(–5, –4) on analytic plane.
2 4 2
2. Find the coordinates of the points whose distance to the x - axis is 5 units and to the
y - axis is 2 units.
3. Draw the triangle whose vertices are A(1, 7), B(–2, 3) and C(2, –3) on analytic plane.
4. If the point M(m, n) is in the 3rd quadrant of the analytic plane, find in which quadrant
the point N(–n, m) is.
5. If the point P(m + 1, m – 6) is in the 4th quadrant of the analytic plane, find the possible
integer values of m.

2. Distance Between Two Points


We will use x0, x1, x2, ... and y0, y1, y2, ... to denote the abscissa and the ordinate of points
respectively in the analytic plane.

Theorem
The distance between points A(x1, y1) and B(x2, y2) is

AB = ( x2 − x1 )2 +( y2 − y1 )2

Coordinate System and Analytic Plane 219


Proof In the figure 5, ∆ABC is a right triangle. y

AC = x2 – x1 B
y2
BC = y2 – y1
by the pythagorean theorem y2 – y1

AB2 = AC2 + BC2 A


y1
x2 – x1 C
2 2 2
AB = (x2 – x1) + (y2 – y1) O x1 x2
x

then AB = ( x2 − x1 )2 +( y2 − y1 )2 or
Figure 5
AB = ( x1 − x2 )2 +( y1 − y2 )2

EXAMPLE 6 Find the distance between A(3, 0) and B(–2, –3).

Solution
AB = ( x2 − x1 )2 +( y2 − y1 )2 = (− 1− 3)2 +(− 3 − 0) 2 = (− 4) 2 +(− 3) 2 = 25 = 5 units.

EXAMPLE 7 Show that ∆ABC with the vertices A(–2, 2), B(1, 5) and C(4, –1) is an isosceles triangle.

Solution Let find the length of the sides of ∆ABC. y


B
AB = 5
(1 + 2)2 + (5 − 2)2 = 9 + 9 = 3 2

AC = (4 + 2)2 +( −1 − 2)2 = 36+9 = 3 5 . A 2

4
BC = (4 − 1)2 +( −1 − 5)2 = 9+ 36 = 3 5 . x
–2
–1
We see that two sides are equal. C

AC = BC so ∆ABC is isosceles.

EXAMPLE 8 Given A(a, 2), B(3, 4) and (–2, 1). If A is at the same distance to the points B and C, find a.

Solution AB = AC

(3 − a2 + 22 = ( a + 2)2 + 12

9 − 6 a + a2 + 4 = a 2 + 4a + 4 + 1

10 a = −8
4
a=
5

220 Analytic Analysis of Lines and Circles


EXAMPLE 9 Find the ordinate of the point on the y - axis which is equidistant to the points A(– 4, 0) and
B(9, 5).

Solution PA = PB y

( −4)2 + y2 = 92 + ( y − 5)2 P(0, y)


2 2 2 2
(– 4) + y = 9 + (y – 5)
5 B(9, 5)
16 + y2 = 81 + y2 – 10y + 25
10y = 90
x
y=9 A(–4, 0) O 9
the point is P(0, 9).

Check Yourself 2
1. Find the distance between the points A(2, –1) and B(–2, 2).
2. Find the length of the sides of triangle MNP whose vertices are at the points M(–1, 3),
N(–2, –3) and P(5, 1).
3. The points K(2, 1) and L(–6, a) are given. If KL = 10 cm find the possible values of a.
4. Α is a point on y - axis whose ordinate is 5 and B is the point (–3, 1). Compute AB.
5. Find the point on y - axis which is equidistant to the points A(–3, 0) and B(4, –1).

3. Midpoint of a Line Segment


Theorem
Let the points A(x1, y1) and B(x2, y2) be endpoints of the line segment AB, and let C(x0, y0)
be midpoint of AB then
x1 + x2 y1 + y2
x0 = and y0 = .
2 2

Proof Let’s take point C on AB such that AC = CB. y

In the figure 6, ∆CAK ≅ ∆BCD by A. S. A similarity


B
y2
theorem AK = CD and CK = BD
then, x0 – x1 = x2 – x0 and y0 – y1 = y2 – y0 y0 C
D

2x0 = x1 + x2 and 2y0 = y1 + y2 y1 A


K
x + x2 y + y2
x0= 1 and y0= 1 x
2 2 O x1 x0 x2

x1 + x2 y1 + y2
So C(x0, y0) = ( , ) Figure 6
2 2

Coordinate System and Analytic Plane 221


EXAMPLE 10 Find the coordinates of the midpoint of AB, if A(–1, –2) and B(–5, 4).

x1 + x2 −1 − 5 y + y2 −2 + 4
Solution x0 = = = −3, y0 = 1 = =1
2 2 2 2
So C(–3, 1) is the midpoint of AB.

EXAMPLE 11 Given the triangle with vertices A(0, 3), B(2, –3) and C(–2, –1).
Draw ∆ABC on analytic plane and find the lengths of the medians.

Solution Let D, E and F be midpoints of the sides of ∆ABC


y
as shown in the Figure 7.
The coordinates of D are A(0, 3)

−2+ 2 −3 − 1
xD = = 0 and = −2, D(0, − 2), so
2 2
E(–1, 1)
Va = AD = (0 − 0)2 +(3+ 2) 2 = 5 units
–2 O F(1, 0)
The coordinates of E are x
–1 1 2
0+ 2 3 −1 –1
xE = = −1 and =1, E( −1, 1) so C(–2, –1)
2 2 –2

D(0, –2)
Vb = BE = (2+1)2 +(–3 – 1) 2 = 5 units and
–3
the coordinates of F are B(2, –3)
0+ 2 3−3 Figure 7
xF = =1 and yF = = 0, F(1, 0) so
2 2

Vc = CF = ( −2 − 1)2 +(–1 – 0) 2 = 10 units.

EXAMPLE 12 Given the triangle ABC with the vertices A(–2, –2), B(1, 8) and C(6, 2). If the points D and
BC
E are midpoints of AB and AC, respectively, show that ED = .
2

Solution First, let’s find the coordinates of D(a, b) and E(c, d). Points D(a, b) and E(c, d) are the
midpoints of AB and AC (Figure 8) so their coordinates are
x1 + x2 −2+1 1⎫
a= = =− ⎪
2 2 2⎪ 1
⎬ ⇒ D( − , 3)
y + y2 −2+8 ⎪ 2
b= 1 = =3 ⎪
2 2 ⎭

222 Analytic Analysis of Lines and Circles


x1 + x2 −2+6 ⎫ y
c= = =2⎪
2 2 ⎪
⎬ ⇒ E(2, 0) 8
B(1, 8)
y1 + y2 −2+ 2 ⎪
d= = = 0⎪
2 2 ⎭

Now, let’s find the length of ED and BC by using


distance formula, and then compare their lengths D(a, b)
C(6, 2)
2
1 ⎫
ED = (2+ )2 +(0 − 3) 2 ⎪ –2
2 x
⎪ O E(c, d) 6
⎪ –2
25 61 61 ⎪⎪
= +9= = A(–2, –2)
4 4 2 ⎬

2
BC = (6 − 1) +(2 − 8) 2 ⎪
⎪ Figure 8

= 25 + 36 = 61 ⎪⎭

BC
Hence, ED = .
2

Note
Let points A(x1, y1), B(x2, y2), C(x3, y3) and D(x4, y4) be D(x4, y4) C(x3, y3)
vertices of a parallelogram ABCD, and P(x0, y0) be
intersection point of diagonals.
P(x0, y0)
Since P(x0, y0) is the midpoint of diagonals.
x1 + x3 x2 + x4
x0 = and x0 = then x1 + x3 = x2 + x4 A(x1, y1) B(x2, y2)
2 2
y1 + y2 y2 + y4
y0 = and y0 = then y1 + y3 = y2 + y4
2 2

EXAMPLE 13 KLMN is a parallelogram and K(2, a), L(1, 4), M(b, 3) N M

and N(3, 2) are the vertices. Find a – b.

Solution The midpoint of KM is also midpoint of NL so


2+b=1+3 and a+3=4+2 K L
b=2 a=3

a–b=3–2=1

Coordinate System and Analytic Plane 223


Check Yourself 3
1. Find the coordinates of the midpoint of AB whose endpoints are A(a + 1, 4 – 2b) and
B(3 – a, 2b – 3).
2. Given ∆ABC with the vertices A(2, 5), B(–2, 3) and C(4, –1). Find the length of the
median passing through A.
3. The points A(–2, –3), B(3, –2) C(x, y) and D(–1, 3) are the vertices of the parallelogram.
Find the coordinates of C.

4. Coordinates of a Point Dividing a Line Segment in a Given


Ratio
a. Coordinates of a point dividing a line segment internally
Let AB be a line segment and C be a point on AB. Point A C B
C divides AB internally and the ratio of the distances of
CA AC
=k
the point C to A and B respectively is = k. (Figure 9) CB
CB
C divides AB internally
If the ratio k is given, the coordinates of C can be found
by using the following theorem. Figure 9

Theorem
Let A(x1, y1) and B(x2, y2) be endpoints of AB and C(xc, yc) be a point on line segment AB
CA x + k ⋅ x2 y + k ⋅ y2
such that = k, ( k ∈ \ + ) then the coordinates of C are xc = 1 and yc = 1 .
CB 1+ k 1+ k

Proof In the figure 10 y

∆AKC ∼ ∆CHB (by A.A.A similarity theorem)


y2 B
AK KC AC
and = = =k
CH HB CB
y2 – yc
AK x – x1 KC yc − y1 x2 – xc
we need = c = k and = = k. yc C
CH x2 – xc HB y2 − yc
yc – y1
x – x1 yc − y1 y1
A
then c =k =k
x2 – xc y2 − yc xc – x1
xc – x1 = k ⋅ x2 – k ⋅ xc yc – y1 = k ⋅ y2 – k ⋅ yc x
O x1 xc x2
xc + k ⋅ xc = k ⋅ x2 + x1
yc + k ⋅ yc = k ⋅ y2 + y2
xc(1 + k) = x1 + k ⋅ x2
yc(1 + k) = y1 + k ⋅ y2 Figure 10
x + k ⋅ x2
xc = 1 y + k ⋅ y2
1+ k yc = 1
1+ k

224 Analytic Analysis of Lines and Circles


EXAMPLE 14 Given the points A(–3, 5) and B(9, –11). Find the coordinates of the point C which divides
CA
AB internally in the ratio = 3.
CB

CA
Solution =3
CB
x1 + kx2 −3 + 3 ⋅ 9 24
x0 = = = =6
1+ k 1+ 3 4
y1 + k ⋅ y2 5 + 3 ⋅( −11) 28
y0 = = = = −7
1+ k 1+ 3 4

so the point is C(6, –7).

Practical Way: 4a

3a a
C(x0, y0) divides AB internally
To find x0: From A to B for 4a, abscissa changes A(–3, 5) C(x0, y0) B(9, –11)
AC
=3
9 – (–3) = 12 units then from A to C for 3a, abscissa CB
12
will change ⋅ 3 = 9 units then Figure 11
4
x0 = –3 + 9 = 6

To find y0: From A to B for 4a, the change in ordinate is –11 –5 = –16 units then from A to
−16
C for 3a, ordinate will change ⋅ 3 = −12 units then y0 = 5 – 12 = –7.
4
So, the point is C(6, –7).

EXAMPLE 15 Given the line segment with the enpoints M(–2, 3) and
5a
N(1, –1). Find the coordinates of the point R which M R N
MR 2 2a 3a
divides MN internally in the ratio = .
MN 5

Figure 12
Solution R ∈ MN ⎫
⎪ MR 2
MR 2 ⎬ ⇒ RN = 3 = k
= ⎪
MN 5 ⎭
Coordinates of point R are.
2 4 2 7
−2+ ⋅1 − 3+ ⋅( −1)
xk = 3 = 3 =−4 yk = 3 = 3 =7
2 5 5 2 5 5
1+ 1+
3 3 3 3
4 7
so the point R is R( − , ) .
5 5
Coordinate System and Analytic Plane 225
Practical Way: 5a

R(x0, y0) divides MN internally; 2a 3a

To find x0: From M to N for 5a, abscissa changes


M(–2, 3) R(x0, y0) N(1, –1)
1 – (–2) = 3 units, then from M to K for 2a, abscissa will
3 6 6 4
change ⋅ 2 = units then x0 = −2+ = − . Figure 13
5 5 5 5
To find y0: From M to N for 5a, ordinate changes –1 – 3 = – 4 units, then from M to K for 2a,
−4 −8 8 7
it will change ⋅2 = units then y0 = 3 − = .
5 5 5 5
4 7
So R( − , ).
5 5

b. Coordinates of a point which divides a line segment externally

Let D be a point on a line AB but out of a line segment

AB, then D divides the line segment AB externally.


D A B
... ...
The ratio of the distances of D to the points A and B DA
= k (k Î IR+)
DA DB
respectively is = k. (Figure 14)
DB
Figure 14
If the value of k is given, the coordinates of D can be
found by using the following theorem.

Theorem
Let A(x1, y1) and B(x2, y2) be two points of a line AB and D(xd, yd) be a point on a line AB
DA
such that = k ( k ∈ \+ ) and D ∉ the line segment AB, then the coordinates of D are
DB
x1 – k ⋅ x2 y1 − k ⋅ y2
xd = and yd = .
1– k 1– k

Proof In the figure 15


∆APD ∼ ∆BHD (by A.A.A. similarity theorem)
AP PD AD
and = = =k
BH HD BD
AP x – x1 PD y − y1
we need = d = k and = d =k
BH xd – x2 HD yd − y2

226 Analytic Analysis of Lines and Circles


then y
xd – x1 yd − y1 D
=k =k yd
xd – x2 yd − y2
xd – x1 = k ⋅ xd – k ⋅ x2 yd – y1 = k ⋅ yd – k ⋅ y2 yd – y2
yd – y1
xd – k ⋅ xd = x1 – k ⋅ x2 B
yd – k ⋅ yd = y1 – k ⋅ y2 y2
H
xd(1 – k) = x1 – k ⋅ x2 xd – x2
yd(1 – k) = y1 – y ⋅ y2 A
y1 P
x1 – k ⋅ x2 y – k ⋅ y2 xd – x1
xd = yd = 1
1– k 1– k x
O x1 x2 xd

Figure 15

EXAMPLE 16 Given the line segment MN with the end points M(–2, 3) and N(4, 5). Find the coordinates
MK 3
of the point K which divides MN externally in the ratio = .
KN 2

Solution 3
−2 − ⋅4
MK 3 2 −8 M(–2, 3) K(4, 5) K(xk, yk)
= = k, xk = = =16
KN 2 3 1
1− −
2 2
3 9
3−
⋅5 −
yk = 2 = 2 =9
3 1
1− −
2 2
so the coordinates of point K are (16, 9).

Practical Way:
For xk: The change 4 – (–2) = 6 units from M 3a

to N. 6 ⋅ 3 = 18 units so xk = –2 + 18 = 16 a 2a
For yk: 5 – 3 = 2 units change from M to N. M(–2, 3) N(4, 5) K(xk, yk)

2 ⋅ 3 = 6 units so yk = 3 + 6 = 9 then K(16, 9)


Figure 16

EXAMPLE 17 In ∆ABC, AK = KB, KS = CS, B(1, 2) and


A
C(4, – 4) are given. Find the coordinates of T.
K
S

B T C

Figure 17

Coordinate System and Analytic Plane 227


In ∆ABC let us use menaleous theorem,
Solution
AK BT CS 1 BT 1 BT
⋅ ⋅ =1 ⇒ ⋅ ⋅ =1 ⇒ = 2.
AB TC SK 2 TC 1 TC

BT
We know the points B(1, 2), C(4, –4) and = 2 , it means T is the point which divides BC
TC
in two proportional parts.
If T is (x, y) then
x1 + k ⋅ x2 1+ 2 ⋅4
x= ⇒ x= ⇒ x=3
1+ k 1+2
y1 + k ⋅ y2 2 + 2 ⋅( − 4)
y= ⇒ y= ⇒ y = –2
1+ k 1+2
so coordinates of T are (3, –2).

Check Yourself 4

1. Given the line segment AB with the endpoints A(1, 5), B(4, 2). C is a point on line
AC 1
segment AB. If = find the coordinates of C.
CB 2
1
2. A point T divides LM externally in the ratio . If the coordinates of the endpoints are
15
L(–1, –5) and M(0, 3), find the coordinates of the point T.
3. The point D(2, –3) is dividing the line segment AB externally such that . If the coordinates
of B are (–1, 3) find the coordinates of the point A.
y
T

4. 2 In the figure, KS = 2 ⋅ ST, points K(–3, 0) and S(0, 2) are given


S
find the distance of point T to the origin.
x
K(– 3, 0) O H

228 Analytic Analysis of Lines and Circles


B. ANALYTIC ANALYSIS OF TRIANGLES
1. Coordinates of Centroid of Triangle
A
Let ∆ABC be a triangle with vertices A(x1, y1), B(x2, y2) and
C(x3, y3), and let D(a, b) be midpoint of BC
x + x3 y + y3
then a = 2 , b= 2 . G(x
2 2 0 ,y
0 )
If G(x0, y0) is the centroid (center of mass) of ∆ABC, B D(a, b) C

x2 + x3 Figure 18
x1 + 2 ⋅
GA 2 x + k⋅a 2 x + x2 + x3
= = 2 so x0 = 1 = = 1
GD 1 1+ k 1+ 2 3
y2 + y3
y1 + 2 ⋅
y + k⋅b 2 = y1 + y2 + y3
and y0 = 1 =
1+ k 1+ 2 3
x1 + x2 + x3 y1 + y2 + y3
Hence G(x0, y0) = G( , )
3 3

EXAMPLE 18 Find the coordinates of the centroid of ∆ABC whose vertices are A(1, 2), B(2, 3) and C(4, 0).

x1 + x2 + x3 y1 + y2 + y3 1+ 2+ 4 2+ 3+0 7 5
Solution The centroid of ∆ABC is G( , ) so G( , ) = G( , ).
3 3 3 3 3 3

EXAMPLE 19 The coordinates of vertices of ∆ABC are A(–a, –b), B(b, a) and C(2b, 2b) and its centroid is
3
G(1, ). Find the coordinates of the vertex A.
5
x1 + x2 + x3 −a + b + 2b 3b − a
Solution x0 = ⇒ 1= ⇒ =1 ⇒ 3 b − a = 3 ...(1)
3 3 3
y1 + y2 + y3 5 −b + a + 2b
y0 = ⇒ = ⇒ 5 = a+ b ⇒ b+ a = 5 ...(2)
3 3 3

From simultaneous solution of (1) and (2),


b = 2, a = 3 and A(–3, –2).

Coordinate System and Analytic Plane 229


Check Yourself 5
1. Find the coordinates of the centroid of ∆ABC whose vertices are A(1, 7), B(–2, –1) and
C(3, 0).
2. The centroid of ∆ABC with the vertices A(–1, 4), B(0, –3) and C(m, n) is G(1, 2).
Find m + n.
y

A
3. If AOB is an equilateral triangle. Find the coordinates of the
centroid.
x
O B(12, 0)

2. Area of Triangle Whose Coordinates of Vertices are Given


Theorem
Let A(x1, y1), B(x2, y2) and C(x3, y3) be coordinates y
y1 A(x1, y1)
of the vertices of ∆ABC, then the area of ∆ABC is
1
A( ∆ABC ) = | x1( y2 − y3 )+ x2( y3 − y1 )+ x3( y1 − y2 )|.
2 y2 B(x2, y2)
C(x3, y3)

D E F
x
O x2 x1 – x2
x1 x3 – x1
x3 – x2
Proof In the figure 19
A(∆ABC) = ∆(ΑBDE) + A(AEFC) – A(BDFC) Figure 19
and notice that all the polygons ABDE, AEFC and BDFC are trapezoid so by the area
formula of trapezoid, we can write
( y1 + y2 )( x1 − x2 ) ( y1 + y3 )( x3 − x1 ) ( y2 + y3 )( x3 − x2 )
A( ∆ABC ) =| + − |
2 2 2
1
= | y1⋅x1 − y1⋅x2 + y2 ⋅x1 − y2 ⋅x2 + y1⋅x3 − y1⋅x1 + y 3 ⋅x 3 −y 3 ⋅x1 −y 2 ⋅x 3 +y 2 ⋅x 2
2
− y3 ⋅x3 + y3 ⋅x2 |

1
A( ∆ABC ) = | x1( y2 − y3 )+ x2( y3 − y1 )+ x3( y1 − y2 )|.
2
230 Analytic Analysis of Lines and Circles
Practical Way:

Write the coordinates of the vertices in the following form

then by using the diagonals which are positively or negatively


1 x1 y1
A(DABC) = ×
2 x2 y2 denoted, write the formula
x3 y3
– + 1
x1 y1 A = |( x1 ⋅ y2 + x2 ⋅ y3 + x3 ⋅ y1 ) − ( x2 ⋅ y1 + x3 ⋅ y2 + x1 ⋅ y3 )|
– + 2
– + and then

1
A = | x1( y2 − y3 )+ x2 ( y3 − y1 )+ x3( y1 − y2 )|.
2
Notice that we got the formula which is exactly same with the one proven above.

EXAMPLE 20 Find the area of ∆ABC whose vertices are A(–2, 4), B(3, 2) and C(2, –3).

Solution Since A(–2, 4) = A(x1, y1)

B(3, 2) = B(x2, y2)

C(2, –3) = C(x3, y3) then


1
A( ∆ABC ) = | x1 ⋅( y2 − y3 )+ x2 ⋅( y3 − y1 )+ x2 ⋅( y1 − y2 )|
2
1 1 27
= | −2 ⋅(2 + 3)+ 3 ⋅( −3 − 4) + 2 ⋅(4 − 2)|= | −10 −21+ 4|= .
2 2 2

EXAMPLE 21 If the points A(–1, –3), B(0, –2a), and C(3a, a) are three different points on the same line,
find a.

Solution Since A, B, and C are on the same line (collinear), they do not form a triangle, that is

A(∆ABC) = 0, A(–1, –3) = A(x1, y1)

B(0, –2a) = B(x2, y2)

C(3a, a) = C(x3, y3) then


1
A( ∆ABC ) = | −1⋅( −2 a − a)+0+ 3a ⋅( −3+ 2 a)|= 0
2
3a − 9a +6 a2 = 0 ; − 6 a+6 a2 = 0 ; a(6 a −6) = 0 ; a = 0 or a=1
since B(0, –2a) and C(3a, a) are different points, a can not be zero, so a = 1.

Coordinate System and Analytic Plane 231


EXAMPLE 22 Find the area of the quadrilateral ABCD with the vertices y

A(–2, –1), B(3, –2), C(3, 5) and D(–3, 4) 5 C(3, 5)


4
D(–3, 4)

2
x
3 1 O 1 2 3
–1
A(–2, –1)
–2
B(3, –2)

Figure 20

Solution As it is seen in the figure 20, the diagonal BD divides the quadrilateral into two triangles ∆ABD
and ∆BCD. Areas of the triangles
1 24
A( ∆ABD ) = | −2 ⋅( −2 − 4)+ 3 ⋅(4+1) − 3 ⋅( −1+ 2)|= =12
2 2
1 42
A( ∆BCD ) = | 3 ⋅(5 − 4)+ 3 ⋅(4+ 2) − 3 ⋅( −2 −5)|= = 21
2 2
A(ABCD) = A(∆ABD) + A(∆BCD) = 12 + 21 = 33.

EXAMPLE 23 In the figure 21, BE = ED = DC, AC is perpendicular to y

y - axis, E(0, 3) and D(2, 0) are given. Find the area of


∆ABC. A C

Solution Let’s choose A(x1, y1), B(x2, y2) and C(x3, y3).

In ∆AEC, OD is parallel to AC and D is the midpoint of EC. O


x
D
1
so OD = ⋅ AC and OE = OA = 3 units that is A(0, 3).
2
On the other hand
E(0, –3)
⎧ x2 + 2 ⎫
⎪ 2 = 0 ⇒ x2 = −2 ⎪
⎪ ⎪
BE = ED ⇒ ⎨ ⎬ ⇒ B( −2, − 6) B
⎪ y2 +0 ⎪
⎪⎩ 2 = –3 ⇒ y2 = −6 ⎪⎭ Figure 21

⎧ 0+ x3 ⎫
⎪ 2 = 2 ⇒ x3 = 4 ⎪
⎪ ⎪
ED = DC ⇒ ⎨ ⎬ ⇒ C(4, 3)
⎪ 3+ y3 ⎪
= 2 ⇒ y3 = 3
⎩⎪ 2 ⎭⎪

232 Analytic Analysis of Lines and Circles


We got the vertices A(0, 3), B(–2, –6), C(4, 3) then the area of ∆ABC is
0 3

1 −2 −6 1 1
A= ⋅ = ⋅|(–6 − 6+12) −( −6 − 24+0)|= ⋅|30|=15 units.
2 4 3 2 2

0 3

Check Yourself 6
1. Find the area of ∆ABC with the vertices A(1, 7), B(–1, 3) and C(2, –4).
2. Find the area of the quadrilateral ABCD whose vertices are A(–2, 1), B(–1, –3), C(4, –1)
and D(3, 4).
3. If the points A(4, 5), B(–1, 1) and C(–2, k) are collinear, find k.
4. If the area of ∆ABC with the vertices A(3, 1), B(–1, –2) and C(m, 1) is 5u2, find possible
values of m.
5. The points M(1, k), N(3, 0) and P(–1, k + 1) are on the same line. Find k.

Coordinate System and Analytic Plane 233


EXERCISES 1.1
A. Analytic of Point 6. The points A(1, –2) and B(–1, 3) are given. Find
1. Plot the following points on analytic plane. the coordinates of the point P which is on the line
AP 3
a. A(2, 3) b. B(–3, 1) c. C(–3, 2) segment AB and = .
PB 2
d. D(5, –3) e. E(0, –4) f. F(–3, 0)
7. In the figure, A(1, 2)
2. Specify the quadrant of the following points. BP is the bisector of
P(x, y)
∠B, BC = 2 ⋅ AB,
a. A(–2, 7) b. B(ñ5, –ñ3)
A(1, 2) and C(–2, –4).
c. C(2ñ2, ñ3) d. D(–1, –ñ3) Find the coordinates B C(–2, –4)
of P.
3. Find the distance between the following pairs of
points given below. 8. If the points M(4, 3), N(1, 1), and P(k, –1) are
collinear, what is k?
a. A(–2, 1) and B(1, –3)
9. In the figure, D C
b. K(ñ3, 3) and L(–2ñ3, 0)
ABCD is parallelogram
c. M(–5, 4) and N(2, –1) AE = EB and AC is a P(x, y)
3 5 diagonal. If A(2, 1)
d. P( − , 2) and R( , –1) and C(–1, –5) are
A E B
2 2
vertices, find the coordinates of P.
e. D(t, 3) and E(t, 7)

f. S( 1+ 3 , 1 − 3 ) and T(
3 − 1 3 +1
)
B. Analytic Analysis of Triangles
,
2 2 2 2 10. Find the coordinates of the centroid of triangles
whose vertices are given below.
4. Given a quadrilateral ABCD whose sides are par-
a. A(0, 0), B(0, 5), C(3, 0)
allel to x and y - axis, if A(3, –2) and C(–4, –7) are
the coordinates of two opposite vertices of the b. A(–1, 1), B(2, 3), C(5, –2)
quadrilateral find the coordinates of the others.
11. G(3, –2) is the centoid of ∆ABC. The sum of the
5. Find the coordinates of the midpoints of a line abscissas of B and C is 5, of the coordinates is 6.
segment MN whose coordinates of endpoints are Find the coordinates of A.
given below.
12. Find the area of ∆ABC whose coordinates of the
a. M(–3, 4) b. M(ñ3, ñ2) vertices are given below.
N(1, 2) N(–2ñ3, –2ñ2) a. A(1, 5), B(–2, 3), C(4, –2)

c. M(n + 2, m – 1) b. A(–4, 2), B(0, –1), C(3, 0)


N(4 – n, 3 – m) c. A(1, ñ3), B(–3, 0), C(4, 0)

234 Analytic Analysis of Lines and Circles


13. If the area of ∆ABC with the vertices A(2, –1), 20. A(–2, –3), B(4, –1), C(3, 2) and D(–1, 4) are the
2
B(1, 4) and C(m, 0) is 3 unit , find m. coordinates of the vertices of ABCD. Find

a. the length of the sides

14. If the points M(4, 3), N(1, 1) and P(k, –1) are b. the coordinates of the midpoints of the sides
collinear, find k. c. the length of the diagonals
d. the area of ABCD

15. M(3, 2), N(1, 1), P(–2, –3) are the vertices of
∆MNP. Point K(m, n) is on the side MN (between 21. In the figure, OA = OB y
d
M and N). If A(∆MKP) = 2 ⋅ (∆KNP) find the P(2, 6) is the point on
P(2, 6)
coordinates of K. line d. Find abscissa of B

point A.
A
x
Miscellaneous Exercises O

16. If the midpoints of the sides of ∆ABC are D(1, 2),


E(–2, 1) and F(–1, –3), find the area of ∆ABC.
22. A quadrilateral with vertices A(–b, –a), B(b, –a),
C(b, a) and D(–b, a) is given. A point P(x, y) is
17. Specify the type of triangles whose vertices are taken on the same plane with ABCD. Show that
given, and compute their areas. PA2 + PC2 = PB2 + PD2
a. A(2, –2), B(–3, –1), C(1, 6)
b. A(–2, –ñ3), B(2, –ñ3), C(0, ñ3) 23. Points A(x1, y1), B(x2, y2), C(x3, y3) and D(x4, y4)
are given. If ABCD is parallelogram, show that
x1 + x3 = x2 + x4 and y1 + y3 = y2 + y4
18. In ∆ABC, A(–2, 1) is the vertex and G(4, –3) is the
centoid. Find the length of the median which
belongs to BC.
24. A triangle ABC with the vertices A(a, b), B(–a, b)
and C(b, –a) is given, (a ≠ 0, b ≠ 0).
19. In the figure, y
A Show that each one of the medians of ∆ABC
BA ⊥ AC,
divides the triangle into two equal parts.
AD = BD
D
B(–3, 0) and
D(0, 3) are given. 25. In the figure, C
x
Find B O C AB ⊥ AC and
D
a. the abscissa of C CD = DB.
b. the coordinates of the centroid B(–1, 5) and
C(5, –3) are given find AD. A B
c. the area of ∆ABC

Coordinate System and Analytic Plane 235


2 ANALYTIC ANALYSIS OF LINES
A. TRIGONOMETRIC RATIOS OF ANGLES
1. Trigonometric Ratios of Acute Angles in Right Triangles
A
the length of the side opposite α BC
sin α = =
the length of the hypothenuse AC

oppasite side to a
se
the length of the side adjacent to α AB e nu
cos α = = th
the length of the hypothenuse AC po
hy
the length of the side opposite to α BC
tan α = = a
the length of the side adjacent to α AB B C
adjacent side to a
the length of the side adjacent to α AB
cot α = =
the length of the side opposite to α BC Figure 22

A
Now, let us consider a right triangle ABC with the acute
angles α and β, that is, m(A) = α, m(C) = β a

From the definitions of trigonometric ratios given above we b


c
can write the following
c a
cos α = ; sin α = b
b b
B a C
Since α + β = 90°, β = 90° – α hence
Figure 23
a
cos β = = sin α, that is cos(90° – α) = sin α
b
c
sin β = = cos α, that is sin(90° – α) = cos α
b
Consequently, if α and β are complementary angles, then cosα = sinβ and sinα = cosβ.
Similarly tanα = cotβ and cotα = tanβ

3. Trigonometric Ratios of Some Special Angles


Trigonometric ratios of frequently used angles of 30°, 60° and B
45° can be calculated by using equilateral triangle and
isosceles right triangle. 45°
ñ2
To calculate the trigonometric ratios of the angle of 45° we use 1

an isosceles right triangle. From the figure, we get 45°


1 2 C 1 A
sin 45° = cos 45° = =
2 2 Figure 25

tan45° = cot45° = 1.

236 Analytic Analysis of Lines and Circles


In the figure, notice the equilateral triangle ABC with the length of one side 2 units. Altitute
of ∆ACB is ñ3 units. From one of the right triangles ABH and AHC, we get
3 A
sin60° = cos 30° =
2
1
cos60° = sin 30° = 30° 30°
2 2 2
1 3
tan 30° = cot 60° = =
3 3
60° 60°
B 1 H 1 C

Figure 26

3. Trigonometric Ratios on Unit Circle


Definition Unit Circle
The circle whose center is at the origin with the y
radius one unit, is called unit circle.
B(0, 1)
In the right triangle OMP,
P(cosa, sina)
PM PM
sin α = = = PM
OP 1 a A(1, 0)
x
A¢(–1, 0) M
OM OM
cos α = = = OM
OP 1
On unit circle the abscissa of the point P is cosα B¢(0, –1)
and the ordinate of the point P is sinα.
P(cosα, sinα).

In the right triangle OQA, y


QA QA B
tan α = = = QA a
OA 1 R(cota, 1)
P Q(1, tana)
In the right triangle ORB,
a
BR BR x
cot α = = = BR O A
OB 1
On unit circle,
the ordinate of the point Q is tanα
the abscissa of the point R is cotα.
Figure 24
Q(1, tanα); R(cotα, 1)

Analytic Analysis of Lines 237


4. Trigonometric Ratios of Obtuse Angles
Now, we shall see how we can state the trigonometric ratios of abtuse angles in terms of the
trigonometric ratios of acute angles.
y
In the figure, 0° < α < 90°
B(0, 1)
Since ∆POP′ ≅ ∆ROR′ we get
OP′ = OR′ and R P
180° – a T(1, tana)
cos(180° – α) = –cosα
a a
PP′ = RR′ and x
R¢ O P¢ A(1, 0)
sin(180° – α) = sinα

On the other hand,
since ∆TOA ≅ ∆T′OA we get
AT = AT′ and tan(180° – α) = –tanα
Figure 28
Conclusion
cos(180° – α) = –cosα
sin(180° – α) = sinα
tan(180° – α) = –tanα

EXAMPLE 24 Find the tangent values of the angles of 120°, 135° and 150°.

Solution tan 120° = tan (180° – 60°) = – tan 60° = – ñ3

tan 135° = tan (180° – 45°) = – tan 45° = –1

3
tan 150° = tan (180° – 30°) = – tan 30° = −
3

Following table shows the trigonometric ratios of some special angles.

Angles
0° 30° 45° 60° 90° 120° 135° 150° 180°
Ratio

sine 0 1 2 3 1 3 2 1 0
2 2 2 2 2 2
3 2 1 1 2 3
cosine 1 0 – – – –1
2 2 2 2 2 2
–
3 3
tangent 0 1 3 undefined – 3 –1 – 0
3 3
cotangent undefined 3 1 3 0 3 –1 – 3 undefined
–
3 3
(Table 1)

238 Analytic Analysis of Lines and Circles


Check Yourself 6
By using unit circle, find the tangent values of following angles.
1. 45° 2. 60° 3. 0° 4. 90°

B. SLOPE OF A LINE

1. Inclination and Slope of a Line


Definition Inclination of a line
Ýnclination of a line is the angle initialted by positive x - axis and terminated by that line.

Definition Slope of a line


Let α be the inclination of a line, then tangent value of α is called slope of the line where
α ≠ 90. It is denoted by m.
y
d
In the figure, the line d intersects with x - axis at A, and α is
the angle formed by x - axis and the line d.
The, α is the inclination of d and tanα is the slope of d. a
x
Slope = m = tanα O A

Figure 29

y y d y y
d d

a a
x x x x
O O O O

If d ^ Oy or d // Ox If d ^ Ox or d // Oy If 0° < x < 90° If 0° < x < 180°


then a = 0°, then m = tan90°, m = tana > 0°, m = tana > 0°,
m = tan0° = 0 m undefined Slope is positive Slope is negative
zero slope No slope

Analytic Analysis of Lines 239


EXAMPLE 25 Find the slopes of the lines whose inclinations are 30°, 45°, 60°, 120° and 150°.

Solution Inclination (α) Slope (m)


3
30° tan 30° =
3
45° tan 45° = 1

60° tan 60° = ñ3

120° tan 120° = – ñ3


3
150° tan 150° = −
3

EXAMPLE 26 Find the inclinations of the lines whose slopes are - 1 and 0.

Solution Since m = –1, tanα = –1


α = 135°
Since m = 0, tanβ = 0
β = 0°

EXAMPLE 27 In the figure, what are the slopes of d1 and d2. d1 y

Solution We have to calculate the tangent values of the angles made


by the lines d1 an d2 with the positive direction of x - axis. d2

Hence, m1 = tanα m2 = tanθ


= tan(180° – 60°) = tan(180° – 150°) 150° a
q 60°
x
O
= –tan60° = tan30°
3
= –ñ3 = Figure 30
3

EXAMPLE 28 In the figure, what is the slope of the line d. y


d

x
O 3

240 Analytic Analysis of Lines and Circles


Solution α is the inclination of d and m = tanα. y
d
since α + θ = 180°
m = tanα = tan(180 – θ) 2
a
q x
m = – tanθ O 3
2
m= − .
3

Check Yourself 7
1. What is the slope of a line whose inclination is 135°.
2. What is the inclination of a line whose slope is –ñ3.
y
d1
d2
3. In the given figure, find the slopes of the lines d1 and d2.

30°
x
–3 O

2. Finding Slope of a Line


If the inclination of a line is known, then slope of the line is tangent of that angle. Otherwise,
to find slope we need to know two points on the line.

Theorem
y2 − y1
The slope of a line passing through the points A(x1, y2) and B(x2, y2) is m = .
x2 − x1

Proof Let α be the inclination of the line d which is passing y


d
through the points A(x1, y1) and B(x2, y2).
B(x2, y2)
y2
As seen in the figure
y2 – y1
A(x1, y1)
m(∠ADO) = m(∠BAC) = α (Corresponding Angles) y1
x2 – x1 C
AC = x2 – x1 and BC = y2 – y1
Then, in the right triangle BAC. D a
x
BC y2 − y1 O x1 x2
tan α = = and m = tan α
AC x2 − x1
y2 − y1 Figure 31
Therefore, m = .
x2 − x1

Analytic Analysis of Lines 241


Remark
If A, B and C are three points on the same line
d
(collinear points) then mAB = mBC = mAC. A B C

Figure 32

EXAMPLE 29 Find the slope and inclination of the line which is passing though the points A(0, –1) and
B(2, 1).

Solution The slope of the line passing through the points A(0, –1) and B(2, 1) is
y2 − y1 1 − ( −1) 2
mAB = = = =1
x2 − x1 2−0 2
Since mAB = tanα = 1, then the inclination of the line is α = 45°.

EXAMPLE 30 If the inclination of the line passing through the points A(1, 2) and B(–1, a) is 120° then find
the value of a.
y2 − y1
Solution mAB = tan120° and mAB =
x2 − x1
= –tan60°
a–2
= –ñ3 –ñ3 =
–2
a – 2 = 2ñ3
a = 2 + 2ñ3

EXAMPLE 31 Show that the given points A(4, 1), B(5, –2) and C(6, 5) are collinear.

Solution We have to show that mAB = mBC


−2 − 1 −3 ⎫
mAB = = = −3 ⎪
5−4 1 ⎪
⎬ so A, B, C are collinear.
−5 − ( −2) −3 ⎪
mBC = = = –3 ⎪
6−5 5 ⎭

EXAMPLE 32 On the analytic plane, A(5, 1), B(1, 3) and P(0, k) are given three points. For what value of
k does PA – PB have its greatest value.

242 Analytic Analysis of Lines and Circles


Solution As seen in the figure for the greatest value of PA – PB y
the points P, B and A must be collinear.
P(0, k)
Otherwise, by triangle inequality PA – PB will be less B(1, 3)
3
than AB.
If mPB = mBA
A(5, 1)
3 − k 1− 3 7 1
= ⇒ k= .
1 − 0 5 −1 2
x
O 1 5

EXAMPLE 33 Given the points A(– 2, 5), B(– 4, 2) and P(0, k). What
y
is the value of k for the smallest value of PA + PB?
A 5 A¢( 2, 5)
Solution Refer to the figure PA + PB has its smallest value
when P is on the same line segment with symmetry P(0, k)
of A with respect to y - axis. )
,2
–4
Hence, P, B and A′ are collinear, B(
2

mPB = mPA′
2−k 5−k x
= –4 –2 O 2
–4 −0 2 −0
4 – 2k = –20 + 4k
Figure 33
– 6k = –24
k=4

Check Yourself 8
1. What is the slope of the line passing through the
points A(–1, 7) and B(0, –3).
2. If the inclination of the line which is passing
through the points A(1, –3) and B(k, 2) is 120°,
find k. y
d

A(2, 5)

B(0, 3)
3. In the figure, the points A, B and K are on the line
d. Find the value of x.
x
K(x, 0) O

Figure 34

Analytic Analysis of Lines 243


3. Parallel and Perpendicular Lines
a. Condition of Parallelism y
d1
On the analytic plane, let d1 and d2 be two parallel
B1
lines. As seen in the figure
A1 d2
m(∠CAA1) = m(∠CBB1) (Corresponding Angles)
a a
The slope of the line d1 is m1 = tan(CBB1) and the x
B O A C
slope of the line d2 is m2 = tan(CAA1)
so m1 = m2.
Consequently, the condition of parallelism is
d1 // d2 ⇔ m1 = m2 Figure 35

EXAMPLE 34 Given the points A(m – 1, 3), B(2m, 1), C(3, m) and D(4, m + 3). If the lines AB and CD
are parallel to each other find m.

Solution Since the lines AB and CD are parallel, their slopes are equal to each other, that is
AB // CD ⇔ mAB = mCD
1– 3 m+ 3 − m
=
2 m − m +1 4 −3
–2 3
=
m +1 1
3m + 3 = −2
5
m=−
3

b. Condition of Perpendicularity
y
Refer to the figure the slope of the line d1 is d2
m1 = tanα1 and the slope of the line d2 is D d1
m2 = tanα2 = –tanβ.
In the right trrangle ABC.
AC
m1 = tan α1 = and
BC
B a1 b a2
BC x
m2 = − tan β = − O A
AC
Let us multiply both equalities side by side, Figure 36
AC BC
m1 ⋅ m2 = ⋅(− ) = −1
BC AC
Consequently the condition of perpendicularity is d1 ⊥ d2 ⇔ m1 ⋅ m2 = –1.

244 Analytic Analysis of Lines and Circles


EXAMPLE 35 Without using Pythagorean Theorem, show that the points A(3, 4), B(–2, –1) and C(4, 1) are
the vertices of a right triangle.

Solution Let us find the slopes of the lines on which the vertices of ∆ABC lie.
−1 − 4 −5
mAB = = =1
−2 − 3 −5
1 − ( −1) 2 1
mBC = = =
4 − ( −2) 6 3
1 − 4 −3 1
mAC = = = –3 Notice that, mBC ⋅ mAC = ⋅ (–3) = –1
4−3 1 3
so, BC ⊥ AC and ABC is a right triangle.

EXAMPLE 36 Given the points M(2k + 1, – 5), N(3, –2) and K(–1, –3). If the lines MN and NK are
perpendicular to each other find k.

Solution Since MN and NK are perpendicular, the product of their slopes is –1.
mMN ⋅ mNK = −1

−2+5 −3+ 2
⋅ = −1
3 − 2k − 1 −1 − 3
3 −1
⋅ = −1
2 − 2 k −4
3 11
= −1; − 8 + 8 k = 3; 8 k = 11; k= .
8 − 8k 8

Check Yourself 9
1. Show that the points A(7, 1), B(1, 4), C(–7, 0) and D(3, –5) are the vertices of the
trapezoid ABCD.
(Hint : You have to prove that there exist two parallel lines on which any two opposite
sides of the trapezoid lie) y

2. In the figure, d1 ⊥ d2. Find the value of K. A(0, k)

C(4, 0)
x
B(–2, 0) O

Analytic Analysis of Lines 245


C. EQUATION OF A LINE
Every straight line can be represented by a linear equation of x and y, conversely, every
linear equation of x and y represents a straight line.

Generally the line equations are written in two different forms such that y = mx + n,
m, n ∈ \ and ax + by + c = 0, a, b, c, ∈ \.

1. Equation of Line In Point-Slope Form


Theorem
The equation of a line which is passing through the point A(x1, y1) with slope m is
y – y1 = m(x – x1)

Proof On the analytic plane, let us draw a line d y d


with slope m and A(x1, y1) be a point on the , y)
P(x
line. B2

Suppose that, P(x, y) ∈ d is a point which


varies along the line. y – y1
y
In the figure m(∠CAP) = m(∠A1DA)
a
(Corresponding Angles) A2 C
A(x1, y1) y1
Since α is the inclination of the line d then D
x
O A1 B1
x1 x – x1
m = tana and in the right triangle CAP we
x
PC y − y1
have m = tan α = then m =
AC x − x1 Figure 36
consequently, y – y1 = m(x – x1) is the equation of d.

EXAMPLE 37 Write the equation of the line which is passing through the point A(–1, 2) with the slope
m = 3.

Solution By substituting m = 3 and A(x1, y1) = A(–1, 2) in the equation


y – y1 = m(x – x1 ) we get.
y – 2 = 3(x – (–1))
y – 2 = 3x + 6
y = 3x + 8 is the equation of the line.
We can also write the equation as 3x – y + 8 = 0.

246 Analytic Analysis of Lines and Circles


EXAMPLE 38 Find the equation of the line whose inclination is 45° and passes through the point K(0, –3).

Solution Since α = 45° then the slope m = tan45° = 1.


By substituting m = 1 and K(0, –3) in the equation
y – y1 = m(x – x1)
y – (–3) = 1 ⋅ (x – 0)
y+3= x and we get
y= x – 3.

EXAMPLE 39 Find the equation of the line which is parallel to the line joining the points K(–1, –4),
L(–5, –10), and intercepting x - axis at 5.

Solution −10 − ( −4) −6 3


mKL = = =
−5 − ( −1) −4 2
3
Since d // KL, md = mKL we get md =
2
Besides, the line d intersects x axis at 5 means that it passes through the point (5, 0).
3
Hence the equation of d with slope md = and passing through (5, 0) is
2
y – y1 = m(x – x1)
3
y–0= ⋅ (x – 5)
2
3 15
y= x− or 3x – 2y – 15 = 0
2 2

EXAMPLE 40 Find the equation of the perpendicular drawn from the point K(–1, 2) to the line joining
A(–1, 4) and B(2, 3).

Solution Slope of the line passing through the points A(–1, 4) and B(2, 3) is
3−4 −1
mAB = =
2 − ( −1) 3
Slope of perpendicular line d to AB is md and
md ⋅ mAB = –1 (condition of perpendicularity)
−1
md ⋅ ( ) = –1
3
md = 3
Therefore, the equation of d is
y – 2 = 3(x – (–1))
y – 2 = 3x + 3
y = 3x + 5 or 3x – y + 5 = 0

Analytic Analysis of Lines 247


EXAMPLE 41 In a triangle ABC whose vertices are A(–1, 1), B(3, 3) and C(1 –1), find the equation of the
line on which the altitude of the triangle ABC through the vertex B lies.

Solution −1 − 1 −2
Slope of AC is mAC = = = −1 .
1 − ( −1) 2 B
Since AC ⊥ BH,
mAC ⋅ mBH = –1
(–1) ⋅ mBH = –1, we get
mBH = 1
Therefore, the equation of d is y – y1 = m(x – x1)
A H C
y – 3 = 1 ⋅ (x – 3) and
d
y = x.

Check Yourself 10

1. Find the equation of the line with inclination 150° and passing through he point
1 1
A( − , ).
2 2
2. Find the equation of the line passing through the point M(–5, 1) and parallel to the line
joining the points K(7, –1) and L(0, 3).
3. Find the equation of the line whose y - intercept is –5 and which is perpendicular to the
line joining the points P(–1, 6) and R(–2, –3).

EXAMPLE 42 If the distance of the y - intercept of the line y = (a – 1)x – a2 from the origin is 9 units, then
find a and draw the graph of the line. (a ∈ \+)

Solution In the line equation y = mx + n, m is slope and n is the y - intercept of a line, so


m = a – 1 (slope) y d

n = – a2 (y - intercept) 4
x
O
If we take – a2 = 9 ⇒ a2 = – 9 is not possible, 9
2
so y - intercept must be – 9.
and if – a2 = – 9 x y
a2 = 9 and a = 3 (a ∈ \+). 0 –9
Then m = a – 1 = 3 – 1 = 2 9
0
Hence the equation of the line is y = 2x – 9. 2
–9

248 Analytic Analysis of Lines and Circles


Note
To find y - intercept of a line, substitute 0 for x in the equation of the line. To find x - intercept
of a line, substitute 0 for y in the equation of the line.

EXAMPLE 43 Draw the graphs of the lines y = –x + 5 and y = –x on the same analytic plane.

Solution Notice that, the slopes of both lines are m = –1, so they y
y = –x + 5
are parallel to each other.
If the graph of a line in the form y = mx is shifted n units 5

up (when n > 0) or n units down (when n < 0) along the y = –x


y - axis without changing the inclination, we get the
graph of y = mx + n.
x
O
We know that y = – x is the second diagonal. Firstly, draw 5
the graph of y = –x and shift the line y = –x, 5 units up
along the y - axis to get the graph of y = –x + 5.

EXAMPLE 44 In the figure ABCD is a square. The lines d1 and d2 y


d1 : y = 3 × x
2
through the origin are passing through the points D and
3 d2
C respectively. If the equation of d1 is y = x and the
2 D C
equation of d2 is y = ax then find the value of a.

Solution 3 3 x
The slope of the line y = x is m1 = . O A B
2 2
In the right triangle DOA,
Figure 41
AD 3
tan(∠DOA) = =
AO 2 y
d1 : y = 3 × x
then AD = 3k and AO = 2k and each of the sides of 2
ABCD is 3k.
d2
From the right triangle COB the slope of d2 can be D 3k C
found.
3k 3k 3k
m2 = tan(∠COB) = .
5k x
O 2k A 3k B
3
m2 =
5
3 3
Therefore, the equation of d2 is y = x and a = .
5 5

Analytic Analysis of Lines 249


EXAMPLE 45 Find all possible values of k so that the points A(–1, 2) and B(3, 4) are on different sides of
the line with equation 2x + 3y + k = 0.

Solution When we write 2x + 3y + k = 0 in the form y


2 k d2
of y = mx + n, we get y = − x – .
3 3
n1
Consider that, slopes of the lines in this from d1
2
is m = – . 4
B
3
Let us draw the lines d1 and d2 with the slope A
2 n
2
2
m=– and passing through the points
3 –1 O 3
x
A(–1, 2) and B(3, 4) respectively.
To find the exact equations of d1 and d2 we
must find the appropriate values of k for each
line.
The point A satisfies the equation of
d2 y
2 k
d1 : y = − ⋅ x − 1 .
3 3
n2
By substituting A(–1, 2) in the equation for d1

(x, y), we get,


2 k
2 = − ⋅( −1) − 1 n1
3 3
k1 2 x
= −2 O
3 3
2 4
k1 = −4. So, d1 is y = − x + .
3 3

Similarly, the point B satisfies the equation of Figure 42

2 k
d2 : y = − ⋅ x − 2 .
3 3

By substituting B(3, 4) in the equation for (x, y), we get,

2 k
4 = − ⋅3 − 2
3 3
k2
= –2 − 4
3
2
k2 = −18. So, d2 is y = − x +6.
3

250 Analytic Analysis of Lines and Circles


And now, for the given condition in the problem, the points A and B will be in different sides
2 k
of the lines d1 and d2 when y - intercepts of the line with equation y = − ⋅ x − is between
3 3
n1 and n2 (Refer to the figure).
k
That is, n1 < − < n2
3
4 k
<− <6
3 3
4 < −k < 18

−18 < k < −4


k ∈ ( −18, − 4)

EXAMPLE 46 In the figure, if the equation of the line d is y

y = 3ñ3 + 3ñ3 + 3. What is the area of the square OABC? d

B A

D
x
C O

Figure 43

Solution Since OABC is a square the point B has the coordinates (– a, a).
This point satisfies the equation of d because B(– a, a) is on the line d.
By substituting B(– a, a) in the equation of d we get
y = ñ3x + 3ñ3 + 3
a = ñ3 ⋅ (– a) + 3ñ3 + 3
a + ñ3a = 3 ⋅ (ñ3 + 1)
a ⋅ (1 + ñ3) = 3 ⋅ (ñ3 + 1)
a = 3.
Hence, the area of OABC is A(OABC) = 32 = 9 unit2.

Analytic Analysis of Lines 251


Check Yourself 11
1. Draw the graphs of the lines y = –2x – 1, y = –2x and y = –2x + 1 on the same analytic
plane and compare them.

2. Find the slope of the following lines

a. x – 3y + 1 = 0 b. –2y = 3x + 1 c. 5 + 4y = –x
y
3. In the figure find the area of the triangle ABO formed by
y = mx
the lines y = 2, y = mx and y - axis.
B A y=2

x
O

2. Equation of a Line in Two - Point Form


Theorem
The equation of the line passing through two fixed points A(x1, y1) and B(x2, y2) is
y − y1 x − x1
=
y2 − y1 x2 − x1

Proof Let α be the inclination of the line d which is passing y


through the pints A(x1, y1) and B(x2, y2). Then the slope d

of the line d is, y2 B

y2 − y1 y2 – y1
md = tan α =
x2 − x1 y1 A a
We know that the equation of the line whose one point x2 – x1

and slope are known is a


x
y – y1 = m(x – x1) O x1 x2

y2 − y1
y − y1 = ⋅ ( x − x1 ). Hence,
x2 − x1
Figure 44
the equation of the line passing through two points is
y − y1 x − x1
=
y2 − y1 x2 − x1

EXAMPLE 47 Find the equation of line which is passing through the point A(– 1, – 3) and B(5, 1)

252 Analytic Analysis of Lines and Circles


Solution By substituting the points A(–1, –3) and B(5, 1) in the formula,
y − y1 x − x1
= we get
y2 − y1 x2 − x1

y + 3 x +1
=
1+ 3 5 +1
y + 3 x +1
=
4 6
3y + 9 = 2 x + 2

2 x − 3y − 7 = 0

EXAMPLE 48 If the points A(2, –5), B(–1, 1), and C(m, 2) are on the same line, then find the value of m.

Solution 1 Firstly, the equation of the line AB is


y+5 x−2
=
1 + 5 −1 − 2
−3y − 15 = 6 x − 12

6 x + 3y + 3 = 0

2 x + y +1= 0 or y = −2 x − 1

Since C(m, 2) lies on this line, it satisfies the equation y = –2x – 1.

so 2 = –2m –1

3 = –2m
3
m= –
2

Solution 2 Recall that the problem can be solved by using slope formula.

Since A, B and C are collinear mAB = mBC


1 – ( −5) 2 − 1
=
–1 − 2 m +1
6 1
=
–3 m + 1
1
m + 1= −
2
3
m=– .
2
Analytic Analysis of Lines 253
EXAMPLE 49 If A(–1, 2), B(2, 1) and C(0, 4) are the vertices of ∆ABC, find the equation of the median of
the triangle through the vertex A.

Solution If N is the midpoint of BC, its coordinates are


2+0 1+ 4 5 A
N( , ) = N(1, )
2 2 2

AN is the median and it passes through the points


5
A(–1, 2) and N(1, ).
2
y−2 x +1 x +1
so its equation is = ; 2y − 4 =
5
− 2 1 − ( −1) 2
B N C
2
and x – 4y + 9 = 0.

Check Yourself 12
1. What is the equation of the line joining the points A(–1, 7) and B(–2, 6)?

2. Find the equation of the line passing through the points A(4, 0) and B(0, 3). Find the
3
value of k, if the line passes through C(k, ).
2

3. The Equation of The Line Whose Intercepts on The Axes


are Known
Theorem
Let a be the x - intercept and b be the y - intercept of the line d, then the equation of d is
x y
+ =1
a b

Proof From the equation of the line whose two points are y
known d

y−0 x−a y x
= ; = +1 B(0, b)
b−0 0−a b −a
x y
+ =1
a b
A(a, 0)
This is called “intercept form” of the equation of the x
O
line d with the intercepts a an b on the axes x and y,
respectively.

254 Analytic Analysis of Lines and Circles


EXAMPLE 50 Find the equation of the line passing through the points A(–5, 0) and B(0, 3).

Solution x - intercept of the line is a = –5 and y – intercept of the line is b = 3, then


x y x y
+ =1 and + =1
a b –5 3
Hence, the equation is 3x – 5y – 15 = 0.

EXAMPLE 51 Find the equation of the line containing the point P(–3, 2) and making opposite signed
intercepts which are equidistant from the origin.

Solution Line d intersects the axes in the second quadrant. y


d
As seen in the figure, point P(–3, 2) satisfies the
equation of d
a
x y
+ =1 .
−a a
P 2
−3 2 5
+ =1; =1; a = 5.
−a a a O
x
–a –3
x y
So the equation of d is + =1 and
−5 5
x – y + 5 = 0.

EXAMPLE 52 Find the equation of a line which passes through y

the point P(–5, 2) and whose segment between


2
the axes is divided by the point P in the ratio . 3k B(0, b)
3
P
2
Solution The value of a: 2k

For 3k, there is 5 units decreasing, for 5k there x


A(a, 0) –5 O
5 25 25
will be 5 ⋅ = decreasing so, a = −
3 3 3
The value of b:
For 2k, there is 2 units increasing for 5k there will
be 5 units increasing, so b = 5.
2k 3k
x y 3x 5y
+ =1 ; + = 1 ; 3 x − 5 y+ 25 = 0. A(a, 0) P(–5, 2) B(0, b)
−25 5 −25 25
3

Analytic Analysis of Lines 255


EXAMPLE 53 The area of a triangle formed by a line and the coordinate axes is 6 unit2 and the length of
the segment intercepted between the axes is 5 units. Find the equation(s) of the lines.

Solution AB = 5 units
A(∆AOB) = 6 unit2
As seen in the figure a and b are the intercepts of the line on the y
axes.
ab B(0, b)
b
A(∆AOB) = = 6 ; ab = 12 and
2
AB2 = a2 + b2 ; a2 + b2 = 25.
(a + b)2 – 2ab = 25 A(a, 0)
x
2
(a + b) = 25 + 2 ⋅ 12 = 49 O a

a + b = – 7 or a + b = 7.

Case I:
a+b=7
a ⋅ b = 12
12
a + = 7 ⇒ a2 − 7 a +12 = 0
a
( a − 3)( a − 4) = 0

12 x y
1) a1 = 3 ; and b1 = = 4 so + =1 or 4x + 3y – 12 = 0 (d1)
3 3 4
12 x y
2) a2 = 4 ; and b2 = = 3 so + =1 or 3x + 4y – 12 = 0 (d2)
4 4 3

Case II:
a + b = –7
a ⋅ b = 12
12
a + = −7 ⇒ a2 +7 a +12 = 0
a
( a + 3)( a + 4) = 0

12 x y
1) a1 = –3 ; and b1 = = –4 so + =1 or 4x + 3y + 12 = 0 (d3)
–3 –3 –4
12 x y
2) a2 = –4 ; and b2 = = –3 so + =1 or 3x + 4y + 12 = 0 (d4)
–4 –4 –3

256 Analytic Analysis of Lines and Circles


EXAMPLE 54 Find the area of the region formed by the lines x – y + 2 = 0, 2x + y – 2 = 0 and
y + 2 = 0.
y
Solution To see the region, first let’s draw the graphs by d2 d1
finding x and y - intercepts of the lines.
A 2
d1 : x – y + 2 = 0 x y
0 2 (0, 2)
m –2 1 n
–2 0 (–2, 0) O
x

d2 : 2x + y – 2 = 0 x y
C(n, –2)
d3
0 2 (0, 2) B(m, –2) D –2

1 0 (1, 0)
d3 : y + 2 and y = –2
B(m, –2) satisfies the equation of d1: m – (– 2) + 2 = 0 and m = – 4, C(n, – 2) satisfies the
equation of d2: 2n + (–2) – 2 = 0 and n = 2. Consequently, it is seen that AD = 4 units and
BC ⋅ AD 6 ⋅ 4
BC = 4 + 2 = 6 units. A( ∆ABC ) = = =12 unit 2
2 2

Check Yourself 13
1. Find the equation of the line whose x and y - intercepts are – 4 and – 3 respectively.
2. A line passing through the point P(2, 3) intersects x and y - axes at the points A and B,
respectively. If PA = 2 ⋅ PB then find the equation of the line AB.
3. The x and y - intercepts of the line are 2 and –3, respectively. Verify that it passes through
the point P(4, 3).
4. If the intercepts of the line 5x + 12y – 60 = 0 are A and B, find AB.

4. The Equations of The Coordinate Axes


The inclination of the x - axis is α = 0°, then the y

slope of the x - axis becomes m = tan0° = 0.


Besides, it is obvious that O(0, 0) is on the x=0
x - axis.
x
Hence, the equation of x - axis is O y=0

y – y1 = m(x – x1)
y – 0 = 0 ⋅ (x – 0)
y=0
Figure 45

Analytic Analysis of Lines 257


On the other hand, the inclination of y - axis β = 90°, then the slope is not defined.
m = tan90° is undefined.
y − y1 y−0
m= ⇒ m=
x − x1 x −0
y
Since the expression is undefined the denominator must be 0. That is x = 0 is the
x
equation of the y - axis.

Remark
Consider that the ordinate of each point on the x - axis is 0. That is for every point (x, y) on
the x - axis, y = 0.

And, the abscissa of each point on the y - axis is 0. That is for every point (x, y) on the
y - axis, x = 0.

5. The Equations of the Lines Parallel to Coordinate Axes


Let’s draw a line through the point B(0, b) and parallel y

to x - axis. y=b B(0, b) P(x, b)


d
The inclination of the line will be α = 0° and its slope
m = tan0° = 0. x
O y=0
y – y1 = m(x – x1)
y – b = 0 ⋅ (x – 0)
Figure 46
y = b is the equation of the line d.

y
Let’s draw another line through the point A(a, 0) and d
parallel to the y - axis. P(x, b)
The inclination of the line will be β = 90° and its slope
will be undefined. A(a, 0)
x
O
y – y1 = m(x – x1)
y – 0 = m(x – a)
x=a
y
m= Figure 47
x−a
= tan90 °
y
To make m undefined, the denominator of the expression must be 0, that is x – a = 0,
x−a
x = a is the equation of the line d.

258 Analytic Analysis of Lines and Circles


EXAMPLE 55 Show the following lines on the analytic plane.

a. y = –3 b. 2x + 4 = 0

Solution a. y = –3 b. 2x + 4 = 0, x = –2

y y

x O x
O –2

y = –3
–3 x = –2

EXAMPLE 56 Show the lines passing through the point A(1, 3) and paral- y

lel to the coordinate axes. 3 y=3

Solution The problem can easily be solved by drawing.

x
O 1

x=1

EXAMPLE 57 Find the equation of the horizontal line passing through P(5, –2).

Solution Since the line is horizontal, it is parallel to x -axis. It cuts y - axis at B(0, –2) and then the
equation of the line will be y = –2 or y + 2 = 0.

EXAMPLE 58 Find the equation of the vertical line passing through K(–7, 3).

Solution Since the line is vertical, it is parallel to y - axis. It cuts x - axis at B(–7, 0) and then the
equation of the line will be x = –7 or x + 7 = 0.

EXAMPLE 59 Find the equations of the diagonals of a rectangle whose sides are on the lines x = –1,
x = 2, y = –2 and y = 4.

Analytic Analysis of Lines 259


Solution Let us draw the graphs of four lines on the y

analytic plane. As seen in the figure, the d1 d2

vertices of the rectangle ABCD are A(–2, 4),


y=4
B(–1, 4), C(–1, –2) and D(2, –2). B A

The line d1 passes through the points A(–1, 4)


and D(2, –2), so the equation of d1 is,
x
O
y − y1 x – x1 y−4 x +1
= and = C D
y = –2
y2 − y1 x2 – x1 −2 − 4 2 + 1
3y – 12 = – 6x – 6 we get
d1: 6x + 3y – 6 = 0 x = –1 x=2
The line d2 passes through the points A(2, 4)
and C(–1, –2) so the equation of d2 is
y−4 x–2
= and –3y + 12 = – 6x + 12
−2 − 4 −1 – 2
– 3y = –6x we get d2: 6x + 3y – 6 = 0

Check Yourself 14
1. What are the equation of the horizontal and vertical lines passing through the point
P(–9, –2).
2. Show the region formed by the lines x – 3 = 0, x + 2 = 0, y + 4 = 0 and y – 1 = 0 on
the analytic plane. Find its area.

D. FINDING SLOPE OF A LINE WHOSE EQUATION IS GIVEN


Recall that, the line equations are in two different forms:
y = mx + n and ax + by + c = 0.
For example, y = 2x – 7 and 2x – y – 7 = 0 are two different forms of the equation of the
same line.
The slope of a line given in the form y = mx + n is m (the coefficient of x).
But to find the slope of a line given in the form ax + by + c = 0, it is useful to convert it to
the form y = mx + n.
ax + by + c = 0
by = –ax – c
a c
y= − x −
b b
a c
Compare y = − x − with y = mx + n
b b

260 Analytic Analysis of Lines and Circles


a c
We get m = − and n = –
b b
a
Consequently the slope of a line in the form of ax + by + c = 0 is m = − .
b

Conclusion
1. The slope of a line in the form y = mx + n is m.
a
2. The slope of a line in the form ax + by + c = 0 is − .
b

EXAMPLE 60 If the equation of the line with the inclination 135° is (2k – 1)x – (k + 2)y + 4 = 0, then
find k.

a
Solution m = tanα and m = −
b
2k − 1 2k − 1
tan135° = − ; − 1= and – k – 2 = 2k – 1
−( k + 2) k +2
– 3k = 1
1
k= − .
3

EXAMPLE 61 Given that the line


y
2
= x − p and the line ax + 5 = 3y are parallel find the value of a.

Solution The slopes of parallel lines are equal, let us find the slopes of the given lines.
y
d1 : =x−p
2
y = 2 x − 2 p and m1 = 2

d2 : ax +5 = 3 y

−a a
ax − 3y +5 = 0 and m2 = =
−3 3

Since m1 = m2
a
2= we get a = 6.
3
Analytic Analysis of Lines 261
EXAMPLE 62 Find the value of m, if the lines 2mx – 3y = 1 and y = 1 – 2x are perpendicular to each other.

Solution The product of the slopes of perpendicular lines is equal to –1.


Let us find the slopes of the lines.

d1 : 2mx – 3y = 1 d2 : y = 1 – 2x
2mx – 3y – 1 = 0 y = –2x + 1
−2 m 2 m
m1 = = m2 = –2
−3 3
2m 3
m1 ⋅ m2 = –1 so ⋅ ( −2) = −1; m =
3 4

EXAMPLE 63 If the lines d1 : 3x + y = 4, d2 : x – ay + 7 = 0 and d3 : bx + 2y + 5 = 0 form three


consecutive sides of a rectangle, find the values of a and b.

Solution As seen in the figure d1 // d3, d1 ⊥ d2 and d2 ⊥ d3 d1


−3
m1 = = −3
1 d2

−1 1
m2 = =
a a
−b b
m3 = =−
2 2
d3
d1 // d3 so m1 = m3
b
−3 = − ⇒ b=6
2
d1 ⊥ d2 so m1 ⋅ m2 = 1
1
−3 ⋅ = −1 ⇒ a = 3.
a

EXAMPLE 64 Find the equation of a line which is perpendicular to the line 2x + 5y + 7 = 0 and
intercepts y - axis at –3.
Solution We know that the product of the slopes of perpendicular lines is equal to –1.
2
m1 ⋅ m2 = –1 and m2 = −
5
2 5
m1 ⋅ ( − ) = –1 ⇒ m1 =
5 2
262 Analytic Analysis of Lines and Circles
The line passes through the point (0, –3). So, the equation is
y – y1 = m(x – x1)
5
y – (–3) = (x – 0)
2
5
y+3= x
2
5
y = x – 3.
2

EXAMPLE 65 Find the equation of a line parallel to the line 2x + 3y = 5 and having the same y - intercept
as x + y + 4 = 0.

2
Solution Slope of the line 2x + 3y = 5 is m = −
3
y - intercept of the line x + y + 4 = 0 is (0, – 4).
2
The slope of parallel lines are equal, then the slope of a line is − .
3
y - intercept is – 4 and the equation is y = mx + n
2
y = − x − 4 or 2x + 3y + 12 = 0.
3

Check Yourself 15
a+ b
1. If the lines x + 4y – 1 = 0 and ax + by + c = 0 are perpendicular to each other find .
b
2. Find the equation of the line which is parallel to the line 2x + 3y = 5 and passes through
the point P(–1, 3).

3. Write down the equation of the line perpendicular to 3x + 8y = 12 and passing through
the point P(–1, –2).
3
4. If the slope of the line (2k – 1)x – 3ky – 1 = 0 is then find the value of k.
4

E. POSITIONS OF TWO LINES


Let lines d1 : a1x + b1y + c1 = 0 and d2 : a2x + b2y + c2 = 0 be given.
a1 b c
1. If = 1 = 1 then lines are coincident, that is d1 = d2.
a2 b2 c2
a1 b c
2. If = 1 ≠ 1 then m1 = m2, that is d1 // d2.
a2 b2 c2
a1 b1
3. If ≠ then m1 ≠ m2, that is lines intersect at one point.
a2 b2

Analytic Analysis of Lines 263


Point of intersection of two lines is equal to the solution set of the system of linear equations
a1x + b1y + c1 = 0 ⎫⎪

a2 x + b2 y + c2 = 0 ⎪⎭

For the equations y = m1x + n1 and y = m2x + n2

y y y
d1 d1 = d2
d2 d1
d2
B y1 A(x1, y1)

a1 a2 a1=a2
x x x
O O O x1
m1 = m2 m1 = m2 m1 ¹ m1
tana1 = tana2 n1 = n2

Figure 49

EXAMPLE 66 Determine the position of the lines x – 3y + 2 = 0 and 2x – 6y + 4 = 0

1 −3 2
Solution = = (The ratios of the coefficients are equal.)
2 −6 4
Therefore, the lines are coincident (same line).

EXAMPLE 67 If the lines x + 2y – 2 = 0 and (k – 1)x + y + 4 = 0 are parallel to each other find the value
of k.

1 2 d1 y
Solution d1 // d2 ⇒ = ⇒ 2 k − 2 =1
k −1 1
3
⇒ k= . d2
2 1
x
Let’s draw the graphs of the lines –8 O 2

1
d1: x + 2y – 2 = 0 d2: x+y+4=0
2
x y x y –4

0 1 0 –4
2 0 –8 0

264 Analytic Analysis of Lines and Circles


EXAMPLE 68 Find the intersection point of the lines x + y + 1 = 0 and 2x – y + 2 = 0.

Solution When the equations are solved simultenously, y


d2
x+y+1=0
2
2x – y + 2 = 0
–––––––––––––––––––
3x + 3 = 0
d1
x = –1 ⇒ –1 + y + 1 = 0
y=0
x
the intersection point of the line is obtained as (–1, 0). O
–1
Let’s find the intersection point by sketching the
–1
graphs of the lines.
d1: x + y + 1 = 0 d2: 2x – y + 2 = 0
x y x y
0 –1 0 2
–1 0 –2 0

y
EXAMPLE 69 In the figure find A(a, b)
a. the coordinates of the point A.
b. the area of the triangle AOB.
C(0, 2)

B(4, 0)
x
Solution First, let’s find the equation of d1 O
d1
x y
+ =1 ⇒ d1 : x + 2 y − 4 = 0 .
4 2 d2 : y = –x
a. Point A(a, b) is the intersection point of d1
and d2. So the simultaneous solution of the equations
x + 2 y − 4 = 0 ⎫⎪
⎬ ⇒ x + 2 ⋅ (–x) – 4 = 0
y = − x⎪⎭ –x – 4 = 0
x = – 4 and y = 4
A(– 4, 4) are the coordinates of the point A.

b. For the area of AOB, since OB = 4 units and altitude belonging to this side is 4,
4⋅4
A(AOB) = = 8 unit2.
2

Analytic Analysis of Lines 265


EXAMPLE 70 Find the intersection point of the lines x – 2y + 6 = 0 and –2x + 4y + 6 = 0

Solution x – 2y + 6 = 0
–2x + 4y + 6 = 0
––––––––––––––––––––––––
2x – 4y + 12 = 0
–2x + 4y + 6 = 0
–––––––––––––––––––––––––
18 = 0 ⇒ S = ∅
d1∩ d2 = ∅ so the lines are parallel.

1 –2 6
Notice the ratios of the cofficients = ≠ , it means m1 = m2 and d1 // d2.
–2 4 6

EXAMPLE 71 Find the intersection point of the lines 3x – 4y + 12 = 0 and – 6x + 8y – 24 = 0.

Solution 3x – 4y + 12 = 0
– 6x + 8y – 24 = 0
––––––––––––––––––––––––
6x – 8y + 24 = 0
– 6x + 8y – 24 = 0
–––––––––––––––––––––––––
0=0 ⇒ S=\
There are infinitely many solutions of the system. So the lines are coincident.
3 − 4 12 1
Notice that, = = =− so d1 = d2.
−6 8 −2 2

Check Yourself 16
Determine mutual positions of the following pairs of lines and find their intersection points.

1. –2x + y + 1 = 0

2y = 4x + 7

2. 7x – 5 = –4y

8y – 10 = –14x

3. x – 5y – 13 = 0
x
– + 3y – 1 = 0
3

266 Analytic Analysis of Lines and Circles


EXAMPLE 72 Given the equations 3x – ky + 6 = 0 and (k – 5)x + 2y – 4 = 0. Find the value of k if
a. the lines are perpendicular.
b. the lines are parallel.
c. the lines are coincident.

Solution a. d1 ⊥ d2 ⇔ m1 ⋅ m2 = –1 b. d1 // d2 ⇔ m1 = m2 c. d1 = d2
−3 k −5 − 3 − ( k − 5)
( ) ⋅( − ) = −1 = 3 −k 6
−k 2 −k 2 = =
k−5 2 −4
3k − 15 = 2 k 6 = –k2 + 5k
k=3
k2 – 5k + 6 = 0
k =15
k1 = 3, k2 = 2.

EXAMPLE 73 If the lines ax + 2y – 4 = 0 and 2x + by + 6 = 0 perpendicularly intersect each other on


y - axis find (a, b).

Solution Since the lines are perpendicular, m1 ⋅ m2 = –1


a 2 a
( − ) ⋅( − ) = −1 ; = −1 ; a = −b.
2 b b
Let P(0, k) be the intersection point of the lines.
So P(O, k) is the common point of the lines and satisfies both equations. Namely,
a ⋅ 0 + 2 ⋅ k – 4 = 0, we get k = 2 and ⎫
⎪ −6
6 ⎬ then 2 = and b = −3
2 ⋅ 0 + b ⋅ k + 6 = 0, we get k = − ⎪ b
b ⎭
We know that a = –b, so a = 3, and (a, b) = (3, –3).

y d1

74
d2
EXAMPLE In the figure the equation of d1 is y = 2x – 3 and d2 is
B
y = x + n.
23
If A(AOCB) = unit2, find the value of n. C
4

x
D O A H

Figure 50

Analytic Analysis of Lines 267


Solution From the given equations let’s find the points A, B, C, D.
C and D are the intercepts of the line y = x + n, that is
for x = 0, y = n, C(0, n)
for y = 0, x = – n, D(– n, 0)
3 3
A is the x - intercept of the line y = 2x – 3, that is for y = 0, x = A( , 0).
2 2
B is the intersection point of the lines. By solving these equations simultaneously.
y = x+ n ⎫
⎬ ⇒ x + n = 2x − 3
y = 2x − 3 ⎭

x = n + 3 ⇒ y = x+ n ⎫
⎬ ⇒ B( n + 3, 2 n + 3)
y = 2n + 3 ⎭
However the given area AOCB is the difference of the areas of ∆ABD and ∆DOC.
That is, A(AOCB) = A(DABD) – A(∆DOC)

3 2n + 3 ⎫
Since AD = n + =
2 2 ⎪

⎪ 1 1
BH = 2 n + 3 ⎬ ⇒ A( AOCB) = ⋅ AD ⋅ BH − ⋅ OD ⋅ OC
⎪ 2 2
OD = OC = n ⎪
⎪⎭ 23 1 2 n + 3
= [( ) ⋅(2 n + 3) − n ⋅ n]
4 2 2
23 4 n2 +12 n +9 − 2 n2
=
2 2
2n2 + 12n – 14 = 0
n2 + 6n – 7 = 0
(n + 7) ⋅ (n – 1) = 0
n = –7 or n = 1,
Since n is positive, n = 1.

Check Yourself 17
1. If the lines (5 – k)y + 2x – 3 = 0 and mx – y = –2 perpendicularly intersect each other
on x – axis, find (m, k).

2. Write the equation of the line passing through the origin and the intersection point of the
lines 2x – y – 4 = 0 and 3x + y – 3 = 0.

268 Analytic Analysis of Lines and Circles


EXERCISES 2
A. Trigonometric Ratios of Angles 7. Find the inclination of the line passing through
1. Find the tangent values of the following acute the points A(–3, 0) and B(0, –ñ3).
angles

a. α = 30° b. α = 45° c. α = 60° 8. If the points A(–1, 1), B(2, 3) and C(k, 5) are
collinear then find the value of k.

2. Find the tangent values of the following obtuse


angles. 9. If the inclination of the line passing through the
points A(–ñ3, 1) and B(k, –2) is 150°, then find k.
a. α = 120° b. α = 135° c. α = 150°

10. The points A(3, –5), B(2, 1) and C(m, –1) are
3. What are the tangent values of the following
given. If AB ⊥ BC, then find m.
angles.

a. 0° b. 90° c. 180°
11. The points A(5, –6), B(3, 2), C(a, 5) and
D(–2, a + 1) are given, if AB // CD then find a.
B. Slope of a Line
4. Find the slopes of the lines whose inclinations are C. Equation of a Line
given below.
12. Find the equations of the lines whose slopes and
a. α = 30° b. α = 120° one point are given below.
c. α = 90° d. α = 0° a. A(0, 0) b. C(3, 4) c. K(6, 0)
m = –1 m = ñ3 m=0
5. Find the inclinations of the lines whose slopes are
given below. 13. Find the equation of each line whose one point
3
a. m = b. m = – ñ3 and inclination are given below.
3
c. m = 1 d. m = 0 a. A(–2, 3) b. B(2, –4)
α = 150° α = 45°
c. C(–3, –4) b. D(7, 1)
6. Find the slopes of the following lines whose two α = 120° α = 90°
points are given below.

a. A(2, –1) b. K(0, –2)


14. Find the equation of the line passing through the
B(–3, 2) L(3, –1)
point A(4, –6) and is parallel to the line joining
c. P(–3, 0) d. M(1, 7) 1
the points B(–2, – ) and C(1, –1).
R(1, –2) N(–1, –3) 2

269 Analytic Analysis of Lines and Circles


15. Write the equation of the perpendicular bisector 24. Find the equation of the locus of all points P such
of the line segment AB, where A(4, 7) and that the slope of the line joining the origin and P
B(2, –1) are the cordinates of the endpoints. is –2.

16. Write the equation of the lines passing through 25. Find the equation of a line passing through the
the point A(5, –2) and parallel to x and y – axes point A(–2, 3) and intercepting x - axis at 4.
respectively.

26. Find the equation of the line passing through the


17. What is the equation of the line passing through
points A(2, –5) and intercepting y – axis at –3.
origin with an inclination 120°.

18. Find the equation of the perpendicular line x y


27. If the line + =1 passes through the points
drawn from the point P(–1, 2) to the line joining a b
A(12, –15) and B(8, –9), find the values of a and b.
A(1, 4) and B(2, 3).

28. Write the equations of the following lines whose


19. Given the points A(–2, 4), B(2, 1) and C(–1, –3)
graphs are given below.
are the vertices of a triangle ABC, find the
a. y b. y
equation of the altitude of the triangle through
the vertex A. 3

2
20. Find the equations of the lines whose two points
are given below.

a. A(–2, 1) b. C(–3, 0) O
x
O
x
–2 2
B(0, 3) D(4, –2)
c. K(0, –2) d. M(1, 7) c. y d. y

L(3, –1) N(–1, –3) 1 –3


x x
O O
21. Given A(– 10, 12), B(6, 4) and C(– 4, – 8) are the
vertices of ∆ABC. Find the equations of the lines –1

passing through the medians of ∆ABC. –2

22. For t ∈ \, find the equation of the locus of all


29. Find the x and y - intercepts of the following lines
points in the form of A(3t – 2, t + 1).
and draw their graphs

23. The points A(–2m, 0) and B(0, m) are given. Find a. 2x – 3y – 4 = 0 b. x + 5y – 2 = 0

the equation of the locus of the midpoint of the c. –3x – y + 1 = 0 d. 3x – 6 = 0


line segment AB. e. 5y + 25 = 0

Analytic Analysis of Lines 270


30. Find the equation of the line whose x and D. Finding Slope of a Line Whose
y - intercepts are –3 and 4, respectively. Equation is Given
40. Find the slopes of the following lines
31. Find the equation of the line which passes
through the point P(3, – 4) and makes a. 2x – 3y + 1 = 0
b. –x + 3y + 1 = 0
a. equal intercepts on the axes
c. –3x + 6y – 7 = 0
b. intercepts equal in length but opposite in sign
on the axes. d. 4 – 2x = 0
e. 5y – 2 = 0
32. Find the equation of the line passing through the
point P(3, –2) and forming positive intercepts on 41. If the lines (2m – 1)x – my + 5 = 0 and
x and y - axes in the ratio 4 : 3. y = mx – 3 are parallel to each other, find m.

33. Find the equation of the lines passing through


42. If the lines 5x – 6y + 7 = 0 and
the point A(–1, –2) and whose sum of x and
mx + (m + 1)y – 5 = 0 are perpendicular to each
y - intercepts is 3 units.
other, find m.

34. Find the equation of the line which passes


y
through the point A(1, –3) and whose y - intercept
43. In the figure, find
is twice as long as x - intercept. A(–1, 4)
the value of a.
35. Write the equation of the line passing through
A(0, 3) and whose product of distances of inter- C(3, 0)
x
B(a, 0) O
cepts from the origin is 2.

36. Find the equation of the line whose x - intercept Figure 52

is 15 and which is perpendicular to the line


3 44. Find the equation of the line passing through the
y= x.
4 point P(2, 4) and having the same slope with the
line 2x – y + 3 = 0.
37. If the area of the triangle formed by the line
5x + 12y + k = 0 and coordinate axes is 30 unit2,
45. If the line passing through the points A(1, 2) and
then find the positive value of k.
B(–3, 4) is parallel to the line kx + 2y + 6 = 0,
then find k.
38. Find the area of triangular region bounded by the
lines x – y + 2 = 0, 2x + y – 2 = 0 and y + 2 = 0.
E. Positions of Two Lines
39. Find the area of triangular region bounded by the 46. If the lines kx + 8y – 6 = 0 and 2x + 4y + c = 0
lines 3x + 2y – 6 = 0, x + 2y – 6 = 0 and x – axis. are coincident, then find the value of k + c.

271 Analytic Analysis of Lines and Circles


47. Given that the lines (m + 2)x + (m – 1)y – 4 = 0 F. First Degree Inequalities In Two
and 2x + y – 3 = 0 are parallel to each other. Unknowns
What is the value of m. 52. Show the solution sets of the following inequalities
by graphing

a. 2x + y – 2 ≥ 0 b. x – 3y + 1 ≤ 0
48. Determine the positions of the following three
lines. c. 3x – y + 4 < 0 d. 5x – 4y – 20 > 0

d1 : –4x + y + 3 = 0
d2 : 12x – 3y – 9 = 0
d3 : 8x – 2y + 1 = 0 53. Show the solution sets of the following systems on
the analytic plane.

a. x + 2y – 2 ≤ 0 b. |x| ≤ 3
49. Find the intersection point of the following pairs
2x – y + 4 > 0 2x – y + 2 ≥ 0
of the lines, and draw their graphs.
c. y < 3x + 4 d. y > –1
a. 5x – 4y + 2 = 0
y < 3x +1 x–y–2≤0
x+y–5=0
x
b. + 3y – 1 = 0
2
x + 2y + 3 = 0 54. Write the appropriate inequality for each of the
3
given graphs below.
c. –x + 2y = –5
a. y b. y
7x – 14y = 35
5
2

50. For what value of k, do the lines


(2k – 1)x + ky – 6 = 0 and –1
x x
kx + (k + 1)y – 4 = 0 intersect each other on O O 1

y - axis.

c. y
51. Find the area of y
the triangle d2
d1
ABC. A(3, 4)
3 A
2 x
O
B C
x
–4 O 5

Figure 53

Analytic Analysis of Lines 272


55. Write the appropriate system of inequalities for 60. 2x + y – 4 = 0
each of the graphs are given below. 3x + 2y – 6 = 0
a. y b. y
ax + y – 1 = 0 are given. If the lines pass through
2 one common point, find the value of a.
1
–2 –5
x
x O
O –1 61. If the equation of a line belonging to the bunch of
–1 lines with the equation
–3
B (k – 2)x + (k + 1)y + k – 1 = 0 is
c. y 2x + by – 2 = 0. Find the value of b.

62. Find the distance of the common point of the


1 lines with the equation
x
1 O
2 1
(2k – 1)x + (k + 1)y + 2k – 1 = 0 to the origin.
3
2
63. Find the common point of the lines with the
equation (3k + 1)x – (2k + 2)y – 7 = 0

56. Show that the points A(–1, 3) and B(4, –1) lie on
the same side of the line 3x – y + 7 = 0. Mixed Exercises
64. Given that the points A(0, 2), B(k, k+ 2), C(6, 10)
and D(n, k) are the vertices of a parallelogram
ABCD. Find the length of BD.
57. Are the points M(2, 3) and N(–1, 5) on the same
side or on apposite sides of the line y = 2x + 5.
65. Find the equation of a line whose y - intercept is
–6 and which is

58. Which of the points A(1, 1), B(–1, 2) and C(–3, 0) a. parallel to the line joining the points A(3, 7)
lie on the same side of the line 4x + 3y = 5 on and B(–2, 0).
which the origin lies? b. perpendicular to the line joining the points
C(–1, 6) and D(–2, –3).

66. Given the points A(–4, –4), B(–8, –2) and C(x, 0)
G. Bunch of Lines
what is x,
59. Given that the lines 2x + y + 4 = 0 and
ax + y – 3 = 0 intersect each other on the line a. if AC – CB has its greatest value
x – y = 0. What is the value of a? b. if AC + CB has its smallest value

273 Analytic Analysis of Lines and Circles


67. In the figure, ABCD is y
C 71. Write the equations of the following lines.
a square. a. y b. y
The equation of the D
B
line is y = ñ3x and
the ordinate of the A 45°
x
30°
x
O O
point D is 4. x
O
Find the coordinates –3
of the point C.
y = ñ3x
Figure 54

72. Find the equation of the line passing through the


68. In the figure d1 and d2 y
points A(2, 1) and the intersection point of the
are perpendicular on
the x - axis. Find the 3 lines 3x – 5y – 10 = 0 and x + y = 0
coordinates of P.
73. Find the equation of the line parallel to the line
d1 –4x + 3y + 3 = 0 and passing through the
x intersection point of the lines 2x + y – 3 = 0 and
O 2
x – 2y + 5 = 0.
d2
P

74. Find the equation of the line perpendicular to the


Figure 55
line y=–2x+5 and passing through the intersection
points of the lines x – y + 1 = 0 and x + y = 0.
69. The graph shows the h (m)

growing of a tree in
75. For what values of k are the points A(–2, –3) and
height related to time.
5 B(1, 2) in different sides of the line x + 2y + k = 0.
What will be the
height of the tree 3

after 6 years? 76. In the figure, OABC is a y


square. Find the area of
t (year)
O 2 the square. 6

Figure 55

70. If the area of the triangle formed by a line and C B

2
positive direction of coordinate axes is 54 unit2 O
x
A
3
and the slope of the line is – then find the
4
equation of the line. Figure 57

Analytic Analysis of Lines 274


y
77. Find the equation of the line passing through the 81. In the figure, the A
B
origin and making an angle of 120° with the line rate of growing of
ñ3x – 3y = 0 in the positive direction. heights of two 3

plants related to 2
time is given.
In 10th year, what is 1
78. In the figure, the y

equation of the y=x


the difference x
B between the O 1
line OB is y = x.
heights of plants. Figure 61
If OA = OB and
A(ñ6, –2ñ3) then
find the coordinates O
x
of the point B.
82. In the figure, the y

ordinate of point A
4 A
A
is 4. What is the
Figure 58 equation of the
line d2? O
2

y
d2
79. In the figure, the d1 –3
d2
equation of d1 is d1
2x + y – 30 = 0 A
Figure 62
and the equation
of d2 is 4x – 3y = 0. O
x
B
What is the area of
the triangle AOB. 83. In the figure, the y

equation of the B
Figure 59
line
d is 3y + 2x = 0 E
A
and
80 .In the figure given y
E(0, 4). If AB ⊥ d,
the points A(–3, 0), x
C
find the ordinate of O
E(0, –1), C(0, 2)
the point B. d
and D(1, 0).
Figure 63
Find the area of he
A D
shaded region. x
O
E
84. A vertex of an equilateral triangle is A(2, 3) and the
B
equation of the opposite side is x + y + 2 = 0. Find
Figure 60 the equations of the other two sides.

275 Analytic Analysis of Lines and Circles


3 FURTHER APPLICATIONS
A. ANGLE BETWEEN TWO LINES
Let us take two lines d1 and d2
d1 : y = m1x + n1 y
d1
d2 : y = m2x + n2 d2
A
The inclination of d1 is α and m1 = tanα.
The inclination of d2 is β and m2 = tanβ. q

The angle between d1 and d2 is ∠BAC and


m(∠BAC) = θ. B b a
x
O C
In the figure 64, α = θ + β, θ = α – β and
tanθ = tan(α – β).
Figure 64
tan α − tan β
In the equation tan( α − β) =
1+ tan α ⋅ tan β
tanα = m1 and tanβ = m2
m1 – m2
that is tan θ =
1+ m1 ⋅ m2
with respect to values of m1 and m2, tanθ can be positive, negative or zero, and if
tanθ > 0 ⇒ θ is acute angle.
tanθ < 0 ⇒ θ is obtuse angle.
tanθ = 0 ⇒ m(θ) = 0, d1 // d2.
tanθ is undefined ⇒ m(θ) = 90°, d1 ⊥ d2.
In general it can be said that the tangent of the acute angle between two lines is
| m1 – m2 |
tan β =
1+ m1 ⋅ m2

Remark
When two lines intersect there are two intersection angles θ and β such that θ + β = 180°.
tanθ = tan(180 – β) = –tanβ
m1 – m2
= –tanβ
1+ m1 ⋅ m2
m2 – m1
⇒ tanΒ =
1+ m1 ⋅ m2

Further Applications 276


EXAMPLE 83 Find the measures of the angles between the lines
d1 : y = –2x + 4 and d2 : y = 3x – 1.
y
Solution m1 = −2 ⎪⎫ m1 − m2 −2 − 3 −5 d1 : y = –2x + 4 d2 : y = 3x – 1
⎬ tan θ = = = =1
m2 = 3 ⎪⎭ 1 + m1 ⋅ m2 1 + ( −2) ⋅ 3 −5

45°
tanθ > 0 so θ is acute angle. 4

q=
and the tangent of the obtuse angle between these two b = 135°
2
lines is tan(180 – θ) = –tanθ = –1.
Now, let’s find the measures of the angles x
O
–1
tanθ = 1 so θ = 45°
θ + β = 180° ; 45 + β = 180° ; β = 135°

EXAMPLE 84 Find the measures of the angles between the lines y =ñ3x – 3 and y = –ñ3x + 3.

Solution y = − 3x − 3 ⇒ m1 = 3 ⎫⎪ m1 − m2 3 − ( − 3) 2 3
⎬ tan θ = = = =– 3
y = − 3x + 3 ⇒ m2 = – 3 ⎪⎭ 1+ m1 ⋅ m2 1+ 3 ⋅( − 3) 1 − 3

tanθ = –ñ3 ⇒ θ = 120°,


and the measure of the acute angle is 180° – 120° = 60°.

EXAMPLE 85 Find the measure of the acute angle between the lines 3x + y – 7 = 0 and x + 2y + 9 = 0

Solution 3x + y – 7 = 0 ; y = –3x + 7 ⇒ m1 = –3
–x – 9 1
x + 2y + 9 = 0 ; y= ⇒ m2 = –
2 2
1 5
| –3+ | | − |
| m1 − m2 | 2 = 2 =
tan θ = =
1+ m1 ⋅ m2 3 5
1+
2 2
tanθ = 1 so and the measure of the acute angle is 45°.

277 Analytic Analysis of Lines and Circles


Check Yourself 20

1. Find the tangent of the acute angle between the lines


2x + y + 1 = 0 and x – 3y – 1 = 0.
y
d1 d2

2. The equation of the lines are d1 : y = ax and d2 : y = 3x. If 45° 45°

the measure of the acute angle between d1 and d2 is 45°, find x


O
a.

A
a
3. In the figure if the coordinates of the points are A(0, 2),
B(1, 0) and C(4, 0) find the tangent of the angle between d1 O
x
B C
and d2. d1
d2

B. DISTANCE OF A POINT TO A LINE


Theorem
Let A(x1, y1) be a point and d : ax + by + c = 0 be a line, then the distance of A to a line d
is
| ax1 + by1 + c |
l=
a2 + b 2

Proof Let the distance of A(x1, y1) to a line y A(x1, y1)


d : ax + by + c = 0 be l = AH. d
a
Take C(x2, y2) = AD ∩ d H

x2 = x1 and y2 = CD C(x1, |CD|)

C is the point on the line ax + by + c = 0 then


a
ax1 + b ⋅ CD + c = 0 O
x
B D
b ⋅ CD = –a ⋅ x1 – c ;

a c
CD = − x1 −
b b
So we got the coordinates of C, Figure 65
a c
C( x1, – x1 – )
b b

Further Applications 278


α is inclination of d and
α = m(∠CBD) = m(∠CAH) (angles with perpendicular lines)
AH
In the right triangle ACH, cos α = ⇒ AH = AC ⋅ cosα ...(1)
AC
Now, let’s find the equivalent expressions for AC and cosα.
a c a c
AC = AD – CD = y1 − ( − x1 − ) ; AC = y1 + x1 + ...(2)
b b b b
we know sec2α = 1 + tan2α
1 a
then cos α = , and tan α = m = −
1+ tan 2 α b

1 1
so cos α = = ...(3)
a a
1+( − )2 1 + ( )2
b b
Let’s substitute (2) and (3) in (1)
AH = l = AH = AC ⋅ cosα

a c
y1 + x1 + ax + by1 + c
a c 1 b b
l = ( y1 + x1 + ) ⋅ = = 1 .
b b a2 1 2 2 a 2
+ b 2
1+ a +b
b2 b

EXAMPLE 86 Find the distance of the point O(0, 0) to the line x – y + 4 = 0.

Solution O(0, 0) = O(x1, y1)

| ax1 + by1 + c | |0 − 0+ 4| 4 4 2
l= = = = = 2 2.
2
a +b 2 2
1 +( −1) 2
2 2

EXAMPLE 87 Find the distance of A(5, 2) to the line 3x – 4y + 5 = 0.

Solution A(5, 2) = A(x1, y1)


| ax1 + by1 + c | |3 ⋅ 5 − 4 ⋅ 2+5| 12
l= = = = 2,4
a 2 + b2 32 + 42 5

279 Analytic Analysis of Lines and Circles


EXAMPLE 88 The distance of A(12, 5) to the line 5x – 12y + 5k = 0 is 10 units. Find positive value of k.

Solution | ax1 + by1 + c | |5 ⋅12 – 12 ⋅ 5+5 k|


l= = =10
a 2 + b2 25+144
|5k |
l= = 10 ; 5k = 13 ⋅10 ; k = 26.
13

Check Yourself 21

1. Find the distance of the point P(–2, 3) to the line 3x + 4y + 9 = 0.

2. Find the distance of the point A(1, 4) to the line y = 3x – 4.

3. If the distance between the point P(3, k) and the line 4x – 3y + 5 is 4 units, find k.

C. DISTANCE BETWEEN TWO PARALLEL LINES


Let d1 : a1x + b1y + c1 = 0
d2 : a2x + b2y + c2 = 0 be two parallel lines.
a b
Since d1 // d2, we can write 1 = 1 = k so. a1 = k ⋅ a2 and b1 = k ⋅ b2
a2 b2
Now, let’s substitute these values into d1
k ⋅ a2x + k ⋅ b2y + c1 = 0;
c
k(a2x + b2y + 1 ) = 0
k
c1
k ≠ 0, so we get d1 : a2x + b2y +=0
k
When we compare d1 with d2, we see that the only difference is in constant number.
In general, parallel lines can be written like

d1 : a1x + b1y + c1 = 0
d2 : a2x + b2y + c2 = 0

Theorem
Let d1: ax+ by + c1 = 0 and d2 : ax + by + c2 = 0 be two parallel lines then the distance
|c − c |
between d1 and d2 is l = 2 1 .
a2 + b 2

Further Applications 280


Proof The distance of any point A(x, y) on line d1 to the line d2 is
A(x, y)
| ax + by + c2 |
l= . d2
a2 + b 2 l

| c2 − c1 |
In the equation of d1, ax + by = – c1 hence l = .
a2 + b 2 d1

Figure 65

EXAMPLE 89 Find the distance between the parallel lines x – 2y + 5 = 0 and 3x – 6y + 9 = 0.

Solution It is important to notice that to find the distance between two parallel lines, first of all the
coefficients of x and y must be equalized.
d1 : x − 2 y +5 = 0 ⎫⎪ 3x − 6 y +15 = 0 ⎫⎪
⎬ ⇒ ⎬ d1 was multiplied by 3.
d2 : 3x − 6 y +9 = 0 ⎭⎪ 3x − 6 y + 8 = 0 ⎭⎪

Now, we have c1 = 15 ⎫⎪ | c1 − c2 | 15 – 9 6 2 5
⎬ ⇒ l= = = = .
c2 = 9 ⎪⎭ 2
a +b 2
9+ 36 45 5

EXAMPLE 90 Find the distance between the parallel lines 3x – 2y + 5 = 0 and –3x + 2y + 8 = 0.

Solution d1 : 3x − 2 y +5 = 0 ⎫⎪ d1 : 3 x − 2 y +5 = 0 ⎫⎪
⎬ ⇒ ⎬
d2 : −3x + 2 y +8 = 0 ⎭⎪ d2 : 3 x − 2 y − 8 = 0 ⎭⎪

So, c1 = 5 ⎫⎪ | c1 − c2 | |5+8| 13
⎬ ⇒ l= = = = 13.
c2 = –8 ⎪⎭ 2
a +b 2 2
3 +(–2) 2
13

Another point of view in solution of problem:


3 5 ⎫
d1 : y = x + = 0 ⎪
d1 : 3x − 2 y +5 = 0 ⎫⎪ 2 2 ⎪
⎬ ⇒ ⎬
d2 : −3x + 2 y +8 = 0 ⎪⎭ 3
d2 : y = x − 4 = 0 ⎪
2 ⎪⎭
3 5 3
then m1 = , n1 = for d1 ; m2 = , n2 = –4 for d2
2 2 2
m1 = m2 and n1 ≠ n2 so d1 // d2.

281 Analytic Analysis of Lines and Circles


The distance between d1 and d2 is same with the distance of any point of one line to other
line.

For example A(0, – 4) is one point on d2, and the distance of A to d1 is


| 3 ⋅ x1 − 2 ⋅ y1 +5| | 3 ⋅ 0 − 2(– 4)+5| 13
l= = = = 13.
2
3 +(–2) 2
13 13

Check Yourself 22

1. Find the distance between the lines 4x – 3y – 5 = 0 and –12x + 9y + 4 = 0

2. The lines x + 2y + 1 and 3x + 6y + k = 0 are parallel lines and the distance between
them is ñ5. Find k.

3. Find the area of a square whose two sides are on the parallel lines 2x + y – 3 = 0 and
4x + 2y + 7 = 0

D. EQUATION OF AN ANGLE BISECTOR


Theorem
Let d1 : a1x + b1y + c1 = 0 and d2 : a2x + b2y + c2 = 0 be two intersecting lines, then the
equation of the angle bisector is
a1x + b1y + c1 a2 x + b2 y + c2
=∓
2 2
a +b
1 1 a22 + b22

Proof Let l1 and l2 be angle-bisectors of ∠A as shown in the d1


y l2 d2
figure 67.
C
Let us take a point B(x, y) on l1.
b b
since B is a point of the bisector, it is at the same a
l1
distance to the lines d1 ad d2, namely BC = BD. A a B(x, y)

If we use the distance formula to find the distance of D


the point B to the lines d1 and d2 O
x

| a1x + b1y + c1 | | a2 x + b2 y + c2 |
we get = .
a12 + b12 a22 + b22 Figure 67

And that’s all for the equation of bisectors.


The equations of bisectors are
a1x + b1y + c1 a2 x + b2 y + c2 a1x + b1y + c1 a2 x + b2 y + c2
= and =– .
2 2 2 2 2 2
a +b
1 1 a +b
2 2 a +b
1 1 a22 + b22

Further Applications 282


EXAMPLE 91 Find the equation of the bisectors of the angles formed by intersecting of the lines
2x – y + 3 = 0 and 2x + y – 1 = 0.

Solution 2x − y + 3 2x + y − 1 y
=∓ 2x + y – 1 = 0 2x – y + 3 = 0
22 +(–1)2 2 2 +12

2 x − y + 3 = ∓ (2 x − y − 1) l1 : y = 2
2
2 x − y + 3 = 2 x + y − 1 ⎫⎪ 2 y = 4 ⎪⎫ 3 1
–2
2
⎬ ⇒ ⎬ x
2 x − y + 3 = –2 x − y +1⎭⎪ 4 x = −2 ⎭⎪ –2 – 1 O
2

y = 2 ... l1 ⎫
⎪ 1
−1 ⎬ l2 : x = – 2
x= ... l2 ⎪
2 ⎭
and it is not surprise to see that l1 ⊥ l2.

EXAMPLE 92 Find the equations of the bisector of the angles y

formed by intersecting of the lines x = 3 and l2


O 3 8
y = –5. x
–2
–2
Solution d1 : x = 3 or x − 3 = 0 ⎫⎪
⎬ 45°
d2 : y = –5
d1 : y = –5 or y +5 = 0 ⎪⎭ –5 45°

x−3 y +5 –8
=∓ ⇒ x − 3 = ∓( y +5) l1
12 − 02 02 +12 d1 : x = 3

⎧⎪ x + y + 2 = 0 ... l1 Figure 68
⇒ ⎨
⎪⎩ x – y – 8 = 0 ... l2 are the equations of bisectors.
as seen at the figure 68.

y
EXAMPLE 93 In the figure ABCD is a square. A(4, 0) and B(0, 6)
C
are vertices of the square. Find the equation of
bisectors of angles formed by lines AB and AD. B(0, 6)
D

x
O A(4, 0)

283 Analytic Analysis of Lines and Circles


Solution Draw [CH] ⊥ Oy. y
C
Since ∆BOA ≅ ∆CHB (A.S.A similarity theorem.)
b
the coordinates of C are (6, 10). a
B(0, 6)
One of the bisectors of BA and AD is AC, because ABCD is
a square and diagonal is also bisector of interior angle. b D

By using two-point line equation, we can write the a


x
equation of AC: A(4, 0) and C(6, 10) so O A(4, 0)
y−0 x−4
= ⇒ 5x − y − 20 = 0.
0 − 10 4 − 6
The other bisector is the line passing through A and perpendicular to AC.
1
Slope of AC = 5, so slope of other bisector = − , by using slope and one-point line equa-
5
tion, we can write
1
y – 0 = − (x – 4) ; 5y = – (x – 4) ; x + 5y – 4 = 0.
5
In fact, we found the equations of the bisectors without using the formula. If you find the
equations of AB and AD, first, you can also use the formula to find the equations of the bisec-

Check Yourself 23
1. Find the equations of the bisectors of the angles between the lines 2x – y + 3 and
x + 2y + 1 = 0. y
d1
C
2. In the figure d1 and d2 are intersecting at D. The points D

E
A(–2, 0), B(2, 0), C(0, 4) and E(0, 1) are given. Find the equa-
tions of the bisectors of angle D. A
x
O B

d2

A
3. In the figure the equation of line d is 4x + 3y – 24 = 0. If BF
and BE are the bisectors of the angle B, find their equations. F
B
x
O
d

Further Applications 284


EXERCISES 3
A. Angle Between Two Lines 8. If the distance of P(3, –2) to the line 4x – 3y + k = 0
1. Find the tangent of the angle between the lines is 4 units, find k.
8x + 6y – 48 = 0 and 3x – 4y + 36 = 0.

9. The points A(1, 3), B(–2, 1) and C(3, –1) are the
2. If the measure of the angle between y = ñ3 and vertices of the tirangle ABC. Find the length of
y = mx is 30°, find m. the altitude of BC.

3. In the figure y
d1 d2 1
the lines 10. If the distance of P( , k) to the line 4x + 3y – 1 = 0
2
d1 : 3x + y = 0 A B
is 2 units, find k.
d2 : 2x – y = 0 C
d3
d3 : ax – y = 0
are given. x
O C. Distance Between Two Parallel
Lines
11. If the distance between the parallel lines
If m(∠AOB) = m(∠BOC) find a. 12x + 9y – 2 = 0 and ax + 3y + c = 0 is 3 units,
a
find the ratio .
c
4. Find the measures of the interior angles of the
triangle whose sides are on the lines x + y – 5 = 0,
x – y + 1 = 0 and y – 1 = 0. 12. Write the equation of lines which are at 4 units
distance to the line 3x + 4y + 10 = 0.

B. Distance of a point to a line


5. Find the distance of the point A(–2, 3) to the line 13. If the distance between the parallel lines
8x + 6y – 15 = 0.
3x + 4y – 6 = 0 and 4x – ky + 4 = 0 is p, find
k + p.
6. If the distance of B(2, 3) to the line 12y – 5x = k
5
is , find k.
13
D. Equation of an Angle Bisector
14. Write the equation of the bisectors of the angles
7. If the distance of a line with equation formed by the lines 3x – 4y + 10 = 0 and
y – 4 = m(x + 2) to the origin is 2, find m. 12x + 5y – 24 = 0.

285 Analytic Geometric


15. Write the equation of the bisectors of the angles 25. In the figure the y
d1
formed by the lines 3y = 2x + 1 and 3x = –2y – 4. lines d1 and d2 are
intersecting at B.
If m(∠ABC) = 45°, A d2
16. Find the equation of locus of the points which are
at the same distance to the lines x + 3y – 1 = 0 A(0, k),
B(–4, –2) and x
and 3x – y + 2 = 0. O C
C(2, 0) then find k.
45°

Miscellaneous Exercises
B
17. Find the angle between the lines joining the
points (0, 0), (2, 3) and (2, –2), (3, 5).
26. In the figure the
d2 y
equation of d1 is
18. Find the measures of the interior angles of the
x + y = 0, d1
triangle whose vertices are the points
A(ñ6, 0)
A(10, 5ñ3 + 2), B(5, 2), C(10, 2).
and d1 // d2.
If ∆ABC is A
x
O
19. Find the angle between the lines whose x and equilateral B
y - intercepts are a, –b and b, –a, respectively. triangle find its
area. C

20. Find the equations of two straight lines passing


through the point (4, 5) and forming an acute y
angle of 45° with the line 2x – y + 7 = 0. d1

27. The lines d1 and


d2 cut each
21. Write the equations of the lines parallel to the
other at A(0, –2). d2
lines –3x + 4y – 16 = 0 at a + 3 units distance to
Find tanα. B(1, 0)
the point P(1, 1). x
O C(3, 0)

22. If the distance of a point on the line y = 2x to the


A(0, –2)
line 3x + 4y – 12 = 0 is 2 units find its coordinates.

28. In the figure y


23. Find the equations of the lines passing through AB ⊥ d2, d1
A(1, 0) and forming an angle of 45° with the line the equation of A
d2
x + y + 1 = 0. d1 is y = 3x and
of d2 is x – y = 0. B

24. Find the lengths of altitudes of the triangle whose If AB = 2ñ2, x


O
sides are given by x – 4y = 5, 4x + 3y = 5, and find the area of
x + y = 1. ∆AOB.

Analytic of Line 286


CHAPTER SUMMARY
• There is one - to - one correspondence between the • Equations of lines are written in two different forms:
points of the analytic plane and the cartesian coordinates. 1) ax + by + c = 0, a, b, c ∈
Points can be represented by two components abscissa
and ordinate, A(x, y). 2) y = mx + n, m, n ∈
• The distance between the points A(x1, y1) and B(x2, y2) is • The equation of the line passing through two fixed points
A(x1, y1) and B(x2, y2) is
AB = ( x2 − x1 )2 +( y2 − y1 )2
y − y1 x − x1
• The coordinates of the midpoint of a line segment AB =
y2 − y1 x2 − x1
with endpoints A(x1, y1) and B(x2, y2) are
• The equation of the line whose x and y - intercepts are
x + x2 y + y2 x y
x0 = 1 and y0 = 1 known is + =1 where a is the x - intercept and b is
2 2 a b
• Given a line segment AB with A(x1, y1) and B(x2, y2). the y - intercept of the line.
C(xc, yc) is the point which divides AB in the ratio • y = 0 is the equation of x - axis
CA
=k
x = 0 is the equation of y - axis.
CB • Slope of the lines whose equations are given in the form
If C divides AB internally of y = mx + n is the coefficient of the variable x.
• Slope of the lines whose equations are given in the form
x + k ⋅ x2 y + k ⋅ y2 a
xc = 1 and yc = 1 of ax + by + c = 0 is m = − .
1+ k 1+ k b
• Given two lines d1: a1x + b1y + c1 = 0 and
If C divides AB externally
d2 : a2x + b2y + c2 = 0
x − k ⋅ x2 y − k ⋅ y2
xc = 1 and yc = 1 a1 b1 c1
1+ k 1+ k 1) If = = then the lines are coincident.
a2 b2 c2
• Given a triangle ABC with vertices A(x1, y1); B(x2, y2), and
a1 b1 c1
2) If = ≠ then the lines are parallel.
x + x2 + x3 y1 + y2 + y3 a2 b2 c2
C(x3, y3); the centroid is G( 1 , )
3 3
a1 b1
1 3) If ≠ then the lines intersect at one point.
and ∆ABC = |x1(y2 – y3) + x2(y3 – y1) + x3(y1 – y2)| a2 b2
2
• The equation a1x + b1y + c1 + k ⋅ (a2x + b2y + c2) = 0
• The angle made by a line with the positive direction of represents the equation of a bunch of lines.
x - axis measured in the counter - clockwise direction is
called inclination of the line. • The regions of the analytic plane separated by a line
• If α is the inclination of a line, tangent of α is called the ax + by + c = 0 is represented by the inequalities
slope of the line, m = tanα. ax + by + c < 0 and ax + by + c > 0.
• The slope of a line passing through the points A(x1, y1) • The tangent of an acute angle between lines y = m1x +
y – y1 | m1 − m2 |
and B(x2, y2) is m = 2 . n1 and y = m2x + n2 is tan θ = .
x2 − x1 1 + m1 ⋅ m2
• If d1 // d2 then m1 = m2 • The distance of a point A(x1, y1) to a line
• If d1 ⊥ d2 then m1 ⋅ m2 = –1 d : ax + by + c = 0 is

• The equation of a line which is passing through the point | ax1 + by1 + c |
l=
A(x, y) with slope m is y – y1 = m ⋅ (x – x1). a2 + b 2

Symmetry 287
• The distance between parallel lines Concept Check
d1 : a1x + b1y + c1 = 0 and d2 : a2x + b2y + c2 = 0 is
1. What is analytic plane?
| c2 – c1 |
l= 2. How can a point be represented on a plane?
a2 + b 2
3. Define a line and find examples from daily life as a
• The equations of an angle bisectors between lines sample to a line.
d1 : a1x + b1y + c1 = 0 and d2 : a2x + b2y + c2 = 0 is 4. Derive distance formula between two points.
5. Derive midpoint formula of a line segment.
a1x + b1y + c1 a2 x + b2 y + c2
=∓ 6. Derive formula for the coordinates of a point which
a12 + b12 a22 + b22 divides a line segment in a given ratio.

• Symmetry of a point A(x0, y0) with respect 7. Find the coordinates of the intersection point of diagonals
of a rectangle on analytic plane.
to a point B(a, b) is A1(2a – x0, 2b – y0)
to x - axis is A2(x0, – y0) 8. Find area of a rectangle.
to y - axis is A3(–x0, y0) 9. Derive formula to find the area of a parallelogram.
to a line x = a is A4(2a – x0, y0) 10. Explain the meaning of the inclination and the slope.
to a line y = b is A5(x0, 2b – y0)
11. Derive the formula for the slope of a line whose two points
to the line y = x is A6(y1, x1) are given.
to the line y = –x is A7(–y1, –x1)
12. What is the relation between the slopes of parallel lines?
• Symmetry of a line ax + by + c = 0 with respect
13. Prove that the product of the slopes of perpendicular lines
to a point B(p, q) is a(2p – x) + b(2q – y) + c = 0 is –1.
to a line ax + by + m = 0 is ax + by + (2m – c) = 0
to the line y = x is bx + ay + c = 0 14. Derive the formula for the equation of a line whose slope
to the line y = –x is –bx – ay + c = 0 and one point are given.
to x - axis is ax – by + c = 0 15. What is the relation between the lines y = mx and
to y - axis is –ax + by + c = 0 y = mx + n?
to a line x = m is a(2m – x) + by + c = 0
16. Derive the formula for the equation of a line whose two
to a line line y = n is ax + b(2n – y) + c = 0
points are given.
17. Explain, why the line x = 5 is parallel to y - axis?
18. What are the conditions of two lines?
19. How can we decide that the angle between two lines is an
acute angle?
20. How can we decide that lines are parallel to one another?
21. What is the procedure to find the area of a square whose
two sides are on parallel lines?
22. Show that the angle bisectors of two intersecting lines are
perpendicular.
23. List the procedure to find the equations of the angle
bisectors.
24. What is symmetry, give some examples to the symmetry.
25. Try to draw symmetric figures with respect to a line and a
point.
26. List the steps to find the symmetry of a point with respect
to a line
27. Verify the formula to find the symmetry of a line with
respect to a parallel line.

288 Analytic Analysis of Lines and Circles


CHAPTER REVIEW TEST 1
1. What is the length of the median passing through 5. With respect to y
d1
vertex A of ∆ABC whose vertices are A(4, 7), the figure, what
B(–1, 2), and C(3, 4)? is tanx? d2
4
A) 5 B) 6 C) 7 D) 8 E) 10

x 2

x
–3 O

6 6 3 4 2
A) B) C) D) E)
17 13 10 9 5
2. If the points A(–2, –1), B(2, k), and C(4, 5) are on
the same line, what is k?

A) 1 B) 2 C) 3 D) 4 E) –4

6. If the lines x + 3y – 8 = 0 and kx + y – 6 = 0


intersect on the line 2x – y – 2 = 0, what is k?

A) 1 B) 2 C) –2 D) 3 E) –3

3. What is m, if the slope of the line


1
2mx + (m –1)y + 1 = 0 is – ?
2
1 1 1 1 1 7. If the distance of P(k, 1) to the line
A) – B) C) D) − E)
3 2 4 4 5 x – 2y + 2 = 0 is 2ñ5, what is the sum of the
possible values of k?

A) 0 B) 1 C) 2 D) –1 E) –2

4. If the centroid of ∆ABC with the vertices


A(–2, 1), B(1, 5), and C(4, –1) is on the line
8. If A and B are x - intercepts of the line
2x + y + k – 1 = 0, what is k?
5x + 12y – 60 = 0, what is AB?
1 5 8 10
A) –1 B) − C) − D) − E) − A) 12 B) 13 C) 14 D) 15 E) 20
3 3 3 3

Symmetry 289
9. If the angle between mx – y + 2 = 0 and 13. If the symmetry of (k – 1)x + 2y – 6 = 0 with
2x + y – 1 = 0 is 45°, which one may be a value respect to y = x is passing through A(2, 1), what
of m? is k?

A) 0 B) –1 C) 2 D) –2 E) 3 A) 1 B) –2 C) 3 D) 4 E) –4

10. Given A(–1, 2), B(4, –3). C is on the line segment


14. In the figure, the length y (length)
AC 3 A
AB and = . of a plant with respect to
AB 5
years is given. What is 5
What is the sum of the coordinates of C?
the length of the plant in
A) 1 B) –1 C) 2 D) –2 E) 3 th 8th year? 2

x
O 3 (year)

A) 9 B) 10 C) 12 D) 15 E) 20
11. Which of the following lines has the greatest
inclination?

A) 2x – y + 1 = 0 B) x + 2y – 1 = 0
C) y = 5x + 4 D) y = ñ3x – 1
15. What is the equation of the line passing through
E) y = x + 3
the intersection point of the lines
(k – 1)x – (2k + 1) y + k – 2 = 0,
and parallel to the line x + 3y – 1?

A) x + 3y + 1 = 0 B) 2x + y – 1 = 0
12. In the figure, y d1 C) x + 3y + 3 = 0 D) 3x + y – 4 = 0
AOBC is a rectangle.
A C E) x + 3y – 4 = 0
d1 : y = 2x and
x
d2 : y = , d2
2
A(0, 8). D

What is the x
O B
area of ∆COD? 16. Given the points A(2, 1), B(–2, 3), C(5, 4), and
D(k, 1). If AC ⊥ BD, what is k?

A) 6 B) 9 C) 12 D) 15 E) 18 A) –4 B) –3 C) –2 D) –1 E) 4

290 Analytic Analysis of Lines and Circles


CHAPTER REVIEW TEST 2
1. If the points P(m, n) and R(x, y) are on the line 5. What is the equation of the locus of points which
m−x are equidistant to the lines x + 3y – 1 = 0 and
4x – 3y – 1 = 0, what is ?
n−y 3x – y + 4 = 0?
4 3 1 3
A) B) C) D) E) 1 A) 4x + y – 1 = 0 B) 2x – 4y + 4 = 0
3 4 3 2
C) 4x + 2y + 3 = 0 D) x + 2y + 4 = 0
E) 2x + 3y + 1 = 0

y
6. In the figure ∆COD is an
equilateral triangle. A
2. If the symmetry of P(3, 5) with respect to A(∆COD) = 12ñ3 and
30°
P(k – 1, 4) is on the y - axis, what is k? m(∠CAD) = 30°. C
What is the equation of AB? D
5 9
A) B) 3 C) 4 D) E) 5
2 2
x
O B

A) y = –ñ3x + 6 B) y = 3x – ñ3
C) y = –ñ3x + 8ñ3 D) y = ñ3x – 8ñ3
E) y = 3x + 8ñ3
3. Given A(–1, 3) and B(4, –2). What is the equation
of the line passing through the origin and
perpendicular to AB? 7. In the figure, P(0, k) is y

moving on the y - axis. B(4, 5)


A) x – y = 0 B) x + 2y – 1 = 0 A(3, 1) and B(4, 5) are
C) y = 2x D) y = x – 1 given. What is k for
P(0, k)
x the smallest value of
E) y =
2 the sum PB + PA? A(3, 1)
x
O

19 17 15 13 10
A) B) C) D) E)
7 5 4 3 3

4. What is the abscissa of the point which is on the 8. If the symmetry of a point on the line
line y = –2x + 5 and at the same distance to he ax + 3y + 1 = 0 with respect to the line
points A(–1, 2) and B(1, 4)? x + y = 0 is A(2, –1), what is a?

A) 1 B) –1 C) –2 D) 2 E) 3 A) 3 B) 4 C) 5 D) –3 E) –4

Symmetry 291
9. If A and B are the x and y - intercepts of the line 13. If the vertices of ∆ABC are B(–3, 3), C(–3, 5) and
x + 2y – 12 = 0, what is the equation of the A is the intersection of the lines
median of hypotenuse in ∆AOB? (m – 1)x + (m + 2)y – m + 4 = 0
what is the area of ∆ABC?
A) x + 2y = 0 B) x – 2y = 0
C) x – y = 0 D) 2x + y = 0 A) 10 B) 15 C) 20 D) 35 E) 40

E) y = 3x

10. In the figure if y

CH ⊥ AB, what is 14. If A(a ⋅ b, a – b) is on the 2nd quadrant of the


A(0, 3)
CH? analytic plane, in which quadrant is B(b2a, b – a)?
H
A) I B) II C) III
B(4, 0)
x D) IV E) at the origin
O

C(0, –2)

A) 10 B) 8 C) 6 D) 5 E) 4
15. A(2, 5), B(4, 1), p(a, o) are given. For the
11. A(1, 2) and B(5, 4) are endpoints of AB. The lines maximum value of PA – PB, what is a?
d : 3x + 4y – 12 = 0 and AB are intersecting at a 9 11
A) 4 B) C) 5 D) E) 6
PA 2 2
point P. What is the ratio ?
PB
1 1 1 1 1
A) B) C) D) E)
19 17 12 8 5

16. y
12. The equation of d is y
d d1
x + y – 6 = 0.
L d2
OABC is a square C
F
and OA = 2 units. B
A
If BH ⊥ KL, what is H x
O D E
BH? C B

K In the figure, CDEF is a square. d1 : y = x is


x
O A passing through C, and d2 through A(–3, 0) and
B(0, 2). What is the area of the square?
2 1
A) 1 B) C) D) ñ2 E) 2ñ2
2 2 A) 16 B) 27 C) 32 D) 36 E) 49

292 Analytic Analysis of Lines and Circles


CHAPTER REVIEW TEST 3
1. In the line d : ax + by + c = 0, a ≠ b ≠ c and 5. If the points A(2, 3) and B(1, –2) are on different
a, b, c are integers. If the area of the triangle sides of the line 3x – y + k = 0, what is the
5c number of integer values of k?
formed by the axes and the line d is , what is
2ab
the value of c?
A) 1 B) 2 C) 3 D) 4 E) 5
5
A) 5 B) 4 C) 3 D) E) 2
2

2. y 6. What is the equation of the line which is parallel


d
to the line 4x – 5y + 6 = 0 and intersecting with
the line y – x – 3 = 0 at y - axis?
B
K A) 5y – 4x = 0 B) 2y – x + 6 = 0
x C) 3y – x + 9 = 0 D) 5y – 4x – 15 = 0
A L O
E) 5y – 4x + 15 = 0
In the figure KL ⊥ LB, KL = LB.
d : 2x – 3y + 30 = 0, A(∆KLB)=?

A) ò26 B) 2ò26 C) 3ò13 D) 2ò13 E) 13

7. What is the equation of the line which is parallel


to the line x + y = 0 and passing through the
intersection points of the buhch of lines
3. The points A(1, –1), B(a, b), and C(9, 9) are
AC 5 x(2 + m) – y(1 – 2m) + 3m = 0?
given. If A, B, C are collinear and = , what
BC 3 A) x + y + 1 = 0 B) 2x + 2y + 3 = 0
are the coordinates of B?
C) 3x + 3y + 4 = 0 D) x + y – 1 = 0
A) (–3, 4) B) (–3, 3) C) (4, –3) E) 5x + 5y + 9 = 0
D) (4, –4) E) (4, 5)

8. What is the area of the triangle formed by the


4. What is the abscissa of the closest point of the
lines x + 2y – 4 = 0, 3x + 2y – 12 = 0, and
line x – 2y + 1 = 0 to the point A(1, 2)?
y - axis?
7 6 4 4 7
A) B) C) D) − E) −
5 5 5 5 5 A) 4 B) 12 C) 16 D) 24 E) 32

Symmetry 293
9. If the symmetry of the origin with respect to 13. If the area of ∆ABC with the vertices A(x, y),
x + y – k is P and OP = 6ñ2, what is k? B(2, 1) and C(3, 2) is 3 units, what is the
equation of locus of the points A(x, y)?
A) 3 B) 4 C) 5 D) 6 E) 9
A) x – y + 7 = 0 B) x + y – 7 = 0
C) x – y – 7 = 0 D) 2x – y + 7 = 0
E) x + 2y – 7 = 0

14. ABCD is a square and y


10. The side BC of the square ABCD is on the line d
A(–1, 0), B(0, 2).
3x – 4y + 2 = 0. If A(2, –3), what is the area of
What is the equation F
ABCD?
of d?
C
A) 64 B) 49 C) 32 D) 24 E) 16
D B

x
E A O

A) x – y + 7 = 0 B) 2x – y + 7 = 0
C) x + 2y + 7 = 0 D) x + y + 14 = 0
11. For a > 0, b > 0 if the area of a triangle formed E) 3x – y + 9 = 0
by the line ax + by + 2 = 0 and the axes is 4,
what is a ⋅ b?
1 1 1 2 1
A) B) C) D) E)
2 3 4 5 8 x y
15. What is the symmetry of + =1 . With respect
x y 3 1
to the line + =1?
2 4
A) x – y – 5 = 0 B) x + 3y – 5 = 0
C) 3x + y – 5 = 0 D) 3x – y + 9 = 0
E) 3x – y – 5 = 0

12. If A(a, b) is a point on the line passing through


the origin and the intersection point of the lines
16. If the vertices of the rhombus are A(0, 0), B(a, b),
2x – y – 4 = 0 and 3x + y – 3 = 0, what is the a+ b
a C(c, 16), and D(0, 10), what is the ratio ?
ratio ? c
b
7 6 5 7 5 5 7 9 7 9
A) B) C) D) E) A) B) C) D) E)
6 7 7 5 6 2 2 2 4 4

294 Analytic Analysis of Lines and Circles


CHAPTER 4

ANALYTIC ANALYSIS
OF CIRCLE
1 EQUATION OF CIRCLES
In the plane geometry, the circle is defined as “the set of all points which are equidistant from
a fixed point.”
On the analytic plane, a circle can be represented by an equation which is the relation between
the points of the circle and the fixed elements, the centers and the radius of the circle.

A. STANDARD EQUATION OF A CIRCLE


Theorem
On the analytic plane, the equation of a circle with the center C(a, b) and the radius r is
(x – a)2 + (y – b)2 = r2.
y
Proof Let P(x, y) be any point of the circle,
C(a, b) is the center and r is the radius of the center.
C(a, b)
Since r = CP, by using distance formula we can write that b r

CP = ( x − a)2 + ( y − b )2 = r
P(x, y)
by taking the square of both sides of the equation,
x
we get (x – a)2 + (y – b)2 = r2. O a

Figure 1

EXAMPLE 1 Write the equation of the circle with the center C(–2, 3) and the radius r = 5.

Solution C(a, b) = C(–2, 3) and r = 5,


The equation of the circle from is (x – a)2 + (y – b)2 = r2 and we find
(x – (–2))2 + (y – 3)2 = 52 ; (x + 2)2 + (y – 3)2 = 25.

2 1
EXAMPLE 2 Find the center and the radius of the circle ( x − )2 +( y +1)2 = .
3 4
2 1
Solution If we compare the equations (x – a)2 + (y – b)2 = r2 and ( x − )2 +( y +1)2 = then we can
3 4
2 1
easily see that C(a, b) = ( , – 1) and r = .
3 2

296 Analytic Analysis of Lines and Circles


Remark
1. The equation of the circle 2. The equation of the circle 3. The equation of the circle
whose center is at the origin whose center is on the whose center is on the
O(0, 0) with radius r is x - axis with the radius r is y - axis with the radius r is
x2 + y2 = r2 (x – a)2 + y2 = r2 x2 + (y – b)2 = r2
y y y

P(x, y) P(x, y)
P(x, y)
r
r r
C(0, b)
x x
O(0, 0) O C(a, 0)

x
O
(x – 0)2 + (y – b)2 = r2
(x – 0)2 + (y – 0)2 = r2 (x – a)2 + (y – 0)2 = r2

Figure 2 Figure 3 Figure 4

EXAMPLE 3 Find the equation of the circle whose center is at the origin and the radius is 3 units. Show
it in the analytic plane. y

Solution C( a, b ) = C(0, 0) ⎫⎪
⎬ and the formula is
r = 3 units ⎪⎭ r=3
x
2
(x – a) + (y – b) = r 2 2 O(0, 0)
O
so (x – 0)2 + (y – 0)2 = 32
x2 + y2 = 9.

EXAMPLE 4 Determine the radius and the center of the circle (x + 3)2 + y2 = 16, and sketch the graph.
y
Solution We write the given equation in the form
(x – (–3))2 + (y – 0)2 = 42 and compare with the equation
4
(x – a)2 + (y – b)2 = r2.
x
O
From here, we get C(a, b) = C(–3, 0) C(–3, 0)

and r = 4 units.

Equation of Circles 297


EXAMPLE 5 Determine the center and the radius of the circle x2 + (y + 2)2 = 7, and sketch its graph.
y
Solution We arrange the given equation in the form
(x – 0) + (y – (–2))2 =(ñ7)2 O
x
Thus, we get C(a, b) = C(0, –2) r=ñ7
r = ñ7 units. C(0, –2)

Check Yourself 1
1. Write the equations of the circles with the centers and the radii given below.

a. C(0, 1), r=2 d. C(3, –1), r=3


b. C(–2, 0), r = ñ2 e. C(4, –4), r = ñ7
2
c. C(0, 0), r = f. C(–ñ2, ñ3), r = ò10
3
2. Find the center and the radius of the circles with the given equations, and then sketch
the graph of each.
1
a. x2 + y2 = 3 c. (x – 2)2 + (y + ñ3)2 =
3
b. x2 + (y + 1)2 = 9 d. (x + 2)2 + y2 = 2

EXAMPLE 6 Find the equation of the circle whose diameter is AB with endpoints A(–3, 6) and B(5, 12).

Solution Since AB is the diameter, the midpoint of AB is the center of


the circle and CA = CB = r B(5, 12)
−3+5 6+12
So, the center C(a, b) = C( , )
2 2
C
= C(1, 9) and the radius
r = CA = ( −3 − 1)2 +(6 – 9)2 = 16+9 = 5 units
A(–3, 6)
We get the equation (x – 1)2 + (y – 9)2 = 25.

EXAMPLE 7 Determine the equation of the circle with the center C(1, –3) and passing through the point
P(–2, 1).

Solution Since the point P(–2, 1) is on the circle, then


r = CP = (1 − (–2)) 2 +(–3 – 1) 2 = 5 units.
The equation of the circle with the center C(1, –3) and radius r = 5 units is written as
(x – 1)2 + (y + 3)2 = 25.

298 Analytic Analysis of Lines and Circles


EXAMPLE 8 Write the equation of the circle with the center C(2, –3) and tangent to the line
4x – 3y + 3 = 0

Solution The distance of the center to the tangent of the circle


is equal to the radius. Hence; by the C(2, –3)
formula of the distance of a point to a line;
r
|4 ⋅ 2 − 3( −3)+ 3| T
r = CT = = 4 units.
42 +( −3)2

Thus, the equation is (x –2)2 + (y + 3)2 = 16. 4x – 3y + 3 = 0

EXAMPLE 9 Find the equation of the circle passing through the points A(–2, 3) and B(–3, 4) whose
center is on the line y = 2x.

Solution Let C(a, b) be the center of the circle. Since the center y=2x
y
is on the line y = 2x, we can write b = 2a, C(a, b) = C(a, 2a)
12
and C(a, b) = C(a, 2a).
On the other hand;
r = CA = CB
r
( a + 2)2 +(2 a – 3) 2 = ( a+ 3) 2 +(2 a – 4) 2 r

a2 + 4a + 4 + 4a2 – 12a + 9 = a2 + 6a + 9 + 4a2 – 16a +16


4
2a = 12 B
3
A
a=6
now let’s find the radius x
–3 –2 6
AC = r = ( a + 3)2 +(2 a – 4) 2 = 9 2 + 8 2 ; r = 145
Figure 5
C( a, 2a) = C(6, 12) ⎫
⎪ 2 2
⎬ ( x – 6) +( y – 12) =145 is the equation of the circle.
r = 145 ⎪⎭

Equation of Circles 299


EXAMPLE 10 Find the equation of the circle which is tangent to the lines y + 1 = 0 and y – 5 = 0 and
whose center is on the line 2x – y = 4.

Solution Since the circle is tangent to the lines y = –1 and y


y = 5, the center will be on y = 2 which is equidistant
y = 2x – 4
from these lines.
y=5 5 (a, 5)
Since the simultaneaous solution of the equations
2x – y = 4 and y = 2 is (3, 2), the center of the circle
is C(3, 2). 2 C(3, 2)
5 − (–1) 6 x
And the radius r = = = 3 units. O
2 2
y = –1 –1 (a, –1)
So, (x – 3)2 + (y – 2)2 = 9 is the equation of the circle.

Check Yourself 2
1. Write the equation of the circles passing through the point P with the center C given below
a. C(0, 0), P(–3, 4) c. C(0, –2), P(2, 3)
b. C(3, 0), P(1, 4) d. C(3, 4), P(0, 0)
2. Write the equations of the circles with the center C and tangent to given lines
a. C(2, 0), 5x – 12y + 10 = 0
b. C(3, –5), 3x – 4y + 12 = 0
3. Write the equation of the circle whose center is on the line 2y – x + 2 = 0 and tangent
to the lines y + 1 = 0 and y – 3 = 0.

4. Write the equation of the circle whose center is on the line 4y + x – 8 = 0 and whose
x – intercepts are A(1, 0), B(3, 0).

Equation of Circles Tangent to the Axes


y
1. Circles Tangent to x - axis
The radius of the circle which is tangent to y - axis with the b
C(a, b)

center C(a, b) is r = |b|, and it’s equation is


r=b
2 2 2
(x – a) + (y – b) = b .
x
a

Figure 6

300 Analytic Analysis of Lines and Circles


EXAMPLE 11 Find the equation of the circle which is tangent to the x - axis and whose center is C(3, –4).

y
Solution It is tangent to x - axis,
3
so r = |b|, r = |–4| = 4 then the equation is x

(x – 3)2 + (y + 4)2 = 42. r=4

–4
C(–4, 3)

2. Circles Tangent to y - axis y

The radius of the circle which is tangent to y - axis with the


center C(a, b) is r = |a|, then it’s equation is b
r=a C(a, b)

(x – a)2 + (y – b)2 = a2.

x
a

Figure 6

EXAMPLE 12 Find the equation of the circle which is tangent to y - axis and whose center is C(–3, 1)
y
Solution It is tangent to y - axis, so r = |a|, r = |–3| = 3 units then
the equation is (x + 3)2 + (y – 1)2 = 32.
C(–3, 1)
1
r=3
x
–3

Equation of Circles 301


y
3. Circles Tangent to Both Axes y=x

The radius of the circle which is tangent to both axes

C(
–r
r

r)
,r

r,
with the center C(a, b) is r = |a| = |b|.

C(
–r r
Hence, the equations of the circles will be as follows: x

r)

in the 1st quadrant (x – r)2 + (y – r)2 = r2

C(
–r
–r

r,
C(

–r
)
in the 2nd quadrant (x + r)2 + (y – r)2 = r2
in the 3rd quadrant (x + r)2 + (y + r)2 = r2 y = –x

in the 4th quadrant (x – r)2 + (y + r)2 = r2


Figure 8
in all cases, the centers of the circles are on the lines y = x or y = –x.

EXAMPLE 13 Write the equation of the circle which is tangent to the coordinate axes with the center
C(2, –2)

Solution Since the circle is tangent to the coordinates, r = |a| = |b|. Therefore, the equation of the
circle with the center C(2, –2) and r = 2 is (x –2)2 + (y + 2)2 = 4.

EXAMPLE 14 Write the equation of the circle which is in the 4th quadrant and tangent to x - axis, y - axis,
and the line 4x + 3y – 6 = 0.

Solution The center is C(a, b) = C(r, –r), since it is tangent to y


the axes in the 4th quadrant.
The distance of C(r –r) to the line 4x + 3y – 6 = 0 is
4x
+
3y

equal to r, because the circle is tangent to the line


–6
=
0

| 4 r − 3r − 6| x
r= ⇒ 5r = |r – 6| O
r r
42 + 32
r
r – 6 = ± 5r M(r, –r)

3
r1 = 1 and r2 = −
2
since r is non-negative, r = 1 and
C(r, –r) = C(1, –1).
We found the equation of the circle as (x – 1)2 + (y + 1)2 = 1.

302 Analytic Analysis of Lines and Circles


Check Yourself 3
1. Find the equation of the circle which is tangent to y - axis in the 3rd quadrant with radius
r = 3.
2. Find the equation of the circle which is tangent to both axes in the 2nd quadrant with
radius r = ñ5.
3. Find the equation of the circle whose center is at the line x + 2y – 4 = 0 and which is
tangent to both axes in the 2nd quadrant.

4. Find the equation of the circle which is tangent to both axes and passing through P(1, 3).

B. GENERAL EQUATION OF A CIRCLE


The standard equation of the circle whose center is C(a, b) with the radius r is
(x – a)2 + (y – b)2 = r2
By expanding this equation we obtain
x2 – 2ax + a2 + y2 – 2by + b2 = r2
or x2 + y2 – 2ax – 2by + a2 + b2 – r2 = 0
Now let us take D = –2a, E = –2b and F = a2 + b2 – r2 and substitute in the equation,
we get x2 + y2 + Dx + Ey + F = 0.

Definition
The equation x2 + y2 + Dx + Ey + F = 0 where D = –2a, E = –2b and F = a2+ b2 – r2 is
called the general equation of a circle.

Remark
In general form of the circle equation

D⎫
1. D = −2 a ⇒ a = −
2 ⎪⎪ D E
⎬ the center C( a, b) = C( − , − )
E⎪ 2 2
E = −2 b ⇒ b = − ⎪
2⎭

D2 E 1
2. F = a2 + b2 – r2 ⇒ r = a2 + b2 − F = + 2 − F ⇒ the radius r = D 2 + E2 − 4F
4 4 2

Analytic of Circle 303


EXAMPLE 15 Find the center and radius of the circle whose equation is x2 + y2 + 6x – 8y + 4 = 0.

D E 6 8
Solution C( − , − ) ⇒ C( − , − ) ⇒ C( −3, 4)
2 2 2 2
1 1 2 1
r= D2 + E2 − 4 F = 6 +( −8)2 − 4 ⋅4 = 100 −16 ⇒ r = 21.
2 2 2

EXAMPLE 16 Find the general equation of the circle whose center is C(–2, 3) and the radius is r = 5.

Solution 1 The general equation of the circle is x2 + y2 + Dx + Ey + F = 0 where


D = –2a, E = –2b and F = a2 + b2 – r2 then
D = –2 ⋅ (–2) = 4, E = – 2 ⋅ 3 = –6, and F = (–2)2 + 32 – 52 = –12
so the equation is x2 + y2 + 4x – 6y – 12 = 0.

Solution 2 Standard equation of the circle is (x – a)2 + (y – b)2 = r2


then (x + 2)2 + (y – 3)2 = 52
x2 + 4x + 4 + y2 – 6y + 9 – 25 = 0
we get x2 + y2 + 4x – 6y – 12 = 0.

EXAMPLE 17 If the circle x2 + y2 + 4x + (1 – k)y + k – 3 = 0 passes through the origin, find its radius.

Solution 1 The origin O(0, 0) is one of the points of the circle, so it’s coordinates satisfy the equation.
02 + 02 + 4 ⋅ 0 + (1 – k) ⋅ 0 + k – 3 = 0 ⇒ k – 3 = 0 ⇒ k = 3
and the equation is x2 + y2 + 4x – 2y = 0.

1 1 2 1
r= D 2 + E2 − 4 F ⇒ r= 4 +( −2)2 − 4 ⋅ 0 ⇒ r= 20 ⇒ r = 5.
2 2 2

Solution 2 For the equation x2 + y2 + 4x – 2y = 0


D E 4 2
the center is C(a, b) = C( − , − ) = C( − , ) = C( −2, 1)
2 2 2 2
and the distance between C(–2, 1) and the origin is the radius
and r = ( −2)2 +12 = 5 .

304 Analytic Analysis of Lines and Circles


EXAMPLE 18 If the circle x2 + y2 + 4x + ky + 3k – 8 = 0 is tangent to y – axis, find k.

Solution The circle is tangent to y - axis with the center C(a, b) then r = |a|.
1 D 1
D2 + E2 − 4F =| − | ; 16 2 + k2 − 4(3 k − 8) = 2
2 2 2
16+ k2 +12 k + 32 =16

k2 − 12k + 32 = 0

( k − 8)( k − 4) = 0 then k1 = 8 and k2 = 4.

EXAMPLE 19 Write the equation of the line passing through these three points K(–1, 0), L(0, 2) and
M(–1, –2).

Solution The general equation of the circle is x2 + y2 + Dx + Ey + F = 0. The coordinates of the


points K, L, M must satisfy this equation because they lie on this circle. So,
for K(–1, 0); 1 + 0 – D + 0 + F = 0 ⎧ –D + F = –1 ...(1)
⎪⎪
for L(0, 2); 0 + 4 + 0 + 2E + F = 0 ⇒ ⎨ 2E + F = –4 ...(2)

for K(–1, –2); 1 + 4 – D – 2E + F = 0 ⎪⎩ –D – 2E + F = 0 ...(3)
If we add (2) and (3) side by side, we get –D + 2F = –9,
and if we solve this equation simultaneously with (1) then we obtain F = –8 and D = –7.
In the equation (2) for F = –8, we find E = 2.
If we put the values of D, E, F to the general equation then we find the equation of the circle
x2 + y2 – 7x + 2y – 8 = 0

Check Yourself 4
1. Find the center and the radius of the circle x2 + y2 + 8x – 10y – 9 = 0.
2. Write the general equation of the circle whose center is C(–2, 3) and which is tangent to
the line 3x + 4y – 16 = 0.
3. Find the radius of the circle x2 + y2 – (8 – m)x + my – (13 + m) = 0 passing through the
point (–2, –1)

4. Find AB if the circle x2 + y2 + x – 3y – 12 = 0 intersects x - axis at A and B.

5. If the circle x2 + y2 – 4x + 8y + k = 0 is tangent to the line x = –1, find k.

Equation of Circles 305


Definition
Given the equation x2 + y2 + Dx + Ey + F = 0,
the expression D2 + E2 – 4F is called the discriminant of a circle.

1. When D2 + E2 – 4F < 0, r ∉ \, and the equation does not represent a circle.


D E
2. When D2 + E2 – 4F = 0, r = 0, and the equation represents only a point C( − , – ).
2 2
3. When D2 + E2 – 4F > 0, r ∈ \, and the equation represents a circle whose center
D E 1
is C( − , – ) and the radius is r = D 2 + E2 − 4 F .
2 2 2

Remark
In the general form of the circle equation, if F < 0, the discriminat D2 + E2 – 4F > 0, so it
always represents a circle.

EXAMPLE 20 Find the values of P so that x2 + y2 – 6x – 4y + P = 0 represents a circle.

Solution To represent a circle, discriminant must be positive.


D2 + E2 – 4F > 0
(–6)2 + (–4)2 – 4 ⋅ P > 0 ⇒ 52 – 4P > 0 ⇒ P < 13.

EXAMPLE 21 If x2 + y2 – 2x + y – m + 1 = 0 represents a point, find m.

Solution If it is just a point, r = 0.


D2 + E2 – 4F = 0 ⇒ (–2)2 + 12 – 4 ⋅ (–m + 1) = 0 ⇒ 4 + 1 + 4m – 4 = 0
1
4m = –1 ⇒ m = – .
4

Remark
The equation in the form
Ax2 + By2 + Cxy + Dx + Ey + F = 0 is called second degree equation in two variables x and y.
For this equation, if

1. A = B ≠ 0

2. C = 0 and
D E 4F
3. ( )2 + ( )2 − ( )2 > 0 , then the equation represents a circle.
A A A

306 Analytic Analysis of Lines and Circles


Note
Notice that when the equation Ax2 + By2 + Dx + Ey + F = 0 is used for the circle equation,
the equality should be rearranged so that A = B = 1.

EXAMPLE 22 If the equation (m – 1)x2 + (2m – 4)y2 + 4x + 6y + 4 = 0 represents a circle, find m.

Solution The coefficients of x2 and y2 must be equal


m – 1 = 2m – 4
m = 3.
Now, let’s check the value of ∆ in the equation 2x2 + 2y2 + 4x + 16y + 4 = 0
4 6 4⋅4
∆ =( )2 + ( )2 − = 5 > 0 so it is really the equation of the circle.
2 2 2

EXAMPLE 23 Show that the equation 2x2 + 2y2 – 8x + 6y + 8 = 0 represent a circle. Determine its
center and radius.

Solution The coefficients of x2 and y2 are equal and the coefficient of xy is O, we can arrange this
equation as
2(x2 + y2 – 4x + 3y + 4) = 0
x2 + y2 – 4x + 3y + 4 = 0
D2 + E2 – 4F = (–4)2 + 32 – 4 ⋅ 4 = 9 > 0.
Since the conditions are satisfied, the equation represent a circle.
D −4 E 3 3
a=− =− = 2 and b = − = − so C(2, − ) is the center.
2 2 2 2 2
1 1 1 3
r= D 2 + E2 − 4 F = ( −4)2 + 32 − 4 ⋅ 4 = 9 = units.
2 2 2 2

EXAMPLE 24 Analyse whether the following equations represent a circle, if so determine their center and
radius.
a. x2 + y2 – 12xy + 5 = 0 b. x2 + y2 + 2x – 4y – 11 = 0

Solution 1 a. There is a term containing xy in the equation of a circle. So x2 + y2 – 12xy + 5 = 0 is


not an equation of a circle.
b. If we compare this equation with the general equation of the circle then we find
D = 2, E = –4 and F = – 11
∆ = D2 + E2 – 4F = 64 > 0

Equation of Circles 307


So, it represents a circle.
D E
The coordinates of the center are a = − = −1 and b = − = 2
2 2
1 1
The radius of the circle is r = ∆= 64 = 4
2 2
Thus the center is C(–1, 2) and the radius is r = 4 units.
Solution 2 We can find the center and the radius by completing the equation of the circle in to
perfect square.
x2 + y2 + 2x – 4y – 11 = 0
x2 + 2x + y2 – 4y – 11 = 0
x2 + 2x + 1 – 1 + y2 – 4y + 4 – 4 – 11 = 0
(x + 1)2 + (y – 2)2 = 1 + 4 + 11
(x + 1)2 + (y – 2)2 = 16
C(–1, 2) and r = 4 units.

3m 3
EXAMPLE 25 If the radius of the circle 3x2 + 3y2 – 3x + 3y –
2
− = 0 is 3 units, find m.
2

m 1
Solution Divide the equation by 3, x2 + y2 – x + y – − =0
2 2
1 m 1 1 1
r= D2 + E2 − 4F ; r = (–1)2 +(1) 2 – 4( − + ) ; r = 1+1 +2 m – 2 = 2 m
2 2 2 2 2
1
r = 3 is given, 2 m = 3 ; 2 m = 36 ; m =18.
2

Check Yourself 5
1. Find the centers and the radii of the given circles

a. x2 + y2 = 16 b. 2x2 + 2y2 – 8x + 12y – 6 = 0


2. Find the values of k so that the equation 2x2 + 2y2 + 2kx + 2ò15y + 2k + 10 = 0
represents a circle.
3. Find the equation of the circle passing through the points A(0, 1), B(0, 3) and C(–1, 2)

4. If the equation (m – 3)x2 + (5 – m)y2 + 4x – 6y + 4 = 0 represents a circle, find the


radius and the center of the circle.

5. If the equation 2x2 + 2y2 – 6x + 8y – 4k = 0 represents a point, find k.

308 Analytic Analysis of Lines and Circles


EXAMPLE 26 Find the center of the circumscribed circle of the triangle formed by the lines
1
y = − x +5, y = 4 x − 4 and x = 0.
4

Solution Since the product of the slopes of the lines y


1
y = − x +5 and y = 4 x − 4 is –1, the triangle A(0, 5)
y = 4x – 4
4
is a right triangle. So, the the center of the
circle is on the hypothenuse.
1
y=– x+5
1 C(0, 1)
4
That is, it is on y - axis. The line y = − x +5 2
4 O
x
intersects y - axis at A(0, 5) and the line
y = 4x – 4 intersects y - axis at B(0, –4).
Due to AB is the diameter, the center is the B(0, – 4)
midpoint of A and B.
1
Therefore, the center is C(0, ) .
2

Equation of Circles 309


EXERCISES 1
A. Standard Equation of A Circle 11. Find the equation of the circle whose center is at
1. The equation (x – 3)2 + (y + 2)2 = 1 is given. the origin with r = ò10.

a. Find the center of the circle 12. Find the equation of the circle tangent to
b. Find its radius x - axis with the center C(3, –4).
c. Show in the analytic plane.
13. Find the equation of the circle tangent to
2. Find the equation of the circle whose diameter is y - axis with the center C(–3, 1).
KT with the endpoints K(–1, 2) and T(5, –6).
14. Find the equation of the circle tangent to both
axes in the 2nd quadrant with the radius r = ñ5.
3. Find the center and the radius of the circle
3 1
( x − )2 +( y + )2 = 7. 15. Find the equation of the circle tangent to both
2 2
axes in the 2nd quadrant and whose center is on
4. Find the equation of the circle with the center the line x + 2y – 4 = 0
C(2, –3) and tangent to the line 4x – 3y + 3 = 0.
16. Find the equation of the circle tangent to both
5. Find the equation of the circle passing through axes and passing through the point P(1, 3).
P(1, 3) with the center C(4, –1).
17. Find the equation of the circle whose center is on
6. Find the equation of the central circle passing the line x – 2y – 6 = 0 and tangent to both axes.
through P(–1, 3).
18. Find the equation of the circle concentric with
7. Find the equation of the circle with the center the circle x2 + y2 + 4x + 3y + 4 = 0 and tangent
C(4, –3) and to y - axis.

a. tangent to the x - axis B. General Equation of A Circle


b. tangent to the y - axis 19. Find the center and the radius of the circle
x2 y2 5
8. If the circle (x + m – 2)2 + (y + n – 3)2 = 25 is + + 3x − y +1= 0.
2 2 2
tangent to the both axes in the 2nd quadrant, find
m
the ratio . 20. Find the general equation of the circle whose
n
center is at C(–3, 5) and which is tangent to
9. Find the equation of the circle whose center is x - axis.
3
C( − , 0) with the radius r = 3ñ2.
2 21. If the equation
(1 – m)x2 + 2y2 – (m – 3)x – 12y – 6 = 0
10. Find the equation of the circle whose center is
3 represents a circle, find the coordinates of the
C(0, 7) with the radius r = .
2 center and the radius.

310 Analytic Analysis of Lines and Circles


22. If the circle x2 + y2 + 4x + (1 – k)y + k – 3 = 0 33. The lines 2x – y + 4 = 0, x + 2y – 8 = 0 and
passes through the origin, find its radius. x - axis form a triangle. Find the equation of the
circumscribe of the triangle.
23. If the equation 3x2 + 3y2 + 18x + 12y – 3k = 0
34. Find the equation of the circle passing through
represents a circle with the radius r = 6 units,
the origin and the points which are the intercepts
find k.
of the line 3x + 4y – 24 = 0.
24. If the equation x2 + y2 – 2x + 3y – 2m = 0 rep-
35. Find the equation of the circle whose center is on
resents a point, find m.
the line 2x – y + 2 = 0 and which is tangent to
the lines x + 1 = 0 and x – 3 = 0.
25. Find the equation of the circle with the center
C(–1, 1) and passing through the center of the 36. Find the equation of the circle whose abscissa of
circle x2 + y2 – 6x + 4y – 2 = 0. the center is –2 and which is tangent to the
parallel lines 2x + y – 3 = 0 and 2x + y + 7 = 0.
26. Find the equation of the circle passing through
the points A(2, –1), B(2, 0) and C(1, 0). 37. Find the equation of the circle whose ordinate of
the center is 4 and which is tangent to the
27. Find the general equation of the circle whose parallel lines 3x – y + 4 = 0 and 3x – y – 2 = 0.
center is on the line 3x – y – 6 = 0 and which is
tangent to both axis in the 4th quadrant. 38. Find m so that the circle
x2 + y2 + 6x – 8y + m = 0
28. Find the general equation of the circle whose will be tangent to x – axis.
center is on the line y = –1 and which is tangent
to the lines y = 1 and y = 7. 39. If PK with the endpoints P(3, –1) and K(a, b) is the
diameter of the circle x2 + y2 – 4x + 6y + F = 0,
29. Find k ⋅ p if the equation find the sum a + b + F.
(2k – 1)x2 + y2 + (p –2)yx + 2x – 2y + 3 = 0
represents a circle.
40. Find the radius of the circle passing through the
points A(3, 1) and B(–1, 3), and whose center is on
30. Find m so that the circle x2 + y2 – 2y + m = 0 the line y = 3x –2.
will be tangent to the line x = 2.
41. Find the general equation of the circle which is
Miscellenaous Exercises tangent to both axis in the 1st quadrant and whose
center is on the line x + y – 4 = 0
31. For ∆ABC, m(∠BAC) = 90°, B(–2, 4) and
C(4, –4) are given. Find the equation of the 42. Find the radius of the circle which is tangent to the
circumscribe of ∆ABC. lines 3x + 4y – 12 = 0 and 6x + 8y + 6 = 0.

32. For ∆ABC, B(1, 0) and C(7, 0) are the vertices, 43. Find the equation of the circle tangent to the lines
st
and A is in the 1 quadrant. Find the equation of x = 3 and x = –3 and whose center is on the line
the inscribed circle of ∆ABC. y = x – 1.

Equation of Circles 311


2 POSITIONS OF LINES AND CIRCLES
A. CORRESPONDING POSITIONS OF A LINE AND A CIRCLE
Let C be a circle with the center C(a, b) and the radius r, and l be a line The distance of the
center to the line l is CH = d in this case, there are 3 different positions with respect to each
other.

l l
C(a, b) l C(a, b) C(a, b)
d=r d
d
r
H
H
H
d>r d=r d<r
line does not intersect line is tangent line in tersects the circle
the circle to the circle at two different points.
dÇc=Æ d Ç c = {H}. d Ç c = {K, L}.

EXAMPLE 27 Examine the positions of the circle (x –2)2 + (y + 3)2 = 25 with the lines
a. 3x – 4y + 12 = 0 b. 3x + 4y – 19 = 0 c. 3x – 4y + 2 = 0

Solution C(2, –3) and r = 5 are given. We must find the distance between C and the lines.
|3 ⋅ 2 − 4 ⋅(–3) + 12|
a. d = =6>5 so, the line does not cut the circle.
32 +(–4)2
| 3 ⋅ 2+ 4 ⋅ (–3) – 19|
b. d = = 5 = r so, the line is tangent to the circle.
32 + 42
|3 ⋅ 2 − 4 ⋅(–3) + 2|
c. d = =4<5 so, the line intersects the circle at two different points.
32 +(–4)2

EXAMPLE 28 If the circle (x – 1)2 + (y – 2)2 = 5 is tangent to the line x – 2y + k = 0 find k.

Solution C(1, 2) is the center and radius r = ñ5.


|1⋅1+ 2 ⋅(–2)+ k| | k − 3|
d=r ⇒ = 5 ; = 5 ; | k − 3| = 5
2
1 +(–2) 2
5

k – 3 = 5 or k – 3 = –5
k = 8 or k = –2.

312 Analytic Analysis of Lines and Circles


EXAMPLE 29 Find the possible values of p so that the circle (x –3)2 + y2 = 9 and the line 4x + 3y + p = 0
intersect at two points.

Solution C(3, 0) is the center and radius r = 3 units.


|3 ⋅ 4+ 3 ⋅ 0+ p| |12+ p|
d= =
16 + 9 5

Since the circle and the line intersect at two points, d < r
|12+ p|
< 3 ; |12+ p|<15
5
–15 < 12 + p < 15
–27 < p < 3 ; p ∈ (–27,3)

Remark
Let us take a circle with center C(m, n) and a
N
radius r, and a line l : ax + by + c = 0.
The distance between the center and the line is r
d = CH. C(m, n)
1. The closest distance between the center and
the line is d = CH. d
r
l : ax + by + c = 0
2. The furthest point ofn the circle to the line
P
is N and that distance is HN = d + r.
H

EXAMPLE 30 Find the nearest and the furthes points on the N

circle (x – 1)2 + (y – 2)2 = 4 to the line


2
3x + 4y + 4 = 0 and find these distances.
C(1, 2)

d
2
Solution C(1, 2) is the center and r = 2 3x + 4y + 4 = 0
P
|1⋅ 3+ 2 ⋅ 4+ 4| 15 H
CH = = = 3 units
2
3 +4 2 5

then the nearest distance PH = CH – r = 3 – 1 = 1 unit


the furthest distance HN = CH + r = 3 + 2 = 5 units.

Positions of Lines and Circles 313


Check Yourself 6
1. Examine the positions of the circle (x –2)2 + (y – 1)2 = 9 with respect to following lines.
a. 2x + y – 2 = 0
b. 3x + 4y + 10 = 0
c. 4x + 3y + 4 = 0
2. Find the length of the part of the line 5x + 12y + 5 = 0 which lies inside the circle
(x – 2)2 + (y – 2)2 = 25.
3. If the distance between the line (x – m)2 + y2 = 4 and the circle
(x –1)2 + (y – 6)2 = m2 is 1 unit, find m.
4. If the line 3x – y + p = 0 is tangent to the circle x2 + y2 = 25, find p.

Positions of a line y = mx + n and a circle x2 + y2 + Dx + Ey + F = 0


If we solve the equation of the line y = mx + n and the equation of the circle
x2 + y2 + Dx + Ey + F = 0 simultaneously, we obtain a second degree equation with one
unknown.

Let ∆ be the discriminant of this second degree equation.

i. If ∆ < 0, then the line does not intersect the circle.

ii. If ∆ = 0, then the line is tangent to the circle. The point of tangency is the solution set
of this equation.

iii. If ∆ > 0, then the line intersects the circle at two different points and these points are
the solution set of the equation.

EXAMPLE 31 Example the position of y = 1 – x and x2 + y2 – 6x + 7 = 0.

Solution Let us substitute y = 1 – x in the circle equation.


x2 + (1 – x)2 – 6x + 7 = 0 ; 2x2 – 8x + 8 = 0 ; x2 – 4x + 4 = 0.
∆ = b2 – 4 ⋅ a ⋅ c ; ∆ = 16 – 4 ⋅ 1 ⋅ 4 = 0 so line is tangent to the circle.
The coordinates of the point of tangency
x2 – 4x + 4 = 0 ; (x – 2)2 = 0 ; x = 2.
The solution of x2 – 4x + 4 = 0 gives us the abscissa of the intersection point, to find the
ordinate, substitute this value in the line equation y = 1 – x.
y = 1 – 2= –1 so, (2, –1) is the point of tangency.

314 Analytic Analysis of Lines and Circles


EXAMPLE 32 Find all values of k if the line x – y + k = 0 is tangent to the circle x2 + y2 = 4.

Solution We write y = x + k in the equation x2 + y2 = 4 then we get


x2 + (x + k)2 = 4 ; x2 + x2 + 2kx + k2 = 4 ; 2x2 + 2kx + k2 – 4 = 0
If the line is tangent to the circle then ∆ must be equal to 0. Thus,
3 2
∆ = 42 – 4 ⋅ 2 ⋅ (k2 – 4) = 0 ; 4 – 8k2 + 32 = 0 ; 8k2 = 36 ; k = ± .
2

EXAMPLE 33 Find the coordinates of the intersection points of the line x – y + 1 = 0 and the circle
x2 + y2 – 3x – y – 1 = 0.

Solution Substitute y = x + 1 in the circle equation


x2 + (x + 1)2 – 3x – (x + 1) – 1 = 0
x2 + x2 + 2x + 1 – 3x – x – 1 – 1 = 0 ; 2x2 – 2x – 1 = 0
∆ = b2 – 4ac = 4 – 4(–1) ⋅ 2 = 12

−b ∓ ∆ 2+ 2 3 3 +1
x1,2 = ⇒ x1 = =
2a 2 ⋅2 2

2 – 2 3 1– 3
x2 = =
2⋅2 2
Now, substitute these values in the equation y = x + 1

3 +1 3 +1 3+3
For x1 = , y1 = + 1=
2 2 2

1– 3 1− 3 3− 3
For x2 = , y2 = + 1=
2 2 2

3 +1 3 + 3 1– 3 3 − 3
so the points are A( , ), and B( , ).
2 2 2 2

EXAMPLE 34 If x2 + y2 + 2x – y + p = 0 is tangent to y - axis, find p.

Solution The circle is tangent to y - axis, so let’s substitute x = 0 in the equation


we get y2 – y + p = 0, an we know ∆ = 0.
b2 – 4ac = 0
1–4⋅p=0
1
p= .
4

Positions of Lines and Circles 315


Check Yourself 7
1. Examine the position of y = –2x + 1 and x2 + y2 – x + 2y + 4 = 0 and find coordinates
of the intersection point(s), if any.
2. If A and B are x - intercepts of the circle x2 + y2 – 5x + 4y + 6 = 0, find AB.
3. Find r if the line 2x – y – 4 = 0 is tangent to the circle x2 + y2 = r2.

4. Find the length of the part of the line y = x + 1 which lies inside the circle
2x2 + 2y2 + 4x – 8y – 10 = 0.

5. If the line y = x – 1 is tangent to the circle x2 + y2 + 2x – 3y + k – 1 = 0, find k.

B. EQUATIONS OF TANGENT AND NORMAL LINES


Definition
A line which meets a circle at only one point is
n : normal
called a tangent to the circle, and this point is
defined as the point of tangency.
P(x0, y0)
The line which is perpendicular to the tangent at
the common point is called the normal of the
t : tangent
circle. C(a, b)

In the figure 9, the line t is tangent to the circle at


P(x0, y0) and the line n is normal to the circle at
P(x0, y0). Figure 9

1. Equations of Tangent and Normal Through a Point on a


Circle y
n : normal
Let P(x0, y0) be a point on the circle
(x – a)2 + (y – b)2 = r2.
t is the tangent passing through P(x0, y0) and n is P(x0, y0)

the normal to the circle at point p. (t ⊥ n)


C(a, b) t : tangent

x
O

Figure 10

316 Analytic Analysis of Lines and Circles


Slope of the normal line:
The normal passes through the point P(x0, y0) and the center C(a, b),
y0 − b
so, its slope is mN = .
x0 − a

Slope of the tangent line:


The tangent line is perpendicular to the normal line, so product of their slopes is equal to –1.
1 x −a
mT ⋅ mN = –1 then mT = − , mT = − 0 .
mN y0 − b

a. Equation of the tangent through a point P(x0, y0)


x0 − a
The slope of tangent is mT = and P(x0, y0) is a point of the tangent so by using the
y0 − b
point - slope form of the line equation y – y0 = m(x – x0), we can write equation of the
x −a
tangent y – y0 = − 0 ⋅( x − x0 ) or (y – y0) ⋅ (y0 – b) + (x – x0) ⋅ (x0 – a) = 0.
y0 − b

b. Equation of the normal through a point P(x0, y0)


y0 − b
The slope of normal is mN = and P(x0, y0) is a point of the normal so by using the
x0 − a
same equation, we can write the equation of the normal.
y −b
y – y0 = 0 ⋅ ( x − x0 ) or (y – y0) ⋅ (x0 – a) – (x – x0) ⋅ (y0 – b) = 0.
x0 − a

EXAMPLE 35 Find the equations of the normal and the tangent to the circle (x + 3)2 + (y – 2)2 = 10 at
the point P(–4, –1).

Solution 1 The center of the circle is C(–3, 2).


y − b −1 − 2 −3
i. Slope of the normal is mN = 0 = = = 3,
x0 − a −4+ 3 −1
and the equation of the normal is
y – y0 = mN(x – x0) ; y + 1 = 3 ⋅ (x + 4) ; y = 3x + 11

Positions of Lines and Circles 317


1 1
ii. Slope of the tangent is mT = − =−
mN 3
So, the equation of the tangent is y – y0 = mT ⋅ (x – x0)
1
y+1=− ⋅ (x + 4)
3
x + 3y + 7 = 0.

Solution 2 P(x0, y0) = P(–4, –1) and C(a, b) = C(–3, 2)

i. The equation of the tangent is


(y – y0) ⋅ (y0 – b) + (x – x0) ⋅ (x0 – a) = 0
(y – (–1)) ⋅ (–1 – 2) + (x + 4) ⋅ (–4 + 3) = 0 ; –3y – 3 – x – 4 = 0 ; x + 3y + 7 = 0.

ii. The equation of the normal is


(y – y0) ⋅ (x0 – a) – (x – x0) ⋅ (y0 – b) = 0
(y – (–1)) ⋅ (–4 + 3) – (x – (–4)) ⋅ (–1 – 2) = 0 ; –y – 1 + 3x + 12 = 0 ; 3x – y + 11 = 0.

EXAMPLE 36 Find the equations of the tangent and the normal to the circle (x + 1)2 + (y – 2)2 = 5 at the
point P(1, 3).

Solution The equation of the tangent passing through P(1, 3) is


(x0 – a) ⋅ (x – x0) + (y0 – b) ⋅ (y – y0) = 0
(1 + 1) ⋅ (x – 1) + (3 – 2) ⋅ (y – 1) = 0
2 ⋅ (x – 1) + 1 ⋅ (y – 3) = 0
2x + y – 5 = 0
The equation of the normal passing through P(1, 3) is.
(y0 – b) ⋅ (x – x0) – (x0 – a) ⋅ (y – y0) = 0
(3 – 2) ⋅ (x – 1) – (1 + 1) ⋅ (y – 3) = 0 ; x – 1 – 2y + 6 = 0 ; x – 2y + 5 = 0.

Check Yourself 8
1. Find the equation of the tangent and the normal lines to the circle (x –2)2 + (y + 3)2 = 25
passing through the point K(5, –1).
2. Find the equation of the tangent and the normal to the circle (x – 1)2 + (y + 2)2 = 20
passing through the point P(3, 2).

318 Analytic Analysis of Lines and Circles


Remark - 1
To find the equation of the normal and the tangent to tangent
y
the central circle x2 + y2 = r2 from a point on the
circle, the same notion can be used

al
rm
no
P(x0, y0)
the solope of the solope of r

the normal the tangent x

y0 x0
mN = mT = −
x0 y0

the equation of tangent: Figure 11

x0
y – y0 = − ⋅ (x – x0) ; y ⋅ y0 – y20 = –x ⋅ x0 + x20 ; y ⋅ y0 + x ⋅ x0 = x20 + y20
y0

y ⋅ y0 + x ⋅ x0 = r 2
the equation of normal:
y
y − y0 = 0 ( x − x0 ) ; y ⋅ x0 − x0 ⋅ y0 = x ⋅ y0 − x0 ⋅ y0
x0

y ⋅ x0 − x ⋅ y0 = 0

EXAMPLE 37 Find the equations of the tangent and the normal to the circle x2 + y2 = 10 at the point
P(3, 1).

Solution The equation of tangent passing through P(3, 1) is


x ⋅ x0 + y ⋅ y0 = r2 ; x ⋅ 3 + y ⋅ 1 = 10 ; 3x + y – 10 = 0
The equation of the normal
x0 ⋅ y – y0 ⋅ x = 0 ; 3 ⋅ y – 1 ⋅ x = 0 ; 3y – x = 0.

Remark - 2

D E
In the circle equation x2 + y2 + Dx + Ey + F = 0, the center is C(a, b) = C( − , − ) , then
2 2
the equation of the tangent to the circle passing through P(x0, y0) is
D E
(y – y0) ⋅ (x0 + ) – (x – x0) ⋅ (y0 + ) = 0,
2 2
and the equation of the normal to the circle passing through P(x0, y0) is
E D
(y – y0) ⋅ (y0 + ) + (x – x0) ⋅ (x0 + ) = 0.
2 2

Positions of Lines and Circles 319


EXAMPLE 38 Find the equations of the tangent and the normal to the circle
3x2 + 3y2 + (m – 2)xy – 3x + 3my – 15 = 0 passing through the point P(–2, 3).

Solution The coefficient of xy must be zero, m – 2 = 0 ; m = 2, and the equation is


3x2 + 3y2 – 3x + 6y – 15 = 0.
Divide the equation by 3, x2 + y2 – x + 2y – 5 = 0.
D E 1 2 1
Then C(a, b) = C( − , − ) = C( , − ) = C( , −1) is the center.
2 2 2 2 2
The equation of the tangent:
1
(y – 3) ⋅ (–2 – ) – (x + 2) ⋅ (3 + 1) = 0 ; 5y + 8x + 1 = 0
2
1
The equation of the normal: (y – 3) ⋅ (3 + 1) + (x + 2) ⋅ (–2 – ) = 0 ; 8y – 5x – 34 = 0.
2

Practical Way to Find the Equation of the Tangent Through P(x0, y0)
a. The equation x2 + y2 = r2 can be written as x ⋅ x + y ⋅ y = r2.
P(x0, y0) is a point of the circle and when we substitute the coordinates of P(x0, y0) into
the equation of circle, we get the equation of the tangent line: x ⋅ x0 + y ⋅ y0 = r2.
b. The equation (x – a)2 + (y – b)2 = r2 can be written as
(x – a) ⋅ (x – a) + (y – b) ⋅ (y – b) = r2 so the equation of the tangent line is
2
(x – a) ⋅ (x0 – a) + (y – b) ⋅ (y0 – b) = r .

c. The equation x2 + y2 + Dx + Ey + F = 0 can be written as


D
x⋅x+y⋅y+ ⋅ (x + x) + E(y + y) + F = 0, so the equation of the tangent line is
2
D
x ⋅ x0 + y ⋅ y0 + ⋅ (x + x0) + E ⋅ (y + y0) + F = 0.
2

EXAMPLE 39 Write the equation of the tangent drawn from A(2, 3) which is on the circle
x2 + y2 + 4x – 8y + 3 = 0.

Solution The equation of the tangent at the point (x0, y0) to the circle x2 + y2 + 4x – 8y + 3 = 0 is
D E
in the form x ⋅ x0 + y ⋅ y0 + ⋅ (x + x0) + ⋅ (y0 + y) + F = 0. Thus,
2 2
4 8
2x + 3y + ⋅ (2 + x) – ⋅ (3 + y) + 3 = 0 ; 2x + 3y + 4x + 2x – 12 – 4y + 3 = 0
2 2
4x – y – 5 = 0 is the equation of the tangent.

320 Analytic Analysis of Lines and Circles


Check Yourself 9
1. Find the equations of the tangent and the normal to the circle x2 + y2 = 25 at the point
P(–ñ5, 2ñ5).
2. Find the equations of the tangent and the normal to the circle x2 + y2 – 4x + 6y – 12 = 0
at the point P(5, 1).
3. Find the equation of the tangent to the circle (x – 6)2 + (y + 4)2 = 9 at the point P(6, –1).

2. Equations of Tangents Drawn From an External Point


There are two tangent lines drawn from an
y = m1x + n1
external point of a circle. Let y = mx + n be the K
equation of a tangent line.
r
1. P(x1, y1) satisfies the equation of the tangent C(a, b)
P(x1, y1)
line y = mx + n. r

2. Condition of tangency of a line to a point L


y = m2x + n2
and the distance formula between a point
and a line are used to find the values of m Figure 12
and n.
When we write the values of m and n in the equation y = mx + n, we get the equations of
the tangent lines:
y = m1x + n2
y = m2x + n2

EXAMPLE 40 Find the equations of the tangent lines to the circle (x – 2)2 + (y – 3)2 = 4 draw from the
external point P(6, 3).

Solution Let y = mx + n be the equation of the tangent t1


line. T

P(6, 3) is on the tangent line, so 2


C(2, 3)
P(6, 3)
3 = 6m + n ; n = 3 – 6m. 2

The distance of C(2, 3) to the tangent line S


t2
y = mx + n or mx – y + n = 0 is
Figure 12
| 2 m − 3+ n |
2=
m2 +1

2 m2 +1 =| 2 m − 3+ n |

Positions of Lines and Circles 321


When n = 3 – 6m is substituted in the equation

2 m2 +1 =|2 m − 3+ 3 − 6 m| ; 2 m 2 +1 =| −4 m| ; 4( m 2 +1) =( −4 m) 2

4 1 3
4m2 + 4 =16 m2 ; 12 m2 − 4 = 0 ; m2 = ; m= ∓ ; m= ∓ .
12 3 3

3 3
Since n = 3 – 6m, for m1 = , n1 = 3 − 6 ⋅ = 3 − 2 3,
3 3
− 3 − 3
for m2 = , n2 = 3 − 6 ⋅( ) = 3+ 2 3.
3 3
Hence the equations of the tangent lines are
3
t1 : y = x+ 3 – 2 3
3
− 3
t2 : y = x + 3+ 2 3
3

EXAMPLE 41 Write the equation of the line passing through the point K(5, 0) which is tangent to the
circle x2 + y2 = 16 at the 1st quadrant.

Solution A is the point of tangency. y

y = mx + n is the equation of the tangent.


K(5, 0) = K(x1, y1) ⇒ y = mx + n
A
0 = 5m + n
r
n = –5m ...(i) K(5, 0)
x
O
OA = 4 and C(a, b) = C(0, 0) then
| y1 − mx1 − n |
OA = ; n2 =16(1+ m2 ) ...( ii)
1+ m2
Solving (i) and (ii) simultaneously, we get Figure 14

4 20
(–5m)2 = 16 + 16m2 ; 9m2 = 16 ; m1 = ± ; n1 = ± .
3 3
The tangent is in the 1st quadrant. Hence,
4 20
y = − x+ is the equation of the line.
3 3

322 Analytic Analysis of Lines and Circles


Check Yourself 10
1. Find the equations of the tangent lines to the circle x2 + y2 = 10 drawn from the
external point P(2, 4).
2. If m1 and m2 are the slopes of the tangent lines to the circle x2 + y2 = 4 drawn from the
external point P(3, 1), find m1 + m2. y

C
3. In the figure d is tangent to the circle at T, and the circle is tangent
T
to x - axis at the origin. The equation of the line is 4x – 3y – 12 = 0, x
O E
find the equation of the circle.
F

C. CORRESPONDING POSITIONS OF TWO CIRCLES


Let C1 and C2 be centers, r1 and r2 be radii of two excentric circles.
C1C2 = d is the distance between the centers of the circles.

1. If d = r1 + r2, circles are tangent to each other C1 r1 r2 C2

externally. d

Figure 15

2. If d > r1 + r2, circles do not intersect. C1 r1 E F r2 C2

Figure 16

E
C1 r1
3. If |r1 – r2| < d < r1 + r2, circles intersect at two r2
points. C2
d
F

Figure 17

Positions of Lines and Circles 323


4. If d = |r1 – r2|, circles are tangent to each other
internally.

r1

C1
C2 r
2
d

Figure 18

5. If d > |r1 – r2|, circles do not intersect and one of


them is completely inside of the other.
C1
C2
d

Figure 19

EXAMPLE 42 If the circles (x – 1)2 + (y + 2)2 = 4 and (x + 2)2 + (y – 2)2 = r2 are externally tangent to
each other, find r.

C1(1, − 2) and r1 = 2 ⎫⎪
Solution The centers and the radii of the circles are ⎬
C2 ( −2, 2) and r2 = r ⎪⎭
The circle are externally tangent, so C1C2 = d = r1 + r2

(1 − ( −2))2 +( −2 − 2) 2 = 2+ r ; 5 = 2+ r ; r = 3

EXAMPLE 43 Find the distance between the nearest points of the circles (x – 2)2 + (y + 3)2 = 9 and
(x + 3) + (y – 9)2 = 16
Solution The centers and the radii of the circles are
C1(2, − 3) and r1 = 3 ⎫⎪
C1 3 E F 4 C2

C2 ( −3, 9) and r2 = 4 ⎪⎭
x

C1C2 = ( −3 − 2)2 +(9 − ( −3))2 =13


C1C2 = r1 + EF + r2 ; 13 = 3 + EF + 4 ; EF = 6 units.

324 Analytic Analysis of Lines and Circles


EXAMPLE 44 If the circles (x – 2)2 + (y – b)2 = 2 and (x + 3)2 + (y – 2)2 = 72 are internally tangent to
each other, find b.
Solution The centers and the radii of the circles are
C1(2, b) and r1 = ñ2
C2(1, 2) and r2 = 6ñ2

C1C2 = d =| r1 − r2 | ; (2 − 1)2 +( b − 2) 2 =| 2 −6 2 |

1 + (b – 2)2 = (5ñ2)2 ; (b – 2)2 = 49


b+2=7 or b – 2 = –7
b = 11 or b = –5

Note r1
r2
2 2
If two intersecting circles satisfy the equality d2 = r1 + r2 they are C2
d
C1
called intersecting perpendicularly.

Figure 20

EXAMPLE 45 If the circles (x – 2)2 + (y + 4)2 = r2 and (x + 4)2 + (y – 4)2 = 64 are intersecting
perpendicularly, find r.
2 2
Solution d2 = r1 + r2
C1(2, − 4) and r1 = r ⎫⎪
2 2
⎬ ⇒ C1C2 = d = (2 − ( −4)) + ( −4 − 4) =10
C2 ( −4, 4) and r2 = 8 ⎪⎭

r2 + 82 = 102 ; r = 6 units.

EXAMPLE 46 If a chord of the circle x2 + y2 – 2x – 4y – 95 = 0 is tangent to the circle


x2 + y2 – 2x – 4y – 59 = 0. Find the length of this chord.

Solution The centers and the radii are


H
C1(1, 2) and r1 =10 ⎫⎪ A B
⎬ 10 8
C2 (1, 2) and r2 = 8 ⎪⎭
C(1, 2)
Notice that the circles are concentric,
CH = 8 and CA = 10
CH ⊥ AB and AH2 = 102 – 82 so AH = 6 units.
AH = HB then, AB = 2 ⋅ 6 = 12 units.

Positions of Lines and Circles 325


Check Yourself 11
1. Find the equation of the circle whose center is C(3, –4) and which is internally tangent
to the circle x2 + y2 = 64.
2. If the circles (x – 5)2 + (y + 2)2 = 5 and (x – 3)2 + (y + 5)2 = r2 are externally tangent,
find r.
3. Find the equation of the circle with radius r = 10 units which is externally tangent to the
circle x2 + y2 = 100 at the point P(–8, 6).
y
4. In the figure the circles with the centers O1 and O2 are
externally tangent to each other. The circle with the center
O1 is tangent to both axes, and the other is to x - axis. The
O2
points A(2, 0) and B(10, 0) are given. Find the equation of O1
the circle with the center O2. x
O A B

326 Analytic Analysis of Lines and Circles


EXERCISES 2
A. Corresponding Positions of A Line 8. In the figure the circle is y

and A Circle tangent to the line at T. P(0, 8)

1. If the line y = 2x + k is tangent to the circle P(0, 8) and OT = 4 units


x2 + y2 = 4 find k. are given. M

Find the equation of the T


circle.
x
O
2. Find the coordinates of the intersection points of
the line y – 3x = 0 and the circle
x2 + y2 + 9x –6y – 20 = 0.
9. The line y = 3 is tangent to the circle
x2 + y2 – 6x + 3k – 2 = 0, find k.

3. If the circle x2 + y2 – 4x + 8y + m = 0 is tangent B. Equations of Tangent and Normal


to y – axis, find m. Lines
10. Find the equations of the tangent and the normal
lines to the circle x2 + y2 + 4x – 8y – 5 = 0
4. If P and K are the x – intercepts of the circle passing through the point P(1, –1).
x2 + y2 – x + 2y – 12 = 0, find PK.

11. Find the coordinates of the x - intercept of the


tangent line passing through P(2, –3) to the circle
x2 + y2 = 13.
5. Find the length of the part of the line
2x + y + 1 = 0, which lies inside the circle
12. Find the equations of the tangents to the circle
(x – 1)2 + (y – 2)2 = 13.
x2 + y2 = 4 drawn from the external point P(3, 1).

6. If the line 3x + y + p = 0 is tangent to the circle 13. Find the equations of the tangents to the circle
(x – 2)2 + (y + 3)2 = 10, find p. (x – 1)2 + (y – 2)2 = 9 drawn from the external
point P(4, 6)

7. If the line 3x – 4y + k = 0 and the circle 14. If A and B are the intersection points of the
2 2
(x – 1) + (y + 2) = 16 intersect each other at tangent lines drawn from the external point P(–3, 7)
two points, find k. to the circle (x – 3)2 + (y + 1)2 = 25, find AB.

Positions of Lines and Circles 327


15. Find the equations of the normal and tangent 22. Find the equation of the circle with the center
lines passing through P(–1, 1) to the circle C(–2, 3) which is internally tangent to the circle
x2 + y2 – 4x + 6y – m = 0. (x + 2)2 + y2 = 64.

y 23. If the circles (x– 2)2 + (y – 3)2 = 36 and


16. In the figure, the circle (x + 1)2 + (y + 1)2 = k2 are internally tangent to
is tangent to y - axis at P(5, 3) each other, find k.
the origin. Find the
equation of the tangent
x
passing through P(5, 3). O C Miscellaneous Exercises
24. Find the measure of the angle formed by the
tangent lines drawn from the external point
P(4, 4ñ3) to the circle x2 + y2 = 16.

17. Find KL, if K and L are the points of tangency of


the tangent line drawn from the point P(5, 12) to 25. If A and B are the points of tangency of the
the circle x2 + y2 = 52. tangents drawn from P(8, 10) to the circle
(x + 12)2 + (y + 5)2 = 225, find the equation of
the line AB.
C. Corresponding Positions of Two
Circles
26. In the figure, the circle y
18. Find the smallest distance between the circles
is tangent to x - axis at
(x – 1)2 + (y + 2)2 = 4 and (x + 5)2 + (y – 6)2 = 25.
P and intersecting with K

the y - axis at K and L.


If the equation of KP is
19. If the circles (x – 3)2 + (y + 1)2 = 9 and
3x + 2y –18 = 0, find L
(x + 1)2 + (y – 2)2 = r2 are externally tangent to x
the equation of the O P
each other, find r.
circle.

20. If the circles x2 + y2 – 4x – 6y – 12 = 0 and 27. If the area between the circles x2 + y2 = 92 and
2 2 2
(x + 1) + (y + 1) = r are intersecting x2 + y2 = r2 is 27π unit2, find r.
perpendicularly, find r.

28. If A and B are the point of tangency of the tangent


21. Find the equation of the circle with the center lines drawn from the external point P(4, –5) to
C(–5, 4) which is externally tangent to the circle the circle (x + 4)2 + (y – 1)2 = 30, find
(x – 1)2 + (y + 4)2 = 16. x and y - intercepts of the line AB.

328 Analytic Analysis of Lines and Circles


3 POWER OF A POINT, RADICAL AXIS, & RADICAL
CENTER
A. POWER OF A POINT
Definition
On the analytic plane, let C(a, b) be the center of a
circle with radius r and Q(x1, y1) be any point and d be T
r
the distance between the point Q(x1, y1) and the center
Q(x1, y1) d
of the circle. The value of d2 – r2 is called the power of
C(a, b)
the point Q(x1, y1) with respect to the circle. The power A
of the point Q(x1, y1) is denoted by the capital letter P, B
that is.
Figure 21
P = d2 − r 2

P = ( x1 − a)2 +( y1 − b)2 − r 2

Property 1. If Q(x1, y1) is at the exterior region of the circle then d > r, we get, d2 – r2 = P > 0.
2. If Q(x1, y1) is on the circle then d = r, we get, d2 – r2 = P = 0
3. If Q(x1, y1) is at the interior region of the circle then d < r, we get, d2 – r2 = P < 0.

Note
QT 2 = QA ⋅ QB and QT 2 = QC 2 – r 2 then d 2 – r2 = QA ⋅ QB

Remark
The length of a tangent segment drawn from the point Q(x1, y1) to the circle is calculated by
QT2 = d2 – r2.
Refer to the figure 21
In the right triangle QTC, QT2 + CT2 = QC2
QT2 + r2 = d2
QT2 = d2 – r2 we know that d2 – r2 = P,
QT2 = P
QT = ñP
Hence, the square root of the power of a point Q(x1, y1) with respect to a circle is equal to the
length of the tangent segment drawn from the point Q(x1, y1) to the circle.

Power of a Point, Radical Axis, & Radical Center 329


EXAMPLE 47 Find the power of the point Q(3, 5) with respect to the circle (x + 1)2 + (y – 2)2 = 16.

Solution P = d2 – r2
P = (x1 – a)2 + (y1 – b)2 – r2
P = (3 + 1)2 + (5 – 2)2 – 16
P = 16 + 9 – 16
P = 9 > 0 so Q(3, 5) is at the exterior region of the circle.

Conclusion
1. The power of the point Q(x1, y1) with respect to the circle
x2 + y2 = r2 is P = x21 + y21 – r2

2. The power of the point Q(x1, y1) with respect to the circle
(x – a)2 + (y – b)2 = r2 is P = (x1 – a)2 + (y1 – b)2 – r2.

3. The power of the point Q(x1, y1) with respect to the circle
x2 + y2 + Dx + Ey + F = 0 is P = x21 + y21 + Dx1 + Ey1 + F = 0.

EXAMPLE 48 Find the power of the point Q(–3, 1) with respect to the circle x2 + y2 = 12.

Solution By substituting the coordinates of the point Q(–3, 1) in the equation Q = x21 + y21 – r2, we get,
P = (–3)2 + 12 – 12
P = –2 < 0. So, the point Q(–3, 1) is at the interior region of the circle.

EXAMPLE 49 What is the power of the point A(5, –4) with respect to the circle (x – 3)2 + (y + 4)2 = 4.

Solution P = (x1 – a)2 + (y1 – b)2 – r2


P = (5 – 3)2 + (–4 + 4)2 – 4
P=4–4
P = 0. So, the point A(5, –4) is on the circle.

330 Analytic Analysis of Lines and Circles


EXAMPLE 50 Find the length of the tangent segment drawn from the point B(1, 2) to the circle
x2 + y2 + 5x + 3y – 7 = 0

Solution Let BT be the tangent segment drawn from the point B(1, 2) to the circle.
Recall that PT = ñP and firstly let us find the power of the point B(1, 2) with respect to the
circle.
P = 12 + 22 + 5 ⋅ 1 + 3 ⋅ 2 – 7
P=1+4+5+6–7
P = 9.
Therefore, BT = ñP
BT = ñ9
BT = 3 units.

EXAMPLE 51 If the length of the tangent segment drawn from the point K(4, –1) to the circle
x2 + y2 + 4x – 6y + k = 0 is 5 units then find k.

Solution If T if the tangent point,


KT = ñP
5 = ñP and P = 25.
P = x21 + y21 + Dx1 + Ey1 + F = 0
25 = 42 + (–1)2 + 4 ⋅ 4 – 6 ⋅ (–1) + k
25 = 36 + k
k = –14.

EXAMPLE 52 If the center of the circle (x – a)2 + y2 = 4 is at the interior region of the circle
x2 + (y – 3)2 = 25, find the value of a.

Solution The center of the circle (x – a)2 + y2 = 4 is C(a, 0).


The power of the point C(a, 0) with respect to the circle x2 + (y – 3)2 = 25 must be
negative because it is at the interior region of the circle.
P<0
a2 + (0 – 3)2 – 25 < 0
a2 – 16 < 0 and we get –4 < a < 4.

Power of a Point, Radical Axis, & Radical Center 331


Note
N
In the figure 22, the points P, K, C and N are collinear. K is
r
the closest point and N is the furthest point of the circle to
C(a, b)
the point P(x1, y1) then
r
1. PK = PC – r is the distance of the nearest point of the
circle, K
2. PN = PC + r is the distance of the furthest point of the
circle to the point P(x1, y1).
P(x1, y1)
Figure 22

EXAMPLE 53 Find the distance of the point P(3, 1) to the closest and the furthest points of the circle
(x + 1)2 + (y – 4)2 = 4.

Solution Refer to the figure 23 N


2
PK = PC – r and, PN = PC + r
C(–1, 4)
= (4 – 1)2 +( −1 − 3) 2 − 2 =5+2
2
= 7 units.
= 9+16 − 2
K
=5–2
= 3 units P(3, 1)

Figure 23

EXAMPLE 54 If the point M is the nearest point of the circle (x – 3)2 + (y + 1)2 = 9 to the point P(–1, 2),
find the coordinates of the point M.
N
Solution PM = PC – r
3
= ( −1 – 3)2 +(2+1) 2 − 3
C(3, 1)
= 16+9 − 3 3

= 2 units. M(x0, y0)

PM 2
M ∈ PC and = k=
MC 3 P(–1, 2)
2
That is, M divides the line segment PC in the ratio. k=
3
2 2 4
−1+ ⋅ 3 2+ ⋅( −1)
x1 + kx2 3 1 3 y1 + ky2 3 3 = 4.
x0 = = = = , y0 = = =
1+ k 2 5 5 1+ k 2 5 5
1+ 1+
3 3 3 3
3 4
So, M( x0 , y0 ) = M( , ).
5 5

332 Analytic Analysis of Lines and Circles


Check Yourself 12
1. Find the power of the point Q(–5, 3) with respect to the circle x2 + y2 = 32.
2. If the power of the point Q(2, –1) with respect to the circle x2 + y2 – 4x – 6y + m = 0 is
5, then find m.
3. If the tangent segment drawn from the point A(–1, 4) to the circle x2 + y2 + 6x – 4y – 3 = 0
touches the circle at the point B, find the length of AB.
4. Find the distance of the nearest and the furthest points of the circle x2 + y2 + 2x – 8y – 8 = 0
to the point P(7, 2).

B. RADICAL AXIS OF TWO CIRCLES


Definition
The set of all points which have equal power with respect to two circles is called radical axis
of the circles.

The radical axis of two circles is perpendicular to the line segment joining the centers of the
circles.
Examine the following figures

Q
Q

T T
S
S
H H
C1 C2 C1 C2

The line QH is the radical axis The line QH is the radical axis
2 2
QT = QS QT 2 = PS2 = PH2
QH ⊥ C1C2 QH ⊥ C1C2

Power of a Point, Radical Axis, & Radical Center 333


Q
T
Q

E S
H
C1 C2
C1 H C1

The line QH is the radical axis The line QH is the radical axis
QH ⊥ C1C2 QT2 = QS2 = QE ⋅ QF
QH ⊥ C1C2

To find the equation of the radical axis of two circles the general equations of the circles are
solved simultaneously.
Let x2 + y2 + D1x + E1y + F1 = 0 and x2 + y2 + D2x + E2y + F2 = 0 be the equations of
given two circles, then
x2 + y2 + D1x + E1y + F1 = 0 ...(1)
x2 + y2 + D2x + E2y + F2 = 0 ...(2)
(1) – (2) : (D1 – D2)x + (E1 – E2)y + F1 – F2 = 0 is the equation of the radical axis.

EXAMPLE 55 Find the radical axis of the circles x2 + y2 + 4x – 2y – 4 = 0 and x2 + y2 – 2x + y – 1 = 0.

Solution x2 + y2 + 4x – 2y – 4 = 0
x2 + y2 – 2x + y – 1 = 0 subtract equations side by side,
––––––––––––––––––––––––––––––
6x – 3y – 3 = 0 and 2x – y – 1 = 0
Every point taken on the radical axis 2x – y – 1 = 0, have the same power with respect to
both circles. For example, point Q(2, 3) is a point on the radical axis and the powers of Q
with respect to the circles are
Q1 = 22 + 32 + 4 ⋅ 2 – 2 ⋅ 3 – 4 = 11 ⎫⎪
⎬ P1 = P2 .
Q2 = 22 + 32 – 2 ⋅ 2 + 3 – 1 = 11 ⎪⎭

334 Analytic Analysis of Lines and Circles


EXAMPLE 56 Given the circles x2 + y2 – 4x – 4y = 0 and x2 + y2 + 6x = 0
a. draw the circles on the analytic plane.
b. show that the radical axis is perpendicular to the line segment joining the centers of the
circles.

Solution a. x2 + y2 – 4x – 4y = 0 y
D E ( −4) ( −4)
C1( − , − ) = C1( − , ) = C1(2, 2)
2 2 2 2
4
1 1 r1
r1 = D 2 + E2 − 4 F = 16 + 16 − 0 = 2 2 units
2 2 B 2 C1
x2 + y2 +6 x = 0 C2
x
– 3 r2 O 2
6 0
C2 ( − , ) = C2 ( −3, 0)
2 2 y=– 5x
2
1
r2 = 36 + 0 − 0 = 3 units.
2
Figure 23
b. Now let’s find the equation of the radical axis,
x2 + y2 – 4x – 4y = 0
x2 + y2 + 6x = 0
–––––––––––––––––––––– 5
–10x – 4y = 0 and y = − x.
2
5
Notice that, the slope of the radical axis is m1 = − .
2
0−2 2
The slope of the line segment joining the centers C1 and C2 is m2 = = .
−3 − 2 5
5 2
m1 ⋅ m2 = ( − ) ⋅ = −1 therefore the radical axis is perpendicular to the line joining the
2 5
centers.

EXAMPLE 57 If the radical axis of the circles x2 + y2 + 4x + 2y + 3 = 0 and


x2 + y2 + 3x – (m – 1)y + m + 1 = 0 passes through the point A(1, 2), find the value of m.

Solution x2 + y2 + 4x + 2y + 3 = 0
x2 + y2 + 3x – (m – 1)y + m + 1 = 0
––––––––––––––––––––––––––––––––––––
x + (2 + m – 1)y + 3 – m – 1 = 0
x + (m + 1)y + 2 – m = 0
The point A(1, 2) satisfies the line x + (m + 1)y + 2 – m = 0
we get, 1 + (m + 1) ⋅ 2 + 2 – m = 0 ; 1 + 2m + 2 + 2 – m = 0 ; 5 + m = 0 ; m = –5.

Power of a Point, Radical Axis, & Radical Center 335


Check Yourself 13
1. Find the equation of the radical axis of the circles
x2 + y2 – 5x + 4y – 11 = 0 and x2 + y2 + 7x – 9 = 0.
2. Find the x and y - intercepts of the radical axis of the circles
2x2 + 2y2 + 4x – 2y + 2 = 0 and x2 + y2 + 6x + 4y + 9 = 0

C. RADICAL CENTER OF THREE CIRCLES


Definition
The point of intersection of the respective radical axes of each pair of circles selected from
the given three circles which have non-collinear centers, is called the radical center of the
circles.

The radical center has the same power with respect to three circles.

d1

C1 C2
P is the radical center of the circles with the
centers C1, C2 and C3.
P
To find the coordinates of the point P it is
enough to find the intersection point of any pair
of radical axes selected from d1, d2 and d3. d3 d2

C3

Figure 23

EXAMPLE 58 Find the radical center of the circles x2 + y2 + x – 2y – 2 = 0, x2 + y2 + 3x + y – 4 = 0,


and x2 + y2 – x + 2y – 6 = 0.

Solution x2 + y2 + x – 2y – 2 = 0 x2 + y2 + x – 2y – 2 = 0 x2 + y2 + 3x + y – 4 = 0
x2 + y2 + 3x + y – 4 = 0 x2 + y2 – x + 2y – 6 = 0 x2 + y2 – x + 2y – 6 = 0
––––––––––––––––––––––––– ––––––––––––––––––––––––– –––––––––––––––––––––––––
–2x – 3y + 2 = 0 2x – 4y + 4 = 0 4x – y + 2 = 0
d1 : 2x + 3y – 2 = 0 d2 : x – 2y + 2 = 0 d3 : 4x – y + 2 = 0

336 Analytic Analysis of Lines and Circles


Since d1, d2 and d3 intersect at one point it is enough to find the intersection point of any pair
of radical axes.
2x + 3y – 2 = 0
x – 2y + 2 = 0
–––––––––––––––––
2x + 3y – 2 = 0
– 2x + 4y – 4 = 0
–––––––––––––––––
6
7y – 6 = 0; y= and
7
6
x − 2⋅ +2 = 0
7
12
x − +2 =0
7
2 2 6
x=− so, P( − , ) is the radical center of three circles.
7 7 7

2 6
The point P( − , ) must satisfy the third radical axis that is,
7 7
4x − y + 2 = 0

2 6 14
4 ⋅( − ) − + 2 = − + 2
7 7 2
−2+ 2 = 0.

Check Yourself 14
Find the radical center of the circles, (x –1)2 + (y + 2)2 = 26, x2 + y2 = 16 and
x2 + y2 + 5x – 6 = 0.

Power of a Point, Radical Axis, & Radical Center 337


EXERCISES 3
A. Power of A point 8. If the length of the tangent segment drawn from
1. Find the power of the point Q(–2, –3) with the point K(3, a) to the circle x2 + y2 = 16 is ò42,
respect to the circle x2 + y2 – 3x + 4y + 10 = 0 find a.

9. If the distance of the nearest and the furthest


2. If the point B(2, 5) is at the exterior region of the
points of the circle x2 + y2 – 4x + 2y – 4 = 0 to
circle x2 + y2 – 2x + 3y + m = 0, find m.
the point P(3, 2) are m and n, respectively, then
m
find the value of .
n

3. Prove that the point K(4, –4) is at the interior


region of the circle x2 + y2 – x + 3y – 17 = 0. 10. In the figure, Q, A and B
B are collinear points.
If 5 ⋅ QA = 3 ⋅ QB,
then find the length of
4. Find the length of the tangent segment drawn the line segment AB. A

from the point A(1, –4) to the circle


x2 + y2 – 5x + 4y + 8 = 0.
Q(5, 1)

5. If the length of the tangent segment drawn from the 11. Find the coordinates of the nearest and the furthest
point Q(–2, –3) to the circle (x + 2)2 + (y – 1)2 = r2 points of the circle (x –3)2 + (y + 4)2 = 1 to the
is 3 units, then what is the radius of the circle? origin.

6. Find the distance of the nearest point of the cir- B. Radical Axis of Two Circles
2 2
cle (x – 2) + (y + 1) = 9 to the point P(–1, 3). 12. Write the equation of the radical axis of each pair
of circles given below.

a. (x – 1)2 + (y + 5)2 = 4 and


(x + 5)2 + (y + 1)2 = 9
7. Find the distance of the furthest point of the
circle x2 + y2 – 16x – 16y + 92 = 0 to the point b. (x + 2)2 (y – 3)2 = 10 and
P(3, –4). (x – 1)2 + (y + 2)2 = 5

338 Analytic Analysis of Lines and Circles


13. Find the equation of the line containing the common D. Mixed Exercises
2 2
chord of the circles x + y – 4x + 2y + 4 = 0 20. Given the points A(–3, –4), B(–5, 3), C(2, –2) and
and x2 + y2 – 2x + 6y + 5 = 0. the circle x2 + y2 – 3y – 25 = 0, find in which
regions of the circle are these points.
14. Find the value of m, if the common chord of the
circles (x – 1)2 + (y + 2)2 = 4 and
x2 + y2 – 4x + 2y + m = 0 passes through the
point A(1, –2).

21. What is the distance of the nearest point of the


15. If the slope of the line passing through the circle x2 + y2 – 16x – 16y + 92 = 0 to the point
intersection points of the circles A(3, –4).

x2 + y2 – 3x + 5y – 6 = 0 and
2x2 + 2y2 + 6x + 2ay – 4 = 0 is 2, find a.

16. Find the x and y -intercepts of the radical axis of 22. If the one of the radical axes of the circles
the circles (x + 3)2 + (y – 2)2 = 8 and x2 + y2 + ax + 3y – 1 = 0 and
x2 + y2 – 3x + 8y + k = 0. 2x2 + 2y2 + 3x + by – 5 =0 is 7x – 5y – 3 = 0,
then find the of b – a.

C. Radical Center of Three Circles


17. Find the radical center of the following circles
23. What are the coordinates of the furthest point of
given by the equations
the circle (x – 2)2 + (y + 3)2 = 225 to the point
x2 + y2 = 2
A(–1, 1)?
x2 + y2 – 4x – 6 = 0
x2 + y2 + 6y – 8 = 0.

18. Find the coordinates of the radical center of the


circles given by the equations 24. If the radical axis of the circles
x2 + y2 – 4x – 4y – 1 = 0 x2 + y2 + x – y – 3 = 0 and
x2 + y2 + 2x – 4y + 1 = 0 3x2 + 3y2 + 12x – 12y + a = 0 is the line y = x,
x2 + y2 + 6x – 2y – 6 = 0. then find a.

19. Find the sum of the coordinates of the radical


center of the circles given below.
x2 + y2 – 4 = 0 25. What is the length of the tangent segment drawn
2
(x – 2) – (y + 1) = 9 2
from the point P(4, 2) to the circle
(x + 3)2 + (y – 1)2 = 16. x2 + y2 – 4x – 6y + 12 = 0.

Power of a Point, Radical Axis, & Radical Center 339

You might also like